Barron's Reading Workbook for the NEW SAT, 1st edition...The new SAT has two required sections—(1)...

753

Transcript of Barron's Reading Workbook for the NEW SAT, 1st edition...The new SAT has two required sections—(1)...

Dedication

DedicatedtomywifeCaitlin,mysonAndrew,andmydaughterEloise—withoutyourloveandsupport,this book would not have been possible. I would like to especially thank my mom, my dad, Andy,Mitchell,Mercedez,Hannah,Alaina,Andrew,andDougfortheirhelpwiththisundertaking.IamgratefultoeveryoneatBarron’s.

Thankssomuchtoallofmystudentsovertheyears—Ihavelearnedfarmorefromyouthanyouhavelearnedfromme.

AbouttheAuthorBrianW.StewartisthefounderandpresidentofBWSEducationConsulting,Inc.,aboutiquetutoringandtestpreparationcompanybased inColumbus,Ohio.Hehasworkedwith thousandsof students tohelpthemimprovetheirtestscoresandearnadmissiontoselectiveschools.BrianisagraduateofPrincetonUniversity (A.B.) and The Ohio State University (M.Ed.). You can connect with Brian atwww.bwseducationconsulting.com.

Copyright©2016byBarron’sEducationalSeries,Inc.

Allrightsreserved.Nopartofthispublicationmaybereproducedordistributedinanyformwithoutthewrittenpermissionofthecopyrightowner.

Allinquiriesshouldbeaddressedto:Barron’sEducationalSeries,Inc.

250WirelessBoulevard

Hauppauge,NewYork11788www.barronseduc.com

eISBN:978-1-4380-6808-4Firste-Bookpublication:March,2016

Contents

Introduction

FAQsAbouttheNewSATReadingTest

DiagnosticTest

Self-AssessmentGuideAnswersExplained

Chapter1:ReadingStrategies

Chapter2:WordsinContext:StrategyandPractice

PracticeExercisesAnswersExplained

Chapter3:GraphAnalysis:StrategyandPractice

PracticeExercisesAnswersExplained

Chapter4:PracticeExercisesofIncreasingDifficulty

ExerciseAAnswersExplained

ExerciseBAnswersExplained

ExerciseCAnswersExplained

PRACTICETESTS

PracticeTest1

AnswersExplained

PracticeTest2

AnswersExplained

PracticeTest3

AnswersExplained

Welcometothee-BookversionoftheReadingWorkbookfortheNewSAT!This e-Bookmay appear differently dependingon the device you are using to view it on.Pleaseadjustyourdeviceaccordingly.Throughout this e-Book, youmay find several hyperlinks that will help navigate you through thecontent,bringyoutohelpfulresources,andallowyoutoclickbetweenquestionsandanswers.

Introduction

FAQsABOUTTHENEWSATREADINGTEST

HowdoestheReadingsectionfactorintotheoverallnewSAT?ThenewSAThastworequiredsections—(1)Mathand(2)Evidence-BasedReadingandWriting.Eachsectionwillbescoredbetween200–800,makingforapotential totalscoreofbetween400-1600.TheReadingsectionand theWritingandLanguagesectionwillcontribute inequalmeasure toyouroverallEvidence-BasedReadingandWritingscore.

WhatisthegeneralformatofthenewSATReadingsection?Itisasinglesectionwiththesefeatures:■Firsttestsection■65minuteslong■52questions(10Fiction,21SocialStudies,21Science).

Whatwillthereadingpassagesbelike?■5passagestotal—eachpassagebetween500and750wordsforatotalofaround3,250words.–Onefictionpassage—aselectionfromU.S.orworldliterature–Twosocialstudiespassages—

onefromsocialscienceandonefromaU.S.foundingdocumentoraselectionfromthe“GreatGlobalConversation”(e.g.,ahistoricalspeechoressay)–Twosciencepassages

■Oneofthepassageswillcomprisetwosmallerpassagesthatyouwillneedtocompareandcontrastinthequestions.

■Twographsaccompanythereadingthatyouwillneedtoanalyze.■Thepassagesrangeindifficultyfromearlyhighschoolleveltoearlycollegelevel.

Whatarethequestionslike?■10WordsinContextquestions

Example:Asusedinline30,theword“advance”mostcloselymeans...

■10CommandofEvidencequestions

Example:Whichoptiongivesthebestevidencefortheanswertothepreviousquestion?

■32Analysisquestions

Examples:

–Whatstatementbestsummarizesthepassage?–Theparagraphinlines21–37primarilyservesto...–Thenarrator’sstatementinlines48–51(“Theprimary...forecast”)mostclearlyimpliesthat...

■Thequestionsforagivenpassagegenerallyappearinthesameorderasthematerialinthepassage(e.g.,Question1isaboutlines1–5,Question2isaboutlines6–9,etc.).

■Thequestionsarearrangedinarandomorderofdifficulty.■Thereare10–11questionsperpassage.

HowisthenewSATReadingTestdifferentfromtheoldSATCriticalReadingTest?

OldSATCriticalReadingTest NewSATReadingTest

Acompletelyseparatesectionforscoringpurposes CombinedwiththeWritingandLanguagesectionfortheoverall“Evidence-BasedReadingandWriting”score

1largetestsection

3shortertestsections

SentenceCompletionquestions—necessarytoknowlotsofdifficultvocabularywords

WordsinContextquestions—moreimportanttobeabletodetermineappropriatewordmeaninginagivensituation

Nographs;no“evidence”questions Graphstoanalyze;demonstratecommandoftextthroughevidence-basedquestions

Questionsrequireconsiderablereadingtime Easiertodeterminewhatisbeingasked

Aquarterpointguessingpenalty NOguessingpenalty

HowcanIusethisbooktoprepare?■IFYOUHAVEONEDAY,lookoverthereadingstrategies,becomefamiliarwiththetestdirectionsandformat,andtryacoupleofpassagesundertimeconstraints.

■IFYOUHAVEONEWEEK,completethediagnostictest,readthestrategychapter,anddotargetedpracticeonthetypesofpassagesandquestionsthatgiveyoudifficulty.

■IFYOUHAVEAMONTHORMORE,doeverythinginthisbook.Startwiththediagnostic,readthestrategychapterverycarefully,andcompletethepracticeexercisesundertimeconstraints,carefullyreflectingonyourapproachasyoupracticefurther.

WhatcanIdobeyondthisbooktoprepare?■PracticewiththeotherBarron’sbooksthathaveexcellentsamplereadingtests,suchasBarron’sNewSAT,Barron’s6PracticeTestsfortheNewSAT,andBarron’sStrategiesandPracticefortheNewPSAT.

■Takerigorouscoursesinschool,suchasA.P.EnglishLanguageandComposition,A.P.U.S.History,andInternationalBaccalaureateLiterature.

■Makereadingadailyhabit—talktoyourlocallibrarianaboutbooksthatmaysuityourinterests,readwell-writtenonlinejournalsandblogs,anddownloadane-readertoyoursmartphonesoyoucanreadgoodbooksnomatterwhereyouare.

■Ifyouareambitious,readpublicationsthatyoufindmorechallenging.TheBestAmericanseriesisgoodforawidevarietyoftexts.Ifyouhavetroublewithfiction,seekoutbooksbyauthorslikeEmilyDickinson,JamesJoyce,andCharlesDickens.Ifyoustrugglewithsocialstudies,readhistoricaldocumentsandpublicationslikeTheEconomistorTheAtlantic.Andifyoufindsciencechallenging,readpublicationslikeScienceNewsandScientificAmerican,andcheckoutonlineresourceslikePubmed.gov.

■UsethefreeofficialpracticeresourcesavailableatKhanAcademy.org.

TheSATReadingTestisatestofyourreadingskill,notanyspecificknowledge.Youwilldowellonthistestifyouimproveyouroverallreadingcomprehensionability,whichisexactlywhatthistextisdesignedtodo.Let’sgetstarted!

**Remember,sincethisisane-Book,alldirectionsontheDiagnosticandPracticeTestsmaylooksimilar to those youmay see on test day, but youwill need towrite your answers and anyotherresponsesseparately.AnswerSheetsareforreferenceonly.Goodluck!

DiagnosticTest

65MINUTES,52QUESTIONS

Directions:Eachpassageorpairofpassagesisaccompaniedbyseveralquestions.Afterreadingthepassage(s),choose thebestanswer toeachquestionbasedonwhat is indicatedexplicitlyorimplicitlyinthepassage(s)orintheassociatedgraphics.

Questions1–10arebasedonthefollowingreadingselection.

ThefollowingisanexcerptfromJaneAusten’sMansfieldPark,1814.Thenovel’sprotagonist,FannyPrice,returnshomeaftermanyyearsoflivingwithherwealthyrelativesatMansfieldPark.

Williamwasgone:andthehomehehadleftherinwas—Fannycouldnotconcealitfromherself—inalmosteveryrespectthe

Line veryreverseofwhatshecouldhavewished.(5) Itwastheabodeofnoise,disorder,and

impropriety.Nobodywasintheirrightplace,nothingwasdoneasitoughttobe.Shecouldnotrespectherparentsasshehadhoped.Onherfather,herconfidencehadnotbeen

(10) sanguine,buthewasmorenegligentofhisfamily,hishabitswereworse,andhismannerscoarser,thanshehadbeenpreparedfor.Hedidnotwantabilities;buthehadnocuriosity,andnoinformationbeyondhisprofession;

(15) hereadonlythenewspaperandthenavy-list;hetalkedonlyofthedockyard,theharbour,Spithead,andtheMotherbank;hesworeandhedrank,hewasdirtyandgross.Shehadneverbeenabletorecallanything

(20) approachingtotendernessinhisformertreatmentofherself.Therehadremainedonlyageneralimpressionofroughnessandloudness;andnowhescarcelyevernoticedher,buttomakehertheobjectofacoarse

(25) joke.Herdisappointmentinhermotherwas

greater:thereshehadhopedmuch,andfoundalmostnothing.Everyflatteringschemeofbeingofconsequencetohersoon

(30) felltotheground.Mrs.Pricewasnotunkind;but,insteadofgainingonheraffectionandconfidence,andbecomingmoreandmoredear,herdaughternevermetwithgreaterkindnessfromherthanonthefirstdayof

(35) herarrival.Theinstinctofnaturewassoon

satisfied,andMrs.Price’sattachmenthadnoothersource.Herheartandhertimewerealreadyquitefull;shehadneitherleisurenoraffectiontobestowonFanny.Herdaughters

(40) neverhadbeenmuchtoher.Shewasfondofhersons,especiallyofWilliam,butBetseywasthefirstofhergirlswhomshehadevermuchregarded.Tohershewasmostinjudiciouslyindulgent.Williamwasherpride;

(45) Betseyherdarling;andJohn,Richard,Sam,Tom,andCharlesoccupiedalltherestofhermaternalsolicitude,alternatelyherworriesandhercomforts.Thesesharedherheart;hertimewasgivenchieflytoherhouseand

(50) herservants.Herdayswerespentinakindofslowbustle;allwasbusywithoutgettingon,alwaysbehindhandandlamentingit,withoutalteringherways;wishingtobeaneconomist,withoutcontrivanceorregularity;

(55) dissatisfiedwithherservants,withoutskilltomakethembetter,andwhetherhelping,orreprimanding,orindulgingthem,withoutanypowerofengagingtheirrespect.Ofhertwosisters,Mrs.Priceverymuch

(60) moreresembledLadyBertramthanMrs.Norris.Shewasamanagerbynecessity,withoutanyofMrs.Norris’sinclinationforit,oranyofheractivity.Herdispositionwasnaturallyeasyandindolent,likeLadyBertram’s;

(65) andasituationofsimilaraffluenceanddo-nothingnesswouldhavebeenmuchmoresuitedtohercapacitythantheexertionsandself-denialsoftheonewhichherimprudentmarriagehadplacedherin.Shemighthave

(70) madejustasgoodawomanofconsequenceasLadyBertram,butMrs.Norriswouldhavebeenamorerespectablemotherofninechildrenonasmallincome.MuchofallthisFannycouldnotbutbe

(75) sensibleof.Shemightscrupletomakeuseofthewords,butshemustanddidfeelthathermotherwasapartial,ill-judgingparent,adawdle,aslattern,whoneithertaughtnorrestrainedherchildren,whosehousewas

(80) thesceneofmismanagementanddiscomfortfrombeginningtoend,andwhohadnotalent,noconversation,noaffectiontowards

herself;nocuriositytoknowherbetter,nodesireofherfriendship,andnoinclination

(85) forhercompanythatcouldlessenhersenseofsuchfeelings.Fannywasveryanxioustobeuseful,and

nottoappearaboveherhome,orinanywaydisqualifiedordisinclined,byherforeign

(90) education,fromcontributingherhelptoitscomforts,andthereforesetaboutworkingforSamimmediately,andbyworkingearlyandlate,withperseveranceandgreatdespatch,didsomuch,thattheboywasshippedoffat

(95) last,withmorethanhalfhislinenready.Shehadgreatpleasureinfeelingherusefulness,butcouldnotconceivehowtheywouldhavemanagedwithouther.

1.Whatbestdescribeswhathappensinthepassageasawhole?(A)Acharacterdiscusseshertroubledthoughtswithclosefamilyandfriends.(B)Acharacteranalyzesherobservationsrelativetoherexperiencesandexpectations.(C)Acharacterreflectsonhowshecouldbeabettercontributortoherimmediatefamily.(D)Acharacterconsidershereconomicstationinastronglyhierarchicalsociety.

2.Fanny’soverallattitudetowardherparentsisbestdescribedas(A)justifiedaffection.(B)unjustifiedjealousy.(C)unwarranteddisrespect.(D)warranteddisappointment.

3.ThefirstparagraphcharacterizesFanny’sfather’sintellectualinterestsas(A)relevantandinteresting.(B)coarseandjoking.(C)overlypragmatic.(D)arrogantlyerudite.

4.Asusedinline35,thephrase“instinctofnature”mostcloselymeans(A)maternalfeeling.(B)desireforsurvival.(C)thirstforacceptance.(D)senseofjustice.

5.ThemorethatFannyisaroundhermother,themorehermothertreatsherwith(A)abuse.(B)affection.(C)inattention.(D)encouragement.

6.Whichoptiongivesthebestevidencefortheanswertothepreviousquestion?(A)Lines19–21(“She...herself”)(B)Lines33–35(“her...arrival”)(C)Lines55–58(“without...respect”)

(D)Lines61–63(“She...activity”)

7.WhenFannyreturnstolivewithherparents,sheiseagertobe(A)anidealisticmartyr.(B)aneconomicalinnovator.(C)anempoweringmentor.(D)ahelpfulcontributor.

8.Whichoptiongivesthebestevidencefortheanswertothepreviousquestion?(A)Lines5–8(“It...hoped”)(B)Lines37–43(“Herheart...regarded”)(C)Lines69–73(“Shemight...income”)(D)Lines87–91(“Fanny...comforts”)

9.Asusedinline46,theword“occupied”mostcloselymeans(A)stayed.(B)resided.(C)dwelled.(D)engaged.

10.Thethirdparagraph(lines59–73)suggeststhatMrs.Norrisis(A)morecapablethanMrs.Price.(B)similarinpersonalitytoMrs.Price.(C)morelethargicthanMrs.Price.(D)lessrespectablethanMrs.Price.

Questions11–20arebasedonthefollowingreadingselectionandaccompanyingmaterial.

TheDownfallofDemocracy?

“ThefutureofthisrepublicisinthehandsoftheAmericanvoter.”—DwightD.Eisenhower

Thestatementabove,madebythe34thPresidentoftheUnitedStates,paintsadismalpicturewhenoneconsidersthetrends

Line oftheAmericanyouth.Thoseundertheage(5) of35,ingeneral,donotvote,donotparticipate

inpolitics,andfrankly,havenointerestortrustinthegovernmentitself.Theretreatofyoungstersfromthepoliticalrealmscanbeseeninelectionturnoutssurely,butit

(10) canalsobenotedintheminisculenumbersofyoungAmericanswhoidentifyaseitherDemocratsorRepublicans.Itisnotonlydisinterestbutalsodistastethatkeepthenextgenerationofrepresentativesfarfrom

(15) thepollboothsandevenfurtherfromoffice.Infact,arecentlypublishedbookRunningFromOffice:WhyYoungAmericansAreTurnedOfftoPoliticsfoundthatonlyabout

tenpercentofhighschoolanduniversity(20) studentswouldevenconsiderrunningfor

publicoffice,withdisdainforfederalpositionsatthehighest.Itmightbeeasytochalkuptheyounger

generationascarelessandunconcerned(25) andaltogethermisdirected,butthat’sthe

easywayout,anditisflawed.Studentsarelargelycivic-minded;theyvolunteer,worryaboutpublicpolicies,andevencongregatedebatesolutionstoeverythingfromenvironmental

(30) issuestohumanrightstohealthcare.SignificanteventslikethewarinIraqorBarackObamarunningforPresidentorgaymarriagerightsmaygenerateaninfluxinyouthparticipation,butoverall,youngvoters

(35) aredisengagedfromAmericandemocracyandlookingatotherwaysoftacklingsociety’sproblems.Itisnotthattheyaremerelyapathetic,butinsteadthattheyhavelostfaithinelectoralpoliticsandarehighlysuspiciousof

(40) partylabels.Ifitisdistrustandsuspicionthatkeep

youngstersaway,theU.S.mustaskwhathaschangedtotriggerthisworryingtrend.Onedoesn’thavetoprepareextensiveresearch

(45) methodstofindthattoday’smillennialsviewthemselvesasinanaltogetherworsesituationthanthatoftheirparentsorgrandparents.Widespreadopinionhastheminabitter,unrewardingjobmarketbankrupted

(50) bytheuniversityandcrushedunderneathastiflingnationaldebt.Theirpathisdifficultanduncertainatbest;behindtheseobstaclesliesaninefficientandwastefulgovernmentthattheyarelessthananticipatorytoinherit.

(55) Theaforementionedbookfoundthat25%ofstudent-agedAmericansshowedabsoluteindifferencetopolitics.Moreworryingwerethe60%whoheldnegativeviewsofpolitics,avoidedthesubjectatallcosts,and

(60) thoughtofallpoliticiansasdeviousanduntrustworthy.MoreandmoreAmericansareshunningawayfromtheloadedlabelsof“conservative”and“liberal,”insteadfindingthemselvessomewhereinthemiddle,

(65) sociallyliberalbutfiscallyconservative.

Alienatedbythetwo-partysystem,young

Americanswouldrathernotbeincludedineitherdisagreeableside.Disinterestedinableakfutureofmore

(70) debtandlessfreedom,andwaryofaligningthemselveswithinpartisanship,today’syoutharedoubtfulofagovernmentthatpromisesfewoftheassurancesitoncepledged.Naturally,thistrendisdisquieting

(75) foranationthatdependsonitsvotersandaninterestinrepresentation,bothofwhichareinastateofdeterioration.Yet,somemayapplaudtheveerfrompartisanship—aphenomenathathasleftmoreundone

(80) thanaccomplished.Still,ifdemocracyistosurvive,somethingmustbedonetoalignthecynicalmillennialswithasystemthatdesperatelyneedstheirinterference.

VoterTurnoutinPresidentialElectionsbyAge

Source:2012U.S.CensusBureau:http://www.census.gov/prod/2014pubs/p20-573.pdf.

11.Whatistheoverallpointoftheessay?(A)Toargueagainstthecontinuationofthetwo-partypoliticalsystem(B)Toraisespecificconcerns

aboutthelackofmillennialpoliticalengagement(C)Tosuggestthatmandatoryvotingwouldalleviatemuchoftheskepticismtowardsthepoliticalprocess(D)Toexplainwhymillennialsaremoreinterestedinpursuingpoliticalofficethanpeoplefrompastgenerations12.Thetoneoftheessayisbestdescribedas(A)panicked.

(B)wavering.(C)concerned.(D)optimistic.

13.Asusedinline23,thephrase“chalkup”mostcloselymeans(A)ascribe.(B)draw.

(C)blame.(D)dispute.

14.TheauthormoststronglysuggeststhatthoseAmericansunderage35aremostlikelytotakewhatapproachtosolvingsocietalproblems?(A)Activeparticipationinthepoliticalprocess(B)Generalapathyandlackofactivity(C)Attackingproblemsoutsideofagovernmentalparadigm(D)Focusingontheirownindividual

interestsaboveall15.Whichoptiongivesthebestevidencefortheanswertothepreviousquestion?

(A)Lines1–4(“Thestatement...youth”)(B)Lines34–37(“young...problems”)(C)Lines43–48(“One...grandparents”)(D)Lines77–80(“Yet...accomplished”)

16.Asusedinline36,theword“tackling”mostcloselymeans(A)beginning.(B)discovering.(C)solving.(D)dismissing.

17.Theprimarypurposeoftheparagraphinlines55–68isto(A)giveevidencepertainingtothelackofinterestinpoliticsonthepartofyoungAmericans.(B)explaintheinherentflawswiththeAmericantwo-partypoliticalsystem.(C)describethespecificreasonswhyyoungpeoplefindpartylabelsdistasteful.(D)analyzewhyolderAmericansaremorelikelytoparticipateinthepoliticalprocessthanyounger

Americans.

18.Whatisthepurposeofthesentenceinlines77–80(“Yet...accomplished.”)?(A)Tostatethethesisoftheessay(B)Togivedetailsaboutthepitfallsofpartisanship(C)Toacknowledgeacontrastingviewpoint(D)Tociteanexpertpointofview

19.Accordingtotheinformationinthegraph,duringwhatyearwerethevoterturnoutratesofAmericansages65andolderandAmericansages18–24closesttooneanother?(A)1992(B)2000(C)2004(D)2008

20.Thevariationintheages18–24voterturnoutcanmostdirectlybeaccountedforbywhatselectionfromthepassage?(A)Lines4–7(“Those...itself”)(B)Lines31–34(“Significant...participation”)(C)Lines57–61(“More...untrustworthy”)(D)Lines74–77(“Naturally...deterioration”)Questions21–31arebasedonthefollowing

readingselectionandaccompanyingmaterial.

Microbiomes

Germsmakeussick.It’sanelementarytruththatweteachtoourchildren.It’swhywewashourhandsbeforeeating.It’swhywe

Line pasteurizeourmilk,andrefrigerateourfood.(5) Whentheydomakeussick,ourabilityto

safelyexterminatethemisnothingshortofamodernmiracle.Beginningwithpenicillinin1928,antibioticsforevertransformedthewaywebothtreatandpreventinfectiousdisease.

(10) Today,moreover,onecanstrolldownanycleaningsupplyaisleatasupermarket,anddiscoverabevyofproductsboastingoftheirbroad-spectrumantimicrobialactivity.Forbetterorforworse,ourcultureof

(15) “germophobia”washard-wonbyitsproponents.Fromthetimeitwasfirstproposedinthe16thcentury,thegermtheoryofdiseasefacedthreehundredyears’worthofinfluentialnaysayers,anditwasnotuntilthe

(20) late1800’sthatthetheorybegantogainthepervasivepublicvindicationitenjoystoday.However,anemergingbodyofresearchindicatesthatwehavebeenperhapsoverzealousinourcrusadetoeradicatethegermsthat

(25) livewithinus.The“humanmicrobiome”referscollectively

tothemicroscopicorganismsthatnaturallycolonizethehumanbody,andtheapplicationofdynamicecologicaltheories

(30) tothisbiomerepresentsarapidlyexpandingfieldofstudy.Comprisedoffungi,viruses,archaea,andperhaps1,000speciesofbacteria,thepopulationofthismicrobiomeisthoughttooutnumberourowncellsbyas

(35) muchastentoone.What’smore,muchlikeourowncells,asignificantportionoftheseorganismsplaycrucialrolesinourmetabolicandimmunologicalprocesses.Forexample,Oxalobacterformigenes,

(40) whichcolonizesthecolon,isaprimarysourceoftheenzymeoxalyl-CoAdecarboxylase,whichallowsustosafelyeliminatedietaryoxalate.Withoutthisenzyme,calciumoxalatesaltstendtoaccumulateinthe

(45) kidneytubules,andeventuallyprecipitateasrenalstones.Othercolonicbacteriacatalyzethereductionofbilirubinintourobilinogen:

areactioncriticaltoourdigestionoffats,andabsorptionoffat-solublevitamins.

(50) Interestingly,manybacteriawithinourgastrointestinaltractsalsodirectlysynthesizeseveralvitaminsinexcessoftheirownmetabolicneeds,and,asaresult,representanimportantsourceofbothvitaminB12,which

(55) isnecessaryfortheproductionofnewredbloodcells,andvitaminK,whichisacofactorinthesynthesisofseveralbloodclottingfactors.Thebenefitswegainfromabalanced,

(60) thrivingmicrobiomearediverse,andweareonlyjustbeginningtoappreciatetheirtruecomplexity,thoughperhapsnosinglefunctionitservesismoresignificantthanitsroleinregulatingourimmunesystems.Thereare

(65) numerousmechanismsbywhichthemicrobiomehelpsprotectusfromdisease.Somespecies,forinstance,secretespecialproteins,knownas“bacteriocins,”thataredirectlytoxictopathogenicbacteria,butharmless

(70) toourowncells.Oneparticularlyimpressivememberofthemicrobiome,Lactobacillus,producesapowerfulbacteriocincalledreuterin,aswellaslacticacidandhydrogenperoxide,whichinhibitthegrowthofdisease-

(75) causingorganismsbyloweringlocalpHanddamaginglipidmembranesrespectively.Ofnolessimportance,thereareanumber

ofmoreindirect,ecologically-orientedwaysinwhichthemicrobiomeconfersprotection

(80) toitshost.Abundantcolonizationofourbodiesbybenignmicroorganisms,forexample,inhibitstheovergrowthofmoredangerousonesthroughthesheerdepletionofmicrobialnutrients.Thisnotionofecological

(85) balancehasbeenofparticularinteresttoscientistsstudyingthemicrobiome,asithingesuponboththevariablediversityofspeciesthatcolonizeanindividual,aswellasfactorsthataffectthedynamismofamicrobiotic

(90) population.Age,geography,diet,andstresshaveallbeenimplicatedininfluencingboththecompositionandbalanceofthemicrobiome.None,however,hasbeenshowntohavesodrasticaneffectastheuseofantibiotics,

(95) which,unlikebacteriocins,tendtobejustasdeadlytodisease-causingbacteriaas

theyaretothosethatcolonizeusnaturally.Admittedly,antibioticssavelives.Yet

ourdecisiontousethemmustbeweighed(100) carefully,asartificiallyupsettingtheecological

balanceofourbodiescanhavedireconsequences.ThereisperhapsnogreaterexampleofthisthanClostridiumdifficile,acolonicbacteriumthatisbothhighlyresistant

(105) toantibiotics,andanincreasingcauseofhospital-acquireddisease.Whenproperlycounterbalancedbyitsneighboringspecies,C.difficileisharmlesstohumans.However,followingtheadministrationofantibiotics,

(110) andtheeliminationofitsecologicalcompetitors,C.diffproliferationproceedsunchecked,resultinginafrequentlyfatalinfectionforwhichveryfeweffectivetreatmentsexist.

The table illustrates variations in microbiota for individuals following either vegetarian oromnivorousdietsacrossthreegeographicaldemographics.

DiversityofHumanMicrobiomes

21.Thegeneralstructureofthepassageisbestdescribedas(A)abroadintroductionfollowedbyspecificillustrations.(B)alogical,point-by-pointargument.(C)aninterestinglookfollowedbyin-depthpersuasion.(D)asequenceoftechnicalexamples.

22.Asusedinline1,theword“elementary”mostcloselymeans(A)scholastic.(B)medical.(C)healthy.

(D)fundamental.

23.Theparagraphinlines14–25mostdirectlyservesto(A)articulatethatwhilesocietyhasnowembracedgermtheory,takingthetheorytoofarmaybedetrimental.(B)arguethatgermophobiahascontinuedtobeamajorobstacletoscientificprogress.(C)pointouttheshortcomingsofgermtheorybypresentingthevalidconcernsofgermophobics.(D)presentthemanywaysthatgermtheoryhasconcreteapplicationstoeverydaylife.

24.Whatistheoverallpurposeoftheparagraphinlines39–58?(A)Torecommendspecificbacteriologicaltreatmentstocommongastrointestinalillnesses(B)To

provideconcreteexamplesoftheutilityofsomebacteriatoourmetabolicandimmunologicalprocesses(C)Toaddresstheobjectionsofthosewhoareinherentlyskepticaltowardstheexistenceofbacteria(D)ToexplainthemetabolicprocesseswherebybacterialeadtothecreationofvitaminsB12andK

25.Whichofthefollowingdoesthepassageexplicitlystateillustratesthepotentialdangerinoverusingantibiotics?(A)Oxalobacterformigenes(B)Bilirubin(C)Lactobacillus(D)Clostridiumdifficile26.Whichoptiongivesthebestevidencefortheanswer

tothepreviousquestion?(A)Lines39–43(“For...oxalate”)(B)Lines46–49(“Other...vitamins”)(C)Lines70–76(“One...respectively”)(D)Lines108–114(“However...exist”)

27.Asusedinline87,theword“hinges”mostcloselymeans(A)fulcrums.(B)analyzes.(C)depends.(D)joints.

28.Theinformationinthetablewouldbemosthelpfultothestudyofwhichofthefollowingconceptsmentionedinthepassage?(A)Germophobia(B)Ecologicalbalance(C)Bloodclotting(D)Pathogenicbacteria

29.Whichoptiongivesthebestevidencefortheanswertothepreviousquestion?(A)Lines14–21(“Forbetter...today”)(B)Lines50–58(“Interestingly...factors”)(C)Lines66–76(“Some...respectively”)(D)Lines84–93(“This...microbiome”)

30.Basedontheinformationinthetable,analysisofthevariationsofwhichtwomicroorganismswouldbemosthelpfulinroughlydeterminingthecountryofresidenceofarandomlyselectedhumantestsubject?(A)ActinobacteriaandBacteroides(B)BifidobacteriaandClostridia(C)Enterococciand

Lactobacilli(D)MethanogensandYeasts

31.Basedontheinformationinthetable,knowingthepercentageofeachofthefollowingmicroorganismswouldbeusefulindeterminingwhethersomeonehadavegetarianoromnivorousdiet,nomatterhisorhergeographicallocation,EXCEPT:(A)Bacteroides.(B)Enterococci.(C)Lactobacilli.(D)Yeasts.

Questions32–42arebasedonthefollowingreadingselection.

Adapted froma 1981 speech toCongress,RonaldReagan states his reasons for a newprogram foreconomicrecovery.1

Mr.Speaker,Mr.President,distinguishedMembersofCongress,honoredguests,andfellowcitizens:

Line OnlyamonthagoIwasyourguestinthis(5) historicbuilding,andIpledgedtoyoumy

cooperationindoingwhatisrightforthisNationthatwealllovesomuch.I’mheretonighttoreaffirmthatpledgeandtoaskthatweshareinrestoringthepromisethatis

(10) offeredtoeverycitizenbythis,thelast,besthopeofmanonEarth.Allofusareawareofthepunishinginflation

whichhasforthefirsttimein60yearsheldtodouble-digitfiguresfor2yearsina

(15) row.Interestrateshavereachedabsurdlevelsmorethan20percentandover15percentforthosewhowouldborrowtobuyahome.Allacrossthislandonecanseenewlybuilthomesstandingvacant,unsoldbecauseof

(20) mortgageinterestrates.Almost8millionAmericansareoutof

work.Thesearepeoplewhowanttobeproductive.Butasthemonthsgoby,despairdominatestheirlives.Thethreatsoflayoff

(25) andunemploymenthangoverothermillions,andallwhoworkarefrustratedbytheirinabilitytokeepupwithinflation.OneworkerinaMidwestcityputittome

thisway:Hesaid,“I’mbringinghomemore(30) dollarsthanIeverbelievedIcouldpossibly

earn,butIseemtobegettingworseoff.”Andheis.NotonlyhavehourlyearningstheAmericanworker,afteradjustingfor

inflation,declined5percentoverthepast5(35) years,butinthese5years,federalpersonal

taxesfortheaveragefamilyhaveincreased67percent.Wecannolongerprocrastinateandhopethatthingswillgetbetter.Theywillnot.Unlessweactforcefully—andnow—the

(40) economywillgetworse.Canwe,whomantheshipofstate,deny

itissomewhatoutofcontrol?Ournationaldebtisapproaching$1trillion.AfewweeksagoIcalledsuchafigure,atrilliondollars,

(45) incomprehensible,andI’vebeentryingeversincetothinkofawaytoillustratehowbigatrillionreallyis.AndthebestIcouldcomeupwithisthatifyouhadastackofthousanddollarbillsinyourhandonly4incheshigh,

(50) you’dbeamillionaire.Atrilliondollarswouldbeastackofthousand-dollarbills67mileshigh.Theinterestonthepublicdebtthisyearweknowwillbeover$90billion,andunlesswechangetheproposedspendingforthe

(55) fiscalyearbeginningOctober1st,we’lladdanotheralmost$80billiontothedebt.Addingtoourtroublesisamassof

regulationsimposedontheshopkeeper,thefarmer,thecraftsman,professionals,and

(60) majorindustrythatisestimatedtoadd$100billiontothepriceofthethingswebuy,anditreducesourabilitytoproduce.TherateofincreaseinAmericanproductivity,onceoneofthehighestintheworld,isamongthelowest

(65) ofallmajorindustrialnations.Indeed,ithasactuallydeclinedinthelast3years.Now,I’vepaintedaprettygrimpicture,but

IthinkI’vepainteditaccurately.Itiswithinourpowertochangethispicture,andwecan

(70) actwithhope.There’snothingwrongwithourinternalstrengths.Therehasbeennobreakdownofthehuman,technological,andnaturalresourcesuponwhichtheeconomyisbuilt.

(75) […]This,then,isourproposal—America’snew

beginning:aprogramforeconomicrecovery.Idon’twantittobesimplytheplanofmyadministration.I’mheretonighttoaskyou

(80) tojoinmeinmakingitourplan.Togetherwe

canembarkonthisroad.

1Citation:RonaldReagan:“AddressBeforeaJointSessionof theCongresson theProgramforEconomicRecovery,”February18,1981.OnlinebyGerhardPetersandJohnT.Woolley,TheAmericanPresidencyProject.http://www.presidency.ucsb.edu/ws/?pid=43425.

32.Theoverallpointofthispassageisto(A)presentscholarlyresearch.(B)surveypopularopinion.(C)argueforacalltoaction.(D)persuadeeconomicthinkers.

33.Thespeaker’stoneisbestdescribedas(A)directandempathetic.(B)haughtyanddismissive.(C)pessimisticandgrave.(D)eruditeandurbane.

34.Asusedinline12,theword“punishing”mostcloselymeans(A)sad.(B)confined.(C)disciplined.(D)severe.

35.ThespeakermostdirectlysuggeststhatunemployedAmericans(A)wouldmuchrathernotbeinthatsituation.(B)clearlyunderstandtheeconomiccausesoftheirtroubles.(C)wishthatforeignaidcouldprovideneededrelief.(D)hopethattheycanreceiveunemploymentbenefitsforanextendedperiod.

36.Whichoptiongivesthebestevidencefortheanswertothepreviousquestion?(A)Lines18–20(“All...rates”)(B)Lines21–24(“Almost...lives”)(C)Lines28–31(“One...off”)(D)Lines57–62(“Adding...produce”)

37.Thequotationinlines29–31(“I’m...off”)servesto(A)giveconcretestatistics.(B)provideanecdotalevidence.(C)separatefactfromopinion.(D)acknowledgelikelyobjections.

38.ThespeakermoststronglysuggeststhattheunderlyingstructureoftheU.S.economyis(A)inflationary.(B)unsound.(C)solid.(D)focused.

39.Whichoptiongivesthebestevidencefortheanswertothepreviousquestion?(A)Lines12–15(“All...row”)(B)Lines21–24(“Almost...lives”)(C)Lines37–40(“We...worse”)

(D)Lines70–74(“There’s...built”)

40.Thespeakerprimarilyusestheparagraphinlines41–56to(A)sharerelevantfirst-handobservations.(B)concretelyillustratetheseverityofaproblem.(C)verbalizetheincomprehensiblecomplexityofaconcept.(D)highlightthewidespreadinterestinaparticularsolution.

41.Lines58–60(“shopkeeper...industry”)areintendedtoillustratethe(A)widespreadimpactofgovernmentregulations.(B)specificjobsthathavebeenlostintherecession.(C)thosewhowillmostbenefitfromtheproposedprograms.(D)primarymembersoftheaudiencethespeakerisaddressing.

42.Asusedinline66,theword“declined”mostcloselymeans(A)decreased.(B)decayed.(C)failed.(D)wilted.

Questions43–52arebasedonthefollowingreadingselections.

Twoscientistspresenttheirviewsoncornsyrup.

PASSAGE1

Sincecomingtoaheadin2004,thehighfructosecornsyrupcrisisanditsroleintheemergentobesityepidemichasfacedunwavering

Line denialfromthefoodindustry;yetthe(5) effortstodefendtheadditiveonscientific

groundshavebeendubiousatbest.Weareallfamiliarwiththepitifulsyllogism:cornsyrupcomesfromcorn,andcornisnatural;cornsyrup,therefore,isnatural.Howevertruethis

(10) maybe,itprovidesnoproofwhatsoeverastocornsyrup’ssafetyforhumanconsumption.Solanine,forexample,iseasilyextractedfrompotatoes,andwhileharmlessinsmalleramounts,onceconcentrateditbecomesa

(15) potentandpotentiallydeadlyneurotoxin.ButIdigress.Letusnotlooktothesourceofcornsyruptodetermineitsnutritionaldemerit,butturninsteadtoitsdirectmetaboliceffectsonourbodies.

(20) Underidealcircumstances,thevastmajorityofsugarinourbloodisderivedfromstarch,whichisbrokenintoglucosebeforebeingreleasedtothebloodstream.

Glycolysisisthenameappliedtoten(25) sequentialchemicalreactionsthatallowus

toeitherliberateenergyfromglucose,ortransformitintofatsforstorageinadiposetissue.Gluconeogenesis,meanwhile,isanoppositeprocessinwhichglucoseisderived

(30) fromnon-carbohydratesubstances,andacloseandefficientregulationofthebalancebetweenglycolyticandgluconeogenicprocessesinresponsetothechangingconcentrationsofglucoseinthebloodis

(35) necessaryforthemaintenanceofhealthfulhomeostasis.Byfarthemostcriticalpointinthisregulation

occursatthethirdstepofglycolysis:inthehormonally-controlledphosphorylation

(40) offructose-6-phospateintofructose-1,6-bisphospate.Whenglucoseisabundant,pancreaticinsulininducestheforwardglycolyticcatalysisofthisreaction,allowingtheproductionoffructose-1,6-bisphospate,

(45) whichinturniscleavedintoglyceraldehyde-3-phosphateanddihydroxyacetonephosphate.Whenglucoseisscarce,pancreaticglucagonblocksglycolysis,andinducesthegluconeogenicproductionoffructose-

(50) 6-phosphate,whichissubsequentlyisomerizedintoglucose-6-phosphate,andreleasedintotheblood.Theprimaryproblem,therefore,with

derivingmajoramountsofdietarysugar(55) directlyfromfructoseratherthanfrom

starchliesinthefactthatthedegradationoffructose—which,uponentryintothecell,issplitimmediatelyintodihydroxyacetonephosphateandglyceraldehyde—completely

(60) bypassesthefirstfourstepsofglycolysis,includingthemostcriticalregulatoryreactionintheentireprocess.Thus,howourbodieshandletheusageoffructoseisutterlydissociatedfromthehormonalcontrols

(65) ofinsulinandglucagon,which,overtime,invariablypredisposesonetoobesity,diabetesmellitus,andahostofotherdangerousmetabolicdisorders.

PASSAGE2

Themediafrenzyandpublicoutcry

(70) thathavesurroundedtheuseofhighfructosecornsyrupasafoodadditiveareasunfoundedasthesimilarlynonsensicalindignationsthateruptedinresponsetotheadventofcommerciallyavailablegenetically

(75) modifiedcropseeds.Despiteongoingproofthatgeneticallymodifiedcropsarenotonlyperfectlysafeforconsumption,butthattheyhaveinfactsavedanestimated600millionpeoplefromstarvationoverthepasttwo

(80) decades,fearsandskepticismtowardthempersistsimplybecausetheyarepopularlyperceivedas“unnatural,”andthus,somehow,unhealthy.Thesesamemisguidedapprehensions

(85) havebeenattheforefrontofthecrusadeagainsthighfructosecornsyrup.Yet,inreality,theprocessofproducingcornsyrupisstrikinglysimilartothecarbohydratemetabolismthatoccursnaturallywithinthehuman

(90) body.First,cornstarchisbrokendownintoglucosebybacterialamylaseenzymes,andglucoseissubsequentlyconvertedtofructoseviaglucoseisomerase.Overall,therecipeishardlyassinisterasitsopponentswould

(95) haveusbelieve.Wemustacknowledge,ofcourse,that

researchhasidentifiedseveralserioushealthrisksassociatedwiththechronicoverconsumptionofsugar,andperhapsoffructose

(100) inparticular.Theserisks,however,arebynomeanslimitedtofoodstuffscontaininghighfructosecornsyrup.Dependingontheformula,cornsyrupcontainsbetween42%and55%fructosebyvolume.Forcomparison,

(105) canesugar,honey,andagavenectar—threepopularsweetenerstoutedas“natural”,andtherefore,morehealthful—contain50%,52%,and85%fructose,respectively.Thus,whileitistruethatfructoseshouldbeconsumed

(110) onlyinmoderation,thesinglingoutofproductsthatcontainhighfructosecornsyrupisnotmerelyinsufficientactiontocurbthefructose-associatedobesityepidemicinourcountry,it’salsopatentlymisleadingto

(115) consumers.

43.Whatistheprimarypurposeoflines12–15(“Solanine...neurotoxin.”)?(A)Topresentpracticalapplications(B)Torefuteaparticularlineofthinking(C)Toclarifyanunfamiliarterm(D)Todrawattentiontoaharmfulprocess

44.Asusedinline16,theword“digress”mostcloselymeans(A)stray.(B)analyze.(C)contradict.(D)reexamine.

45.Accordingtolines37–52,bodilyregulationofglucoselevelsisbestsummarizedas(A)artificial.(B)dynamic.(C)arbitrary.(D)static.

46.Theauthorofpassage1mostdirectlysuggeststhatthelong-termconsumptionoffructosewillleadto(A)anincreasinglywell-regulatedhormonalbalance.(B)asignificantincreaseinneurotoxinsinthebloodsupply.(C)agreaterlikelihoodofdevelopinghealthailments.(D)nosignificantchangestobodilyprocesses.

47.Whichoptiongivesthebestevidencefortheanswertothepreviousquestion?(A)Lines13–15(“while...neurotoxin”)(B)Lines41–47(“When...phosphate”)(C)Lines47–52(“When...blood”)(D)Lines62–68(“Thus...disorders”)

48.Asusedinline81,theword“persist”mostcloselymeans(A)persevere.(B)mislead.(C)continue.(D)affect.

49.Theauthorofpassage2mostlikelyuseslines102–108(“Depending...respectively”)inorderto(A)demonstratethatcornsyrupisespeciallyharmfultoconsumers.(B)showthatcornsyrupisundeservinglysingled-outforcriticism.(C)arguethatfructoseisbutonereasonthatcornsyrupismaligned.(D)illustratethatmanyfoodstuffscontaingreatquantitiesofsugar.

50.Itcanmostreasonablybeinferredthatthetwoauthorswoulddisagreewiththosewhodeclaredafoodtobehealthysimplybecauseitis(A)“engineered.”(B)“geneticallymodified.”(C)“natural.”(D)“metabolized.”

51.Whichoptiongivesthebestevidencefortheanswertothepreviousquestion?(A)Lines6–12(“Weare...consumption”)andlines75–83(“Despite...unhealthy”)(B)Lines16–

19(“Let...bodies”)andlines86–90(“Yet...body”)(C)Lines24–28(“Glycolysis...tissue”)andlines100–104(“These...volume”)(D)Lines47–52(“When...blood”)andlines90–93(“First...isomerase”)52.TheauthorsofPassage1andPassage2primarilyanalyzeexamplesfromwhatgeneralareastomaketheirrespectivecases?

(A)Passage1analyzesexamplesinternaltothehumanbody,andPassage2analyzesexamplesexternaltothehumanbody.

(B)Passage1analyzesexamplesexternaltothehumanbody,andPassage2analyzesexamplesinternaltothehumanbody.

(C)Bothfocusonexamplesinternaltothehumanbody.(D)Bothfocusonexamplesexternaltothehumanbody.

Ifyoufinishbeforetimeiscalled,youmaycheckyourworkonthissectiononly.Donotturntoanyothersection.

SELF-ASSESSMENTGUIDEUsethistabletodeterminewhichtypesofquestionswilldemandmoreofyourattention:

QuestionType QuestionNumbers

WordsinContext 4,9,13,16,22,27,34,42,44,48

GraphAnalysis 19,(20),28,30,31

CommandofEvidence 6,8,15,(20),26,29,36,39,47,51

Sentence-levelAnalysis 5,7,14,18,25,35,37,41,43,49

Paragraph-levelAnalysis 3,10,17,23,24,38,40,45,46

WholePassageAnalysis 1,2,11,12,21,32,33,50,52

SCORINGAPPROXIMATIONThistablegivesyouanestimateofhowyourperformanceontheReadingsectionwillcontributetoyouroverallEvidence-BasedReadingandWritingscore.Keepinmindthateachtestwillbecurved,makingthe number of questions needed for a particular score dependent on the test that day. This is the bestestimate we can give you based on (1) previous SAT curves and (2) the fact that guessing is nowpermittedontheSAT.

Questionsoutof52answeredcorrectlyEstimatedoverallsectionscore(between

200–800)

800

52

750

49

700

46

650

43

600

40

550

37

500

33

450

30

400

26

350

20

300

13

250

7

200

0

NReadingStrategies

1

owthatyouhavetakentheDiagnosticReadingTest,let’sreviewandpracticesomekeystrategiesthatwillhelpyoudoyourbestontheSATReadingTest.Thischaptercontains:

■16KeyStrategies■“PuttingitAllTogether”within-depthguidanceonhowtoattackthedifferenttypesofquestionsyou

willencounter■Additionalpassagesforfurtherpractice.

READINGSTRATEGIES

1 RealizethattheSATisnotthetypeoftestyouareusedto.

IfyouapproachtheSATReadingsectioninthesamewayyouapproachaschool-basedassignmentthatassessestextrecall,youwillhavesomemajordifficulty.HerearesomefundamentaldifferencesinhowyoushouldtackletypicaltestsandtheSATReadingTest:

TypicalSchool-BasedReadingTests SATReadingTest

Thetestsarealmostalwaysclosed-book,soyouneedtoreadandrereadtobecertainyouremembereverythingthatmightbetested.

TheSATisopen-book—youdon’thavetoknowanythingaheadoftimeormemorizethepassage.Youcangobacktothepassageasoftenasyouneed.

Testquestionsareoftenaboutspecificfacts ,soitisvitalthatyoumemorizedetailsanddefinitionsasyouread.

Testquestionsaremoreoftenaboutinference ,purpose ,andbigideas ,sofocusongeneralparaphrasinginsteadofspecificmemorizing.

Occasionally,therearemistakesonatestwithacoupleofrightanswers toaquestion.Itiseasyforateachertosimplygiveeveryoneafreepointifthereisanerror.

SATquestionsareverywellwritten,andtherewillbejustonecorrectanswertoeachquestion.TheCollegeBoarddoesnotwanttothrowouttheresultsforhundredsofthousandsoftest-takers,sotheyinvesttremendousresourcesintoensuringthatthequestionshaveanswersthatare100%correct.

Ifyouhavetoldyourselfthatyouare“abadtest-taker,”askyourself:“AmItakingtheSATinthesamewayItakeaschool-basedtest?”Ifso,theproblemisnotwithyoubutwithyourstrategy.

2 Takeyourtime.

TheSATReadingTesthasonlyabout3,250wordsofreadingpassages,and52questions,butgivesyouafull65minutestofinish.Mosttest-takerswillfindthattheSATReadingsectionisverymanageabletocomplete—youneedtoreadatapaceofabout150wordsperminute,whichisaboutasquicklyasmostpeoplecantalk.Youwilllikelydoyourbestifyouusethefullamountoftimetoreadthepassageswell,andthinkthroughthequestionscarefully.Giventhecomplexityofthequestions,youwillbebetterservedifyoudo thequestionsone timewellasopposed to rushing through them,makingcarelesserrors,andquickly“checking”overyourwork.Since thepassagesallhave10–11questions,youcanpaceyourselfby takingabout13minutesper

passage.Thiswouldinvolvetakingabout5minutestoreadthepassageandabout8minutestodotheaccompanying questions. Here is a table of how youmight want to allocate your time for a typical

Readingsectionasawhole(youcanadjustthisbasedonyourpersonalsituation,butthisbreakdownwillworkformanystudents):

ATotalof5Passages,52Questions,65Minutes

Passage1,Fiction,10questions

8minutesansweringquestions

5minutesreading

Passage2,SocialScience,10–11questions 8minutesanswering

questions

5minutesreading

Passage3,Science,10–11questions

8minutesansweringquestions

5minutesreading

Passage4,SocialStudies(GreatDocument),10–11questions 8minutesanswering

questions

5minutesreading

Passage5,Science,10–11questions

8minutesansweringquestions

5minutesreading

Whatcanyoudoifyouhavedifficultyfinishing?

■FOCUSONTHEPASSAGESTHATAREEASIERFORYOU.Everyquestionisworththesame,sopickyourbattles.IfyouknowthatFictionisalwaysmoredifficultforyou,goaheadandguessonthatpassageandskipit.ThesamegoesforScienceorSocialStudies.Thetestwillalwayshave1Fiction,2SocialStudies,and2Sciencepassages,soyoucanlikelyplanwhatyouwillskipbeforeyoutakethetest.Youcanalsomakeadecisionatthebeginningofthetestbytakingaquicklookatthepassagetitlesanddescriptions.Dothepassagesthatseemeasiestandmostinterestingfirst—savetheonesthatlookdifficultandboringfortheend.

RememberthatthereisNOGUESSINGPENALTYonthenewSAT,soifyouareunsureaboutaquestionoryouarerunningoutoftime,besuretobubbleinaletter.

■FOCUSONTHEQUESTIONSTHATAREEASIER.ThequestionsthatwilllikelytaketheleastamountoftimearetheWordsinContextquestions(askaboutthemeaningofaword)andtheCommandofEvidencequestions(askaboutwhatlinesgivethebestevidenceinsupportofthepreviousquestion).Vocabularyquestionstakelittletimebecauseyoutypicallyonlyneedtoconsiderthecontextimmediatelyaroundthegivenlines.Anevidencequestionispairedwiththequestionthatimmediatelycomesbeforeit—figureoutthequestionthatcomesimmediatelybeforetheevidenceone,andpaycloseattentiontowhereyoufoundsupportinthetextforyouranswer.Thatway,youwillbeabletoanswertwoquestionswithaboutthesameeffortittakestodoonequestion.

3 Tryreadingthepassagesbeforedoingthequestions.

Moststudentswillfinditusefultoreadthroughthepassagesbeforedoingthequestions.Why?Becausethemajorityofthequestionsinvolveanalysisofthetext.IftheSATReadingsectioninvolvedmostlytextrecallquestions, itwouldmakesense to review thequestionsbefore reading thepassagesoyouknewwhattolookforasyouread.SincetheSATmainlyhasanalyticalquestionsinvolvinginference,function,suggestion,tone,andpurpose,itwillbemorehelpfultoputyourenergyintodevelopingastronginitialunderstanding ofwhat iswritten. Even questions that refer to a handful of lineswill almost certainlyrequire that you grasp how these lines fit into the passage as awhole. If, however, you have troublestayingfocusedwhenyouread,youmaywant toquicklyskimthequestionsfirstsoyouhaveageneralideaofwhattolookfor.Justbesurethatifyoudothis,youarestillpayingattentiontothebigpictureofthepassageasyouread.Whateveryoudo,decidebeforethedayofthetestwhichapproachbettersuitsyou.

4 Focusontheoverallmeaningofthepassage(s)asyouread.

Youshouldbeable to restate the“gist”ofwhatyouhave read—don’tworryaboutmemorizingdetailsfromthepassage.Youcanchangeyour focusdependingon thepassage type inorder tomaximizeyourcomprehension:

■FICTION&LITERATURE—Readthefirstparagraphortwoabitmorecarefully,andreadthe

remainderofthepassagenormally.Thiscanhelpyoufullyunderstandthecharactersandsettingbeforeyoumoveintotherestofthestory.

■NON-FICTION(SOCIALSTUDIES&SCIENCE)—Readthefirstparagraph,firstsentencesofeachparagraph,andlastparagraphabitmorecarefully,andtherestnormally.Non-fictionistypicallymorestructuredthanfiction,sothesepartswilltypicallygiveyoumorecriticalinformation,suchasthethesisoftheessayandgeneraltopicsofeachparagraph.

YouareNOTexpectedtohaveanybackgroundknowledgeonanyofthetopicsinthereadingpassages.Everythingyouneedtoanswerthequestionswillbegiventoyouinthetextand,ifapplicable,theaccompanyinggraphics.

■PASSAGE1&PASSAGE2—Readthesewithafocusontheoverallmeaning,butpaycloseattentiontotheoverallrelationshipbetweenthetwopassages.Why?Becausetherewillbeseveralquestionsthatinvolvecomparingthesimilaritiesanddifferencesbetweenthetworeadingselections.

Beforeeachpassage,thereisaverybriefsummarythatwillgiveyousomeinformationaboutwhatyouareabouttoread.Besuretoreadthisbeforereadingtheactualpassage,asitwillhelpyoupreviewthegeneralmeaningofwhatfollows.Ifanyof the topicsareunfamiliaror thepassage languageseemstoolofty, don’t be alarmed. If you carefully read thepassages, youwill have the informationnecessary toanswerthequestionswell—theSATmakersdonotexpectyoutobeamasterofallpotentialtopicsandpotentialwritingstyles.

5 Considermakingsmallnotesandannotationsasyoureadthepassage.

Youareabletowritealloveryourtestbooklet,sodonothesitatetojotdownsomebriefnotesasyouread. Since you can take about fiveminutes to read the passage, some of this time can be devoted tomakingashortsentencethatsummarizeseachparagraph.Donotfeelcompelledtodothisifnote-takingdoeslittletohelpyoufocus.JustbemindfulthatitissomethingworthconsideringgiventheSATReadingtimeconstraints.

6 Comebacktoquestionsiftheyseemoverlydifficult.

Thefirstquestionsafterapassagewilltypicallybeabouttheoverallmeaningofthepassage.Ifyouhavenotfullygraspedtheoverallmeaning,comebacktothegeneralquestionsafterhavingdonemorespecificquestions. If you find yourself stuck on a question, come back to it so that you can allow yoursubconsciousmindtoprocessthepossibilities.Onceyoucomebacktothequestionwithfresheyes,youwilloftensurpriseyourselfathowwellyoucanthinkthroughitatthatpoint.

Carelessmistakesarestillmistakes!Don’tallowcarelessnesstosabotageyourperformance.

7 Covertheanswersasyoureadthequestions.

On factual recall tests, checking out the answers before you have formulated an answer can help younarrowitdown.WiththecriticalthinkingquestionsontheSAT,incontrast,youwilloftenfindyourselfmisledbypersuasivebutultimately incorrectanswers.Takecontrolof thequestionsanddon’t let themcontrolyou.

8 Underlineandcirclekeywordsasyoureadthequestions.

Skipping a keywordwhile reading a questionwill likely lead to awrong answer. Instead of quicklyreadingthroughthequestion,andthenhavingtorereadit,readitonetimewellandunderlineandcirclethe most important words as you do so. This will ensure that you do not miss wording critical tounderstandingwhatthequestionisasking.Examplesofthesetypesofkeywordsarenot,primary,infer,suggest,etc.YouareabletowriteontheSATtestpages,sotakeadvantageofit!

9 Createyourowngeneralanswerbyconsideringthecontext.

TheprimaryreadingskilltestedontheSATisyourabilitytoparaphrase(putinyourownwords)whatyou read.Prior to lookingat thechoices, createabroad ideaofwhat theanswercouldbebeforeyoulook.Wheneverpossible,takealookatthecontextrelatedtothequestionsothatyouhavealltherelevantinformation available. For those questions where it is difficult to come up with an answer prior toconsidering the choices, donot “jump” to an answeror eliminate an answerwithoutpatiently thinkingthroughallthepossibilities.

10 Gobacktothepassageasoftenasyouneed.

Mosttestswetakeareclosed-book—theSATReadingsectionisopen-book.Ifyouhadanopen-booktestinschool,youwouldsurelyuseyourtextbookandnotestohelpyouanswerthequestions.WithsomanySATquestionsgivinglinereferencesandkeywords,itmakessensetousethetextwhenevernecessary.

Sincethequestionsarealmostalwaysintheorderthattheyareinthepassage,itisreallyeasytocheckbackwiththepassageasyouworkthrougheachproblem.

11 Usepassageevidencepossibilitiestohelpwiththepreviousquestion.

Thereare10questionsontheSATReadingsectionthatwillaskyoutoselectwhatevidenceinthetextsupportsthepreviousquestion.Ifyouarehavingdifficultywithaquestionthatisfollowedbyanevidencequestion, lookat thelinesofthepassagetowhichtheevidencequestionrefers.Considerthefollowingquestion.

Whichoptiongivesthebestevidencefortheanswertothepreviousquestion?

(A)Lines6–10(“Heintroduced...lives”)(B)Lines37–40(“Whilethis...scientific”)(C)Lines47–50(“Furthermore...space”)(D)Lines65–69(“Despite...expensive”)

Ifthequestionthatcomesbeforethisoneisgivingyoutrouble,lookattheselectedevidencelinesinorder to help you focus what you check out in the passage. The evidence you need for the previousquestionwillbeinoneofthesespots.

12 Theanswerswillbeeither100%correctortotallywrong.

Asingleword can contaminate an answer,making it completelywrong.Whenyounarrow the choicesdown to two options, don’t just look for the “best” answer—look for the “flawless” answer. Try toquicklydebatewithyourself thecorrectnessor incorrectnessofeachanswer,knowingthat there isonethatisdefinitelycorrect,andthreethataredefinitelywrong.TheCollegeBoardhasputagreatdealofeffort intocreatingthequestionsyouwillseeontheSAT,soyoucansafelyassumetheywillbeoftheveryhighestquality.

13 Focusonmeaning,notmatching.

Onordinaryschooltests,weareoftenusedtomatchingthechoiceswithfactswerecallfromtheassignedreadingorthein-classlecture.OntheSAT,thefactthatananswerhaswordingthatmatchespartsofthepassagetextisnoguaranteethatitiscorrect.Thereisnothingwrongwithpickingananswerbecauseitdoeshavewordingthatisinthepassage;justdon’tpickanansweronlybecauseithasmatchingwording.Becertaintheoverallmeaningofananswergivesthecorrectidea.

14 Workonpickinguponcontextclueswithworddefinitions.

TheSATreadingsectionnolongerhasthesentencecompletionquestionsofthepreviousversion.Whilememorizingvocabularywillstillhelpyouprepare,youshouldespeciallysharpenyourskillsinpickinguponthemeaningsofwordsbasedoncontext.Evenifyouknowthedefinitionsofwords,youwillneedto determinewhich definition ismost applicable in the particular situation. For example, aword like“compromise”canmeanverydifferentthings.Ifyouare“compromising”withyourfriendaboutwhattodo over the weekend, that shows a willingness to meet someone halfway. If your immune system is“compromised,”youaremorelikelytobecomesick.Buildonthisskillbymakingahabitoftryingtopickupontheappropriatedefinitionsofwordsgiventheircontextinbooksandarticles.Formuchmorein-depthtreatmentofthewordmeaningquestions,checkouttheseparatein-depthexercise.

TheNewSATReadingfocusesonpickingupwordmeaningbasedonthesurroundingcontext,notonmemorizinghundredsofobscureworddefinitions.Whilehavingagreatvocabularyisstillhelpfultotestperformance,itisnotnearlyashelpfulasitoncewas.

15 Justbecauseyoudonotknowaword’smeaningdoesnotmeanitiswrong.

One of the most frequent mistakes students make on word meaning and questions that happen to useelevatedvocabularyisgoingwithawordthat“sortofworks”simplybecausetheyknowthemeaningoftheword.Ifyounarrowtheoptionsdowntotwo,oneforwhichyouknowthemeaninganddoesn’tquitefit,andtheotherforwhichyoudonotknowthemeaning,gowiththewordyoudonotknowsinceithasthepotentialtobe100%correct.

16 Whenuncertainaboutyourstrategy,givetheSATthebenefitofthedoubt.

On poorlywritten tests, tricks and gimmicks can help you succeed—such shortcutswill not help you

perform well on the new SAT. The new SAT is going to be an extraordinarily well-constructedassessment,giventheamountoftimeandresourcestheCollegeBoardhasdevotedtoitsoverhaul.Asaresult,donotwasteyourtimeandenergywhiletakingtheSATlookingforflawsinthetest.Instead,givethe SAT Reading section the benefit of the doubt and focus on how you can improve your readingcomprehensionandcriticalthinkingskills.

PUTTINGITALLTOGETHERNow,let’sputourstrategiesintopracticewithsomesamplepassagesandquestions.ThefirstpassageisanonfictionSciencepassage.Whilereading,doreadallofthepassage,butfocusabitmoreonthefirstparagraph,topicsentences(i.e.,firstsentenceofeachparagraph),andthelastparagraph.Sinceitisnon-fiction,thesepartswilllikelytakeongreatersignificanceinconveyingthebroadideaofthepassage.Youmayalsotryannotatingorunderliningasyoureadtoseeifitimprovesyourfocusandcomprehension.Trytotakeabout5minutestoreadthispassage.

TimeTravel

Timetravelhaslongintriguedus;it’senoughtospurawholesub-genreofsciencefiction.It’smind-bogglingtoconsideralloftheimplicationsoftravelingthroughtimeandhavingfreewill.It’seasytoseethattheuniverseasweknowitwouldberather

Line unstableif,forinstance,youcouldtravelbackwardsintimeandkillyourowngrandfather.(5) Butisitpossible?

Tobegintounderstandthepossibilitiesoftime,wefirstneedabriefintroductionofspacetime.We’reallfamiliarwithourthree-dimensionalworld,butweneedtoconsiderafourthdimensionaswell—time.Timepasses.Therefore,youcansitstillinachairnottravelinginthree-dimensions,buttravelinginspacetime.Wethinkoftimeas

(10) passingforward.StephenHawkingexplainsthisasthree“arrowsoftime”.Thethermodynamicarrowoftimepointsfromatimeoflowentropy(highorganization)toatimeofhighentropy(loworganization/highchaos).Itpassesfromaglassofwatersittingonatabletoashatteredglassandapuddleofwateronthefloor.There’sapsychologicalarrowoftime:werememberthepast,butnotthefuture.Finally,there’sacosmological

(15) arrowoftime.Theuniverseisexpanding(thoughthisarrowcouldreverseinthefuture).

Wetendtothinkoftimeasanabsolute:thereare60secondsineveryminute,andmy60secondsshouldbethesameasyour60seconds.However,Einstein’stheoryofrelativitydefiesthiswithtimedilation.Whenabodyapproachesthespeedoflight,

(20) timeeffectivelyslowsdown.Therefore,ifobserverstravelingatdifferentfractionsofthespeedoflightweretoholdclocks,theclockswouldbetickingatdifferentspeeds.Timeisrelativetotheobserver,ratherthanabsolute.Considerwhat’sreferredtoasthetwinparadox:oneidenticaltwinstaysonearthandtheothertravelsnearthespeedoflightinaspaceship.Thetwinaboardthespaceshipwillexperiencetimedilation,and

(25) thuswillagelessquicklythanthetwinathome.Furthermore,ifthetwin’sjourneywerelongenough,thetwincouldreturntoearthtofindthateveryoneheorsheonceknewwaslongdead.Essentially,thiswouldbetravelingintothefuture.

Thisisn’ttheexcitingkindoftimetravelfromthesci-fibooksandmovies,though.Wewouldpreferourtimetraveltobeinstantaneousandnotlimitedtothefuture.So

(30) far,we’veexaminedonlylineartimetravelintheformofslowingtimedown.Butdoestimehavetopasslinearly?Isitpossiblethattherecouldbeloopsinspacetimeleadingtothepastandfuture?Onepossiblecandidateforsuchtravelisthepresenceofworm-

holes.Whilewormholesalsoseemthestuffofsci-fi,theirbasisisactuallyinapaperwrittenbyEinsteinandNathanRosen,wheretheyreferto“bridges”inspacetime.They

(35) believedbridgestobeextremelyunstableandthusonlytemporary.Theideabehindthesebridges/wormholesisthattherearetheoreticaltunnelsbetweentwofarapartlocationsinspacetime.Thedistanceofthewormholewouldn’tnecessarilyhavetocorrespondtothedistancebetweenthetwolocations.Thereisevidencethatsuchwormholescouldtheoreticallyexist,butthatisbeyondthescopeofthispaper.We’llsuffice

(40) ittosaythatevenifwefindsuchwormholes,we’dhavetofigureouthowtostabilizetheminordertoutilizethemfortimetravel.

StephenHawkingcurrentlybelievestimetravelintothepasttobeimpossibleformanyreasons.Onelessthanscientificreasonisthathumanstendtolove“spillingthebeans.”Ifsomeoneinthefuturehadfiguredouthowtotimetravelintothepast,heor

(45) shelikelywouldhavetraveledbackandtoldus!Sowillweeventuallybeabletotimetravelwithoutlimit?Ican’tsay;mypsychologicalarrowoftimedoesn’tpointthatway.Ohwell,it’sallrelativeanyway.

Witheachofthesequestions,takethesegeneralsteps:

■Coveryouranswerchoicesasyouconsiderthequestion.■Underlineandcirclekeywordsasyoucarefullyreadthequestion.■Createananswerinyourownwordsbasedonthecontext(oratleastpatientlyconsidertheanswers

withoutjumpingtoanythingoreliminatinganythingprematurely).■Carefullyevaluatetheanswers,pickingonlyananswerthatis100%correct.

1.Asusedinline10,theword“passing”mostcloselymeans(A)living.(B)moving.(C)throwing.(D)succeeding.

2.TheauthormoststronglysuggeststhatmostpeoplewouldreacttoEinstein’snotionsoftimetravelwith(A)optimism.(B)derision.(C)disappointment.(D)bewilderment.

3.Whichoptiongivesthebestevidencefortheanswertothepreviousquestion?(A)Lines13–14(“There’sa...future)”(B)Lines19–21(“Whena...speeds”)(C)Lines25–27(“Furthermore...future”)(D)Lines28–29(“Thisisn’t...future”)

4.Theprimarythemeofthepassageis(A)informedcontemplation.(B)skepticaldismissal.(C)imaginativemusing.

(D)factualpresentation.

Thenextexampleisafictionpassage—thiswillbe thefirst typeofpassageyouwillencounteron theSATReadingTest.Whenreadingthis, takethingsmoreslowlyat thebeginningsoyouareclearonthesetting,characters,andearlypartoftheplot.Youwanttoavoidgettinghalf-waythroughthepassageonlyto realize that you haven’t understoodwhat you have read up to that point. Readingmore slowly canactuallyhelpyoucompletethetaskmorequickly,solongasyouunderstandthingswellthefirsttime.

ThefollowingpassageisfromIrinaPetrov’s,MorethanManySparrows.ThesettingisruralRussia1917.Kolyaistheheirtoalargeestate,uponwhichAnnaandherfamilyreside.

Already,thefirewasdying.Kolyawatched,stonily,silently,asthewarm,orangeribbonsbegantounravel,andvanishbeneaththecinders.Tofeedthethingseemedpointless—almostcruel—andhewonderedifperhapsitwasbettertostampitoutthantoletitstarve.Hetoo,afterall,washungry.Hetoo,afterall,wascold.

Line(5) Alreadythatwinter,hehadfedithalfthebooksinhisgreatgrandfather’slibrary.

IthadeatenupalltheNapoleonicsetteesandtablesthatonceadornedhisancestralhome.He’devenofferedithismother’sbelovedmandolin,lettingthestringsonwhichshe’dpluckedhissomberlullabiescatchfire,snap,andturntoash.Hewatchedithappen,andfeltnothing.Nothing,thatis,butwarm.Itwaswinter,andsentimentality

(10) wasnotinseason—norhaditbeenformanymonths.Besides,noonestilllivinginthehouseknewhowtoplayit.

“Willthisbeenough?”Annaenteredthebarrenparlor,shivering,anddraggingarocococradlebehindher,“It’snotlarge,butIthinkit’sdry.”

Kolyanodded,watchingherwordsturntoplumesofpalesteamasshespoke.Much(15) likethelullabies,thecradlehadlongoutliveditspurpose.Withpalehands,hebroke

freethefirstturnedbar,andsnappeditoverhiskneebeforecondemningthesplinteredendstothefurnace.Annasat,andwarmedherhands.

Shewasright:thewoodwasdry.Anddrywood,lately,wasworthmoretothemthanrubies.Therains,Kolyarecalled,hadcometooearlythatautumn.Downinthevillage,

(20) whereAnna’sfamilylived,he’doverheardthepilgrimsinthestreet,withtheirsilverybeards,andmud-cakedsandals,speakingofagreatrasputitsa*tothesouth.They’dsaiditwasanillomen.They’dsaidtherewerestirringsinthecapital.Kolya,atthetime,hadsmiled.Itpleasedhimtobelongtosoremoteaprovince,cutofffromtheuglinessofcitiesandthesouthernroads.

(25) Buteverydayuntiltheyfroze,therainsgrewheavier,andthenewsstillworse.AprincedisappearedinMoscow.Afarmer’swagonwentmissinginthemire.Theyfoundhishorsethefollowingmorning,burieduptoitsbridle,thoughhe,likeasailorlostatsea,hadvanished.Itwasn’tlongbeforethecartsquitcomingaltogether,andinOctober,withthewaron,thetrainquitcomingaswell.Therewasnotimberfromthe

(30) taiga,nograinfromthesteppes,norsunflowerseedsfromthefertile,far-offsouth.AndKolya,forthefirsttime,cametoknowthecostoflivingattheedgeoftheworld.

“We’rerunningoutoftinder,youknow,”Annaturnedtohim.“Howmuchlongerdoyoureallythinkwecanstayhere?”

Kolyafrowned,“Theentirehouseistinder—wecantearouttheraftersifweneed(35) to,”hesaidashelaidmorewoodonthefire,andhungacastironkettleoverthecoals.

“Solongastherootcellarholds,Ithinkwecanstay.”

“Buthowlongwillthatbe?”Annacreasedherbrow.

Shewasanxious.Herfatheroughttohavereturnedfromhistraplinefourdaysago,butthesnowshadbeenheavierthanexpected.Probably,hewasjustcaughtonthe

(40) othersideofthepassuntilthestormcleared—probably.Kolyastirredthekettleslowly,mixingtogethertheblood-redbeets,andtheice-whitecabbage.Thenightlystewswithwhichthey’dstartedoutthewinterhadgivenwaytoathin,translucentsoup.Itwouldnotbelong,Kolyamused,beforetheywerelittlemorethanseasonedwater.

“Idon’tknow,”hereplied,raisingarabbitbonefromthebottomofthepot—the(45) onlyremnantofherfather’slastouting.HeladledoutabowlforAnna,“Atleast,noone

willgohungrytonight.”

Shedidnotsmile,butleanedbacktogazeattheintricatemillworkandmuralsoverhead.

“WhenIwasyoung,youknow,”shesighed,“Ioftendreamtoflivinghere.Dolokhov(50) Palace—itseemedsomysteriousuponthehill.Idreamtoflavishwinterballs,ofbeing

courtedbyboyswhowouldcallme‘Countess’.”

Kolyasmirkedgrimly,takingabowlforhimself,andsatbesideher.

“Itisadreamcometrue,then,CountessAnna.”

Sheshivered.Evennow,shecouldneithersmile,normeethisgaze.

(55) “Perhapsitis,PrinceNikolai.ButnowI’dmuchratherbeawake.”

*‘Seaofmud,’asemiannualoccurrencewhentheunpavedroadsofRussiabecomedifficulttotraverse

1.Theauthorusestheword“probably”inthesentenceinlines39–40(“Probably...probably)toemphasizeAnna’s(A)confidenceandself-assurance.(B)anxietyanduncertainty.(C)belligerenceandhostility.(D)depressionandlackoffaith.

2.Kolya’spointofviewpresentedintheparagraphsinlines18–31canmostfullybedescribedas(A)generallyoptimistic.(B)consistentlypessimistic.

(C)shiftingfromappreciationtomelancholy.(D)shiftingfromdespairtohope.

3.Whathappensinthepassageasawhole?(A)Ayoungcoupleresolvestotakemeasurestoimprovetheirlives.(B)Twothinkerssharetheirviewsoncontemporarypoliticsandeconomics.(C)Twochildrenmuseonthestatusoftheirlovedones.(D)Twocharactersconsiderthepastandtheirpresentsituation.

Next up is a Passage 1 and Passage 2 selection—two authors will express their views on the samegeneraltopic.TherewillbeoneselectionlikethisontheSATReading.YoumaywanttoreadPassage1firstandanswerthequestionsaboutit, thenreadPassage2andanswerthequestionsaboutit,andthenanswer the comparative questions that come at the end of the series of questions. If you read bothpassagesbeforeansweringanyofthequestions,besurethatyouconsiderhowthepassagesaresimilaranddifferentfromoneanotherbecausetherewilldefinitelybequestionsthatwillhaveyouanalyzethepassages’ relationship. This passage also has a graph that relates to the information in the passage—graphswillaccompanytwoofthepassagesintheSATReading.

TwocontemporarywriterssharetheirthoughtsontheElectoralCollegeoftheUnitedStates.

PASSAGE1

Theelectoralcollegehasreceivedmuchattentionlately,primarilyforbeingantiquatedandabsurd.However,Iwouldarguethatit’sactuallymuchmoreabsurdthanmostrealize.Historyclassprovidesuswitharatherfragmentedunderstandingoftheelectoralcollege.

Line(5) Weteachinschoolsthatapresidentialcandidatemustreceiveanabsolutemajority

oftheelectoralcollegevotestowinthepresidentialelection,butwedon’tteachthemechanicsofhowacandidateactuallyreceivesthesevotes.Aparty’selectorsarechoseninavarietyofways,butthey’retypicallyreputablemembersoftheparty.Whenacitizencastsavoteduringthepresidentialelection,heorsheisactuallyvotingfor

(10) aparticularparty’selectorstocasttheirvotesforthepresidentialcandidatetowhomtheyhavepledged.

Whatwecertainlydon’tteachinschoolsisthattheseelectorsinmanystatesaren’tevenlegallyobligatedtoactuallychoosethecandidatewithwhomthey’reallied!Asof2000,therewere24stateswithoutlawspreventingthis,foratotalofapossible257

(15) roguevotes.Infact,therehavebeenaminimumof157casesofsuch“faithlessvoting.”Until2008,Minnesota’selectorsvotedanonymously,thusthenumberoffaithlessvotesisalmostcertainlyhigher.Eveninthestateswheretherearepenaltiesforfaithlessvoting,thepenaltiestendtobeminimalandaren’ttypicallyenforced.Thus,theonlythingskeepingelectorshonestarethehonorcodeandthedesiretostayreputable

(20) membersoftheirparties.

Asidefromfaithlessvoting,therehavealsobeencaseswheretheelectorsdiedinbetweenbeingchosenaselectorsandcastingtheirvotes.Obviously,thisisn’tafrequentoccurrence,butstillaproblemthatwewouldn’thavetocontendwithinapopularvotesystem.

(25) It’shardtounderstandwhywemaintainasystemwithsomanyglaringproblemsandveryfewredeemingqualities.Manycitetraditionandtheinsightofourforefathersasreasonsforpreservingtheelectoralcollege,buttheelectoralcollegeisn’tevenintactinitsoriginalform,sincethesecondplacecandidatenolongerservesasvicepresident.It’swaypasttimewecutourlossesandadmitthatwe’vebeendefendinga

(30) ridiculousarrangementfordecades.Ifweweren’tashamedofit,maybewe’dactually

beteachingitsintricaciesinschool.

PASSAGE2

TheironfabricofClassicAmericanEssencehadagoodrun.Itweathereditswaythroughthecenturies,bendingwiththeebbofturmoilandstretchingwiththeflowoftimethatbroughtustothispoint.But,theinfrastructurewasnevermeanttobeapermanent

(35) solution;itwasastopgap,aframeworktobepatchedupandtorndownandremodeledasthecountryanditspeoplegrew.Somewherealongtheline,though,weturnedourattentiontoothermatters,andweforgotthatbasicpremiseofoxidation:eveniron,yousee,corrodes.

Washington,Franklin,Jefferson,Adams:greatmen,oneandall.Yet,onlymen—(40) flawedandmisinformed,limitedbythepervasivemisconceptionsoftheeraand

hamstrungbytheabsenceofelectricity,Internet,RedBull,andfishoil.Nonetheless,witheachmonumentbuiltandmountainfacecarved,witheachcoinmintedanddollarbillprinted,wecametoseethemlessasmortalsthandemigods,untilwecametothepointofnoreturnwhenourforefathersbecameOurForefathers.Now,seemingly

(45) everypoliticaldebatehingesontheinfallibilityoftheseframers,andprogressisoftenthwartedbytheunimaginativebutomnipotentfour-wordobjection—“buttheConstitutionsays....”asifanythingwrittenbyquillandvelluminsidelead-paintedwallscouldeverbeindisputableinitsabsolutecertainty.

Morethan200yearsofcomplacencyhaveleftuswithsomethingresemblinglessa(50) federalgovernmentthanayardsaleofantiquatedinstitutions,withnonemoredusty

thantheelectoralcollege.Considerthatit’stheoreticallypossibletoreceivejust11votes,haveyouropponentreceive200million,andstillwintheelectionundertheelectoralcollege.

Suchisthenatureofthe“winnertakeall”systeminwhichwinningthe11most(55) populousstatesbyaslittleasonevoteisenoughtoovercomelandslidedefeatsinthe

other39statesandtheDistrictofColumbia.

Whateveritsinitialpurposemayhavebeen,perhapsit’stimetoeschewtheesotericinfavorofsimplearithmetic.Letusturnourattentiontoprogress,lestourferventinsistenceontraditionleaveusobsolete.

1912U.S.PresidentialElectionResults

Source:http://uselectionatlas.org/

1.Therelationshipbetweenthepassagescanbestbedescribedaswhichofthefollowingstatements?(A)Passage1andPassage2bothcitepoliticalauthorityfigurestomaketheircases.(B)Passage1focusesmoreonvotingtechnicalitieswhilePassage2focusesonhistorical

foundations.(C)Passage1hasmoreofapiousviewoftheFoundingFathersthandoesPassage2.(D)Passage1focusesmoreonpoliticaldishonestywhilePassage2focusesoneconomic

repercussions.

2.WhichstatementfromPassage2ismostconsistentwiththestyleofargumentprimarilymadeinPassage1?(A)Lines36–38(“Somewhere...corrodes”)(B)Lines41–44(“Nonetheless...Forefathers”)(C)Lines46–47(“anything...certainty”)(D)Lines51–53(“Consider...College”)

3.Someonewhodisagreedwithbothpassageauthorscouldmakewhichofthefollowingstatements,basedoninformationinthegraph,mosteffectivelytoargueforherviewpoint?(A)Thedatashowsthattheelectoralcollegeisinherentlyflawed,sincethewinnerneednoteven

comeclosetowinningthepopularvote.(B)Theelectoralcollegeensuresnationalunityevenwhentherearewidelydifferentpoliticalviews.(C)Theelectoralcollegepreventssomecandidateswithsignificantsupportfromhavinganyvoicein

executiveleadership.(D)Thegraphshowshowelectorscaneasilychangetheirintendedvotes,sometimeswithcomplete

anonymity.

ADDITIONALPRACTICEPASSAGESTheSATReadingTesthasmoremodernscienceandmodernsocialsciencewritingthanithadinthepast.Trythesenexttwopassagesforadditionalpracticewiththetypesofpassagesandquestionsyouarelikelytoencounterontestday.

ANaturalSynthetic

In1970,NormanBorlaugwasawardedtheNobelPeacePrizeandcreditedwithsavingoverabillionpeoplefromstarvation.InwhatisnowcalledtheGreenRevolution,Borlaugledtheresearchanddevelopmentoveratwo-decadespan

Line beginninginthe1940’stodramaticallyincreaseagriculturalproductionworldwide.(5) HeintroducedthesyntheticfarmingmethodsalreadycommonintheUnitedStates

andBritaintoaglobalmarket,focusingparticularlyonthedevelopingworld,andsucceededinhikingfoodproductionandsavinglives.Borlaug’sinitiativecallsforcelebration.

Yet,itisthesesameagriculturaltechniques—thoseassociatedwithconventional(10) farming—thathavegottenabadrapoverthelasttwenty-fiveyears,causingthe

organicfoodmarkettosoartoawhopping$63billionby2012.Theselaborsaving,high-yieldingtechniquesbeganinthelate18thcenturyandwereperfectedfornearlytwocenturiesbeforeworryspreadthattheyseriouslyharmedthesoilandallowedtoxicchemicalstoenterthefoodsupply.Inthe1940’s,whileBorlaugwasbusyfeeding

(15) theworld’simpoverished,AlbertandGabrielleHoward—bothaccomplishedbotanists—weredevelopingorganicagriculture.Organicfarmingistheprocessbywhichcropsareraisedusingonlynaturalmethodstomaintainsoilfertilityandcontrolpests.

Inthecurrentfoodmarket,GMO’s,orgeneticallymodifiedorganisms,turnnosesfasterthansaturatedfatsandsodapop.Instead,organicfarmersrelyoncroprotation,

(20) greenmanure,andbiologicalpestcontrol,whileexcludingsyntheticfertilizers,pesticides,andgrowthhormones.Organicagricultureissaidtopromotesustainability,openness,health,andsafety,anditsstandardsarecloselyregulatedbytheInternationalFederationofOrganicAgricultureMovements.TheIFOAMbasesthefoundationoforganicfarmingontheminimaluseofoff-farminputsandonmanagement

(25) practicesthatrestore,preserve,andimproveecologicalharmony.Whilethisstrategysoundsmore“conventional”andeffortlessthanwhatisnowcoinedconventionalfarming,organicagricultureisactuallyquitescientific.

Ecologically,organicfarmingisdesignedtopromoteandenhancebiodiversity,soitmustcombinescientificknowledgeandtechnologiestostimulatenaturallyoccurring

(30) biologicalprocesses.Forinstance,organicfarmingusespyrethrin,anaturalpesticidefoundinthechrysanthemumflower,todeterpests,andpotassiumbicarbonatetocontroldiseaseandsuppressunrulyweeds.Furthermore,whereconventionalfarmingfocusesonmassproductionofeachindividualcrop,organicfarmingencouragespolyculture,ormultiplecropsbeingraisedinthesamespace.Toreplacenutrients,organic

(35) farmingreliesonthenaturalbreakdownoforganicmatterbymicroorganismslike

mycorrhiza,whichformsasymbioticrelationshipbetweenfungiandplantroots.Toreplenishnitrogen,greenmanureiscreatedbyleavinguprootedcroppartstowitheronafield,andisthenusedasacovercroptofixnitrogenintothesoil.

Thesciencedoesn’tstopwiththecrops.Onfarmswithlivestock,thefieldofagroecology—(40) whichincludesorganicagriculture—attemptstoprovideanimalswithnatural

livingconditionsandfeed.Justlikeinplants,organicfarmingrejectsanygrowthhormonesorgeneticengineeringinanimals.TheUSDAhasspecificregulationsinregardtoorganiclivestock,demandingthattheanimalsreceiveonlyorganicfeedandarepasturedratherthancaged.

(45) Despitethepopularityoforganicfoods,manyarguethattheconcernsoverconventionalfarmingarealuxuryoftherich.Organicfarmingyieldsfarlessthanconventionalmethods,usesmorelandandmorelabor,andis,therefore,moreexpensive.Whenpricesriseandproductionfalls,itisthepoorthatsuffer.WiththeUnitedNationsreporting870millionpeopleworldwidesufferingfromchronicmalnutrition,organic

(50) farmingfacesatoughargumentagainstthecapital-intensive,prolificconventionalmeans.Sciencehasalotmoreworktodobeforeorganicagriculturalmethodscanfeedtheworld.

AverageCashCropPrices

1.Theauthorgenerallybelievesthatsyntheticagricultureis(A)aninferioralternativetothemoresustainableorganicagriculture.(B)thehealthiestindividualoptionforconsumers,bothwealthyandpoor.(C)amisunderstoodpotentialsolutiontoglobalfoodshortages.(D)solidlygroundedinscience,unlikeorganicagriculturaltechniques.

2.Asusedinline5,theword“common”mostcloselymeans(A)widespread.(B)lowly.(C)communal.(D)corporate.

3.Anunstatedassumptionoftheauthoristhat(A)organicagriculturehelpspromotediversityofcrops.(B)agricultureisthewaythatmostpeoplewillacquirefood.(C)manypeopleintheworldhavealackofsustenance.(D)organicagricultureisrelativelypopular.

4.Basedonthepassage,whencomparedtoanon-organicfarmfield,anorganicfarmfieldwillmostlikelybemore(A)productive.(B)diverse.(C)mature.(D)centralized.

5.Whichoptiongivesthebestevidencefortheanswertothepreviousquestion?(A)Lines5–7(“Heintroduced...lives”)(B)Lines25–27(“Whilethis...scientific”)(C)Lines32–34(“Furthermore...space”)(D)Lines45–47(“Despite...expensive”)

6.Theparagraphinlines28–38functionstocountertheclaimthatorganicagriculture(A)lacksascientificapproach.(B)isrelativelycostly.(C)hasnutritionalmerit.(D)causeswidespreadstarvation.

7.Asusedinline36,theword“symbiotic”mostcloselymeans(A)environmentallysignificant.(B)hierarchicallypredatory.(C)mutuallybeneficial.(D)agriculturallyresilient.

8.Lines39–41servetodemonstratethat(A)animalsthathavebeenraisednaturallyaremorecontent.(B)providinganimalswithunnaturalfeedisdeleterioustotheirhealth.(C)agroecologyisthefirstattempttounifysciencewithanimalhusbandry.(D)organictechniqueshavealsobeenappliedtoraisinganimals.

9.Basedonthegraph,thedifferenceinpricebetweentheorganicversionofacropandtheconventionalversionofthecropisgreatestwith(A)corn.(B)wheat.(C)soybeans.(D)noneofthese,asitdependsontheparticularyearconsidered.

10.Theinformationinthegraphgivesthemostdirectevidenceinsupportofaclaimmadein(A)lines9–11(“Yet...2012”).(B)lines19–21(“Instead...hormones”).

(C)lines21–23(“Organic...Movements”).(D)lines46–47(“Organic...expensive”).

11.Generalizingbasedonthegraph,onecouldestimatethatorganicproduceisapproximately(A)halfasexpensiveasconventional.(B)onefourthasexpensiveasconventional.(C)twiceasexpensiveasconventional.(D)fourtimesasexpensiveasconventional.

TheSlums

You’redrivingandsuddenly,outofnowhere,youendupinaverybadpartoftown.Itlooksverydifferentfromtheneighborhoodsyouhavejustpassed—thestreetsareroughandpot-holed,thewindowsareboardedup,moststoresarecloseddownand

Line theonesthataren’tarecoveredinspraypaint.Youmightturnthecararoundorlock(5) yourdoorsnervouslyoraskyouriPhonetoreroute.Andyouruneaseisn’tunwarranted:

theseareashavethehighestcrimerates,thehighestunemploymentrates,thelowestgraduationrates,thelowestmedianincomes,etc.Buthowdiditgetthisway?

Theanswermaylieinresidentialsegregation,orthesortingofpopulationgroupsintoareastoshapelivingenvironments.Residentialsegregationoccursonracial,ethnic,

(10) andincomescales;buthistorically,AfricanAmericansarethemostresidentiallysegregated.Theideaistoisolateneighborhoodcontextstomakecertainareasmoreappealingtospecificpopulationgroups.Andwhileitmaynotbeobvioushowgrouping“like”populationscancreateslums,itisproventoproducenegativesocioeconomicoutcomesforminoritygroups.

(15) LetusfirstconsidertheexampleofBaltimore,whereacenturyofpublicpoliciessegregatedmetropolitanlandscapesandturned“blackneighborhoods”intoovercrowdedslums.In1910,residentialsegregationordinancesrestrictedAfricanAmericanstocertainblocksandcreatedfederalhousingsubsidiesthatdirectedlowincomeblackfamiliesawayfrommiddleclasssuburbs.Additionally,favorable

(20) subsidiesweredeniedtosubdivisionsthatrefusedtoexcludeAfricanAmericansentirely.Realestateagenciesbegandirectingprospectivebuyersand/orrenterstorestrictiveareasandbanksbeganredlining,orfailingtoinsuremortgagesinblackneighborhoods.

Theresultofthesepoliciesisclear.Thefederalgovernmentprohibitedblackfamilies(25) fromaccumulatinghousingequityandcoercedblacksintohomeswithhigher

interestrates,whilelocaljurisdictionsforcedpublichousingintodesignatedareasandcreatedovercrowded,under-resourcedneighborhoods.Theschoolssaweducationdeteriorateasteacherswereoverwhelmedwithremediationanddiscipline,andunabletospendthenecessarytimeongrade-levelcurriculum.Exclusionaryzoning

(30) anddiscriminatorypoliciescreatedcommunitiesofpovertywithwhitesfleeingthearea,businessesclosingdown,andfutureinvestmentscollapsing.Withoutemploymentopportunities,thepoorbecamepoorer,anxious,andmoreisolated.

InChicago,asimilarpatternoccurred.Gentrification,orrenovationtoimprovepropertyvalues,forcedthepooroutofcertainareasandsteeredthemtowardothers.

(35) Homeownerswererestrictedfromsellingtoblacksandwhenblackfamiliesdidpurchasehomes,itwasatanaverageofone-thirdhigherinterestrate.ThehousingstarvedAfricanAmericanswerepushedintosegregatedneighborhoodsviaurbanrenewalplans,andmunicipalresourceswerereservedforwhiteneighborhoods.Unemploymentratessoared,publicschoolsystemshadmorestudentsthantheir

(40) budgetsallowed,understaffedpolicedepartmentsrespondedtoviolencewith

violence,andimpoverishedblacksturnedtocriminalactivitytomakemoneyandgainprotection.

St.LouisCountyhasahistoryofstate-sponsoredsegregationthatisdirectlyresponsibleforpolice-communityhostilityinitssurroundingareas.Zoningruleshave

(45) continuouslyclassifiedwhiteneighborhoodsas“residential”andblackneighborhoodsas“commercial”or“industrial,”leavingthelatterstarvingforresourcesandamplelawenforcement.Discriminationpracticesstillallowlandlordstorefusetoaccepttenantswithsubsidizedrents,whilerelocationassistanceprogramsofferincentivestofamiliesthatcomplywiththecity’spopulationshifts.Inessence,federalregulationsenforce

(50) thecontinuanceof“ghettos”orslumsthatareundoubtedlyconnectedtodistrustandviolentprotests.

Whethercausedbyexclusionaryzoning,publichousingenforcement,gentrification,whiteflight,oracombinationofallthesefactorsandmore,ahistoryofresidentialsegregationhaspurposelyimposedareasofpoverty,unemployment,andlack

(55) ofnecessaryresourceslikehealthcare,education,andsecurity.Theeffectsspeakforthemselveseverytimeyouaccidentallydriveintotheslums.Moreover,oncethosepoliciesturnneighborhoodsintoovercrowdedslums,populationsbegintoassociatetheAfricanAmericanswholivetherewithslumcharacteristics,erroneouslymistakingthecausesfortheeffects.Isitanywonderthattheinhabitantsfindthemselvesangry,

(60) desperate,anddistrustful?

1.Theauthor’soverallapproachisbestdescribedas(A)presentingfirst-handhypotheticalsituationstodemonstratetheseverityofaproblem.(B)analyzingnationwidestatisticaldatatogeneralizesociologicalpatterns.(C)systematicallyaddressinglikelyobjectionstoaprincipalclaim.(D)usingspecificcasestudiestohelpreadersmoredeeplyunderstandasubject.

2.Basedonthefirstparagraph(lines1–7),itismostreasonabletoinferthattheauthorbelievesherreaders(A)donotthemselvesliveinaslum-likeneighborhood.(B)arenaturallyconcernedaboutthewelfareoftheinnercity.(C)areinterestedinpotentiallybecominginvolvedincrime.(D)havecellphonesthatarenotequippedwiththelatestsoftware.

3.Lines8–9(“orthe...environments.”)mostdirectlyfunctionto(A)giveanexample.(B)provideaconcretedetail.(C)addressanobjection.(D)defineaterm.

4.Thepassagemoststronglyimpliesthatslumswereformed(A)accidentally.(B)belligerently.(C)intentionally.(D)randomly.

5.Whichoptiongivesthebestevidencefortheanswertothepreviousquestion?(A)Lines5–7(“Andyour...etc.”)(B)Lines31–32(“Without...isolated”)(C)Lines52–55(“Whether...security”)(D)Lines57–60(“populations...distrustful”)

6.Asusedinline21,theword“prospective”mostcloselymeans(A)wealthy.(B)potential.(C)perceived.(D)privileged.

7.Thepassagesuggeststhatslum-likeneighborhoodsaremorelikelytobewhatwhencomparedtonicerneighborhoods?(A)heavilypoliced.(B)mixed-use.(C)zoned.(D)individualistic.

8.Whichoptiongivesthebestevidencefortheanswertothepreviousquestion?(A)Lines1–4(“You’re...paint”)(B)Lines5–7(“Andyour...way”)(C)Lines39–43(“Unemployment...protection”)(D)Lines44–47(“Zoning...enforcement”)

9.Asusedinlines40–41,thephrase“respondedtoviolencewithviolence”mostdirectlymeans(A)policerespondedtocriminalactswithharshreprisals.(B)policeandcitizenswereengagedinopencivilwar.(C)aneyeforaneyemakesthewholeworldgoblind.(D)theviolenceofbothcriminalsandpolicenegatedoneanother.

10.Lines52–55(“Whether...security.”)servetounderscorethattheauthorbelievesthat(A)amultitudeofcomplexcauseshaveledtoaclearresult.(B)aseriesofclearcauseshaveledtoacomplexresult.(C)asingularcausehasledtoasingularresult.(D)asingularcausehasledtoacomplexresult.

AWordsinContext:StrategyandPractice

2

substantialchangeto theredesignedSATis the testingofwordmeaningwithincontextrather thanthroughastandardquizzingofvocabulary.Therewillbe10questionsontheReadingsectionthatare

likethis.Whilethischangeavoidstheesotericandgrandiosedictionofpasttests,itrequiresyoutoreadcloselyandconsidercontextaroundwidelyusedwordsandphrasestointerpretthemostfittingmeaningor impliedmeaning.Thiscanbeparticularlychallengingsince theSATwillpurposelyusewordswithmeaningsthatdependonhowtheyareused.Yet,thiscanalsobeadvantageous—awell-preparedstudentwillhavepracticedusingthesurroundingcontextofawordtodecipheritsintendedmeaning.Moreover,thewordswillgenerallybesimplerandmorewidelyused.So,successnolongerrestsonwhetheryouarefamiliarwiththeword.Ifyoufindthispartofthetestespeciallydifficult,youshouldmakeuseofawide variety of scholarly magazines, articles, and journals as your command of language will onlyimprovewithadeeperunderstandingofhowauthorscrafttheirargumentsandwhichwordstheychoosetoemploy.Whenyoucanthinklikeawriterandusecontextualevidencetoderivemeaning,youarebestequippedforthispartoftheSAT.Thischapterincludes:

■8TipsforApproachingWordsinContextQuestions■Step-by-StepPracticeofEasy,Moderate,andDifficultProblems■50PracticeProblemstoTryIndependently■AnswerExplanationsThatExplainHelpfulThinkingProcessesSTRATEGIESFORAPPROACHINGTHESATWORDSINCONTEXTQUESTIONS

1 ConsiderContextualEvidenceReadthesentencefirst,andthenagain,putting“blank”wheretheunderlinedwordis.Attheveryleast,lookinthepreviousandfollowingsentencesforhintsregardingtheauthor’sintendedmeaning.

2 CoverAnswerChoices

Itcanbeverydistractinganduseupmoretimeifyouconsider theanswerchoices tooearly.Sincethewords testedwill oftenbe commonlyusedwordswith several differentmeanings, the answer choiceswilloftenbewritteninwaysthatcaneasilyconfuseyoubecauseyouhavemostlikelyseentheindicatedwordusedinavarietyofcontexts.

3 CreateaGeneralSynonym

Before looking at the choicesgiven, replace the “blank”withyourowngeneral synonym.This canbebroadorvagueorsimplified—don’tworrytoomuchaboutthat.Instead,usethisexercisetomakesureyouhavesome ideaof the intendedmeaningbeforeyou jump into thechoicesgiven.This toolwillbeparticularlyusefulineliminatingotheroroppositemeaningswithinthechoices,andsaveyoutime.

4 UseProcessofElimination

Uncoverthechoicesandeliminatethoseanswersthatsoundnothinglikeyoursynonym.Ifthereisawordyouarenotfamiliarwith,leaveitopenforfurtherconsideration.Youneverwanttonegateananswerjustbecauseyoudon’tknowthedefinitionofit.Likewise,ifyougetitdowntoachoiceyouarefamiliarwithbutisonly“sortofright,”andachoicethatsoundsappropriatebutisunknowntoyou,youshouldgowiththeunfamiliarwordsinceithasthepotentialtobe100%correct.

5 ReadtheSentencewiththeRemainingChoicesThenewformatisnotonlygoingtotestvocabulary,butusageaswell.Especiallywhenyoucannotdecidebetweentwoanswers,readingthesentencewiththeanswerchoicereplacingtheoriginalword(s)canoftenclearitupforyou.Itallowsyoutoconsiderthemeaning,butalsotohearthesentenceanddecideiftheusageisproper.Certainly,iftheauthorisusingaverbandyoureplaceitwithanounthathasarelatedmeaning,mouthingthesentenceoutsilentlywillhelpyounoticethatmistake.

6 PickYourAnswerBasedonMeaning

Studentsmake two commonmistakes on these types ofword choice problems.First,without coveringtheiranswersandcreatingageneralsynonym,theyjumpto“matching”words,orthosethatareassociatedwith theunderlinedword,butdon’tfit themeaningof thesentence.Second,withoutpayingattentiontotheirownsynonym, theybecomedistractedbywords that“makesense” in thesentence,butchange themeaningoftheindicatedword.Itisnotenoughtomatchwordswiththeirdefinitions,asthewordstestedwilloftenderivetheirmeaningfromthecontextsinwhichtheyareused.Additionally,itisnotenoughtopick aword that is logical in the sentence; it has tomake senseand fit themeaning of the underlinedword.

7 ReadtheSolutionsinThisBook

Tomake those finishing touches and eliminate carelessmistakes, or to figure outwhy you continue tomakethesametypeofmistake,refertotheanswerexplanationsattheendoftheexercise.Thisbookisdesigned to be a unique resource for you, and acknowledging that youwere incorrect is not nearly asimportant as understandingwhyyouwere incorrect.Go to the solutions so that you can improve yourSAT-thinking.

8 Read,Read,Read

Inthepreviousformat,itmadesensetostudyhundredsofobscurevocabularywordsandhopethatyouwouldcomeuponthatsamerare,specializedlanguageonthetest.However,thenewformatisdesignedtotestpracticalandwidelyusedwordsthatwillcontinuetocomeupincollegeand/oryourcareer.So,youcanhelpyourselfdramatically—forthetestandlifemoregenerally—whenyoubecomefamiliarwiththesewordsandthewaystheyareused.Startwithnewspapers,magazines,andscholarlyjournals.

STEP-BY-STEPPRACTICE

EasyExamples

1.ThevacationhadtheoppositeeffectonJim.Hedespisedtheupcomingdaysandwasdepressedatthethoughtofhisrecentbreak-up.AlthoughhemarchedintoworkfifteenminutesearlyonMondaymorning,hewasfeelingveryblue.(A)energetic(B)dispirited(C)indigo(D)rejuvenated

CorrectAnswer: (B) To approach this question, consider the context clues. Vacation is supposed toreplenishandrelaxyou,butithadanoppositeresult.WordslikedespisedanddepressedmakeiteasytoguessatJim’scondition.Althoughisacontrastingword,sohowhefeelsmustcontradicthisgetting toworkearlyandprepared.(C)referstotheactualcolorblue,sowecaneliminatethis.Andthen(A)and(D)areantonymstowhatthewriteristryingtoconvey.

2.Thadwalkedthroughthetunneltoloudapplause.Hemountedthepitchingmoundandbreatheddeeplytocalmhisnerves.Hethrewhardandfast,butwasoffhismark.Thecatcherhadtorunandretrievethewildball.(A)inaccurate(B)verynervous(C)notonthemound(D)careless

CorrectAnswer:(A)ThecontextallowsustounderstandthatThadisanervousbaseballplayergettingreadytothrowthefirstpitchofthegame.Sincetheballwaswild,wecaninferthattheintendedmeaningis(A),inaccurate.Choices(B)and(C)couldbetrueaboutThad,butdon’trepresentthemeaningoftheindicatedwords.Choice(D)impliesthathewasn’tattentiveorconcerned,whichweknowisfalse.

3.Thebosssaidtocallitaday.Therainmadetheworkimpossibleandhalfthematerialshadn’tbeendeliveredontime.Hewastoofrustratedtothink!(A)dowhatyoucan(B)startthework(C)appreciatethesunlight(D)endthetask

CorrectAnswer: (D) This common idiommeans to “declare the end of a task.” The rain and lack ofmaterialshint at the impossibilityofgetting the jobdone.Notice that (B) is theoppositemeaning, (C)referstodaylight,and(A)couldmakesense,butdoesn’tfitthemeaning.Rememberitisn’tenoughthatthesentencewouldreadokaywiththisnewmeaning,italsohastofittheauthor’smeaning.

ModerateExamples

1.Whenthespeakerwasdone,thecrowdroseandclappedvigorously.Yet,atthequestionandanswersession,attentivespectatorsraisedseveralpoints—particularlythatthespeech’smorallessoncame

offascondescendingandwasgenerallyunfounded.(A)increasedsalary(B)elevated(C)putforward(D)nourished

CorrectAnswer:(C)Thislineisreferringtoviewerswhobroughtup,orpresented,pointsthatweakenedthespeaker’scredibility.Therefore,themeaningisthattheyputforwardseveralpoints.(A)isthetypeofraiseonemightgetatajob.(B)isthedefinitionforraisewhenitreferstoliftingsomethingup.And(D)indicatesbringingupandcaring for someone, likeamothermight raiseachild.Makesure to read thesentencewiththechoicesreplacingtheunderlinedportionifyouarestrugglingbetweentwo.

2.Ellenfoundtherestauranttobeadimeadozen.Itlackeddistinctionandthefoodwasmerelyaverage.(A)unique(B)expensive(C)common(D)belowpar

CorrectAnswer:(C)Since therestaurant“lackeddistinction”andwas“merelyaverage,”wecan inferthatthisrestaurantwasalotlikeothermediocreplacestoeat.So,(A)isoutrightawaysinceitimpliesthatitisone-of-a-kind.(B)iswrongbecausethisidiomdoesnotactuallyrefertoanythingmonetary.Andinchoosingbetween(C)and(D),rememberthat“average”ismorelikecommonthanbelowparis.

3.Inscienceclass,thetwofriendsfranticallymixedtheirchemicalsolutionstogether.Theyhadfailedtomeetovertheweekendandtheirassignmentwasnowhereclosetobeingfinished.Whentheyreceivedafailinggrade,theteacherremarkedthatthesolutiontotheirproblemcomesfromadequatepreparation.(A)resolution(B)mixture(C)choice(D)origin

CorrectAnswer:(A)Wecanputourownsynonyminhereprettyeasily.An“answer”toaproblemistheintendedmeaning,so(A)worksbest. (B)refers to thefirstusageofsolution,amixtureof liquids. (C)denotes an option. And (D) inaccurately signifies that the teacher is referencing the cause—a lack ofpreparation—ratherthanthesolution,adequatepreparation.

DifficultExamples

1.Sammycamehomeeuphoricandthoroughlyamused.Fromtheparadetotheiranimateddinner,sheandTanyahadenjoyedeverymomentoftheirdaytogether.Thatgirlisariot,shethoughttoherself,smilingatthefuntheyhadhad.(A)aloudprotest(B)theone(C)anuproar(D)anentertainingperson

CorrectAnswer: (D)Riot is a word with several definitions, so be careful in your approach to thisquestion. Choices (A), (C), and (D) are allmeanings ofa riot, but only (D) captures this sentence’s

precise meaning. (B) changes the meaning, and so is an inappropriate word choice.Uproar literallymeans“alouddisturbanceorchaos.”

2.Mr.Thompsondecidedtotakethebookoffthesyllabuspermanently;hewouldhavetoreplaceitwithanotherautobiography.Thebook’sextremeviewssurpassedliberalandborderedonradicalism.Hecouldalreadyheartheparentsarguinginhisoffice.(A)non-fictionalaccount’s(B)author’s(C)volume’s(D)politician’s

CorrectAnswer:(B)Sincethisisreferringtoextremeviews,wecaninferthatitisnottheactualbookbeing discussed. Furthermore, as it is autobiography, it is safe to assume these views are those of theauthor,asin(B).(A)and(C)bothrefertothebookitself,whichisincapableoftakingperspective.And(D)assumestheauthorisapoliticianandchangesthemeaningofthesentence.

3.Leeassertedthatamovielikethathasacertaineffectonyou.Afterwatchingit,hehadnotonlyjoinedtheuniversity’sUNICEFchapter,buthadalsobegunvolunteeringattheYMCA.(A)isconsequential(B)createsachangein(C)heals(D)resultsinbenefitsfor

CorrectAnswer: (B) To approach this, look at contextual evidence. Lee is changed by the movie; itcaused him to do things hemight not normally have done. So, the line is referring to a consequentialchangeinLee.(B)fitsthemeaningandisusedproperly.(A)isincorrectbecauseit isinappropriatetosayamovie“isconsequentialyou.”Andwhileitmightdo(C)and(D)forLee,itdoesn’thavetoinordertohaveaneffect.So,thesechoicesdon’tpreciselyreproducethatmeaning.

PRACTICEEXERCISES

Instructions:Beloware50questionsrepresentativeofwhatyoumightseeontheactualSAT.Theyvaryindifficulty,butallrequireyoutochoosetheappropriatemeaningbasedoncontextandwordchoice.Foreach question, choose the answer that most closely fits the meaning of the underlined portion of thesentence.

1.Tocelebrategettinganewjobatalawoffice,Mikedecidedtospendtimewithhisfamily.Theyallwenttotheparkandhadacookout.Mikehadayouthfulnatureandlovedplayingwithhisniecesandnephewsonthejunglegym.(A)environment(B)humor(C)personality(D)structure

2.Spencerwasveryinterestedinwhoallofherfriendshadacrushon.Thepryinggirlaskedthemsomanypersonalquestionstotrytofigureitout.(A)nosy

(B)interference(C)curious(D)lifting

3.Will’sparentsknewhewasaverysmartchild.Hemasteredconceptsinschoolmuchmorequicklythanotherstudents.However,hisgradesdidnotreflecthisintelligence.Hislackadaisicalnaturewashisdownfallashenevercompletedhishomeworkassignmentsontime.Perfecttestscoreswerenotenoughtocompensateformissingassignments.(A)lazy(B)drowsy(C)moronic(D)passionate

4.Johnnywasafraidtojumpoffthedivingboard,sohissistercalledhimachicken.Heknewhewouldn’tgethurtbuthewasstillscared,soheturnedaroundandclimbedbackdowntheladder.(A)weakling(B)fowl(C)petrified(D)coward

5.Theproudboytalkedabouthisaccomplishmentsonlywhenasked;heneverboasted.(A)haughty(B)glorified(C)conceited(D)confident

6.ThediscounteddressfellapartthefirsttimeEmilytriedtoputiton.Shecouldn’tbelievethestitchingwasdonesopoorly.Nexttime,Emilythought,Iwillgotoaboutique.(A)rebated(B)overlooked(C)shabby(D)cheap

7.Juanwassurprisedbywhatheheard,butdiscountedtheinformationaslies.Heknewmanyofhispeerswerejealousofhissuccess,buthedoubtedtheywouldstoopsolow.(A)reduced(B)forgot(C)concession(D)dismissed

8.Inchemistryclass,westartedaunitlearningaboutpH.Acidsoftenhaveacorrosivecomponenttothem.Wehadtomixsomechemicalstogetherandendedupwithanacidthatwassostrongitburnedaholethroughthecontainer!(A)weighty(B)potent(C)tenacious(D)athletic

9.Whenthereportercalledtheactressforastatement,herassistantsaidshewasnotreadilyavailable.Thereporterdoubtedthisashelaterfoundouttheactresshadspentthedayloungingaroundthepool.(A)couldnotbereached(B)wasoutofthecountry(C)wasindisposed(D)wouldbecognizant

10.Aftersheputawhoopeecushionunderhisseatatdinner,Joeysetupapranktoexacthisrevengeonhisoldersister.Itwastimeforhertofeelembarrassed.(A)avenge(B)imposehisretribution(C)enacthiscruelty(D)titfortat

11.Inconsideringtheprosecutor,thejudgedecidedtogiveleewayonsomemistakesasitwashisfirstcase.Shecouldrememberfumblingthroughheropeningargumentthefirsttimeshewasincourt.(A)provideroomtogrow(B)evokecarteblanche(C)tolerateamarginoffreedom(D)grantamnesty

12.Theaudiencewaitedwithbatedbreathfortheclimaxofthemovie.Theentirestoryhadleduptothefinalbattlebetweenthevillainandthehero.Itwasnervewrackingtoseewhowouldcomeouttriumphant.(A)apprehension(B)impatiently(C)stagnantinhalation(D)cessation

13.BennieandJanelikedtoboatontheriveronhotsummerdays.Thecurrentwaspowerful;itpulledtheboatdownstreamincrediblyquickly.(A)contemporary(B)waterway(C)flowingwater(D)cutting-edgetide

14.EmilyhadreservationsaboutbeinginvitedtoKayla’shouse.Thelasttimetheyhungoutithadn’tgoneverywell.Infact,EmilyhadstormedoutangrilyafterKaylamadesomerudecomments.(A)detainments(B)territories(C)doubts(D)skeptical

15.WhileMr.Davis,theartteacher,approvedofcreativity,hecouldn’tsanctionthestudentsexpressingthemselvesintheformofgraffiti.Stylistically,hecouldappreciatethepieces,buttheywerestillvandalism.(A)endorse(B)consentwith

(C)confirmthat(D)boycott

16.Andrewcouldn’thelpit.Hewasstillharboringbadfeelingsabouthisex-girlfriendlongaftertheirbreakup.Therewassimplynoexcuseforhowshehadtreatedhim.(A)disregarding(B)camouflaging(C)entertaining(D)cherishing

17.SinceLizwastryingtoloseweight,shepickedoutleanturkeyatthegrocerystore.Shewrinkledhernoseatthethoughtofhowtastelessthemeatwouldbecomparedtothebeefsheusuallybought.(A)sinewy(B)slant(C)low-calorie(D)nutritious

18.Eloisewassoexcitedaboutherbirthday.Shekeptaskingherparentsabouttheparty,buttheycarefullyskirtedherquestions.(A)bordered(B)detouredabout(C)evaded(D)avoidanceof

19.Eventhoughthehomeworkwasdifficult,Mariadidn’tmindworkingonit.Sheknewinthelongrunitwouldhelpherbemorepreparedforcollege.(A)waswillingly(B)tookoffense(C)couldn’theed(D)adhered

20.Thepawnshopownercarefullyinspectedthering.Whilethepatronwasconvinceditwasrealsilver,theownerdeterminedittobemadeofbasemetals.(A)menial(B)metallic(C)artificial(D)groundwork

21.Theshiningsunwasawelcomebreakfromthemiserableweeksofwinter.IthadbeengloomyandcloudysinceSeptember!(A)dormancy(B)obstruction(C)fracture(D)hiatus

22.Wrensleepilyopenedoneeyeandputapillowoverhisears.Thebirdsweremakingsucharacketitwasimpossibletostayasleep.Heneededhissleepbeforethebigtennismatch.

(A)turmoil(B)clamor(C)roaring(D)swindle

23.Melindawasontheleftinrespecttothestage.Shegaveagreatperformanceandeveryonewasimpressedwithhertalent.(A)deference(B)regard(C)point(D)admiration

24.Mel’strackrecordwasfraughtwithbothawardsandhonors,butalsocountlessreferencessayingshewasconstantlylate.Whensheshowedup,herworkwasimpeccable.However,herspaceynaturegenerallymadeherdistractedandshesometimeswouldn’tevencomeintotheoffice.(A)indicator(B)updatedresume(C)pastperformance(D)achievement

25.Edwin’sadmissiontothemoviewasfreebecausehewasstillundertheageoffive.Oncehewassixorolder,itwouldcost$5forhimtoseemovies.(A)withoutcost(B)comped(C)footloose(D)emancipated

26.JosiahsufferedatthehandsoftheAPExam.Whilehehadbeenstudyingformonthsandfeltaspreparedaspossible,itwasstillaverydifficulttest!(A)gavehisbesteffortto(B)challengedwillfully(C)experienceddifficultywith(D)waswoundedby

27.Eventhoughhegotalotofhisinformationfromcomedynewsshows,Billywasalwaysuponcurrentevents.Theshows,surprisingly,presentedalotofaccurateinformationwithapunchline.(A)unseasonedwith(B)atthesummitof(C)up-to-datewith(D)dashingon

28.Thetrialhadbeenatastandstillformonths.Thatwas,untilnewevidencewasbroughtintoplayforthejurytothinkabout.Followingthis,thevotewasnearlyunanimous.(A)presentedforconsideration(B)becameexpositional(C)scrutinizedforactivity(D)contemplated

29.GrandpaJoewasverysetinhisways.Forthelasttwentyyears,hehadgottenupat7:00o’clock

everymorning,andgonetothelocaldinerforcoffeeandeggsbeforework.Sidneydoubtedanythingaboutthisroutinewouldchangeinthenexttenyears.(A)unwillingtochange(B)steadilydependent(C)againstmodernity(D)disobligingtocustom

30.Theprojectwasgreatintheory.However,outcomesarenotalwaysasquickasintended.Thepolicywritersranintodifficultywithpeoplebeingunreceptivetothenewideas.Assuch,theirprogresshadplateaued.(A)fellintodeepwater(B)wereprovokedwithobjection(C)encounteredresistance(D)confrontedhostility

31.Theplotwascoveredinweedsandotherdebris.Itwasgoingtotakethefamilyhoursandhourstoclearit.(A)storyline(B)space(C)graveyard(D)conspiracy

32.Threehundredpeoplewerepresentforthepresentationonnuclearenergy.Someofthespeakerswereinfavor;others,opposed.Itmadeforaveryinterestingsetofspeakersthatcoveredalmosteverysideoftheissue.(A)introduced(B)inattendance(C)awarded(D)existing

33.Thefightwasabouttobegin.Everyfiberintheboxer’smusclestensedinanticipation.Thiswasgoingtobethefightofthecentury.(A)stressedout(B)swayed(C)arching(D)clenched

34.Aneducationisusuallythoughttohaveintrinsicvalueratherthanextrinsic.Thatis,itsworthcomesfromtheloveofknowledgeratherthansomethingmonetary.However,withtherisingcostoftuition,onecouldmaybearguethatthereisafiscalpricerelatedtoeducation.(A)merit(B)price(C)principles(D)standard

35.Carlstruggledtoregainhisbalance,butitwastoolate.Hepitchedforwardandtumbleddownthehill.(A)inclined

(B)stroked(C)fell(D)threw

36.Whileshereallyneededtransportation,thelowqualityofthecardroveKellitosellit.Shesimplycouldn’taffordtheconstantrepairsitrequired.Ifshecouldtakethebusforjustafewmonths,shehopedshecouldsaveupforanicercar.(A)steered(B)whirled(C)cruised(D)forced

37.Thedoglaidonherjeansandpantedheavily.Itwassuchahotday;hewastryingsohardtocooldown!(A)gaspedforair(B)drooled(C)fellasleep(D)lusted

38.Monicalaughedathercabinmate’sideas.Whiletheyweresomewhatbasedonfact,hertheorieswereessentiallybunk.Pseudoscienceisnotasubstituteforactualscience.(A)indubitable(B)conflicted(C)cot(D)nonsense

39.Walkerdidn’tmindmowingthegrass.Theyardforhistinyapartmentwasonlyaboutthreefeetbythreefeet.Itonlytooktenminutes!(A)threefeet(B)lawn(C)measurement(D)garden

40.Simonwasrearedinasmalltownbyparentswhowerefarmers.Simon’sroomwastowardthebackofthelarge,barnstylehouse.Helovedgoingbacktovisitthefarm.(A)grown(B)raised(C)behind(D)discovered

41.LivecomedyshowswerealwaysHannah’sfavorite.Shereallyenjoyedtheideathatanythingcouldhappenduringthelivelybanter!(A)subsist(B)premeditated(C)improvised(D)energetic

42.Thestormwasblowingtheboatallovertheplace.“Roger!Manthehelm!”thecaptainshoutedovertheloudwind.Rogerrantothesteeringwheelandrightedtheboatbeforeittippedover.(A)friend(B)hurry(C)operate(D)male

43.Thenewspaperprintedfalseinformationinthedailyheadlines.Asaresult,thereporterinchargeofthestorywasfiredandforcedtofindotherwork.(A)journalist(B)article(C)publishingcompany(D)chronicle

44.Rossineededtobookatickettotheauthor’stour.Hehadgreatlyenjoyedthenovel;itwassodifferentthananythingelsehehadeverread.(A)fiction(B)present(C)reserve(D)arrest

45.Thecrowdstartedtoripplewithexcitement.Thefamoussingerwassteppingoutofhercartoenterthevenue.Andrecranedhisnecktotrytogetaglimpseofherbeforeshedisappearedagain.(A)stretchedout(B)magnified(C)constricted(D)massageddown

46.Thecanoewasgoingtooquicklytowardtherapids.Yusefwasbankingontheshoretoslowthemdown.Whiletheyhitthebankhard,itslowedtheboatdownenoughthatYusefandhisfriendwereabletojumpoutandpullthecanoeoffthewater.(A)hittingoff(B)countingon(C)runninginto(D)depositing

47.Emiliaknewthecarwasinbadshape,butshewasunpreparedfortheexorbitantquotetofixit.Shewouldhavetostartworkingdoubleshiftstocoverthecost.(A)finalcost(B)referencedexcerpt(C)citation(D)estimatedprice

48.Pathadallthenotesaboutthecaseinasuitcasethathecarriedwithhimeverywhere.Hewasverynervousaboutwhatmighthappeniftheinformationweretofallintothewronghands.(A)baggage(B)trial

(C)sampling(D)defendant

49.TheextraordinarilystrongmanmightbeagoodenoughweightliftertogototheOlympics!Hehadbeentrainingtodososincehewasayoungman.(A)usedto(B)powerfulforce(C)dominated(D)maypossibly

50.Whenthemarchingbandwonthecompetition,thedrumlinebeatoutacelebratoryrhythmwhilethehornplayerschantedtheschoolname.(A)played(B)pummeled(C)subdued(D)exhausted

AGraphAnalysis:StrategyandPractice

3

nother major change to the redesigned SAT is assessing your ability to analyze quantitativeinformationasportrayedingraphs,tables,andcharts.TheSATnowhasquantitativeanalysisasa

part not just of the math, but of the entire Evidence-Based Reading andWriting section. Five of thequestionsontheReadingsectionwillbegraphinterpretation.Theskillsassessedinthesequestionsareapplicable not just in the physical sciences, but in social sciences, finance, computer science, andanything else that involves using statistics and numbers as evidence. While you will not need anybackground knowledge to answer these questions, taking rigorous courses in the sciences and socialscienceswillhelpyoudoyourbest.Thischapterincludes:

■5TipsforApproachingGraphAnalysisQuestions■32PracticeProblemstoTryIndependently■AnswerExplanationsThatBreaktheQuestionsDownStep-By-StepSTRATEGIESFOR

APPROACHINGTHESATGRAPHANALYSISQUESTIONS

1 Useonlytheevidenceprovided

Since you are not required to use background knowledge on questions, do not allow yourself to pickanythingthatisnotdirectlysupportedbytheinformationinthegraph.Evenifachoicemaybetruebasedonyourmemory,donotchooseitunlessthereisevidencerightinfrontofyou.

2 Becertainthatyouransweris100%correctAsintheSATasawhole,theresimplywillnotbepartiallyrightanswers.Ifananswerismostlycorrectbuthassomesmallpartincorrect,throwitoutandconsidertheotherpossibilities.

3 Carefullyconsiderthegraphlabelsandaxes

Theinformationpresenteddependsgreatlyonthestartingpointsandscalesontheaxes.Takethetimetobesureyouknowexactlywhatisbeingpresented—graphlabelsanddescriptionsareessentialtodoingthis.

4 Don’tjumptoyouranswer

Since the questions arewell-crafted and the answers are quite persuasive, take the time necessary tobreakthequestionsdown.TheSATisnotarapidrecalltestthatyoumustfinishquickly—itisacriticalthinkingtestthatdemandspatienceandcare.

5 Referbacktothetextwhennecessary

Manyofthegraphanalysisquestionswillaskyoutoconnectinformationinthegraphtoinformationinthetext.Beawarethatyouwilloftenneedtogobackandforthbetweenthetextandthegraph.

PRACTICEEXERCISESBeloware32questionsrepresentativeofwhatyoumightseeontheactualSAT.Theyvaryindifficulty,but all require you to choose the appropriate answer based on what is given in the graph. For eachquestion,choosetheanswerthatisbestsupportedbytheinformationprovided.

LeadingCausesofTraumaticBrainInjuries

Source:CentersforDiseaseControl

1.Basedontheevidenceinthegraph,whichofthefollowingistrue?(A)Peoplearefourtimesaslikelytobeinvolvedinafallthanassaulted.(B)Increasedseatbeltusewoulddecreasethenumberoftraumaticbraininjuriesfrommotorvehicle

accidents.(C)21%ofthecausesoftraumaticbraininjuryareunknown.(D)“Other”and“Assault”accountforapproximatelyone-thirdoftraumaticbraininjuries.

2.Accordingtothegraph,15outof100traumaticbraininjuriesarecausedby(A)motorvehicleaccidents.(B)unintentionalblunttrauma.(C)assault.(D)falls.

LikelihoodaJobWillBeDonebyaMachine

Source:Npr.org

3.Whichjobismostlikelytobedonebyamachineaccordingtothegraph?(A)Umpire(B)Bookkeeper(C)Microbiologist(D)BusDriver

4.Usingtheinformationinthegraph,whichstatementistrue?(A)GPSandautomationwillleadtobusdriversbeingobsolete.(B)Careerswithahighlikelihoodofamachineperformingthejobhavepooremploymentoutlooks.(C)Ahistorianislesslikelytohavetheirjobperformedbyamachinethanajanitoris.(D)Morepeopleareemployedasumpiresthananyotherprofession.

5.Fillintheblank:Thejobofa/an_________isthreetimesaslikelytobeperformedbyamachineasthatofa/an_________.(A)umpire,athlete(B)microbiologist,physician/surgeon(C)historian,bookkeeper(D)microbiologist,lawyer

U.S.BeefConsumption

Source:ers.usda.gov

6.A“bestfitline”ofthedatapresentedinthegraphwouldshowthatU.S.consumptionofbeefovertimehas(A)increased.(B)decreased.(C)remainunchanged.(D)peakedin2007.

7.Americansconsumedthefewestpoundsofbeefin(A)2003.(B)2014.(C)2011.(D)2006.

8.Basedontheinformationinthegraph,whatcanbesaidaboutU.S.consumptionofbeefbetween2006and2007?(A)Americansconsumedsubstantiallylessbeef.(B)Americansconsumedsubstantiallymorebeef.(C)Consumptionwasnearlyunchanged.(D)Duetoanincreaseinpopulation,eachAmericanconsumedlessbeef.

World’sTallestMountains

9._______isapproximatelythreetimesastallas________.(A)MountEverest,ChoOyu(B)Lhotse,DhaulagiriI(C)K2,Makalu(D)Noneoftheabove

10.Whichstatementisbestsupportedbytheevidenceinthegraph?(A)Theworld’stallestmountainsareinTibet.(B)30%ofmountainnamesalsocontainanumber.(C)NangaParbatisalessdangerousmountaintoclimbthanLhotse.(D)MountEverestistheworld’stallestmountain.

AverageCostofHigherEducationintheUnitedStates

Source:Nces.ed.gov

11.Thecostdifferencebetweenfour-yearpublicandfour-yearprivateinstitutionswasthesmallestduringwhichyears?(A)2012–13(B)1982–83(C)2001–02(D)1992–93

12.Whichstatementistruebasedontheevidenceprovidedinthegraph?(A)Four-yearprivateinstitutionsexperiencedthegreatestsingleyearincreaseincostbetween

1982–83and2001–02.(B)Onaveragein2012–13,aprivatefour-yearinstitutioncostjustundertwiceasmuchasapublic

four-yearinstitution.(C)Thecostofpublicandprivatefour-yearinstitutionshasincreasedsince1982–83.(D)Thecostoffour-yearinstitutionswilldecreasein2013–14.

13.Whichofthefollowingstatementsisnotsupportedbytheinformationinthegraph?(A)Thecostoffour-yearpublicinstitutionswaslowerthanthecostoffour-yearprivateinstitutions

ineveryyearfrom1982–2013.(B)Accountingforinflation,thecostoffour-yearpublicinstitutionswasmorein1982–83thanin

2001–02.(C)Thecostofprivatefour-yearinstitutionsincreasedtheleastbetween2008–09and2009–10.(D)Thecostofpublicfour-yearinstitutionsremainedunder$10,000until2003–04.

LiteracyRates

Source:CIAWorldFactbook

14.Whichcountryhasafemaleliteracyrateunder50%?(A)Ethiopia(B)Haiti(C)NorthKorea(D)Cambodia

15.InAfghanistan,thefemaleliteracyrateis(A)morethanthemaleliteracyrate.(B)roughlyhalfthemaleliteracyrate.(C)approximatelytwo-thirdsthemaleliteracyrate.(D)thesameastheoverallliteracyrate.

16.Accordingtothegraph,whichofthefollowingistrue?(A)InGreece,morewomencanreadthanmen.(B)ThemaleliteracyrateinNorthKoreaismorethanthefemaleliteracyrate.(C)IranhasagreateroverallliteracyratethanMexico.(D)70%ofthecountriespresentedhaveamaleliteracyrategreaterthantheoverallliteracyrate.

200studentsweresurveyedabouttheirpiepreferences,andthepercentagesofpiesthatwereconsideredthe“favorite”byrespondentsaregraphedbelow:

PiePreference

17.Thefavoritepieofapproximately25%ofpeopleis(A)cherry.(B)pumpkin.(C)apple.(D)peach.

18.Morepeopleprefer____________piethan____________pie.(A)chocolate,apple(B)keylime,peach(C)cherry,coconutcream(D)chocolate,cherry

TotalRevenueforKhanGrocery

19.InFebruary,whatwastheapproximatetotalrevenueforbeverages?(A)$45,000(B)$50,000(C)$28,000(D)$12,000

20.InJanuary,whatpercentoftotalrevenuewasmadeupofproduce?(A)50%(B)30%(C)25%(D)20%

21.Whatmonthhadthelowesttotalrevenue?(A)March(B)October(C)May(D)August

HoursSpentWatchingTV

22.BetweenwhichweekswasthegreatesttotalhourdecreaseinhoursspentwatchingTV?(A)Weeks3and4(B)Weeks4and7(C)Weeks7and8(D)Weeks5and6

23.HowmanytotalhourswerespentwatchingTVbetweenWeeks1and8?(A)45(B)8(C)46(D)15

24.Basedontheevidenceinthegraph,whichofthefollowingistrue?(A)ThefewestnumberofhourswasspentwatchingTVinWeek4.(B)ScreentimeisahealthconcernforAmericanchildren.(C)Week4wasthebusiestwithotheractivities.(D)AnewTVcontributedtotheincreaseinhourswatchedinWeek7.

GroundCovering

25.Betweenwhichmonthsisthepercentageofsedimentcoveringthegroundthegreatest?(A)JanuaryandFebruary(B)AugustandSeptember(C)JulyandAugust(D)NovemberandDecember

26.Accordingtothegraph,whatcanbesaidaboutsedimentasagroundcovering?(A)Itremainsroughlythesamedepththroughouttheyear.(B)ItisthickestinFebruary.(C)ItisthinnestinAugust.(D)ItincreasesindepthfromAugusttoDecember.

27.WhatisthedepthininchesoficeinApril?(A)8(B)2(C)10(D)5

EyeColorsinMs.Smith’sClass

28.WhichofthefollowingstatementsisNOTsupportedbyinformationinthegraph?(A)Ms.Smithhasalargeclass.(B)One-quarterofMs.Smith’sclasshasblueeyes.(C)Twiceasmanystudentshavegreeneyesashavehazeleyes.(D)Mostofthestudentshavebrowneyes.

29.Usingthegraph,whatistrueabouttherelationshipbetweenthenumberofstudentswithgreeneyesandthenumberofstudentswithbrowneyes?(A)Thenumberofstudentswithgreeneyesequalsone-sixthofthenumberofstudentswithbrown

eyes.(B)Greeneyesarearecessivetraitandmoreuncommonthanbrowneyes.(C)Hazeleyesareavariantofgreenandshouldbeincludedinthegreeneyestotal.(D)Forevery10studentswithgreeneyestherearesixstudentswithbrowneyes.

PopulationStructure

30.Whichcountryhasthehighestpercentageofpeopleage0–14?(A)India(B)Moldova(C)Armenia(D)Zimbabwe

31.InArmenia,peopleage15–24makeupapproximatelywhatpercentageofthetotalpopulation?(A)10%(B)15%(C)35%(D)75%

32.Whichofthefollowingistruebasedontheinformationinthegraph?(A)Norwayhasthegreatestpercentageofpeopleage65andolder.(B)Norwayhasthebesthealthcareforolderadults.(C)MostNorwegianschoosenottohavechildren.(D)Norwaywillsoonfeeltheeconomiceffectsofitslowbirthrate.

PracticeExercisesofIncreasingDifficulty4

PRACTICEEXERCISEAThesepassagesandquestionsarerepresentativeoftheeasierpassagesyouwillfindontheactualSAT.Usethesetopracticenewstrategiesandtobuildyourconfidence.Takeabout13minutesforeachpassage(includingreadingandansweringthequestions).Detailedanswerexplanationsfollow.

PassageA1

CharlesDickens’sGreatExpectationswasfirstpublishedin1861.Pip,apoororphanwhoiscaredforbyhissisterandherhusband,meetstheyounggirlwhowillbecomethelifetimeobjectofhisaffectionswhilesimultaneouslybecomingawareofhislowlypositioninthecastesystem.

ImusthavebeenabouttenyearsoldwhenIwenttoMissHavisham’s,andfirstmetEstella.

MyunclePumblechook,whokeptacornchandler’sshopinthehigh-streetoftheLine town,tookmetothelargeold,dismalhouse,whichhadallitswindowsbarred.For(5) milesroundeverybodyhadheardofMissHavishamasanimmenselyrichandgrim

ladywholedalifeofseclusion;andeverybodysoonknewthatMr.Pumblechookhadbeencommissionedtobringheraboy.

Heleftmeatthecourtyard,andayounglady,whowasveryprettyandseemedveryproud,letmein,andInoticedthatthepassageswerealldark,andthattherewasa

(10) candleburning.Myguide,whocalledme“boy,”butwasreallyaboutmyownage,wasasscornfulofmeasifshehadbeenone-and-twenty,andaqueen.SheledmetoMissHavisham’sroom,andthere,inanarmchair,withherelbowrestingonthetable,satthestrangestladyIhaveeverseen,orshalleversee.

Shewasdressedinrichmaterials—satinsandlaceandsilks—allofwhite—orrather,(15) whichhadbeenwhite,but,likeallelseintheroom,werenowfadedyellow.Hershoes

werewhite,andshehadalongwhiteveildependentfromherhair,andbridalflowersinherhair;butherhairwaswhite.Isawthatthebridewithinthebridaldresshadwitheredlikethedress.

“Whoisit?”saidtheladyatthetable.

(20) “Pip,ma’am.Mr.Pumblechook’sboy.”

“Comenearer;letmelookatyou;comeclose.Youarenotafraidofawomanwhohasneverseenthesunsinceyouwereborn?”

“No,ma’am.”

“DoyouknowwhatItouchhere?”shesaid,layingherhands,oneupontheother,(25) onherleftside.

“Yes,ma’am;yourheart.”

“Broken!”Shewassilentforalittlewhile,andthenadded,“Iamtired;Iwantdiversion.Play,play,play!”

Whatwasanunfortunateboytodo?Ididn’tknowhowtoplay.

(30) “CallEstella,”saidthelady.“CallEstella,atthedoor.”

Itwasadreadfulthingtobebawling“Estella”toascornfulyoungladyinamysteriouspassageinanunknownhouse,butIhadtodoit.AndEstellacame,andIheardhersay,inanswertoMissHavisham,“Playwiththisboy!Why,heisacommonlabouringboy!”

(35) IthoughtIoverheardMissHavishamanswer,“Well?Youcanbreakhisheart.”

Weplayedatbeggarmyneighbour,andbeforethegamewasoutEstellasaiddisdainfully,“HecallstheknavesJacks,thisboy!Andwhatcoarsehandshehas!Andwhatthickboots!”

Iwasverygladtogetaway.Mycoarsehandsandmycommonbootshadnever(40) troubledmebefore;buttheytroubledmenow,andIdeterminedtoaskJoewhyhehad

taughtmetocallthosepicturecardsJackswhichoughttobecalledknaves.

ForalongtimeIwentonceaweektothisstrange,gloomyhouse—itwascalledSatisHouse—andonceEstellatoldmeImightkissher.

AndthenMissHavishamdecidedIwastobeapprenticedtoJoe,andgavehim£25(45) forthepurpose;andIleftoffgoingtoseeher,andhelpedJoeintheforge.ButIdidn’t

likeJoe’strade,andIwasafflictedbythatmostmiserablething—tofeelashamedofhome.

Icouldn’tresistpayingMissHavishamavisit;and,notseeingEstella,stammeredthatIhopedshewaswell.

(50) “Abroad,”saidMissHavisham;“educatingforalady;faroutofreach;prettierthanever;admiredbyallwhoseeher.Doyoufeelthatyouhavelosther?”

Iwassparedthetroubleofansweringbybeingdismissed,andwenthomedissatisfiedanduncomfortable,thinkingmyselfcoarseandcommon,andwantingtobeagentleman.

1.Thepassagecanbestbesummarizedaswhichoneofthefollowingstatements?(A)Aboyhasinterestinginteractionsatanoldwoman’shouseandreflectsontheseexperiences.(B)Aboyseducesagirlintofallinginlovewithhimfortheyearstocome.(C)Awomanteachesayoungboyaboutthemeritsofapprenticeship.

(D)Agirltravelsabroadforhereducation,leavinghercompanionbehindtofendforhimself.

2.Thepassageisgenerallyorganized(A)frommosttoleastimportantdetails.(B)chronologically.(C)spatially.(D)throughasequenceofflashbacksandpresent-dayreflection.

3.Thesecondparagraph(lines3–7)servestoexplain(A)whyPipwantedtobeagentleman.(B)whyMissHavishamdesiredcompanionship.(C)howPipcametobeatMissHavisham’s.(D)howPipcametofallinlovewithEstella.

4.Asusedinline12,theword“resting”mostcloselymeans(A)suppressing.(B)dreaming.(C)sleeping.(D)laying.

5.WhatbestdescribesMissHavisham’sappearance?(A)Typical(B)Unusual(C)Colorful(D)Vivacious6.Whichoptiongivesthebestevidenceforthe

answertothepreviousquestion?(A)Lines3–4(“My...barred”)(B)Lines11–13(“She...see”)(C)Lines27–28(“Broken...

Play”)(D)Lines31–32(“Itwas...doit”)7.HowdoesPipfeelabouthiscurrentsocialandeconomiccircumstances?

(A)Dissatisfied(B)Content(C)Serene(D)Entertained8.Whichoptiongivesthebestevidencefortheanswertothepreviousquestion?

(A)Lines8–10(“Heleft...burning”)(B)Lines23–26(“No...heart”)(C)Line35(“Ithought...heart”)(D)Lines52–54(“Iwas...gentleman”)9.Theparagraphinlines39–41highlightsPip’sfeeling(A)asenseofbelonging.

(B)aneedtoshowoff.(C)outofplace.(D)readytoargue.

10.Asusedinline46,theword“afflicted”mostcloselymeans(A)diseased.(B)strengthened.(C)emboldened.(D)troubled.

PassageA2

PASSAGE1

ThefirstisaspeechgivenbySojournerTruthin1851attheWomen’sConventioninAkron,Ohio.(Asahistoricaltext,thisusesantiquatedlanguage.)ThesecondispartofCarrieChapmanCatt’s“AddresstotheCongressonWomen’sSuffrage”in1917.

Ain’tIaWoman?

“Well,children,wherethereissomuchrackettheremustbesomethingoutofkilter.Ithinkthat‘twixtthenegroesoftheSouthandthewomenoftheNorth,alltalkingaboutrights,thewhitemenwillbeinafixprettysoon.Butwhat’sallthisheretalkingabout?

Line(5) Thatmanovertheresaysthatwomenneedtobehelpedintocarriagesandlifted

overditches,andtohavethebestplaceeverywhere.Nobodyeverhelpsmeintocarriages,orovermud-puddles,orgivesmeanybestplace!Andain’tIawoman?Lookatme!Lookatmyarm!Icouldhaveploughedandplanted,andgatheredintobarns,andnomancouldheadme!Andain’tIawoman?Icouldworkasmuchandeatasmuch

(10) asaman—whenIcouldgetit—andbearthelashaswell!Andain’tIawoman?Ihavebornethirteenchildren,andseenthemmostallsoldofftoslavery,andwhenIcriedoutwithmymother’sgrief,nonebutJesusheardme!Andain’tIawoman?

Thentheytalkaboutthisthinginthehead;what’sthistheycallit?[Intellect,somebodywhispers]That’sit,honey.What’sthatgottodowithwomen’srightsornegroes’

(15) rights?Ifmycupwon’tholdbutapint,andyoursholdsaquart,wouldn’tyoubemeannottoletmehavemylittlehalfmeasure-full?

Thenthatlittlemaninblackthere,hesayswomencan’thaveasmuchrightsasmen,‘causeChristwasn’tawoman!WheredidyourChristcomefrom?WheredidyourChristcomefrom?FromGodandawoman!ManhadnothingtodowithHim.

(20) IfthefirstwomanGodevermadewasstrongenoughtoturntheworldupsidedownallalone,thesewomentogetheroughttobeabletoturnitback,andgetitrightsideupagain!

Andnowtheyisaskingtodoit,themenbetterletthem.

Obligedtoyouforhearingme,andnowoldSojournerain’tgotnothingmoretosay.”

PASSAGE2

(25) Doyourealizethatinnoothercountryintheworldwithdemocratictendenciesis

suffragesocompletelydeniedasinaconsiderablenumberofourownstates?Therearethirteenblackstateswherenosuffrageforwomenexists,andfourteenotherswheresuffrageforwomenismorelimitedthaninmanyforeigncountries.

Doyourealizethatwhenyouaskwomentotaketheircausetostatereferendum(30) youcompelthemtodothis:thatyoudrivewomenofeducation,refinement,achievement,

tobegmenwhocannotreadfortheirpoliticalfreedom?

Doyourealizethatsuchanomaliesasacollegepresidentaskingherjanitortogiveheravoteareoverstrainingthepatienceanddrivingwomentodesperation?

Doyourealizethatwomeninincreasingnumbersindignantlyresentthelongdelay(35) intheirenfranchisement?

Yourpartyplatformshavepledgedwomensuffrage.Thenwhynotbehonest,frankfriendsofourcause,adoptitinrealityasyourown,makeitapartyprogram,and“fightwithus”?Asapartymeasure—ameasureofallparties—whynotputtheamendmentthroughCongressandthelegislatures?Weshallallbebetterfriends,weshallhavea

(40) happiernation,wewomenwillbefreetosupportloyallythepartyofourchoice,andweshallbefarprouderofourhistory.

“Thereisonethingmightierthankingsandarmies”—aye,thanCongressesandpoliticalparties—“thepowerofanideawhenitstimehascometomove.”Thetimeforwomansuffragehascome.Thewoman’shourhasstruck.Ifpartiesprefertopostpone

(45) actionlongerandthusdobattlewiththisidea,theychallengetheinevitable.Theideawillnotperish;thepartywhichopposesitmay.Everydelay,everytrick,everypoliticaldishonestyfromnowonwillantagonizethewomenofthelandmoreandmore,andwhenthepartyorpartieswhichhavesodelayedwomansuffragefinallyletitcome,theirsinceritywillbedoubtedandtheirappealtothenewvoterswillbemetwithsuspicion.

(50) Thisisthepsychologyofthesituation.Canyouaffordtherisk?Thinkitover.

1.Passage1asawholeemphasizeswhatqualitiesofwomen?(A)Theirstrengthandindependence(B)Theirsignificantdependenceonmen(C)Theirdesiretobe

homemakers(D)Theirwillingnesstofightinwars2.Asusedinline3,theword“fix”mostcloselymeans(A)repair.

(B)predicament.(C)securing.(D)settling.

3.Lines15–16(“Ifmy...full?”)mostdirectlysuggestthat(A)itwouldbewrongtodenyhertheopportunitytobeallshecanbe.(B)menhavenobusinessinvolvingthemselvesinculinaryactivities.(C)carefulmeasurementsareneededwheninvestigatingtheseissues.(D)barrierstofemaleadvancementhadbeenlargelyremoved.

4.HowdoesSojournerTruthprimarilyusereligiousteachingstomakeherargument?(A)Sheusesthemtodemonstratethestrengthoffemales.(B)Sheemphasizestheneedforwomentoquietlypray.

(C)Shecontradictsthemwithherpersonalexperiences.(D)Shebelievesthatshespeaksonbehalfofdivineforces.

5.Whichoptiongivesthebestevidencefortheanswertothepreviousquestion?(A)Lines6–7(“Nobody...woman”)(B)Lines13–14(“Then...honey”)(C)Lines20–22(“Ifthe

...again”)(D)Lines23–24(“And...say”)6.Asusedinline30,theword“compel”mostcloselymeans(A)suggest.

(B)force.(C)demonstrate.(D)list.

7.Passage2arguesthatwomen’ssuffrageshouldhappen(A)afteralongdelay.(B)whenallforeigncountrieshavedoneso.(C)afterpoliticalpartieshaveconsideredtheissuethoroughly.(D)immediately.

8.Whichoptiongivesthebestevidencefortheanswertothepreviousquestion?(A)Lines26–28(“There...countries”)(B)Lines34–35(“Do...enfranchisement”)(C)Lines36–

38(“Your...us”)(D)Lines42–44(“There...come”)9.Lines45–46(“Theidea...may”)canbestbeparaphrasedaswhichoneofthefollowingstatements?

(A)Womenwhodonothavetherighttovotearefinallystartingtocomearoundtotheidea.(B)Thosepoliticianswhoopposewomen’ssuffragewilllikelyfacemajorpoliticalconsequences.(C)AcompleteoverthrowoftheU.S.governmentisabouttooccur.(D)Policymakersneedtohaveabettergraspofthetruethreatstointernalsecurity.

10.Theauthorsofbothpassagesseeksocial(A)hierarchy.(B)repression.(C)equality.(D)isolation.

11.TheaimsofPassage1andPassage2are,respectively,(A)moregeneralandmorefocused.(B)morecasualandlessgoal-oriented.(C)lessopinionatedandmoredetailed.(D)lessintenseandmorehistorical.

PassageA3

Earthquakes

In2010,adevastatingearthquakeinHaitileftover100,000peopledeadandthousandsmoreinjured.Hundredsofthousandsofhomesandbusinessesweredestroyedinamatterofminutes.SeveralfactorscontributedtothelossoflifeandpropertyinHaiti,includingsocioeconomicsandthemagnitudeofthequake.

Line(5) Butwhatevencausesanearthquakeandwhatmakesitsosevere?Thinkaboutsitting

stillatyourdesk.Evenifyouaren’tactivelymoving,yourbodyisstilltechnicallyinmotion.Thisisduetoplatetectonicsandtheconstantshiftofthoseplatesonthesurfaceoftheearth.Whentensionbuildsupbetweentheseplates,theysometimesfractureandfault.Thisfracturingandfaultingiswhatcausesanearthquake.

(10) Specifically,therockshaveatremendousamountofpotentialenergybuiltupbetweenthemastheypressoneachother.Whentheserocksbreakor“fault,”itcausesaseismicwavethatmakesthe“groundshake”characteristicofanearthquake.Peoplecansometimesfeelearthquakes,butothertimesthey’resosmallthatonlyspecializedmachinesdetectthem.Thesemachinesarecalled“seismographs.”

(15) EarthquakesaremeasuredontheRichterscale,whichisnamedforitsinventor,CharlesFrancisRichter.TheRichterscalegoesfrom0to10plus,withlowernumbersrepresentingsmallerquakes,andhighernumbersrepresentinglargerquakes.Peoplerarelyfeelanearthquakewithamagnitudeof2orlower.Thesearethetypesthatneedtobepickedupbyaseismographortheywillgolargelyunnoticed.

(20) Anearthquakewithamagnitudeof3.0–3.9iscalleda“minorearthquake.”Thesecanusuallybefeltbymostpeople,butveryrarelycauseanydamage.Oftentimes,peopleendupattributingthesesmallquakestoothercauseslikelargetruckspassingorlargepublictransitvehicles.Earthquakesonthehigherendofthisrangemaycauseslightshakingofhouseholdobjects.

(25) Onceanearthquakereachestherangeof4.0–4.9,itisalmostalwaysfeltbythemajorityofpeople.These“lightearthquakes”willcausemarginallymoresevereshaking,andmayknockobjectsoffshelves.Thesequakesarestillhighlyunlikelytocauseanydamage,otherthantoknockedovertchotchkes.

Anearthquakewithamagnitudeof5.0orhigherwilldefinitelybefeltbypeople,(30) andatthislevel,therewillstarttobedamagetobuildings.“Moderateearthquakes,”

thosewithamagnitudeof5.0–5.9,rarelycausedamagetowell-constructedbuildings,butsometimesolderoneswillexperiencecrackedfoundations,electricaltrouble,andsinking.

Ina“strongearthquake,”aquakewithamagnitudeof6.0–6.9,damagewillbeseen

(35) inmostbuildingsunlesstheyarebuilttobe“earthquakeresistant.”Thedamageseeninthesebuildingswillbesimilartothosedonebymoderatequakesexceptitwillbemoresevereandhappentomorebuildings.

Typically,therewillnotbecompletecollapseofbuildingsuntilthelevelofa“majorearthquake.”Theseearthquakeswithamagnitudeof7.0–7.9canbefeltmanymiles

(40) fromtheepicenteroftheearthquakeandcancausethousandsofdollars’worthofdamage.

“Greatearthquakes”arethosewithaRichterlevelof8.0orhigher.Majordamagewilloccureventoearthquakeresistantbuildings.At9.0orhigher,therewillbepermanentchangestothegroundgeography.Therewillalsobeneartototaldestructionof

(45) mostbuildingsinthearea.

TheearthquakeinHaitihadamagnitudeof7.0.However,thedamagewasmoreseverethanonewouldexpectduetothetypesofbuildingscommoninHaiti.Tomakebuildings“earthquakeresistant,”asmentionedabove,ismuchmoreexpensivethanbuildingtypicalstructures.ThisisanunimaginableluxurytomanyHaitianswholive

(50) onlessthan$2aday.Additionally,thelackofinfrastructureledmanybuildingstobeinviolationof“codes”meanttokeeppeoplesafeintimesofdisaster.

DatesandLocationsofMajorEarthquakes

Year Location Magnitude

1138 Aleppo,Syria

8.5

1556 Shaanxi,China

8.0

1908 Messina,Italy

7.1

1927 Xining,China

7.9

1960 Valdivia,Chile

9.5

1976 Tangshan,China

8.2

2004 Sumatra,Indonesia

9.2

2008 Sichuan,China

8.0

2010 Port-au-Prince,Haiti

7.0

2011 Tohoku,Japan

9.0

2015 KathmanduandPokhara,Nepal

7.8

1.Whatisthepointofthepassage?(A)ToeducatethereaderaboutHaiti’seconomyandculture(B)Todiscussindetailthemajor

earthquakesthroughouthistory(C)Toinformaboutearthquakescienceandaspecificearthquake(D)Toreviewplatetectonicsandthescienceofseismographs2.Assumethatamoderateearthquakedamaged30%ofthebuildingsbuiltbefore1960inTownA.Iftheearthquakehadbeenastrongearthquakeinstead,thepassagemoststronglysuggeststhatwhatwouldbetheresultingdamage?

(A)Morethanhalfofthebuildingswouldfalldown.(B)Fiftypercentofthebuildingswouldbedamaged.(C)Thetownwouldexperiencenodamage.(D)Therewouldbepermanentchangestothegeography.

3.Accordingtothepassage,whatcanbedonetominimizeearthquakedamagetobuildings?(A)Newbuildingsshouldreplaceoldbuildings.(B)Buildingsshouldnotbebuiltonfaultlines.(C)Buildingscanbebuilttobeearthquakeresistant.(D)Earlyrepairsafterearthquakesareessential.

4.Whichoptiongivesthebestevidencefortheanswertothepreviousquestion?(A)Lines25–28(“Oncean...tchotchkes”)(B)Lines29–33(“Anearthquake...sinking”)(C)

Lines34–37(“Ina...buildings”)(D)Lines38–41(“Typically...damage”)5.Asusedinline24,“slight”mostnearlymeans(A)mild.

(B)quick.

(C)luminous.(D)irrelevant.

6.Accordingtothepassage,whydidthe2010earthquakeinHaitiresultinmoredamagethanwouldtypicallybeexpectedforearthquakesofsimilarmagnitude?(A)Earthquakesonislandstendtobemoreseverethanonthemainland.(B)ManybuildingsinHaitiwerenotbuilttocodestandardsorearthquakeresistant.(C)Haitiislocatedinanareathatissusceptibletomajorearthquakes.(D)TheseismographinHaitiwasfaultyandincorrectlymeasuredthemagnitude.

7.Whichoptiongivesthebestevidencefortheanswertothepreviousquestion?(A)Lines1–3(“In2010...minutes”)(B)Lines25–27(“Once...shelves”)(C)Lines42–44

(“Great...geography”)(D)Lines47–51(“Tomake...disaster”)8.Asusedinline42,“great”mostnearlymeans(A)numerous.

(B)high.(C)boundless.(D)big.

9.Accordingtothechart,thegreatestmagnitudeearthquakeoccurredinwhichcountry?(A)China(B)Chile(C)Japan(D)Haiti10.Usingtheinformationinthepassageandchart,which

countrieshaveexperiencedpermanentgeographicalchangesduetoearthquakes?(A)Chile,China,Italy(B)Indonesia,Japan,China(C)Haiti,Chile,Italy(D)Japan,Chile,Indonesia

11.TheRichterscaleisabase-10logarithmicscaleinwhicha3.0magnitudeearthquakehasashakingamplitude10timesgreaterthana2.0magnitudeearthquake,anda5.0magnitudeearthquakewouldhaveashakingamplitude1,000timesgreaterthana2.0magnitudeearthquake.Giventhisinformationandtheinformationinthechart,howmuchgreaterinshakingamplitudewasthe2011Japaneseearthquakethanthe2010Haitianearthquake?

(A)10times(B)100times(C)1,000times(D)10,000times

PassageA4

Buyer’sRemorse:theEuropeanUnionandtheGrexit

ErnestHemingwayonceboastedthathecouldtellastoryinsixwords:“Forsale:babyshoes,neverworn.”Nonetheless,I’mnotimpressed;askmetosummarizesomethingasvastastheglobalwholeofthe20thCentury,andIthinkIcandohimfourbetter:

Line Europefought.100years,eightPopes,andtwoworldwarsallboileddowntojust(5) thosewords.Europe.Fought.

Suchavolatileconnectedhistorymakesitallthemorefascinatingthattheentiretyofthecombativecontinentwasabletoredressitsrespectivegrievances,applythesalvetodecades-oldfesteringwounds,shuckoffferventnationalism,andjointogetherinmaritalblissasaveritableEuropeanUnion.

(10) Butthehoneymoon—ashoneymoonsarewonttodo—hasended.Theinitialendorphinrushofunitingtowardagreaterpurposehaslongpassed,andallofEuropenowfindsitselfinsomethingverymuchlikeinternationalrelationshipcounseling.“Hecan’tmanageourfinances,”Germanybemoansasthereluctantbreadwinner.“SherefusestohelpnowthatIneedhermost,”Greeceexclaims.“Listentoyoutwo!Youhavenoideaof

(15) thesordidsortofthingsthatwe’veseen!”therestofthecontinentmarvels,obligedtoplayarolesomewherebetweencharacterwitnessandneutralarbiterinthisgeopoliticallovers’quarrel.Yet,asthesayinggoes,“breakingupishardtodo.”Nowthatalllowhangingfruitromanticmetaphorshavebeenexhausted,atthatcrossroadsiswherewenowfindourselves.

(20) Tomorrow,Greecewillgobeforeitscreditorstolearnitsfate:eithertherestofEurope(read:GermanyandChancellorAngelaMerkel)willextenda£1.5billionloantotheGreekssothattheymightpayoffapreviousInternationalMonetaryFundfloat,orthisidyllicMediterraneanTitanofyorewillfinallymeetitsend,defaultingonitsdebtandhoppingthenexttraintowardtheghosttowncalledEuropeanBanishment.

(25) Suchanexit(dubbedGrexitbythemedia,intheireminentwit)mightwellbethefirstdominotofallinaseriesofdevelopmentsthatcoulddestabilizetheregionandthreatenthevalidityandvitalityoftheE.U.henceforth.

Consequentialpossibilitiesabound.Forone,shouldGreecereceivesaidfundingandbepermittedtoremain,atwhatpointdoesGermanytireofpayingchildsupport?

(30) ThenaturalconclusiontothatfatigueinBerlinwouldbeaharriedrummagingthroughtheatticinsearchofleftoverDeutschemarks,desperatetoreplacetheEuroandnostalgicfortheautonomyofyesteryearwhencurrencywastheirownandnotsomepervertedfiscaltragedyofthecommons.Moreover,theprecedentissetforfurtherdisqualificationwithaGrexit;perhapsSpain,Portugal,Ireland,orItalymightbethe

(35) nextoneleftwithoutachairwhenthemusicstops,resignedtotheirfateaswallflowersontheoutsidelookingin.

Yet,perhapsthemosttroublesomepossibilityisthatanisolatedGreecewouldbeanimpressionableGreece—desperatebothforalliesandaccesstotheircoffers.CurrentGreekoptimisticsentimentisthatRussiamightdonitsshiningarmorandrescuethe

(40) fledglingcastawayswithagodsendofaloan.But,givenPutin’srecentsleightofhandinCrimea,anysuchlendingmaynotbesomuchanactofcharityasaTrojanHorse;Vladimir’sKremlinfriendsareacraftybunch,andtheirendgameisopaque.Alas,suchistheproblemwithdecipheringulteriormotives:theyoftenaren’tclearuntilthehistorybooksgotoprint.Thetrilliondollarquestionis,whentheinkdries,willthe

(45) E.U.belistedinthechaptersofcurrentevents?Or,willitberelegatedtotheannalsofacademia,itsskeletonbutadiplomaticcasestudyofoilandwater,itsasheslittlemorethanaKennedySchoollectureontheperilsofcollaborationbetweensquarepegsandroundholes?

1.Whatistheoverallpointofthepassage?(A)ToexplainthecurrentchallengesoftheEuropeanUnionandponderitsfuture(B)Tomakethe

casethatGreeceshouldhaveneverbeenadmittedtotheEuropeanUnion(C)Toconvincethereaderthatacontinent-widecurrencyisarejectionofsoundeconomictheory(D)TodemonstratehowRussiaisoftenhelpfultoflounderingcountries2.Whatisthepurposeofparagraph1withrespecttothepassageasawhole?

(A)TogivethereaderabriefoverviewofEuropeanhistory(B)TodemonstratethattheauthorisasuperiorwritertoHemingway(C)Toshowthatevenlongstoriescanbeconciselysummarized(D)Todrawthereader’sinterestbyplacingthetopicofthepassageinaglobalandhistoricalcontext3.Whichoptiongivesthebestevidencefortheanswertothepreviousquestion?

(A)Lines6–9(“Such...Union”)(B)Lines10–12(“Butthe...counseling”)(C)Lines20–24(“Tomorrow...Banishment”)(D)Lines40–42(“But...opaque”)4.Inparagraph3(lines10–19),whatprimarypurposedoesthepersonificationofthecountriesserve?

(A)Togivethereaderfurtherinsightintothethoughtsofthecountries’leaders(B)TodemonstratehowthecountriesmightdiscussextendingaloantoGreece(C)Touseametaphortofurtherexplaintheconflictbetweenthecountries(D)Tofurtherpredicttheconsequencesofmilitaryconflict5.Asusedinline16,“arbiter”mostnearlymeans(A)pundit.

(B)legate.(C)helper.(D)referee.

6.Withwhichofthesestatementswouldtheauthormostlikelyagree?(A)TheEuropeanUnionisinnovativeandservesasasoundmodelforothercontinents.(B)WithouttheEuropeanUnion,Greecemaymakedesperatediplomaticdecisions.(C)GermanyandChancellorMerkelwanttoexittheEuropeanUnionandreturntoDeutschmarks.(D)IftheGrexitoccurs,GreecewillcertainlyexploitRussia’sfinancesandnaturalresources.

7.Whichoptiongivesthebestevidencefortheanswertothepreviousquestion?(A)Lines6–9(“Such...Union”)(B)Lines17–19(“Nowthat...ourselves”)(C)Lines30–33

(“Thenatural...commons”)(D)Lines37–38(“Yet...coffers”)8.Lines25–27(“Such...henceforth.”)primarilysuggestthattheauthorbelievesthataGreekexitfromtheEuropeanUnion(A)isallbutcertain,somustbeembraced.

(B)couldhaveunforeseenandnegativeconsequences.(C)willcausethedownfallofEuropeancivilization.

(D)shouldbeavoidedatallcosts.

9.Asusedinline33,“perverted”mostnearlymeans(A)deviant.(B)foreign.(C)unfortunate.(D)premeditated.

10.Whichofthefollowingwouldmostaccuratelyparaphraselines44–48(“Thetrillion...holes”)?(A)WillthecountriesoftheEuropeanUnionendtheirconflictspeacefullyorbyresortingtoan

expensivearmsrace?(B)WilltheEuropeanUnionbecomeobsoleteandonlyreadaboutintextbooksasalessoninthings

thataredysfunctional?(C)WilltheKennedySchoolgivefrequentlecturesabouttheEuropeanUnionanditssuccess?(D)WilltheEuropeanUnionextendaloantoGreecetoensurefuturesuccessandprosperityforall

countries?

PassageA5

TheValueofEngineering

Oneofthegreatestvocationsonecouldpursueisengineering.Engineersplayacentralroleintransformingourworldintoasafer,moreenjoyableplacetolive.Considerjustonearea,whichisveryclosetoeachofus:ourownhealth.Biomedicalengineers

Line havedesignedincubatorsthatsustainthelivesofprematurebabies,devicesand(5) procedurestodiagnoseconditionsandfightdiseases,andequipmenttorestorebodily

functionssuchasbeingabletowalkortosee.Agriculturalandbiologicalengineersdevelopmedicines,createbetterwaysofgrowingandprotectingourfoodsupply,and,alongwithenvironmentalengineers,helptocreateacleaner,saferenvironmentinwhichtobreathe,growfood,andhavesafedrinkingwater.Evenifyouhavenotmet

(10) suchanengineerbefore,theyhavesetanexampleofhowtobegoodstewardsofourresources,andwehaveallbenefittedfromwhattheyhavedoneforus.

Whatexactlyisengineering?Itisthepracticalapplicationofscientificknowledge.Therearemanydifferentareasofengineering.Dr.SeusshelpedtoopenourmindstoallthepossibilitiesofwhatwecandoinlifeinhisbookOh,ThePlacesYou’llGo!

(15) Engineersmakethisareality,bothliterallyandfiguratively.Intheareaoftransportation,engineersdesignvehiclestohelpmanypeopledailygetfromoneplacetoanother,aswellasairplanes,spaceshuttles,andsubmarinesthatenableustoexplorethefarreachesoftheplanetandbeyond.Morebroadly,therearefourmajorbranchesofengineering,includingchemical,civil,electrical,andmechanicalengineering,each

(20) ofwhichhasanumberofsubdisciplines.Thentherearedifferentinterdisciplinaryareasofengineering.Fromthesmallestscope,wherenanoengineersandbiomolecularengineersdesignatthelevelofatomsandmolecules,tothelargestscope,wherecivilengineersdesignbridgesandbuildings,theoptionsaremany.

Beinganengineerisintellectuallystimulatingandprovidescontinualchallenges.(25) Youwilllearnnotonlyhowthingswork,butalsowhydifferentmaterialssuchas

woods,metals,orplasticshavethepropertiesthattheydo,dependingontheiratomicstructureandhowtheywereformed.Youbecometrainedtoidentifywhatproblemsmayexistandhowtouseyourknowledge,skills,andcreativitytofigureoutinnovativesolutionstofixthem.OneinterestingchallengeoutlinedbytheNationalAcademy

(30) ofEngineeringistofigureouthowtocontrolafusionreactiontoprovideuswithenergyinamoreefficient,economic,andenvironmentallyfriendlyway.Orhowaboutdiscoveringhowtodiagnoseandtreatconditionspeoplehave,basedonindividualdifferences?Quicklyassessingsomeone’sgeneticprofileandhavingawaytodeliverpatient-specificmedicationstopreciselocationsusingnanoparticlesareimportant

(35) engineeringchallengesthatcouldimprovemanypeople’slives.

Itishardtoimaginewhatlifewouldbelikewithoutthecontributionsengineershavealreadymade.Theyhavenotjusthelpedustosurvive,behealthy,explore,and

movearoundbetter,buttheyhavealsomadelifemoreenjoyablethroughadvancesinareassuchascommunications,computing,andsports.Computerengineers,for

(40) example,havehelpeddevelopdevicesandsoftwarethatwecanusetomakeandsharedocumentsandhomevideos,listentoourfavoritemusic,andtalkwithco-workers,friends,andfamilymembersacrosstheglobe.Insports,differentengineershavemadesystemsanddevicesthatprovideuswithbetter,saferequipment,communicationsthatenableteamstointeractbetterandgamestobetelevised,andenvironmentsand

(45) infrastructurethatimprovetheplayingandwatchingofgames.

Itistruethateveryonehasuniquedesires,soyoumaypreferavocationinsomeotherarea.Butevenourdesiresthemselvescanbechangedandneedtobedeveloped,sowhynottryengineeringonasmallerlevelwithaprojectaroundthehouseorfollowanengineeraroundforawhile?Itmightjustbecomethekindofthingyouwouldenjoy

(50) doingevenifyouwerenotgettingpaid,andperhapsyoucouldbecomethenextOrvilleorWilburWright,comingupwithawholenewdesignthattakesustonewheights.Consideralsothefruitsofengineers’labors:iftheyhavebenefittedyousomuch,whynotdosomethingthatcouldbenefitothersinkind?Attheveryleast,weneedtobeawarethatwithoutthecontributionsofengineers,ourliveswouldbemoreimpoverished,

(55) sowehavealotforwhichtobethankful.

1.Whatistheoverallthemeofthepassage?(A)Toconvincethereadertopursueengineeringandabandonothercareergoals(B)Todemonstrate

thevalueofengineeringandhighlightthefield’scontributionstosociety(C)Togivein-depthexamplesofhowbiomedicalengineeringhasimprovedlifeforeveryone(D)Toprovidecontextforhowevensmalldo-it-yourselfhomeprojectsshouldbeapproachedwithanengineer’smindset2.Asusedinline10,“stewards”mostnearlymeans(A)attendants.

(B)curators.(C)assistants.(D)guardians.

3.Withwhichstatementwouldtheauthormostlikelyagree?(A)Everyonecanincorporateengineeringintohisorherlifeasahobbyorpersonalinterest.(B)Biomedicalengineeringisthemostusefulbranchofengineeringandmostworthyofacademic

pursuit.(C)Dayton,Ohio,ishometotheworld’sgreatestengineeringminds,includingOrvilleandWilbur

Wright.(D)Collegesshouldincreasetheirscholarshipsforstudentsstudyingengineering.

4.Whichoptiongivesthebestevidenceforthepreviousquestion?(A)Lines21–23(“Fromthe...many”)(B)Lines29–31(“Oneinteresting...way”)(C)Lines47–

50(“Buteven...paid”)(D)Lines50–53(“youcould...likekind”)5.WhydoestheauthorreferenceDr.Seussandhisbook“OhThePlacesYou’llGo!”inParagraph2(lines12–23)?

(A)Toreminiscencewiththereaderaboutabelovedchildhoodbook(B)Tosetupatransitiontodiscussthenumerousengineeringsubspecialties(C)Tosignaltothereaderthattheauthorisswitchingtodiscussionofanewtopic(D)Tousehyperbolewhileoutliningthevarioustypesofengineering6.Whatisthepurposeoflines21–23?

(A)Toconvincestudentsthatengineeringisapplicabletoanyfield(B)Toinformthereaderofthe

rangeofengineeringcareers(C)Tomakethecasethatengineeringisanintellectuallystimulatingcareer(D)Toreviewthefourmainbranchesofengineering7.Whichparagraphgivesthemostspecificevidenceinsupportoftheauthor’sstatementinline4thatengineeringhasmadetheworlda“moreenjoyableplacetolive?”

(A)Paragraph2(lines12–23)(B)Paragraph3(lines24–35)(C)Paragraph4(lines36–45)(D)Paragraph5(lines46–55)8.WhydoestheauthormentiontheNationalAcademyofEngineeringchallengeinlines29–31?

(A)Toinformthereaderaboutthedifficultyofcontrollingafusionreaction(B)Tolamentaboutthelackofclean,renewableenergysources(C)Toencouragethereadertoenterthecontest(D)Tociteanauthoritytoaddcredibilitytohiscase9.Whatisthepointofparagraph4(lines36–45)?

(A)Tomakethereaderfeelthatengineeringisanaccessiblecareer(B)Toshowhowlifewouldbedifferentwithoutengineers(C)Tohighlightthecontributionsofcomputerandsportengineers(D)Togiveexamplesofhowengineers’contributionsareusedindailylife10.Asusedinlines54–55,thephrase“moreimpoverished”mostnearlymeans(A)emptier.

(B)insolvent.(C)unproductive.(D)indigent.

PRACTICEEXERCISEBThesepassagesandquestionsaregenerally representativeofaverage-difficultypassagesandquestionsyoucouldfaceontheSAT.Takeabout13minutesforeachpassage(includingreadingandansweringthequestions).Detailedanswerexplanationsfollow.

PassageB1ThisisanexcerptfromCharlotteBronte’sJaneEyre,writtenin1847.Jane,previouslyagovernessatThornfield Hall, is engaged to marry the wealthy homeowner Mr. Rochester. She fretfully relaysunexpectedeventstohimthathaverecentlyoccurredinhisabsence.

“Idreamtanotherdream,sir:thatThornfieldHallwasadrearyruin,theretreatofbatsandowls.Ithoughtthatofallthestatelyfrontnothingremainedbutashell-likewall,veryhighandveryfragile-looking.Iwandered,onamoonlightnight,through

Line thegrass-grownenclosurewithin:hereIstumbledoveramarblehearth,andthere(5) overafallenfragmentofcornice.Wrappedupinashawl,Istillcarriedtheunknown

littlechild:Imightnotlayitdownanywhere,howevertiredweremyarms—howevermuchitsweightimpededmyprogress,Imustretainit.Iheardthegallopofahorseatadistanceontheroad;Iwassureitwasyou;andyouweredepartingformanyyearsandforadistantcountry.Iclimbedthethinwallwithfranticperiloushaste,eager

(10) tocatchoneglimpseofyoufromthetop:thestonesrolledfromundermyfeet,theivybranchesIgraspedgaveway,thechildclungroundmyneckinterror,andalmoststrangledme;atlastIgainedthesummit.Isawyoulikeaspeckonawhitetrack,lesseningeverymoment.TheblastblewsostrongIcouldnotstand.Isatdownonthenarrowledge;Ihushedthescaredinfantinmylap:youturnedanangleoftheroad:I

(15) bentforwardtotakealastlook;thewallcrumbled;Iwasshaken;thechildrolledfrommyknee,Ilostmybalance,fell,andwoke.”

“Now,Jane,thatisall.”

“Allthepreface,sir;thetaleisyettocome.Onwaking,agleamdazzledmyeyes;Ithought—Oh,itisdaylight!ButIwasmistaken;itwasonlycandlelight.Sophie,I

(20) supposed,hadcomein.Therewasalightinthedressing-table,andthedoorofthecloset,where,beforegoingtobed,Ihadhungmywedding-dressandveil,stoodopen;Iheardarustlingthere.Iasked,‘Sophie,whatareyoudoing?’Nooneanswered;butaformemergedfromthecloset;ittookthelight,helditaloft,andsurveyedthegarmentspendentfromtheportmanteau.‘Sophie!Sophie!’Iagaincried:andstillitwassilent.I

(25) hadrisenupinbed,Ibentforward:firstsurprise,thenbewilderment,cameoverme;andthenmybloodcreptcoldthroughmyveins.Mr.Rochester,thiswasnotSophie,itwasnotLeah,itwasnotMrs.Fairfax:itwasnot—no,Iwassureofit,andamstill—itwasnoteventhatstrangewoman,GracePoole.”

“Itmusthavebeenoneofthem,”interruptedmymaster.

(30) “No,sir,Isolemnlyassureyoutothecontrary.TheshapestandingbeforemehadnevercrossedmyeyeswithintheprecinctsofThornfieldHallbefore;theheight,thecontourwerenewtome.”

“Describeit,Jane.”

“Itseemed,sir,awoman,tallandlarge,withthickanddarkhairhanginglongdown(35) herback.Iknownotwhatdressshehadon:itwaswhiteandstraight;butwhether

gown,sheet,orshroud,Icannottell.”

“Didyouseeherface?”

“Notatfirst.Butpresentlyshetookmyveilfromitsplace;shehelditup,gazedatitlong,andthenshethrewitoverherownhead,andturnedtothemirror.Atthat

(40) momentIsawthereflectionofthevisageandfeaturesquitedistinctlyinthedarkoblongglass.”

“Andhowwerethey?”

“Fearfulandghastlytome—oh,sir,Ineversawafacelikeit!Itwasadiscolouredface—itwasasavageface.IwishIcouldforgettherolloftheredeyesandthefearful

(45) blackenedinflationofthelineaments!”

“Ghostsareusuallypale,Jane.”

“This,sir,waspurple:thelipswereswelledanddark;thebrowfurrowed:theblackeyebrowswidelyraisedoverthebloodshoteyes.ShallItellyouofwhatitremindedme?”

(50) “Youmay.”

“OfthefoulGermanspectre—theVampyre.”

“Ah!—whatdiditdo?”

“Sir,itremovedmyveilfromitsgaunthead,rentitintwoparts,andflingingbothonthefloor,trampledonthem.”

1.Whatoptionbestsummarizesthepassage?(A)Acharactersharestroublingportentsforthefuture.(B)Acharacterseekscounselforherterrifyingdreams.(C)Twocharactersdeterminehowtoovercomesupernaturalforces.(D)Twocharactersrecounttheirrespectivetravels.

2.Mr.Rochester’soverallattitudetowardsJaneisbestdescribedas(A)dismissiveimpatience.(B)skepticaldepression.(C)anxioustrepidation.(D)respectfulcuriosity.

3.Asusedinline7,theword“retain”mostcloselymeans(A)remember.(B)collect.(C)holdonto.(D)sacrifice.

4.Inherdream,howdidJanefirstperceivethepresenceofMr.Rochester?(A)Throughsight(B)Throughhearing(C)Throughsmell(D)Throughtouch5.Whichoptiongives

thebestevidencefortheanswertothepreviousquestion?(A)Lines7–9(“Iheard...country”)(B)Lines9–12(“Iclimbed...summit”)(C)Lines14–16(“I

bent...woke”)(D)Lines43–45(“Fearful...lineaments”)6.ThemostlikelypurposeofJane’sstatementinline18(“All...come”)isto(A)giveadescription.

(B)demonstrateherrespect.(C)describeacharacter.(D)provideatransition.

7.Asusedinline23,theword“form”mostcloselymeans(A)entity.(B)creation.(C)system.(D)clothing.

8.Lines26–28(“Mr....Poole”)mainlyserveto(A)vividlydescribeanapparition.(B)describeresidentsofThornfieldHall.(C)anticipateandaddressalikelyobjection.(D)recounttroublingmemoriesfromadream.

9.ThefrighteningintruderasdescribedbyJane(A)stronglyresemblesahouseholdservant.(B)isquiteuniqueinitsterribleattributes.(C)isweakandabouttosuffercollapse.(D)isratherlikelytotransformintoabat.

10.Whichoptiongivesthebestevidencefortheanswertothepreviousquestion?(A)Line3(“Veryhigh...looking”)(B)Lines22–23(“Iasked...closet”)(C)Lines43–44

(“Fearful...face”)(D)Line51(“Of...Vampyre”)

PassageB2

ChemistryofCooking

Wetendtothinkofcookingasanart,butmuchofitsbasisactuallycomesfromchemistry.Let’stake,forinstance,theexampleofcookingmeat.Whydowebotherwithcookingmeat?Forone,itkillsthebacteriathatcanliveinmeatandbeharmfultous.Butadditionally,itmakesthemeatmuchmoretender—easiertoeatandeasiertodigest.

Line(5) Typically,proteinisthesecond-highestcomponentofmeatbehindwater.Proteins

haveseverallevelsoforganization.Aprotein’sprimarystructureistheorderinwhichtheaminoacidsarejoinedbytheirpeptidebonds.Aprotein’ssecondarystructureismadeupoflocalinteractionsoftheprimarystructure.Secondarystructureincludesalphahelices,betasheets,turns,andloops.Tertiarystructureisformedwhenvarious

(10) secondarystructuresinteract,typicallyoverlongdistances.Finally,quaternarystructureistheinteractionofdifferentproteinsubunits.Proteinsfoldtightlyincomplexwaysthatareenergeticallyandstericallyfavorable.Sowhathappenstothiscomplexorganizationwhenmeatisheated?Theseinteractionsbecomeweaker.Proteinsdenature,meaningtheirinteractionsweakenandtheirquaternary,tertiary,andsecondary

(15) structuresbreakdown.Insteadoftightlyfoldedproteins,theybecomelooseandstretchedout.Thisdenaturationiswhatmakesmeatmoretender.However,continuingtocookmeatafterthisinitialdenaturationservesonlytoremovewater,makingthemeattougheranddrier.Inparticular,thedenaturationofcollagenmakesmeatmoretender.Collagenisthemostabundantproteininanimalconnectivetissue.Tougher

(20) cutsofmeattendtohavemoreconnectivetissue,andthusmorecollagen.

Butheatingproteinsisn’ttheonlywaytodenaturethem:theycanalsobedenaturedbyaddingcertaindenaturingsubstances.Manyofthesesubstances,likestrongacidsandbases,youwouldn’twanttoaddtoyourfood;however,onecommondenaturingagentissalt.Thisiswhyyoumaywanttobrineatoughercutofmeatin

(25) additiontocookingit.Brininginvolvessoakingsomethinginasolutionofsaltwater.Anotherbenefitofbriningisthatwhenthemeatabsorbsthesalt,thisdrawswaterintothemeattodilutethesalt.Thus,briningalsoservestokeepmeatmoist.Somechefswilladvisesearingtheoutsideofacutofmeatbeforecookingitthroughtolockinthemoisture.However,chemistrydoesn’tsupportthisapproach:steamisequallycapable

(30) ofescapingthroughasearedcrustasitisthroughnon-searedmeat.

Ifyou’veevercookedredmeat,youknowthatasitcooks,itturnsbrown.Redmeatisredbecauseofitshighmyoglobincontent.Myoglobinisanoxygen-storingproteinfoundinmusclecells.Itisassociatedwithanironatom.Beforethemeatiscooked,theironatomisinthe+2oxidationstate.Cookingitremovesanelectron,thuschangingittothe

(35) +3oxidationstate.Thistransformsthecolortobrown.Ontheotherhand,whitemeatdoesn’tturnbrownbecauseitdoesn’thavenearlyasmuchmyoglobintobeoxidized.

Werarelypausemid-recipetoconsiderthechemistryofcooking,butunderstandingthechemicalreactionsoccurringinourfoodwillhelpustobecomebettercooks.Isn’tthatsomefoodforthought?

MeanPercentYieldofMeatCookedUnderDifferentMethods

1.ItcanbereasonablyinferredfromthepassagethatwhichoftheseproteinstructuresisLEASTimpactedbyheating?(A)Primary(B)Secondary(C)Tertiary(D)Quaternary2.Whichoptiongivesthebestevidencefor

theanswertothepreviousquestion?(A)Lines7–9(“Aprotein’s...loops”)(B)Lines9–10(“Tertiary...distances”)(C)Lines10–12

(“Finally...favorable”)(D)Lines13–15(“Proteins...down”)3.Thesentenceinlines12–13(“Sowhat...heated”)servesto(A)defineaterm.

(B)explainaneffect.(C)provideatransition.(D)analyzeanobservation.

4.Asusedinline13,theword“weaker”mostcloselymeans(A)defeated.(B)loosened.(C)pathetic.(D)fatigued.

5.Accordingtothepassage,whichofthesecookingapproacheswouldhavethemostnegligibleeffectonthetendernessofmeat?(A)Cookingpastproteindenaturation(B)Briningitinsaltwater(C)Searingitbeforefurther

cooking(D)Usinganacidorbasetodenature6.Whichoptiongivesthebestevidencefortheanswertothepreviousquestion?

(A)Lines16–18(“However...drier”)(B)Lines22–24(“Many...salt”)(C)Lines26–27(“Another...salt”)(D)Lines29–30(“However...meat”)7.Asusedinline39,thephrase“somefoodforthought”mostcloselymeans(A)anappetizingsituation.

(B)anextstepforresearchers.(C)aplanforactionablechange.(D)somethingworthconsidering.

8.Thepassageexplicitlystatesthatthesubstancemostdirectlyresponsibleforthebrowningofmeatis(A)collagen.(B)myoglobin.(C)quaternarystructure.(D)zirconium.

9.Basedontheinformationinthegraphandonlines19–20(“Collagen...collagen”),whattypeofcookingmethodwouldmostlikelybemostappropriateforatoughcutofmeat?(A)Roasting,givenitsmoderateyield.(B)Braising,givenitspotentialtoeliminateagreaterproportionoftheundesirablepartsofthemeat.(C)Broiling,sincevirtuallyallofthemeattissuewouldbepreserved.(D)Allthreemethodswouldbeequivalentasfarastheirappropriateness.

10.Accordingtothegraph,broiledmeat’syieldisabouthowmuchgreaterthanbraisedmeat’syield?(A)Threetimes(B)Twotimes(C)Fiftypercentmore(D)Athirdmore

PassageB3Twopassagesfrom2015writtenaboutthePhilosophyofEducation

PASSAGE1

Teachershaveonetruejob:toinspire.EducatecomesfromtheLatinwordeducaremeaning“bringoutandleadforth;”therefore,itisthedutyoftheteachertoopenthemindsofstudentsandtoguidethemintonewplaces.Itisnotnearlyasimportant

Line todivulgeknowledgeasitistostimulateadesireandloveforthethingitself.Teaching,(5) whendoneproperly,allowsasecondchanceforstudentstocontemplatethose

valuesandbeliefsthathavebeenimpartedtothem.Inventuringtothesenewplaces,teachershavetheopportunitytounleashactiveandindependent-thinkinghumans.

Considertheaveragestudent,constantlytoldwhattobelieveandhowtoact—a(10) roboticimitatorofculturalvaluesandsocietalstandards.Yet,theteacherhasthe

powertomakeallthedifference.Theteacheristheopenerofthedormantmind,thebringeroftheepiphany,thesupplierofthemomentofchange.Aclassroomthatfailstochallengeastudent’sfundamentalprinciplesandcreateahungerforlifetimelearningfailsaltogether.Whetherteachingthehumanitiesorthesciences,theeducator

(15) istheonewholaysoutapuzzleandpilotsinstancesofprofound,sublimeenlightenment.Astudentluckyenoughtostudyunderatrueteacherisneversatisfied,butinsteadisrelentlesslylookingforanswers.

Itisthroughinspirationthatateacherteachesastudenttothink.Educatingisnotthebusinessofmemorizationbutthebusinessofinquiry.Thestudent-teacher

(20) dichotomyisoneofreciprocatedwondermentwitheachpartyundergoingcontinuousimprovement.Andaneffectiveinstructorismerelytheoneresponsibleforthatcuriosityinthemindsofhisorherstudents.ItwasRobertFrostwhosaid,“Iamnotateacher,butanawakener.”

PASSAGE2

IbecameateacherbecauseitwasthenoblestprofessionIcouldthinkof.Getthis:(25) Iimpartknowledge.Iamresponsibleforinstillingthefoundationinthenextgeneration

ofprofessionalsandleaders.Itallsoundsveryromantic,butthejobcomesdowntoanabilitytosimplifyandexplainthoseconceptsthatmakeupthecorecurriculum.IfIcannotcommunicateclearlywithmystudents,ifIcannotcometotheirlevelandmaketheinstructionrelevantandaccessible,thenIfail.

(30) InthewordsofRalphWaldoEmerson:“Themanwhocanmakehardthingseasyistheeducator.”Andthatismyjob.Myphilosophyreliesongivingyourchildrenthebestpossiblestart.Iamtheopenerofdoors.Ibelieveinprovidinganoptimallearningenvironmenttoattackthecoresubjectsrigorously,allowingeachstudenttowalkout

feelingconfidentandprepared,whetheritbeforgraduationtests,college-entrance(35) exams,orthejobmarket.Somyskillisjustthis:Ibridgethegapbetweenstudentand

textbook.

Asacompassionateandpassionateeducator,Iknowthecompetitionyourchildfaces.Iaminterestedinbreakingdownthebarriersthatintimidatestudentsandprovidingthepaththatwillgetthembettercollegeoffers,bettercareers,andmostsignificantly,

(40) abetterqualityoflife.Todothis,Iadoptaperspectiveofpeakpreparationatallcosts.Myclassroomisresponsibleforinstillingtheconceptsandskillsthatwillbetestedandevaluated,anditisaresponsibilitythatIdonottakelightly.Itisthesestandardsandbasics—whicheverystudentofminewillcometoknowandunderstand—thatoftendetermineastudent’sfuturesuccess.BecauseItakemypositionas

(45) educatorsoseriously,youcantrustmetohaveyourchildreadyforthenextstep.

ValuedSkillSets

Surveyof100randomlyselectedcollegeprofessorsand100randomlyselectedemployers.

1.Asusedinlines6–7,thephrase“venturingtothesenewplaces”mostcloselymeans(A)travelingtoanewlocation.(B)investinginanewbusiness.(C)introducingnewideas.(D)questioningone’spastchoices.

2.Basedonlines16–17(“Astudent...answers”),ateacherismostresponsibleforimpartingwhichvirtuetohisorherstudents?(A)Knowledge(B)Truth(C)Diligence(D)Curiosity3.Theauthorofpassage2wouldmostlikely

beinterestedinthesurveyresponses,asportrayedinthegraph,of(A)justthecollegeprofessors.

(B)justtheemployers.(C)thecollegeprofessorsandemployersequally.(D)neitherthecollegeprofessorsnortheemployers.

4.Whichoptiongivesthebestevidencefortheanswertothepreviousquestion?(A)Lines24–26(“Getthis...leaders”)(B)Lines32–35(“Ibelieve...market”)(C)Lines41–42

(“Myclassroom...lightly”)(D)Lines42–44(“Itis...success”)5.Asusedinline26,theword“romantic”mostcloselymeans(A)idealistic.

(B)passionate.(C)loving.(D)emotional.

6.Lines37–41moststronglyimplythattheauthorofPassage2placesapremiumon(A)physicalexertion.(B)intellectualcontemplation.(C)rigorousexpectations.(D)logicalreasoning.

7.Basedontheinformationinthegraph,whichskillsetsareconsideredtobemoreimportanttosuccessincollegethanintheworkforce?(A)Communication/interpersonalandfield/technical(B)Leadership/decisionmakingand

knowledge/technical(C)Field/technicalandleadership/decisionmaking(D)Creativity/generativeandknowledge/conceptual8.Therelationshipbetweenthetwopassagesisbestdescribedas(A)Passage1ismorereasonableandPassage2ismoreemotional.

(B)Passage1ismorehumbleandPassage2ismorearrogant.(C)Passage1ismoredreamyandPassage2ismorecontemplative.(D)Passage1ismoreloftyandPassage2ismorepractical.

9.TheintendedaudiencesofPassage1andPassage2,respectively,aremostlikely(A)globalandcosmopolitan.(B)broadandfocused.(C)specializedandscholarly.(D)narrowandwide.

10.HowwouldtheauthorofPassage1mostlikelyrespondtothestatementinlines42–44(“Itis...success”)?(A)Withdisgust(B)Withconcern(C)Withambivalence(D)Withappreciation11.Whichoption

givesthebestevidencefortheanswertothepreviousquestion?(A)Lines1–2(“Educate...forth”)(B)Lines6–8(“Inventuring...humans”)(C)Lines12–14(“A

classroom...altogether”)(D)Lines16–17(“Astudent...answers”)

PassageB4

IsIttheHeartortheBrain?

Isawyoufromacrosstheroom,andIknewimmediately.Mypulsebegantorace;Istartedtosweat;Icouldbarelybreathe.FromthefirstmomentthatIlaideyesonyou,Iwasconvincedthatyouweretheone.Onourfirstdate,thechemistrywasobvious.We

Line laughedandsmiledandheldhandsandtalkedforhours.Icouldn’tsleeporeatoreven(5) payattentionatwork.Ijusthadtobewithyou.

Withinone-fifthofasecond,yourphysicalappearanceandbodylanguagecausedanexcessivereleaseofdopamineinmybraincreatingfeelingsofexcitementandhappiness.Wemadeeyecontactfor8.2seconds;yourpheromoneswereindistinguishablefrommymother’s.Then,yourvoicetriggeredmybrainmechanismforgeneratinglong-term

(10) attachment;and,forthenextseveraldays,myneurotransmitterssentobsessivemessageaftermessage,accompanyingthoughtsofyouwithfeelingsofeuphoria.Oncevasopressinandoxytocinreachedmyreceptors,IknewthatIcouldneverbewithoutyou.

Althoughtheformerdescriptionof“fallinginlove”isfarmoreprevalent,itisthelatterthathasscientistsspeculatingthatloveismorebiologicalthancultural.Helen

(15) Fisher,AmericananthropologistandprofessoratRutgersUniversity,studiesromanticinterpersonalattractionandstatesthatlovebeginswithafocusononepersonthatresultsinobsessivethinking.Inheressay,“TheRealitiesofLoveatFirstSight,”Fisherexplainshowweevaluateanindividualwithinthreeminutesofmeetingthembasedontheirphysicalattributes,voice,andwords.First,wedecideiftheyareattractive,

(20) thenweevaluatetheirclothingandstature,beforemovingontothesoundandtoneoftheirvoice.Lastly,weconsiderwhattheyactuallysay.Sheconcludesthatwechoosepartnerswhosebiologicalchemicalscomplementourown.

Fisher’sresearchhasevengonefurthertoexplainthesciencebehindfallinginlove.Shetheorizesthathumanshavethreebrainsystemsforloving:lust,whichis

(25) associatedwithtestosterone;attraction,whichislinkedtodopamine;andattachment,whichisbroughtonbyincreasedlevelsofoxytocinandvasopressin.Whilethesethreesystemscansurelyoverlap,theyalsoexistseparatelyfromoneanotherwhich,forFisher,explainswhylovingmorethanonepersonatatimeisquitepossible.However,Fisherisn’tconvincedthatloveisentirelybiological.TheEconomistdescribesthe

(30) phenomenonlikethis:“achemicalstatewithgeneticrootsandenvironmentalinfluences.”

Yet,theconnectionbetweenloveandbrainchemicalsisfartoocertaintoignore.PsychologyTodayrendersloveathree-stepscientificprocess.First,areleaseofthehormonedopaminecauses“feelgood”emotions.Inthisstage,wefeelattractedtoan

(35) individualandassociatethemwithfeelingsofintensejoy.Next,ourneurotransmitters—norepinephrineandphenylethylamine—leadtofocusedattentionontheobject

ofourattractionandfeelingsofgiddinessthatmakeithardtosleepanddecreaseourappetite.Lastly,ourbrainrewardsystemisactivated,sendingchemicalmessagestovariouspartsofthebodythatelevatemoodandmakeanaddiction-likeurgetobewith

(40) thatperson.Biologically,thatistheextentoflove.

Comparinglovetoachemicaladdictionisactuallyaquitecommonmetaphor,withinpoetryandmusic,butitalsohappenswithinthesciences.DeepakChopra,physicianandpublicspeaker,postulatesthatlovestemsfromanimpulseandbehaveswithinourbrainverysimilarlytoadrugaddiction.StephanieOrtigueisaprofessor

(45) atSyracuseUniversityanddescribestheaddiction-likequalitiesasaresultoftwelvedifferentareasofthebrainworkingintandemtoreleaseeuphoria-inducingchemicalssuchasadrenaline,dopamine,oxytocin,andvasopressin.Oxytocin,knownas“thelovehormone,”iswhywefeelcalmandcuddlywithoursignificantothers.

Whenitcomesto“loveatfirstsight,”biologistsareassertingthatitactually(50) doeshappen—inabout11%ofloveencounters,theattachmentmechanismwithin

thebraincanbetriggeredwithinthefirstfewminutesofmeetingsomeone.TheHuffingtonPostcontendsthattheoptimalenvironmentforloveatfirstsighttakesplaceinencounterswithabout8secondsofcontinuouseyecontact;incaseswhereindividualsstaredintooneanother’seyesfor4.5secondsorless,thechancesforlove

(55) weredismal.Itwouldappearthatnewresearchhaslovedowntoascience.

WhenDidIFallinLove?

1.Whatistheoverallpointofthepassage?(A)Toargueinfavorofascientificbasisformatchmaking(B)Topresentthelatestfindingsonthe

scienceoflove(C)Topresenttwosidesonamajorphilosophicalquestion(D)Togiveevidenceinsupportofsocietalreform2.Thedifferentperspectivesrepresentedbythefirstandsecondparagraphsaregenerallydescribedaswhat,respectively?

(A)ethical,scientific(B)trivial,important(C)authentic,misguided(D)subjective,objective3.Asusedinline3,theword“chemistry”mostcloselymeans(A)attraction.

(B)emotion.(C)collegiality.(D)biology.

4.Theauthormakesthemostbroaduseofwhichofthefollowingtobuildhercase?

(A)Personalanecdote(B)Dismantlinglogicalfallacies(C)Rhetoricalquestioning(D)Referencetoauthoritativesources5.Itcanbereasonablyinferredfromthepassagethatwhichofthefollowingchemicalsismostimportanttoalong-lastinglovingrelationship?

(A)Oxytocin(B)Testosterone(C)Dopamine(D)Norepinephrine6.Whichoptiongivesthebestevidencefortheanswertothepreviousquestion?

(A)Lines6–10(“Within...attachment”)(B)Lines23–26(“Fisher’s...vasopressin”)(C)Lines35–38(“Next...appetite”)(D)Lines44–47(“Stephanie...vasopressin”)7.Asusedinline22,theword“complement”mostcloselymeans(A)praise.

(B)supplement.(C)reveal.(D)interactwith.

8.Lines41–42(“Comparing...sciences”)moststronglyserveto(A)expressthewideuseofacomparison.(B)highlightthecontributionsoftheartstothesciences.(C)underscoreaflawedassumption.(D)useliterarytechniquesforaddedstylisticvariety.

9.Basedontheinformationinthegraph,itismostlikelythatthesampleofpeopleresearchersreferredtoinlines49–51(“Whenit...someone”)wasa(A)groupofonlymen.(B)groupofonlywomen.(C)groupofbothmenandwomen.(D)groupofscholarsinthesubject.

10.Afterapproximatelyhowmanymonthsintotheircurrentrelationshipwouldthemajorityofmensurveyedinthegraphstatethattheywereinlovewiththeirpartner?(A)Firstsight(B)1month(C)3months(D)6months11.Theinformationinthegraphmostdirectly

contradictswhatevidencefromthepassage?(A)Fisher’spointofviewinlines13–22(B)Fisher’stheoriesinlines23–31(C)ThefindingspresentedinPsychologyTodayoutlinedinlines32–40(D)Theauthoritativeviewspresentedinlines41–48

PassageB5

SagesandFools

Theentiretyofhumaninteractionfitsunderjustthreeumbrellas.Inthefirst,bothpartiesemergefromtherubbleworsethanbefore;theperpetratorofamurdergetslifeinprisonandthevictim’sdayisruined,sotospeak.Inthesecond,everybodywins;

Line I’mterribleatcookingandyou’reabysmalathousework,soyoumakeuspizzawhileI(5) attackthefungusunderyourcouch.Andasforthethird,well,acynicwillsaythatthis

isbyfarthelargestumbrella—ifitwereapiechart,we’retalkingeverythingbuttheàlamode.Wecallthisinteractioncategorythezerosum,becauseanythinggainedmustcomeatthelossofanother.Putanotherway,yourtragedyismywindfall.

WelcometotheNewWallStreet.

(10) Pokerplayersarefondofsayingthatifyoulookaroundthetableandcan’tspotthefool,runawayasfastasyoucan;you’reit.Thesamegoesfortheworldoffinance,whichhasquicklybecomeasadversarialasaduelathighnoon.Suchisthenatureofequities,futures,bonds,andderivatives(tonameafew):anytransactionhasbothawinnerandaloser.Eitherthebuyerhaspurchasedaninstrumentthatisundervalued

(15) andwillbeworthmoretomorrow,orthesellerhasunloadedabloatedinstrumentthatwillfallbackclosertoits“true”marketvaluationinthenearfuture.Eitherway,dollars(bothunrealizedandactual)willflowfromonepockettoanother.

Thus,withinsuchacontentioussystem,itistotheadvantageofallparticipantstotradewiththefool.And,ifthefoolcan’tbereadilyfound,trustthatonewillbe

(20) enticed.LeadinguptotheCrashof2008,foolswereaplentyaswildspeculationwastheflavordujourandsoundfinancialtheorywasdisregarded.Nowherewasthisactofleapingbeforeyoulookasflagrantasinthemortgageindustry,whereloanswererepeatedlyandsystematicallyextendedtoAmericanswithlittletonochanceofrepayingtheinterest,letalonetheinitialcapital.Knownas“subprime,”high-risk

(25) borrowerswithpoorcreditscoreswereextendedmoneyatinterestratesfarabovepar.Thebanks’deludedthoughtprocessbehindthiswasthattheinterestratesweresohighthat,evenifaninordinatelylargepercentageoftheborrowersdefaulted,hugeprofitswerestillguaranteed.Besides,worsecomestoworst,thebankstillownsthehouse.Itwasafoolproofplan.

(30) Untilitwasn’t.Thehousingbubblehadfinallyburst.Itwasn’tjustthatafewpeopledefaulted;rather,foreclosureswereeverywhereyoulooked.And,evenworse,whenthebankscamebytocollectthekeys,thehouseswerenowworthonlyfractionsofwhatthebankslentforthem.Consequently,theylostbillions.

Here’sthekicker:whiletheAmericanfinancialsystemnearlycollapsed,somehad(35) seenthefoolcomingfrommilesaway,secretlycheeringwitheachmisstepbythesavings

andloanindustry.So,whiletheguardiansofthemoneyboxescontinuedspeculating

onwhattheythoughttobediamonds,asmallhandfulonthefringesknewthatitwasnothingbutcoal.Andtheyprofitedspectacularly.

Throughaconvoluted,abstruseinstrumentknownasa“creditdefaultswap,”these(40) sageswereabletomakefortunesoffofthemortgageimplosions,laughingalltheway

tothebank(wherenobodyelsemostcertainlywaslaughing).Itwasanadroitplaybythesages,recognizinganobscureopportunityandcapitalizingontheperfectstorm.But,suchwerethejustdesertsofeachparty,andsuchistheblackandwhitedualityofthesystem.Oneisdestinedtolosehisshirt,andanotherexpandshiswardrobe.

1.Whichchoicebestsummarizesthepassage?(A)Anexplanationofhowstructuralflawsinthefinancialsystemledtoaneareconomiccollapse

(B)Ananalysisofdishonestyinbankingandwhatmeasuresconsumersshouldhavetakentoprotectthemselveswhenthehousingbubbleburst(C)Apresentationofhowfoolishconsumersfallpreytopredatory“sage”investors(D)Anaccountofhowuniversalprofitabilitywouldhavepreventedthe2008stockmarketcrash2.Thenarratorindicatesthatbanksandlendersthoughtsubprimeloanswouldendupprofitablebecause(A)mostborrowerspaybacktheirloans.

(B)highinterestrateswouldoffsetdefaults.(C)housingpricesremainsteadythrougheconomiccycles.(D)“creditdefaultswap”wasasafetynet.

3.Whichchoiceprovidesthebestevidencefortheanswertothepreviousquestion?(A)Lines5–8(“Andas...windfall”)(B)Lines14–17(“Either...another”)(C)Lines26–29

(“Thebanks’...plan”)(D)Lines39–41(“Through...laughing”)4.Asusedinline15,theword“bloated”mostcloselymeans(A)overvalued.

(B)distended.(C)full.(D)waterlogged.

5.Accordingtotheparagraphinlines18–29,whatfactorscontributedtotheCrashof2008?(A)Greedybanksattemptedtoprofitoffunsuspectinghomeowners.(B)Thereweretoomanyspeculatorsandsoundfinancialtheorywasrejected.(C)“Zerosum”interactionsdonotworkineconomictheory.(D)StockswereundervaluedandWallStreetwasadversarial.

6.Inline30“Thehousingbubblehadfinallyburst”meanswhathappened?(A)Homeswereworthlessthanwhentheyweremortgaged.(B)Homesweredestroyedacrossthecountry.(C)Nonewhomeswerebeingbuilt.(D)Homepricesremainedsteadyforseveralyears.

7.Inline36,“guardiansofthemoneyboxes”mostnearlyrefersto(A)fringeinvestors.(B)theAmericanconsumer.(C)mainstreambanks.(D)theFederalgovernment.

8.Asusedinline39,theword“abstruse”mostcloselymeans(A)sinister.

(B)perplexing.(C)subrosa.(D)weighty.

9.Thenarratorwouldagreethatallofthefollowingareexamplesof“zerosum”EXCEPT(A)sellinganovervaluedstockandmakingaprofit.(B)using“creditdefaultswap”tocapitalizeondefaultedmortgages.(C)orderingapizzadinnerwhileanotherpersoncompleteshousework.(D)defaultingonaloananddischargingthebalanceinbankruptcycourt.

10.Whichchoiceprovidesthebestevidencefortheanswertothepreviousquestion?(A)Lines3–5(“Inthe...couch”)(B)Lines26–29(“Thebanks’...plan”)(C)Lines34–36

(“Here’s...industry”)(D)Lines39–41(“Through...laughing”)

PRACTICEEXERCISECThese passages and questions are generally representative of the toughest passages and questions youcouldfaceontheSAT.TheyaredesignedtohelpyoupushyourlimitsandtomaketheactualSATseemeasier.Takeabout13minutesforeachpassage(includingreadingandansweringthequestions).Detailedanswerexplanationsfollow.

PassageC1ThefollowingpassageisfromAPortraitoftheArtistasaYoungManbyJamesJoyce,1916.StephenDedalus, Joyce’sprotagonist, yearns tobeanartistbutwas raisedandeducated to join theclergy.Belowhecontemplatesthedivergingpathsbeforehimafterapriestwarnshimofthepermanenceofhisintendedholyposition.

Ashedescendedthestepstheimpressionwhicheffacedhistroubledself-communionwasthatofamirthlessmaskreflectingasunkendayfromthethresholdofthecollege.Theshadow,then,ofthelifeofthecollegepassedgravelyoverhisconsciousness.

Line Itwasagraveandorderedandpassionlesslifethatawaitedhim,alifewithoutmaterial(5) cares.Hewonderedhowhewouldpassthefirstnightinthenovitiateandwithwhat

dismayhewouldwakethefirstmorninginthedormitory.ThetroublingodourofthelongcorridorsofClongowescamebacktohimandheheardthediscreetmurmuroftheburninggasflames.Atoncefromeverypartofhisbeingunrestbegantoirradiate.Afeverishquickeningofhispulsesfollowed,andadinofmeaninglesswordsdrovehis

(10) reasonedthoughtshitherandthitherconfusedly.HislungsdilatedandsankasifhewereinhalingawarmmoistunsustainingairandhesmeltagainthemoistwarmairwhichhunginthebathinClongowesabovethesluggishturf-colouredwater.

Someinstinct,wakingatthesememories,strongerthaneducationorpiety,quickenedwithinhimateverynearapproachtothatlife,aninstinctsubtleandhostile,and

(15) armedhimagainstacquiescence.Thechillandorderoftheliferepelledhim.Hesawhimselfrisinginthecoldofthemorningandfilingdownwiththeotherstoearlymassandtryingvainlytostrugglewithhisprayersagainstthefaintingsicknessofhisstomach.Hesawhimselfsittingatdinnerwiththecommunityofacollege.What,then,hadbecomeofthatdeep-rootedshynessofhiswhichhadmadehimlothtoeatordrink

(20) underastrangeroof?Whathadcomeoftheprideofhisspiritwhichhadalwaysmadehimconceivehimselfasabeingapartineveryorder?

TheReverendStephenDedalus,S.J.

Hisnameinthatnewlifeleapedintocharactersbeforehiseyesandtoittherefollowedamentalsensationofanundefinedfaceorcolourofaface.Thecolourfaded

(25) andbecamestronglikeachangingglowofpallidbrickred.Wasittherawreddishglowhehadsooftenseenonwintrymorningsontheshavengillsofthepriests?Thefacewaseyelessandsour-favouredanddevout,shotwithpinktingesofsuffocatedanger.WasitnotamentalspectreofthefaceofoneoftheJesuitswhomsomeoftheboys

calledLanternJawsandothersFoxyCampbell?

(30) HewaspassingatthatmomentbeforetheJesuithouseinGardinerStreetandwonderedvaguelywhichwindowwouldbehisifheeverjoinedtheorder.Thenhewonderedatthevaguenessofhiswonder,attheremotenessofhisownsoulfromwhathehadhithertoimaginedhersanctuary,atthefrailholdwhichsomanyyearsoforderandobediencehadofhimwhenonceadefiniteandirrevocableactofhisthreatened

(35) toendforever,intimeandineternity,hisfreedom.Thevoiceofthedirectorurginguponhimtheproudclaimsofthechurchandthemysteryandpowerofthepriestlyofficerepeateditselfidlyinhismemory.Hissoulwasnottheretohearandgreetitandheknewnowthattheexhortationhehadlistenedtohadalreadyfallenintoanidleformaltale.Hewouldneverswingthethuriblebeforethetabernacleaspriest.Hisdestiny

(40) wastobeelusiveofsocialorreligiousorders.Thewisdomofthepriest’sappealdidnottouchhimtothequick.Hewasdestinedtolearnhisownwisdomapartfromothersortolearnthewisdomofothershimselfwanderingamongthesnaresoftheworld.

Thesnaresoftheworldwereitswaysofsin.Hewouldfall.Hehadnotyetfallenbuthewouldfallsilently,inaninstant.Nottofallwastoohard,toohard;andhefeltthe

(45) silentlapseofhissoul,asitwouldbeatsomeinstanttocome,falling,falling,butnotyetfallen,stillunfallen,butabouttofall.

1.Thedilemmafacedbythenarratorinthepassageasawholeisbestcharacterizedasachoicebetween(A)embracingalifeofpriestlyholinessoralifeofcriminaldepravity.(B)followinganorderlyprescribedpathoranintuitivelyauthenticone.(C)listeningtohissensorypassionsorthedemandsoflogicalconsistency.(D)attendingtomemoriesofreligiousritualsormemoriesofworldlydecadence.

2.Theoverallorganizationofthepassageisbestdescribedas(A)logicallyargumentative.(B)spatiallyprogressive.(C)graduallyconvergent.(D)strictlychronological.

3.Theprincipalmanifestationsofthenarrator’sanxietiesarewhatthoughtprocessesinthefirstandsecondparagraphsrespectively?(A)Consciousperceptionandunconscioussensation(B)Extroverteddiscussionandintroverted

analysis(C)Analyticalreasoningandintuitivereflection(D)Sensorymemoryandimaginativeprediction4.Howwouldthenarratormostlikelycharacterizehowmostpriestshandletheiremotions?

(A)Byrepressingthem(B)Byembracingthem(C)Byeliminatingthem(D)Byexpressingthem5.Whichoptiongivesthebestevidencefortheanswertothepreviousquestion?

(A)Lines8–10(“Atonce...confusedly”)(B)Lines18–20(“What...roof”)(C)Lines23–24(“Hisname...face”)(D)Lines26–27(“Theface...anger”)6.Asusedinline21,thephrase“apartineveryorder”mostnearlyexpressesthenarrator’sfeelingsof(A)acceptancebyhispeers.

(B)emotionalisolation.(C)apatheticindifference.(D)optimisticdecisiveness.

7.Thetypeofeducationthatbestfitsthenarratorismostlikely(A)religiouslyformal.(B)self-directed.(C)literary.(D)scholarlylecture.

8.Whichoptiongivesthebestevidencefortheanswertothepreviousquestion?(A)Lines13–15(“Some...acquiescence”)(B)Line18(“Hesaw...college”)(C)Lines28–29

(“Wasit...Campbell”)(D)Lines41–42(“Hewas...world”)9.Asusedinline41,thephrase“tothequick”mostcloselymeans(A)withspeed.

(B)verydeeply.(C)intelligently.(D)inisolation.

10.Thetoneofthefinalparagraphisoneof(A)resignation.(B)terror.(C)corruption.(D)decreasing.

PassageC2

Surfactants

Whilesignificantstructuralandfunctionaldifferencesexistbetweenthevariousclasses,asurfactant,simplyput,describesanycompoundcapableofreducingthesurfacetensionbetweenaliquidandoneothersubstance.Surfacetension,onewillrecall,

Line referstothetendencyofliquidmoleculestocoalescewithoneanother,thusminimizing(5) theircollectivesurfacearea.Thisphenomenonisthephysicalprincipleunderlying

afamiliaradage,“oilandwaterdonotmix.”Phrasedmoreprecisely,oils,whichcontainprimarilynonpolarhydrocarbonbonds,areimmisciblewithaqueoussolutions,meaningtheywillnotspontaneouslydissolveinwater,whichconsistsofhighlypolarhydrogen-oxygenbonds.Instead,oilswilltendtoformafilmoverpolarsolvents,while

(10) surfacetensionservestostabilizethisfilmattheoil-waterinterface.

Becausetheyareamphiphilic—meaningtheypossessbothpolarandnonpolardomains—surfactantsmayinteractwithbothcomponentsofthisinterface,andinterferewiththeelectrochemicalforcesthatmaintainitsintegrity.Duetothisuniqueproperty,surfacetensionloweringagentshavefoundahostofapplicationsindiverse

(15) commercialproducts,andareusedinparticularasemulsifiers,foamingagents,anddetergents.

Anemulsionismerelyamixtureoftwonormallyimmiscibleliquids.ThewordemulsionderivesfromaLatinrootmeaning“tomilk,”and,asaneasilyhomogenizedmixtureoffatswithinanaqueoussolutionofsugars,proteins,andminerals,milkitself

(20) isaquintessentialexampleofanemulsion.Moreover,withoutthesurfactantactivityofthecomplexlipidsitcontains,thefatglobulesdispersedthroughoutagivenvolumeofmilkwouldcoalesceintoafilmonitssurface.Similarly,thesurfactantsfoundinfoamingagentsdecreasethetendencyofsoapbubblestocoalesce,andareresponsibleforthelatheringeffectfoundinmanyhygieneproductssuchastoothpasteand

(25) shampoo.

Soapitself,interestinglyenough,canalsobeconsideredasurfactant.Principally,soapisasaltconsistingofapositivelychargedsodiumionandanegativelychargedfattyacid.Importantly,thestructureofanionizedfattyacidincludesanonpolarhydrocarbon“tail,”andapolar,carboxyl“head.”Thepolarheadallowsthefattyacid

(30) topartiallydissolveinwater,whilethenonpolartailfacilitatesitsinteractionwithothernonpolarcompounds,suchasoils.Thus,byinterruptingsurfacetension,soapallowsoiltobewashedawaywithwater.

Attheriskofoversimplifying,soapsarecreatedbyexposingtriglyceridesgatheredfromeitherplantoranimalsourcestoastrongbaseinaprocesscalledsaponification.

(35) Thebasehydrolyzestriglyceridestoformglycerolandamphipathicfreefattyacids.Theglycerol,inturn,isremoved,andthefattyacidsarecomplexedwithsodium.

Whilethewordssoapanddetergentaresometimesusedinterchangeablyincommonparlance,oneshouldnotethatdetergentsarenotsynthesizedbysaponification.

Structurally,detergentsdifferfromsoapsonlyinthecompositionoftheirpolar(40) heads.Thatis,whereassoapscontainanionizedcarboxylgroup,detergentscontain

anionizedsulfonate.Thesignificanceofthisalterationistwofold.First,detergentcompoundsarefarlesspronetoprecipitateandbecomeineffectiveinhardwater.Hardwater,ofcourse,referstowaterthatisrichindissolvedcalciumandmagnesiumasaresultofexposuretolimestone,anditispresentinanestimated80%ofAmerican

(45) households.Second,thesulfonatecomponentofdetergentsmakestheirdegradationproductsfarmoretoxictotheenvironmentthanthoseofsoaps.Owingbothtotheirlowcostofproductionandtotheirimpressiveutility,detergentsareproducedandsoldonascalethatdwarfsallothercommerciallysynthesizedsurfactants.Notsurprisingly,thishasbecomeacauseofgrowingconcernwithregardtothepotential

(50) impactsonaquaticecosystems,aswellasonhumanhealth,asexposuretodetergentderivativeshasbeenconvincinglyimplicatedinseveralendocrineandreproductivedisorders.

Thoughthiscontroversyisheated,complex,andunlikelytobesettledintheforeseeablefuture,ithasalsosparkedsignificantsupportforafascinatingfieldofbiotechnology

(55) thatdealswiththesurfactantsproducedendogenouslybylivingorganisms,andparticularlythoseproducedbymicrobes.Withregardtotheirstructure,theseso-called“biosurfactants”arehighlydistinctfrombothsoapsanddetergents,andyetseveralpromisingpreliminarystudieshaveshownthemtobefunctionallyviablealternativestomoreconventionalcleaningproducts.Theadvantageliesinthehigh

(60) biodegradabilityandbiologicallybenigncharacterofbiosurfactants.Theobstacle,ofcourse,liesinthenightmarishlogisticsofisolatingthemonalarge,industrialscale.

SurfaceTensionofSoapBubbles

Thegraphillustrateschangesinsurfacetensionovertimeinmilliseconds(ms)forsoapbubblesmadewithvaryingconcentrationsofsurfactant.Thetimelabelonthex-axisisexponentialandiscompressedasthevaluesincreased.

1.Theauthormostlikelyusesthephrase“oilandwaterdonotmix”inline6to(A)explaintheprocessinwhichtwoimmisciblesubstancesareemulsified.

(B)implythatmostlaypeoplecannotunderstandthetopicofthisarticle.(C)showthatsubstancesthatdonothavetriglyceridescannotundergosaponification.(D)connecttheesotericanalysistoacommonlyunderstoodphenomenon.

2.Asusedinline13,theword“integrity”mostcloselymeans(A)rectitude.(B)solidarity(C)cohesion.(D)decadence.

3.Accordingtothepassage,whatpropertyofsurfactantsismostresponsiblefortheirwidespreadhumanapplications?(A)Theyareproducedbymicrobes.(B)Theyareamphiphilic.(C)Theyareanemulsion.(D)Theirsaponification4.Whichoptiongivesthebestevidencefortheanswertotheprevious

question?(A)Lines13–16(“Dueto...detergents”)(B)Lines17–20(“Anemulsion...emulsion”)(C)Lines

33–35(“Atthe...acids”)(D)Lines53–56(“Though...microbes”)5.Itcanmostreasonablybeinferredfromthepassagethattherelativeamountsoftheseman-madesurfactantsarecurrentlywhat,fromleasttogreatest?

(A)Biosurfactants,detergents,soaps(B)Biosurfactants,soaps,detergents(C)Soaps,detergents,biosurfactants(D)Detergents,soaps,biosurfactants6.Theauthor’soveralldescriptionofsoapsanddetergentsisthattheyare(A)commonlythoughtofasinterchangeable,buthavingimportantdifferences.

(B)oneandthesameinsofarastheirchemicalproperties,suchasmolecularstructure.(C)differentwithrespecttotheircapacitytomixinemulsions.(D)majorobstaclestothewidespreadacceptanceofbiosurfactants.

7.Whichoptiongivesthebestevidencefortheanswertothepreviousquestion?(A)Lines16–20(“Theword...emulsion”)(B)Lines28–31(“Importantly...oils”)(C)Lines37–

38(“While...saponification”)(D)Lines56–59(“Withregard...products”)8.Asusedinline53,theword“settled”mostcloselymeans(A)firm.

(B)resolved.(C)disturbed.(D)mobilized.

9.Basedontheinformationinthegraph,ifsoapbubbles(liketheonesmeasuredinthegraph)withaconcentrationof8.5%surfactantweremeasured10msaftertheircreation,thesurfacetensioninmN/mwouldbeclosestto(A)30.(B)40.(C)50.(D)60.

10.Accordingtotheinformationinthegraph,anincreaseinsurfactantpercentagefromwhattowhatwouldmostlikelyresultinthelargestrelativeincreaseinsurfacetension?(A)From0.5%to1.5%(B)From4.5%to5.5%(C)From8%to9%

(D)Allofthesewouldresultinequivalentsurfacetensionincreases.

11.Whatisthemostlogicalreasonwhytheauthorusedalogarithmicscaleonthex-axisofthegraph?(A)Ithelpsgiveamoreaccuratecompilationofthedatathanwouldatypicallinearscale.(B)Itenablesreaderstomoreeasilyseethedirectlyproportionalrelationshipbetweenthevariables.(C)Itimpressesthereaderbecauseoftheauthor’sobviousmasteryofadvancedmathematical

reasoning.(D)Itmakesiteasiertovisualizethechangesinsurfacetensionovertheever-increasingordersof

magnitudeoftime.

PassageC3Below is the beginning excerpt fromMeditations on First Philosophy, by René Descartes, 1641, inwhichhemusesaboutthenatureofknowledge.

MEDITATIONI.OftheThingsofWhichWeMayNowDoubt

1.SEVERALyearshavenowelapsedsinceIfirstbecameawarethatIhadaccepted,evenfrommyyouth,manyfalseopinionsfortrue,andthatconsequentlywhatIafterwardbasedonsuchprincipleswashighlydoubtful;andfromthattimeIwasconvinced

Line ofthenecessityofundertakingonceinmylifetoridmyselfofalltheopinionsI(5) hadadopted,andofcommencinganewtheworkofbuildingfromthefoundation,ifI

desiredtoestablishafirmandabidingsuperstructureinthesciences.Butasthisenterpriseappearedtometobeoneofgreatmagnitude,IwaiteduntilIhadattainedanagesomatureastoleavemenohopethatatanystageoflifemoreadvancedIshouldbebetterabletoexecutemydesign.Onthisaccount,IhavedelayedsolongthatIshould

(10) henceforthconsiderIwasdoingwrongwereIstilltoconsumeindeliberationanyofthetimethatnowremainsforaction.Today,then,sinceIhaveopportunelyfreedmymindfromallcares[andamhappilydisturbedbynopassions],andsinceIaminthesecurepossessionofleisureinapeaceableretirement,Iwillatlengthapplymyselfearnestlyandfreelytothegeneraloverthrowofallmyformeropinions.

(15) 2.But,tothisend,itwillnotbenecessaryformetoshowthatthewholeofthesearefalse—apoint,perhaps,whichIshallneverreach;butasevennowmyreasonconvincesmethatIoughtnotthelesscarefullytowithholdbelieffromwhatisnotentirelycertainandindubitable,thanfromwhatismanifestlyfalse,itwillbesufficienttojustifytherejectionofthewholeifIshallfindineachsomegroundfordoubt.Norforthis

(20) purposewillitbenecessaryeventodealwitheachbeliefindividually,whichwouldbetrulyanendlesslabor;but,astheremovalfrombelowofthefoundationnecessarilyinvolvesthedownfallofthewholeedifice,Iwillatonceapproachthecriticismoftheprinciplesonwhichallmyformerbeliefsrested.

3.AllthatIhave,uptothismoment,acceptedaspossessedofthehighesttruthand(25) certainty,Ireceivedeitherfromorthroughthesenses.Iobserved,however,thatthese

sometimesmisledus;anditisthepartofprudencenottoplaceabsoluteconfidenceinthatbywhichwehaveevenoncebeendeceived.

4.Butitmaybesaid,perhaps,that,althoughthesensesoccasionallymisleadusrespectingminuteobjects,andsuchasaresofarremovedfromusastobebeyondthe

(30) reachofcloseobservation,thereareyetmanyotheroftheirinformations(presentations),ofthetruthofwhichitismanifestlyimpossibletodoubt;asforexample,thatIaminthisplace,seatedbythefire,clothedinawinterdressinggown,thatIholdinmyhandsthispieceofpaper,withotherintimationsofthesamenature.ButhowcouldIdenythatIpossessthesehandsandthisbody,andwithalescapebeingclassedwith

(35) personsinastateofinsanity,whosebrainsaresodisorderedandcloudedbydarkbiliousvaporsastocausethempertinaciouslytoassertthattheyaremonarchswhentheyareinthegreatestpoverty;orclothed[ingold]andpurplewhendestituteofanycovering;orthattheirheadismadeofclay,theirbodyofglass,orthattheyaregourds?Ishouldcertainlybenotlessinsanethanthey,wereItoregulatemyprocedureaccording

(40) toexamplessoextravagant.

1.Descartes’overallattitudetowardsknowledgeaspresentedinthepassageisbestdescribedas(A)dogmatic.(B)credulous.(C)skeptical.(D)popular.

2.Descartesuseslines6–11toexpresswhy(A)hebelievesthatthefoundationsforknowledgeareerrorridden.(B)theintellectualprojectheistacklingissoimportant.(C)hismentalandphysicalhealthhavebeguntodecline.(D)hehaschosenthispointintimetowritethiswork.

3.Asusedinline8,theword“advanced”mostcloselymeans(A)increasinglycomplex.(B)faralongintime.(C)withgreatskill.(D)significantlyimproved.

4.Descartes’minimalthresholdfordismissingaknowledgeclaimisifitis(A)completelyinerror.(B)moderatelywrong.(C)evenslightlyflawed.(D)anyclaimtoknowledge.

5.Whichoptiongivesthebestevidencefortheanswertothepreviousquestion?(A)Lines5–6(“ifI...sciences”)(B)Lines13–14(“Iwill...opinions”)(C)Lines18–19(“itwill

...doubt”)(D)Lines37–38(“orclothed...gourds”)6.Lines28–33primarilyillustrateDescartes’thinkingabout(A)thepitfallsofhumanperception.

(B)thesuperiorityoflogicalreasoning.(C)theimportanceofproperobservationaltools.(D)thespectrumofsensorycertainty.

7.ThepassagestronglyimpliesthatDescartesbelievesthatthestructureofknowledgeisbestdescribedas(A)hierarchical.(B)disconnected.(C)indubitable.(D)nonexistent.

8.Whichoptiongivesthebestevidencefortheanswertothepreviousquestion?(A)Lines5–6(“commencing...sciences”)(B)Lines9–11(“Onthis...action”)(C)Lines19–21

(“Norfor...labor”)(D)Lines32–33(“Ihold...nature”)9.Descartesusesthephraseinlines35–36,“sodisorderedandcloudedbydarkbiliousvapors,”toillustratewhathebelievestobe

(A)thereligioussourceofdemonicpossession.(B)thephysicalsourceofhallucinogenicvisions.(C)thepsychologicalsourceofchronicdepression.(D)theenvironmentalsourceofmathematicallogic.

10.Asusedinline40,theword“extravagant”mostcloselymeans(A)luxurious.(B)conservative.(C)psychological.(D)excessive.

PassageC4

AlternativeSplicing

WithJamesWatsonandFrancisCrick’slandmarkarticleonthedoublehelicalstructureofDNAnowmorethanahalf-centuryold,thesheervolumeofknowledgewehavesinceamassedregardingtheregulationandexpressionofgeneticmaterialis

Line staggering,andcontinuestoexpanddaily.Yetforallthathasbeenaccomplishedin(5) thestudyofgenetics,therecomesnowandagainadiscoverytounderscorejusthow

manymysterieswehaveyettounravel.

Historically,wehavedefinedageneasaregionofDNAresponsibleforencodingandregulatingtheexpressionofadiscrete,heritabletrait.Theuseofregulatinghereisofnosmallimportance,astheprotein-codingsequenceitselfrepresentsonlya

(10) fractionoftheDNAcontainedwithinagivengene.A“promoter”region,forinstance,doesnotdirectlycontributetothemRNAtranscript,butinsteadprovidesbindingsitesfortranscriptionfactorproteins,andfunctionsassortofan“on”or“off”switchfortheexpressionofthegene’scorrespondingtrait.Similarly,“silencer”and“enhancer”regionscanalsobindregulatoryproteins,andhelptofine-tunetheprecisedegreeto

(15) whichagenewillbeexpressedundervariouscircumstancesandinresponsetovaryingstimuli.However,perhapsthemostimplicitlyfascinatingnon-codingregionsofDNAarethoseembeddedwithintheprotein-codingregionitself.

Duringtranscription,nucleotidesarepolymerizedintoastrandofmRNAwhosesequenceiscomplementarytothatofthetemplateDNA.This“pre-mRNA”typically

(20) containsseveralregionsofnon-codingmaterial,or“introns,”thatmustbeexcisedpriortotranslationoftheprotein-codingregions,whicharereferredtoas“exons.”Inacomplexprocessknownassplicing,theintronsareremovedanddegraded,whiletheadjacentendsofexonsareadjoined,andtraffickedoutofthenucleustotheendoplasmicreticulum,whereproteinsynthesiscanatlastbegin.

(25) Predictably,mutationsthataffectagene’ssplicingpatternmayprecipitateseverefunctionalimpairmentstoitsencodedprotein,andsomestudieshaveestimatedthatasmanyashalfofalldisease-causingmutationsinhumans—includingthoseresponsibleforAlzheimer’sdisease,Parkinson’sdisease,andcertainformsofcysticfibrosis—areultimatelyaresultofalteredsplicing.Furthermore,anincreasingbodyofresearch

(30) hasalsoreportedpatternsofalteredsplicinginawidevarietyofcancercells,thoughitremainstobeseenastowhetherthesechangescontributetooncogenesis,oraresimplysymptomaticofdysregulatedgrowth.

Withsuchgrimpotentialforgeneticmisstep,onemightwonderhowevolutioncouldhaveeverfavoredthedevelopmentofsuchaprecariousandseeminglysuperfluous

(35) systemofgeneexpressioninthefirstplace.Theanswertothisliesinthefactthatalternativesplicingpatternsarenotexclusivelypathological,butcananddooccur

underphysiologicalcircumstancesaswell.Thatistosay,throughtightlycontrolledchangestothedifferentialremovalofintronsandretentionofexons,twoidenticalstrandsofpre-mRNAcan,ultimately,codefortwoentirelydifferentproteins.

(40) Calcitoningene-relatedpeptide,orCGRP,wasamongthefirstproteinsidentifiedasaproductofphysiologicalalternativesplicing.Whereascalcitoninisawell-knownhormoneproducedbythemedullarycellsofthethyroidgland,andisinvolvedintheregulationofcalciumlevelsintheblood,CGRPisbelievedtomediatepainsensationswithincentralandperipheralneurons.Despitetheiruniquestructuresandvastlydiffering

(45) functions,bothproteinsarenonethelessencodedbythesamegene.

Thediscoveryofphysiologicalalternativesplicingcameasachallengetoourtraditionalunderstandingofgenes,whichheldthateachcodingregionwasresponsiblefortheexpressionofasingleprotein.Today,ofcourse,weknowthislineofthoughttobeanelegantbuterroneousoversimplification.Scientistshavedemonstratedthat

(50) thevastmajorityofanimalgenesparticipateinalternativesplicingtooneextentoranother;farfromamerebiochemicalcuriosity,itisavitalbiologicalstrategytomaximizetheeconomyofgeneticmaterial,whichmustbelaboriouslyreproducedwitheachcelldivision,whilemaintaininganimmensediversityintheprotein-encodingcapacityofagenome.Inanextremeexample,thegenomeoftheinsectspecies

(55) Drosophilamelanogastercontainsabout15,000genes.Yet,throughalternativesplicing,onesingleDmelanogastergene—knownasDSCAM—hasbeenshowntoencodeabout38,000differentproteins.

Thetableillustratesthecorrespondingexonsforthepre-mRNAtransciptandninealternativesplicingisoformsofthetheα-tropomyosingene.

mRNASplicingIsoformsofα-tropomyosin

1.Whatistheoverallpurposeofthispassage?(A)Tomakeanargument(B)Tointroduceatopic(C)Topresentopposingviews(D)Togive

medicaladvice2.Thefirstparagraphoftheessay(lines1–6)primarilyservesto(A)payhomagetothegreatscientistswhomadethediscoverieshighlightedinthepassage.

(B)putthesubjectofthepassageinawidercontext.(C)illustratehowlittleisknownaboutaparticulartopic.(D)expresstheoptimismtheauthorhastowardscientificadvancement.

3.Asusedinline3,theword“amassed”mostcloselymeans(A)inquired.(B)fused.(C)accumulated.(D)weighed.

4.Asusedinline20,theword“excised”mostcloselymeans(A)removed.(B)translated.(C)coded.(D)mutated.

5.Lines29–32(“Furthermore...growth.”)moststronglysuggestthattheauthorofthepassagebelievesthatscientificthinkingregardingthecontributionofalteredsplicingtocanceris(A)malignant.(B)skeptical.(C)assured.(D)unsettled.

6.Basedonlines54–57,whichexpressiongivesthemostlikelyrangeofvaluesforthetotalnumberofproteinsinaDrosophilamelanogastergenome?(A)Numberoftotalproteins=15,000+38,000(B)Numberoftotalproteins=38,000–15,000(C)Numberoftotalproteins≤38,000×15,000(D)Numberoftotalproteins>38,000/15,000

7.Whichofthefollowingstatementsaccuratelydescribestherelationshipbetweenthepre-mRNAandalltheotherisoformsportrayedinthetable?(A)Thepre-mRNAissplicedintothedifferentisoformsinthesequenceprovidedfromtopto

bottom.(B)Theotherisoformsrepresentalternativesplicingpatternsderivedfromtheoriginalpre-mRNA.(C)Thepre-mRNArepresentsacombinationoftheotherisoforms’exonswhentheymutateduring

cellularreproduction.(D)Theotherisoformsusealternativesplicingtocreatethegivensequenceofpre-mRNA.

8.Whichoptiongivesthebestevidencefortheanswertothepreviousquestion?(A)Lines10–13(“A‘promoter’...trait”)(B)Lines13–16(“Similarly...stimuli”)(C)Lines18–

19(“During...DNA”)(D)Lines37–39(“Thatis...proteins”)9.Scientistswhothoughtalongthelinesoutlinedinlines49–54(“Scientists...genome”)wouldmostlikelyhavewhatopinionabouttheinformationinthegiventable?

(A)Supportive,becauseitdemonstrateshowalternativesplicingcanhelpmaximizetheeconomyofgeneticmaterial.

(B)Supportive,becauseitshowstheprocesswherebymRNAtransformsintoapotentialvarietyofisoforms.

(C)Unsupportive,becausethisdataillustratesthepotentialpitfalls,suchascancerousmutations,associatedwithalternativesplicing.

(D)Unsupportive,becauseitmentionsexonsbutignoresintronsandtheirdeletionduringthealternativesplicingprocess.

10.Basedonthepassage,whatwouldsomeonewhobelievedinthefirsthistoricaltheoryofDNAmostlikelythinkwouldbethenumberofuniqueproteintranscriptsthatwouldresultfromthereplicationoftheα-tropomyosingeneoutlinedinthetable?(A)1(B)3(C)9(D)12ormore11.Whichoptiongivesthebestevidencefortheanswertothepreviousquestion?(A)Lines1–4(“With...daily”)(B)Lines4–6(“Yetfor...unravel”)(C)Lines29–32

(“Furthermore...growth”)(D)Lines46–48(“Thediscovery...protein”)

PassageC5AnEnglishprofessorandaMasterofFineArtscandidatesharetheirthoughtsonliterarystandards.

PASSAGE1

Thereisandmustremainastandardbywhichgoodwritingismeasuredandacknowledged.Takeamomenttoconsiderthealternative,andyou’llsurelycometoagreewithme.Withoutstandard,anythingandeverythingcouldbeconsidered“literature.”

Line Moreso,itwouldchangefrompersontopersonandplacetoplacebasedsolely(5) ontherudimentarypreferenceofvariedindividuals.Withoutaclearideaofwhatis

meantby“literary,”allwritingisachaoticmessofopinionandidiosyncraticinterestwithgenreromancebeingjustasviableasthoserareworksofartthatembodythehumanexperience,raisesignificantsocialandpoliticalquestions,andremaininthereaders’mindslongafterthebookisfinished.Withoutmeasure,theTwilightseriessits

(10) rightnexttotheworksofToniMorrison.

Letusthinkaboutwhatmakesimportantwriting.Writing,likeallgreatformsofart,hasthepowertomakeusseetheworldmoreclearly.Itis,whendoneeffectively,acarrierofhistoryandtruth,ascriptofhumanitythatcanbefelt.Itislasting,orasEzraPoundonceremarked,“newsthatstaysnews.”Anditmovesus.WhatImean

(15) isthatliteratureplayswiththebigquestions,searchesforthebigthings.Itpursuesbeauty,purpose,andmeaninginanaestheticwaythatarousesemotion.Thecanonisacknowledgedassuperiorandofartisticmeritnotbecauseeveryonelikesreadingthestuff,butbecauseitheightensourunderstandingoflifeandrattlesourcomfortlevels.

Inessence,readingliteraturemakesusbetterhumans,andcertainly,notallwriting(20) candothat.Tosaythatwritingcannotbemeasuredorthatthereisnotaclearstandard

isabsurd.Whileyoumaynotlikeeverythingdeemed“literary,”itsurelyhasthepowertomakeyouthinkandfeelandwonder.Itisthisexplorationofuniversaltruthsthatintensifiesourunderstandingofhumanityandstirssomethingdeepwithinusthatmakesliterature.WhileyoumaylaughorcryorshoutwhilereadingHarryPotter,

(25) thatinitselfcannotclassifyitasoneofthegreats.

PASSAGE2

IusedtorevelatmyanxietyafterturninginanassignmentinmyfirstyearsofmyCreativeWritingdegree.Onemoment,Iwasquitesurethatmyworkwasgenius.Andanother,Iwasthemostdim-wittedsimpletontoeverputpentopaper.Ihadabsolutelynoideawhethermyfictionwouldcomebackwithan“A”oran“F”stampedon

(30) it—nocluehowtheprofessormightdecidebetweenthetwo.Often,I’dpulldecentgrades,butmoanaloudwhentheinstructorpickedoutmyveryfavoritesentence—theonethatwasgoingtomarkmethenextVonnegutorKerouac—andcrosseditoutinredink.Rethinkthisshe’dscribbleunderneath.Ittookmetwoyearsandtheonsetofcarpaltunneltorealizethatthereisnorealwaytoknowwhat’sgood,andthatwhat’s

(35) goodisentirelysubjective.

Totestmytheory,Isubmittedafewpoemsfromprevioussemesters(highlyfrowneduponbytheuniversity,butnecessaryformyexperiment)inhopesofgettingasecondopinion.Ifoundthatmygradesvariedonlyimperceptibly,butmoreinterestingly,instructorfeedbackborderedonpolarity.Andso,Irealizedthereisnotrue

(40) standardofmeasurementforwriting,notcreativewritingatleast.Onceyouventurepastthe“thesisstatement”and“logicalreasoning”and“coherentorganization”ofthepurelyacademicwriting,theconcessiononwhatisgoodisreallynonexistent.

Sure,wemightbeabletoagreethatsomethingparticularlyterribleisjustthat,andwemightbeabletonodourheadstoapiecethatisparticularlybrilliantandsay

(45) that,attheveryleast,itisn’tterrible.Butoverall,manywilladorelanguagethatothersdetest,andsomewillgaspappreciativelyatametaphorthatmakesthemassesvomit.Ifindenchantingwhatyoufinddull,andsoitgoes.AndMaryWright(fallsemester)willfindthesameimage“ineffective,”whichTobiasDalton(springsemester)calls“delightfulandprovocative.”AndsoIsay,toeachtheirown.Whatisthoughtful,good,

(50) andstirringiswithoutimpartiality,contingentnotonlyonthereader,butthereader’smood,location,andevenonwhatthereaderhasrecentlyread.Therefore,writewhatyouwillandreadwhatyouwish,andifyoulikeit,thendeclarewithauthoritythatitisindeedexceptional.

1.HowwouldtheauthorofPassage1mostlikelyrespondtosomeonewhocontendedthatthequalityofliteratureisdirectlyrelatedtotheintensityofthereader’semotionalresponse?(A)Agreewithitwholeheartedly.(B)Arguethatamoreprecisestandardisneeded.(C)Dismissthestatementoutright.(D)Arguethatemotionsareirrelevanttoliteraryanalysis.

2.Whichoptiongivesthebestevidencefortheanswertothepreviousquestion?(A)Lines4–5(“Moreso...individuals”)(B)Lines11–12(“Letus...clearly”)(C)Lines20–21

(“Tosay...absurd”)(D)Lines24–25(“While...greats”)3.Towhatideadoestheword“alternative”(line2)mostlikelyrefer?

(A)Aestheticbeauty(B)Artisticinterpretation(C)Literaryrelativism(D)Linguisticimpartiality4.Asusedinline17,theword“merit”mostcloselymeans(A)quality.

(B)literature.(C)artwork.(D)honesty.

5.Whichofthefollowing,iftrue,wouldpresentthegreatestchallengetotheargumentofPassage2?(A)SomepeopleadoreShakespeare,whileothersdonotcareforhiswork.(B)Well-trainedliterarymindsareabletousemoresophisticatedlanguagetogivetheirviewsonthe

qualityofdifferenttexts.(C)Americanbookreadershiphassteadilydeclinedinthepastthreedecades.(D)Thewriter’sacademicevaluatorsgradedinahurried,haphazardway.

6.Lines40–42(“Once...nonexistent.”)servetoacknowledgethattheauthorofPassage2believesthat

(A)writingforscholarlyjournalsislikeotherliteraryformsintherandomnessofitsquality.(B)thereareatleastsomewritingqualitiesthatincertaincontextsaremoreobjective.(C)thosewhohaveconfidenceinscientificobjectivityneedonlyreviewliteraturetoseetheerrorof

theirways.(D)therearethreekeycomponentstoahigh-qualitypieceofwriting.

7.TheauthorofPassage2wouldcontendthatshewouldbemorelikelytoreceiveabettergradefromacertaininstructorifheorshe(A)happenedtobeinapleasantmoodandnicesettingwhilegrading.(B)hadrigoroustrainingbyreviewingpaststudentessaysonthetopic.(C)receivedpositivefeedbackonthefairnessofhisorhergradingfrompaststudents.(D)waswell-versedinthedifferenttypesofrubricsthatcouldbeusedforevaluation.

8.Whichoptiongivesthebestevidencefortheanswertothepreviousquestion?(A)Lines27–28(“One...paper”)(B)Lines30–33(“Often...underneath”)(C)Lines43–45

(“Sure...terrible”)(D)Lines49–51(“What...read”)9.Asusedinline50,theword“contingent”mostcloselymeans(A)grouped.

(B)dependent.(C)random.(D)disappointed.

10.Theauthorsofbothpassageswouldmostlikelyagreewithwhichofthefollowingstatements?(A)Therootofliteraryqualityiswhetheritcanexpresstimelesstruths.(B)Weshouldagreetodisagreeonwhetherliteraryqualityisafactoropinion.(C)Wecancertainlyagreethattherearesomeliteraryworksthatarehorrible.(D)Interestinhigh-qualityliteraturehasmadesignificantprogressinrecentyears.

11.Whichoptionbestexpressestheoverallrelationshipbetweenthetwopassages?(A)Passage1arguesfortheexistenceofliteraryobjectivity,whilePassage2arguesforthe

opposite.(B)Passage1assertstheprimacyofreadingliterature,whilePassage2assertsthatwritingisthe

onlygatewaytounderstanding.(C)Passage1contendsthatgoodliteraturemakesreadersuncomfortable,whilePassage2contends

thatgoodliteratureiswhatismostpopular.(D)Passage1focusesonthe“greatbooks,”whereasPassage2focusesonexcellenceinpoetic

expression.

ReadingPracticeTests

PracticeTest1

65MINUTES,52QUESTIONS

Directions:Eachpassageorpairofpassagesisaccompaniedbyseveralquestions.Afterreadingthepassage(s),choose thebestanswer toeachquestionbasedonwhat is indicatedexplicitlyorimplicitlyinthepassage(s)orintheassociatedgraphics.

Questions1–10arebasedonthefollowingreadingselection.

BelowisthebeginningofHarrietBeecherStowe’s1852novel,UncleTom’sCabin.Herownremarkonthechapterisasfollows:“inwhichthereaderisintroducedtoamanofhumanity.”

LateintheafternoonofachillydayinFebruarytwogentlemenweresittingovertheirwine,inawell-furnishedparlourinthe

Line townofP----inKentuckyinthemidstofan(5) earnestconversation.

“ThatisthewayIshouldarrangethematter,”saidMr.Shelby,theowneroftheplace.“Thefactis,Tomisanuncommonfellow;heiscertainlyworththatsumanywhere;steady,

(10) honest,capable,managesmyfarmlikeaclock.Yououghttolethimcoverthewholeofthedebt;andyouwould,Haley,ifyou’dgotanyconscience.”“Well,I’vegotjustasmuchconscience

(15) asanymaninbusinesscanaffordtokeep,”saidHaley,“andI’mwillingtodoanythingto‘bligefriends;butthisyer,yesee,istoohardonafeller,itreallyis.Haven’tyouaboyorgalyoucouldthrowninwithTom?”

(20) “Hum!—nonethatIcouldwellspare;totellthetruth,it’sonlyhardnecessitymakesmesellatall.”Herethedooropened,andasmallquadroonboy,remarkablybeautifulandengaging,enteredwithacomicair

(25) ofassurancewhichshowedhewasusedtobeingpettedandnoticedbyhismaster.“Hulloa,JimCrow,”saidMr.Shelby,snappingabunchofraisinstowardshim,“pickthatup,now!”Thechildscampered,withallhis

(30) littlestrengthaftertheprize,whilehismasterlaughed.“Tellyouwhat,”saidHaley,“flinginthatchap,andI’llsettlethebusiness,Iwill.”Atthismomentayoungwoman,obviously

thechild’smother,cameinsearchofhim,(35) andHaley,assoonasshehadcarriedhim

away,turnedtoMr.Shelbyinadmiration.“ByJupiter!”saidthetrader,“there’san

articlenow!YoumightmakeyourfortuneonthatonegalinOrleans,anyway.WhatshallI

(40) sayforher?What’llyoutake?”“Mr.Haley,sheisnottobesold.Isayno,

andImeanno,”saidMr.Shelby,decidedly.“Well,you’llletmehavetheboy,though.”“Iwouldrathernotsellhim,”saidMr.

(45) Shelby;“thefactis,I’mahumaneman,andIhatetotaketheboyfromhismother,sir.”“Oh,youdo?La,yes,Iunderstandperfectly.

Itismightyunpleasantgettingonwithwomensometimes.Ial’ayshatesthese

(50) yerscreechin’times.AsImanagesbusiness,Igenerallyavoids‘em,sir.Now,whatifyougetthegaloffforadayorso?thenthething’sdonequietly.It’salwaysbesttodothehumanething,sir;that’sbeenmyexperience.”

(55) “I’dliketohavebeenabletokickthefellowdownthesteps,”saidMr.Shelbytohimself,whenthetraderhadbowedhimselfout.“AndEliza’schild,too!IknowIshallhavesomefusswiththewifeaboutthat,andfor

(60) thatmatter,aboutTom,too!Somuchforbeingindebt,heigho!”Theprayer-meetingatUncleTom’sCabin

hadbeenprotractedtoaverylatehour,andTomandhisworthyhelpmeetwerenotyet

(65) asleep,whenbetweentwelveandonetherewasalighttaponthewindowpane.“GoodLord!what’sthat?”saidAuntChloe,

startingup.“Mysakesalive,ifitaintLizzy!Getonyourclothes,oldman,quick.I’m

(70) gwinetoopenthedoor.”Andsuitingtheactiontotheword,thedoorflewopen,andthelightofthecandlewhichTomhadhastilylighted,fellonthefaceofEliza.“I’mrunningaway,UncleTomandAuntChloe—carrying

(75) offmychild.Mastersoldhim.”“Soldhim?”echoedboth,holdinguptheir

handsindismay.“Yes,soldhim!”saidElizafirmly.“Icrept

intotheclosetbymistress’sdoorto-night,(80) andIheardmastertellmissusthathehad

soldmyHarryandyou,UncleTom,bothtoatrader,andthatthemanwastotakepossession

to-day.”Slowly,asthemeaningofthisspeech

(85) cameoverTom,hecollapsedonhisoldchair,andsunkhisheadonhisknees.

1.Whichchoiceprovidesthebestsummaryofwhathappenedinthepassage?(A)Adealisreluctantlymadeandthereactionsofthoseaffectedaregiven.(B)Aslavesuccessfullyplotsanescapefromanoppressivesociety.(C)Amanstrugglestochoosebetweenwhatishumaneandwhatisprofitable.(D)Aphilosophicaldiscussionisheldbetweenaslave-ownerandaslave-trader.

2.Haleyisbestcharacterizedasa/an(A)humaneempathizer.(B)financialamateur.(C)aggressivenegotiator.(D)passivemediator.

3.Asusedinline31,thephrase“flingin”mostcloselymeans(A)include.(B)hurl.(C)relate.(D)involve.

4.Mr.Shelby’streatmentofthechildinlines27–32isbestdescribedas(A)purposelydeceitful.(B)unintentionallyinhumane.(C)openlybelligerent.(D)tenderlyimpartial.

5.Asusedinline38,theword“article”mostcloselymeans(A)agreement.(B)report.(C)word.(D)item.

6.ThepassagemoststronglyimpliesthatTom’sreactiontohearingofMr.Shelby’splansforhimisoneof(A)unanticipatedpeacefulness.(B)delightedrelief.(C)surpriseddespondency.(D)playfulmockery.

7.Whichoptiongivesthebestevidencefortheanswertothepreviousquestion?(A)Lines58–61(“Iknow...heigho”)(B)Lines62–66(“Theprayer...pane”)(C)Lines73–75(“I’m...him”)(D)Lines84–86(“Slowly...knees”)

8.ItcanreasonablybeinferredthatMr.Shelbyplacesthehighestvalueonwhichcharacter?(A)Tom(B)Eliza(C)Eliza’sson(D)Chloe

9.Whichoptiongivesthebestevidencefortheanswertothepreviousquestion?(A)Lines8–13(“Thefact...conscience”)(B)Lines27–32(“Hulloa...will”)(C)Lines37–42(“By...decidedly”)(D)Lines67–70(“Good...door”)

10.The“lighttap”madebyElizainline66suggeststhatshe(A)fearedunwanteddetection.(B)respectednightlyrituals.(C)washesitanttosharebadnews.(D)understoodhermisdeeds.

Questions11-21arebasedonthefollowingreadingselectionandaccompanyingmaterial.

TheWoesofConsumerism

Ah,butyouseemyfriend,Ifailtobe“hipster”withoutthecheckeredshirt,bowtie,andskinnyjeans.

Line Myfree-spiritedbestfriendisonly“boho”(5) withherheadbandandfringebag.Andmy

“preppy”sisterisrarelyseenwithoutherbeanbootsandstripedcardigans.Oneday,IwilldriveaMercedes-Benzbecause,wellwhywouldn’tI?Andwhenallofmyaspirations

(10) andthelabelsattachedtothemcauseinmeagreatmigraine,IwilltakeTylenolbeforestoppingattheneighborhoodStarbucksformydailymochalatte.Youseewherethisisgoingbecauseyouaresoperceptiveand

(15) undoubtedlyaMillennial,knownforyourskepticism,feelingsofself-importance,andimmunitytothepatheticpropagandathatsoeasilytrickedthepreviousgenerations.But,areyouimmunetobeingtricked?

(20) Youwoulddowelltostudythefollowinglistofdefinitionsbeforecontinuing.TheywereallfoundviaasimpleGooglesearchfortherespectiveterm.

■capitalism:aneconomicandpolitical(25) systeminwhichacountry’stradeand

industryarecontrolledbyprivateownersforprofit,ratherthanbythestate

■consumerism:asocialandeconomicorderandideologythatencouragesthe

(30) acquisitionofgoodsandservicesinever-increasingamounts

■propaganda:information,especiallyofabiasedormisleadingnature,usedtoinfluenceanaudienceandfurtheran

(35) agenda■advertising:themarketingcommunicationusedbycompaniestopersuadeanaudiencetopurchasetheirproductsand/orservices

(40) mereexposure:apsychologicalphenomenoninwhichpeopledevelopapreferenceforthingsthroughfamiliarity

■affectiveconditioning:thetransferoffeelingsfromonesetofitemstoanother

(45) toencouragethepublictoassociateaproductwithpositivity

Certainly,youhaveascendedbeyondthemanipulationaccompanyingsuchridiculoussubculturelabelsasthosementionedabove.

(50) Youhaveseenitandhearditall.Advertisingisallaroundyou;weliveinacommercialworld—acapitalisteconomywithanunparalleledattachmenttoconsumerism.Butthoseslylittledevilsarefarfrombringing

(55) youtothedarkside.Orsoyouthink.Nobodywantstofeeleasilyinfluenced.

Yet,Ibegyoutohearmeout:advertisingiseverywherebecauseitworks.U.S.companiesspendanannual$70billionintelevision

(60) ads,andthisisbeforewetakealookatothermediumsofadvertisinglikeradios,magazines,websitecookies,andeventhoseterriblesocialmedia“sponsored”ads.Thetruthiswedon’tliketofeelmanipulated,but

(65) weare.Advertisingisbyitsnatureaformofpropagandainthatitchangesperceptionswithlimitedinformation—itisneitherobjectivenorcomplete.Advertisingismeanttodoafewthings.

(70) First,itinformsthepublicofaproduct’sexistence(noharminthat,right?).Next,itis

meanttobuildbrandrecognition—asconsumers,wewanttotrustandrecognizethenamesbehindourproducts.Third,advertising

(75) createslifestyleidentification.Theproductissomehow“likeyou”;itsayssomethingaboutyou;youcanrelatetotheproductanditsotherconsumers.Theworldofadvertisingspendsalotofmoneyandtimedeveloping

(80) strategiestoaccomplishthesethreegoals.Wewillfirstlookatlogicalpersuasion,or

theexpositionoffactsaboutproducts.Thistechniqueinitselfisquiteharmless.Wecannotbeinformedconsumerswithout

(85) information.Yet,Iurgeyoutobeskepticalofeventhemoststraightforwardadvertising.Mereexposureisaneffectivetoolinleavinglastingimpressionsonthepublic.Withyouronlybasisasrecognition,youwouldbe

(90) surprisedtoseehowquicklyyouchooseoneproductoveranother,evenatahighercost.Perhapsmoredangerousisthestrategyof

nonrationalinfluence,inwhichadvertisingschemescircumventconsumers’conscious

(95) awarenessbydepictingafunorpleasantscenequiteunrelatedtotheproductitself.Here,affectiveconditioningallowsyoutoassociatepositivefeelingswithspecificproducts.Forinstance,acommercialmight

(100) flashimagesofcolorfulflowers,sunshine,puppies,etc.;andevenyearslater,yoursubconsciouswillrecallthese“feelgood”imageswhenyouspotthatproduct.PsychologyTodayfoundthatthistypeofadvertising

(105) lowersbrainactivityandcauseslessrestraintintheconsumer.Accordingtothestudy,youare70–80%morelikelytobuyaninferiorproductwhenyouhavepaireditwithpositivefeelings.

(110) Justremember,mywaryconsumer,tousecautioninasocietyofdistortionandillusion.Advertisingcanbesubtle,butpowerful.Andifyouthinkyouareunaffected,thinkagain.

FactorsInfluencingConsumerBehavior

11.Itismostreasonabletoinferthattheauthorbelievesthemembersofherreadershipthinktheyare(A)wealthierthantheirpeers.(B)abovetheinfluenceofadvertising.(C)morepoliticallyastutethantheirelders.(D)destinedtoleaveagreatlegacytotheiroffspring.

12.Whatisthemostlikelyreasonthattheauthorchosetobegintheessayasshedidratherthanbeginningwiththesecondparagraph?(A)Tohelpherreadersfirstunderstandthepitfallsofadvertisingbeforeseeingitsbenefits(B)Togiveexamplesoftermsusedincontextpriortothosesametermsbeingdefined(C)Toprovideamoderncasestudybeforeadiscussionofhistoricalprecedents(D)Tobuildrapportwithherreadersbeforedelvingintoatechnicalanalysis

13.Asusedinlines1–6,thewords“hipster,”“boho,”and“preppy”mostcloselymean(A)progressiveyouth.(B)fashionsubcultures.(C)independentthinkers.(D)unconventionallabels.

14.Theauthormoststronglysuggeststhattheoverallattitudethatconsumersshouldhavetowardsadvertisingshouldbe(A)grateful.(B)subconscious.(C)distrustful.(D)bellicose.

15.Whichoptiongivesthebestevidencefortheanswertothepreviousquestion?(A)Lines1–9(“Ah...wouldn’tI”)(B)Lines50–55(“Advertising...think”)(C)Lines85–91(“Yet...cost”)(D)Lines97–103(“Here...product”)

16.Whatismostlikelythepurposeoflines58–63(“U.S....ads”)?(A)Toexplaintheprocessbywhichadvertisingiscreated

(B)Tocritiquecorporationsformisleading,unethicalpractices(C)Togiveconcreteevidencetoillustratetheimpactofadvertising(D)Todemonstrateinstanceswhenconsumersfeelconsciouslymanipulated

17.Asusedinline61,theword“mediums”mostcloselymeans(A)middles.(B)ranges.(C)methods.(D)medians.

18.Accordingtothegraph,culturalfactorsareapproximatelywhatpercentagegreaterintheirinfluencethanthecombinationofpersonalandpsychologicalfactors?(A)30%(B)40%(C)50%(D)60%

19.Basedontheinformationinthegraphandthepassage,anadvertiserwishingtoeffectivelyuseaffectiveconditioningwouldmostlikelyshowwhatsortofascenetoadvertiseacar?(A)Thecarbeingdrivenbyafamouscelebritythroughabeautifulseasidelandscape(B)Thecardrivershownlisteningtoeducationalprogramsonthecar’ssatelliteradio(C)Acarbuyerlookingatdifferentcarpricetagsandfindingtheonebeingadvertisedtobethe

cheapest(D)Anengineerofthecarcarefullydescribingitsuniquedesignfeatures

20.Whichoptiongivesthebestevidencefortheanswertothepreviousquestion?(A)Lines99–103(“Forinstance...product”)(B)Lines103–106(“Psychology...consumer”)(C)Lines106–109(“According...feelings”)(D)Lines110–113(“Just...again”)

21.Whichofthefollowingmodificationstothegraphwouldmakeitmorehelpfultoaconsultingfirmadvisingclientsfromawiderangeofindustriesastohowtobestspendtheiradvertisingdollars?(A)Givingagreaterdegreeofprecisioninthepresentedresultsbyreportingthefactorpercentages

tothenearesthundredthofapercentinsteadofawholepercent(B)Reportingthepercapitaincomelevelsofthepersonssurveyed(C)Addingabreakdownoftherelativeinfluenceofthethreefactorswithrespecttogeneral

categoriesofproducts,suchasentertainment,finance,andconsumergoods(D)Providingthissamefactorbreakdown,butdoingsowithrespecttodifferentgeographical

regions

Questions22–32arebasedonthefollowingreadingselectionandaccompanyingmaterial.

Hemoglobin

ThispassageisadaptedfromA.F.Young,“PathogenesisofHemoglobinopathies.”

Therearefewbiochemicalcompounds

asfamiliartousashemoglobin,andastheprimarytransporterofoxygeninourblood,

Line thecelebrityofthiscuriouslittlecompound(5) isnotwithoutjustcause.Vitaltoalmost

everyknownvertebrate,hemoglobinappearswithintheveryfirstweekofembryogenesis,andwhileitsrolemaynotchangethroughoutdevelopment,itsmolecularstructureundergoes

(10) aseriesofsignificanttransformations.Withintheredbloodcell,hemoglobin

existsasafour-subunitcomplex,or“tetramer,”eachsubunitofwhichismadeupofone“heme”metalloprotein,andoneofseveral

(15) varietiesof“globin.”Comprisedofironandacarbon-nitrogenring,hemeisresponsibleforboththeoxygen-bindingcapacityofhemoglobin,andfortheredcolorationofblood.Globin,meanwhile,referstoafolded

(20) chainofpolypeptides,anditisthecombinationofthesechainsthatimpartseachtypeofhemoglobinwithitsuniquecharacteristics.Inhumans,sixglobinchainsareexpressed

sequentiallythroughoutdevelopment.(25) Embryonichemoglobin,orHbE,iscomposed

oftwoζchainsandtwoεchains,bothofwhichareexpressedexclusivelyduringtheembryonicperiod.Inthefetalperiod,anothertetrameroftwoαchainsandtwo

(30) γchainsemerges,andpersistsforthefirstsixmonthsofpostnatallife.Duetoitshighaffinityforbindinggases,thisfetalhemoglobin,orHbF,isabletoextractoxygenfromlow-affinitymaternalhemoglobin,andthus

(35) playsacrucialroleintheoxygenationoffetaltissues.LikeHbF,thefinaltwophysiologichemoglobins,HbAandHbA2,alsorequireapairofachains,anddifferonlyinbeingcoupledtotwoβchains,andtwoδchains,

(40) respectively.Typically,bothHbAandHbA2aresynthesizedatfairlystableconcentrations,thoughHbAisproducedinfargreaterabundance.Giventhetremendousimportofthese

(45) complexes,itshouldhardlybesurprisingthaterrorsintheirproductioncanyielddevastatingresults.Whatmaybesurprising,

however,isthattheseerrors—includingsickle-celldiseaseandthalassemia—are

(50) amongthemostcommonofallinheritedgeneticdisorders,withanestimated7%oftheworld’spopulationascarriers,two-thirdsofwhomresideinAfrica.Thalassemiadescribesagroupofdisorders

(55) inwhicheithertheαorβchainisquantitativelyreduced.Dependingonthemutation,thesedefectscanpresentwithawiderangeofanemia-relatedsymptoms,andareparticularlyprevalentthroughoutAfrica,

(60) SoutheastAsia,andtheMediterranean.Thisgeographicaldistributionisanythingbutrandom.Manystudieshavedemonstratedthattheproductionofsuboptimalhemoglobinconfersadegreeofprotectionagainst

(65) malaria,apotentiallydeadlyinfectiousdiseasecausedbymembersofthePlasmodiumgenus,whichparasitizeredbloodcells.Itfollows,then,thatwhereasinmanyregionsthroughouttheworldthalassemiamay

(70) merelyconstitutedisease,inthosewheremalariaisendemic,itrepresentsafavorableevolutionaryadvantage.Owingtoaredundancyinthehuman

genome,therearefourcopiesoftheαglobin(75) gene,withtwoα-codingregionsoneach

copyofchromosome16.Forthisreason,thespectrumofseverityinαthalassemiaisparticularlybroad.Forinstance,deletionofasinglegenewillresultinacarrierstate,and

(80) isunlikelytocauseclinicallyacutesymptoms.Deletionofallfour,meanwhile,leadstoaprecipitationduringthefetalperiodofnonfunctionalγtetramers,alsocalledHbBarts,andisuniversallylethalinutero.

(85) Similarly,adeletionofthreecopiestypicallyresultsinaseriousbutsurvivableanemia,andischaracterizedbytheformationofHbBartsinthefetalperiod,andnonfunctionalβtetramers,termedHbH,throughout

(90) adulthood.Predictably,adeletionoftwocopiesproducesastillmilderanemia,butcanbesubclassifiedbasedonwhetherthedeletionsoccuronthesamechromosome,

termed“cis”deletion,oronopposite(95) chromosomes,termed“trans”deletion.The

transsubtypeappearsmorecommonlyin

theMediterranean,whilecisismoreoftenfoundinAsia.Notably,ithasbeensuggestedthatthecisdeletionmaycontributetothe

(100) relativelyhigherratesoffailedpregnanciesobservedinthispartoftheworld.Likeitsαcounterpart,βthalassemiaalso

impairstheproductionofHbA.However,thesymptomsofβthalassemiawillnotbecome

(105) evidentuntilafterthefirstsixmonthsoflife,whentheconcentrationofHbFwanestoacriticalthreshold.Often,acompensatoryupregulationintheexpressionofHbA2occursinaffectedindividuals,theeffectsof

(110) whichcanbepharmaceuticallyaugmentedbyadrugcalledhydroxyurea,whichinducestheexpressionofHbFinchildrenandadults.

Thegraphshowsvaryingconcentrationsofglobinchainsduringhumandevelopment.

22.Thegeneralpurposeofthispassageisto(A)makeanargument.(B)raisevitalquestions.(C)introduceaconcept.(D)callforacourseofaction.

23.Asusedinline4,theword“celebrity”mostcloselymeans(A)notoriety.(B)infamy.(C)personage.(D)festivity.

24.Basedonlines11–22andtheinformationinthegraph,whatmakesthehemoglobinvarietiesdistinct?(A)Whetherthereisacarbon-nitrogenring

(B)Whetherthereisaredcolorationoftheblood(C)Variationinthearrangementofmetalloproteinchains(D)Variationinthearrangementofpolypeptidechains

25.Whichoptiongivesthebestevidencefortheanswertothepreviousquestion?(A)Lines11–15(“Within...globin”)(B)Lines15–16(“Comprised...ring”)(C)Lines16–19(“heme...blood”)(D)Lines19–22(“Globin...characteristics”)

26.Theparagraphinlines44–53moststronglysuggeststhatitissurprisingthat(A)thevastmajorityofinheritedgeneticdisordersoccurduetomalfunctionsinhemoglobin

production.(B)biochemicalcompoundssoimportanttohumandevelopmentcansofrequentlyhaveerrorsin

theirproduction.(C)suchacriticalpartofhumanhealthissusceptibletowidespreadpandemiccontagion.(D)sickle-celldiseaseandthalassemia,despitebeinggeneticallyinheriteddiseases,canbe

responsibleforthemajorityofearlydeathsinAfrica.

27.Asusedinline50,theword“common”mostcloselymeans(A)communal.(B)lowly.(C)widespread.(D)famous.

28.Thepurposeoflines60–72istoconnect(A)hemoglobindevelopmenttomigrationpatterns.(B)culturalcharacteristicstoevolutionarytraits.(C)geographicparticularitiestoevolutionaryadaptation.(D)environmentalforcestomedicalinnovations.

29.Lines73–78(“Owing...broad”)mostdirectlyimplythattheintensityofthalassemiawouldbemoreuniformiftherewere(A)fewercopiesoftheαglobingeneinhumans.(B)thesamenumberofcopiesoftheαglobingeneinhumans.(C)morecopiesoftheαglobingeneinhumans.(D)moreinformationisneededthanisgivenintheselectedsentences.

30.Theparagraphinlines73–101suggeststhattherelationshipbetweenthenumberofglobingenesdeletedandtheseverityofdiseaseis(A)inverse.(B)proportional.(C)equivalent.(D)random.

31.Whatevidencefromthepassagegivesthebestjustificationastowhythegraphbeginsalongthex-axiswhereitdoes?

(A)Lines5–7(“Vital...embryogenesis”)(B)Lines19–22(“Globin...characteristics”)(C)Lines47–53(“What...Africa”)(D)Lines81–84(“Deletion...utero”)

32.Basedonthegraph,aroughmeasurementofwhichofthefollowingglobinchainswouldgivetheclearestindicationthatachildwasborntwomonthspremature?(A)Alpha(B)Beta(C)Gamma(D)Zeta

Questions33–42arebasedonthefollowingreadingselection.

This excerpt from “The American Forests,” was part of John Muir’s 1897 campaign to save theAmerican wilderness. He would later be called the godfather of the American environmentalmovement.

TheforestsofAmerica,howeverslightedbyman,musthavebeenagreatdelighttoGod;fortheywerethebestheeverplanted.

Line Thewholecontinentwasagarden,andfrom(5) thebeginningitseemedtobefavoredabove

alltheotherwildparksandgardensoftheglobe.[…]Sotheyappearedafewcenturiesago

whentheywererejoicinginwildness.The(10) Indianswithstoneaxescoulddothemno

moreharmthancouldgnawingbeaversandbrowsingmoose.EventhefiresoftheIndiansandthefierceshatteringlightningseemedtoworktogetheronlyforgoodinclearingspots

(15) hereandthereforsmoothgardenprairies,andopeningsforsunflowersseekingthelight.Butwhenthesteelaxeofthewhitemanrangoutinthestartledairtheirdoomwassealed.Everytreeheardthebodefulsound,

(20) andpillarsofsmokegavethesigninthesky.Isupposeweneednotgomourningthe

buffaloes.Inthenatureofthingstheyhadtogiveplacetobettercattle,thoughthechangemighthavebeenmadewithoutbarbarous

(25) wickedness.Likewisemanyofnature’sfivehundredkindsofwildtreeshadtomakewayfororchardsandcornfields.Inthesettlementandcivilizationofthecountry,breadmorethantimberorbeautywaswanted;andin

(30) theblindnessofhunger,theearlysettlers,claimingHeavenastheirguide,regardedGod’streesasonlyalargerkindofperniciousweeds,extremelyhardtogetridof.Accordingly,withnoeyetothefuture,these

(35) piousdestroyerswagedinterminableforestwars;chipsflewthickandfast;treesintheirbeautyfellcrashingbymillions,smashedtoconfusion,andthesmokeoftheirburninghasbeenrisingtoheavenmorethantwo

(40) hundredyears.AftertheAtlanticcoastfromMainetoGeorgiahadbeenmostlyclearedandscorchedintomelancholyruins,theoverflowingmultitudeofbreadandmoneyseekerspouredovertheAllegheniesinto

(45) thefertilemiddleWest,spreadingruthlessdevastationeverwiderandfartherovertherichvalleyoftheMississippiandthevastshadowypineregionabouttheGreatLakes.Thencestillwestwardtheinvadinghordeof

(50) destroyerscalledsettlersmadeitsfierywayoverthebroadRockyMountains,fellingandburningmorefiercelythanever,untilatlastithasreachedthewildsideofthecontinent,andenteredthelastofthegreataboriginal

(55) forestsontheshoresofthePacific.Surely,then,itshouldnotbewondered

atthatloversoftheircountry,bewailingitsbaldness,arenowcryingaloud,“Savewhatisleftoftheforests!”Clearinghassurelynow

(60) gonefarenough;soontimberwillbescarce,andnotagrovewillbelefttorestinorprayin.Theremnantprotectedwillyieldplentyoftimber,aperennialharvestforeveryrightuse,withoutfurtherdiminutionofitsarea,

(65) andwillcontinuetocoverthespringsoftheriversthatriseinthemountainsandgiveirrigatingwaterstothedryvalleysattheirfeet,preventwastingfloodsandbeablessingtoeverybodyforever.

(70) Everyothercivilizednationintheworldhasbeencompelledtocareforitsforests,andsomustweifwasteanddestructionarenottogoontothebitterend,leavingAmericaasbarrenasPalestineorSpain.Inits

(75) calmermomentsinthemidstofbewilderinghungerandwarandrestlessover-industry,

Prussiahaslearnedthattheforestplaysanimportantpartinhumanprogress,andthattheadvanceincivilizationonlymakesit

(80) moreindispensable.[…]Sofarourgovernmenthasdonenothing

effectivewithitsforests,thoughthebestintheworld,butislikearichandfoolishspendthriftwhohasinheritedamagnificent

(85) estateinperfectorder,andthenhaslefthisrichfieldsandmeadows,forestsandparks,tobesoldandplunderedandwastedatwill,dependingontheirinexhaustibleabundance.Nowitisplainthattheforestsarenot

(90) inexhaustible,andthatquickmeasuresmustbetakenifruinistobeavoided.

33.Theoverallpointofthepassageisto(A)tellastory.(B)surveycurrentknowledge.(C)makeanargument.(D)describeanenvironment.

34.Muir’stoneinthepassageisbestdescribedas(A)urgentandearnest.(B)arrogantandcondescending.(C)optimisticandcheerful.(D)hopelessanddepressed.

35.Asusedinline19,theword“sealed”mostcloselymeans(A)fastened.(B)settled.(C)authenticated.(D)killed.

36.Basedontheinformationinthepassage,itisreasonabletoinferthatintheyear1897,whichregionoftheUnitedStateshadthegreatestabundanceofunharvestedforests?(A)TheAtlanticCoast(B)TheMiddleWest(C)TheMississippiValley(D)ThePacificRegion

37.Whichoptiongivesthebestevidencefortheanswertothepreviousquestion?(A)Lines34–42(“Accordingly...ruins”)(B)Lines43–45(“Multitude...West”)(C)Lines47–48(“rich...Lakes”)(D)Lines49–55(“Thence...Pacific”)

38.Muiruseslines59–62(“Clearing...prayin”)tomakeappealsthatfocusonthethemesof(A)nationalism,militarism,andexpansionism.(B)environmentalism,scholarship,andpiety.(C)economics,leisure,andreligion.(D)justice,individualism,andtruth.

39.MuirdescribestheoverallapproachtoforestmanagementbytheU.S.Governmentatthetimethispassagewaswrittenas(A)hands-off.(B)legalistic.(C)progressive.(D)interventionist.

40.Whichoptiongivesthebestevidencefortheanswertothepreviousquestion?(A)Lines1–3(“The...planted”)(B)Lines40–42(“After...ruins”)(C)Lines56–59(“Surely...forests”)(D)Lines81–84(“Sofar...spendthrift”)

41.MuirsuggeststhattheUnitedStatesshouldemulatethephilosophyofwhichcountry?(A)Palestine(B)Prussia(C)Spain(D)Georgia

42.Asusedinline91,theword“ruin”mostcloselymeans(A)undoing.(B)hostility.(C)devastation.(D)ignorance.

Questions43–52arebasedonthefollowingreadingselections.

AlternativeEnergy—twoauthorsconsiderthestateofalternativeenergysolutions.

PASSAGE1

Nooneissurehowmuchavailableoilisleft,butconsideringouroilreservestookhundredsofmillionsofyearstoform,timeto

Line depletionislittlemorethanablinkofaneye.(5) Sowehavetwooptions—stoprelyingonoil

oruseitupandwatchtheensuingchaos.Onepromisingalternativefuelsource

isethanol.Ourancestorshavebeenfermentingorganicmattertomakeethanol

(10) forthousandsofyears.Todayethanolis

primarilyconsumedinalcoholicbeverages,butwhynotalsouseittopowerourcars?Thisalternativefuelismadebyfermentingcropssuchaswheat,corn,andsugarcane.

(15) Oneglucosemoleculeisbrokendowntoformtwoethanolmoleculesandtwocarbondioxidemolecules.Becauseitismadefromorganicmatter,itisrenewable—abigprocomparedtooil.Anotherbenefitisthat

(20) it’sdomesticallymade,sowedon’thavetorelyonothercountriesforit.Unfortunately,it’sslightlymoreexpensivepermilethangasoline.Additionally,becauseitsproductionusescrops,widespreadimplementationmay

(25) causeanincreaseinsomefoodprices.Anotherpromisingalternativeisbiodiesel.

Biodieselismadeoutofanimalfats,plantfats,andevenusedgreasefromrestaurants.Theglycerolbackboneisremovedfrom

(30) thefat,breakingthefatintothreeseparatechains,whicharethenreactedwithanalcoholtoformthebiodiesel.Thistypeofchemicalreactioniscalledatransesterification.Likeethanol,biodieselisalsorenewableand

(35) domesticallyproduced.It’salsocompletelynontoxicandbiodegradable.Unfortunately,likeethanol,it’salsomoreexpensive.Whiletheymaybemoreexpensive,bothofthesefuelsourcesproducefewergreenhousegases

(40) thanregulargasoline.Acoupleextradollarsisasmallpricetopayfortheenvironmentalfriendlinessandself-sufficiencythatthesealternativeswouldprovide.Ourcurrentalternativesmaynotbeperfect,

(45) butthat’snoreasontobediscouraged.Thetimeforalternativefuelexplorationisnow.Whywaitforoiltorunoutwhensuperiorsourcesofenergyarealreadyavailableandmorearewithinreach?

PASSAGE2

(50) Thereisanaturaltendencytoconfusechangewithprogress.Thisisperfectlyunderstandable,especiallyconsideringthatwewentfrominventingelectricitytoperfectingaviationtoreachingthemoonallinatime

(55) periodanalogoustojustablinkinthegrandschemeofhumanhistory.Suchprodigiousleapshaveleftushungryformoreleaps,andtherearebenefitstorestlessness,evenifentropic;throwenoughaimlessdartsin

(60) everydirectionandyou’llfindabull’seye,evenifbyaccident.But,suchleapshavealsoleftusskeptical

againstinaction,andnowthereisaproclivitytomistakethestatusquoforthestagnation

(65) ofstandingstill.CallittheNewCokeeffect,wheresocietytakesthreemisguidedstepsbackinitsinterminableurgencytokeepmovingforward.Thatsaid,Iwillbethefirsttoadmitthat

(70) thefuturelivelihoodofanindustrializedworldmostlikelyhingesonchange,namelythediscoveryofaneffective,inexpensivesourceofrenewableenergy.But,asthefederalgovernmentwastesbillionshere

(75) andbillionstherethrowingmoneyathopelesscompanieswithhaplessexecutives(Solyndra,forinstance),Ican’thelpbutfeellikerenewableenergyisNewCoke.Certainly,wehavenotyetperfectedourenergeticways

(80) andmeans,butwhyarewesoobsessedwithdiscardingwhatwehavenow?Principally,despitedecadesofapocalyptic

forecastingofpeakoil,petroleumoutputisashealthyasever.Infact,petroleumcompanies

(85) areleavingtheindustrynotbecauseoilreservesaredwindling,butratherbecauseoilproductionissomassivethatdemandisfallingconsiderably.Caseinpoint:oiliscurrentlysellingatathirdthecostofbottled

(90) water.So,yes—thedaymostlikelywillcome

whenthewellsrundry.But,untilthen,letuscelebrateourgoodfortuneandbethankfulforwhatwehave.

43.Theauthorofpassage1moststronglyimpliesinparagraph1(lines1–6)thatthechoiceofwhethertopursuealternativeenergyis(A)multi-faceted.(B)obvious.(C)ambiguous.(D)premature.

44.Asusedinline30,theword“breaking”mostcloselymeans(A)flouting.(B)eliminating.(C)separating.(D)categorizing.

45.TheauthorofPassage1suggestsinlines44–49(“Ourcurrent...reach”)thatextensiveresearchintoalternativeenergyresourcesshouldbegin(A)inthecomingcenturies.(B)inthecomingdecades.(C)inthecomingyears.(D)immediately.

46.Lines59–61(“throw...accident”)canbestbeparaphrasedas(A)systematic,focusedresearchwillleadtoasuccessfulresult.(B)amateurresearchersshouldbeputonequalfootingwithacademicresearchers.(C)nearlyallusefulrecentinnovationshavecomeastheresultofchaoticcreativity.(D)ifyoutryenoughdifferentthings,somethingwilleventuallywork.

47.Asusedinline82,theword“apocalyptic”mostcloselymeans(A)pessimistic.(B)technical.(C)asymmetric.(D)deceitful.

48.TheauthorofPassage2primarilyusestheexampleinlines88–90(“Casein...water)to(A)showhowwaterpricesreflectrelativelyhighdemandforit.(B)illustratehowoilpricesreflectrelativelylowdemandforit.(C)explainhowoilhascometobemoreplentifulthanwater.(D)demonstratewhyconsumersfindfewerusesforoilthanwater.

49.TheauthorofPassage1wouldmostlikelystatethattheauthorofPassage2needstomakewhatimportantclarificationtohisstatementinlines82–84(“Principally...ever”)?(A)Towhatextentthisappliestojustdomesticpetroleumproduction(B)Whetherthepetroleumproducedisorganicandrenewable(C)Ifthepetroleumproductionwillgenerategreenhousegases.(D)Ifthepetroleummentionedherewillbemoreorlessexpensivethanethanol

50.Whichoptiongivesthebestevidencefortheanswertothepreviousquestion?(A)Lines7–10(“Onepromising...years”)(B)Lines19–21(“Another...forit”)(C)Lines37–40(“While...gasoline”)(D)Lines40–43(“Acouple...provide”)

51.WhatevidencefromPassage1wouldtheauthorofPassage2mosteffectivelyusetosupporthisstatementinlines78–81(“Certainly...now”)?(A)Lines1–4(“Noone...eye”)

(B)Lines15–19(“Oneglucose...oil”)(C)Lines21–25(“Unfortunately...prices”)(D)Lines29–32(“Theglycerol...biodiesel”)

52.Whichstatementbestsummarizestheoverallrelationshipbetweenthetwopassages?(A)Passage2andPassage1areindirectoppositiontoeachotherwhenitcomestothequestionof

theassociationofpetroleumwithgreenhousegasemissions(B)Passage2explorestheassociationofpetroleumwithcontemporarypopularculturefarmorethan

doesPassage1(C)Whilebothpassagesareconcernedaboutpetroleumdepletion,Passage1advocatesimmediate

actionandPassage2callsforpatience(D)Whilebothpassagesareinterestedinalternativeenergysolutions,Passage2focuseson

governmentfundingandPassage1focusesonscientificinnovation

Ifyoufinishbeforetimeiscalled,youmaycheckyourworkonthissectiononly.Donotturntoanyothersection.

SCORINGAPPROXIMATIONThistablegivesyouanestimateofhowyourperformanceontheReadingsectionwillcontributetoyouroverallEvidence-BasedReadingandWritingscore.Keepinmindthateachtestwillbecurved,makingthe number of questions needed for a particular score dependent on the test that day. This is the bestestimate we can give you based on (1) previous SAT curves and (2) the fact that guessing is nowpermittedontheSAT.

Questionsoutof52answeredcorrectlyEstimatedoverallsectionscore(between

200–800)

800

52

750

49

700

46

650

43

600

40

550

37

500

33

450

30

400

26

350

20

300

13

250

7

200

0

PracticeTest2

65MINUTES,52QUESTIONS

Directions:Eachpassageorpairofpassagesisaccompaniedbyseveralquestions.Afterreadingthepassage(s),choose thebestanswer toeachquestionbasedonwhat is indicatedexplicitlyorimplicitlyinthepassage(s)orintheassociatedgraphics.

Questions1–10arebasedonthefollowingreadingselection.

This excerpt is the beginning ofF. ScottFitzgerald’sThis Side of Paradise, published in 1920. Thebook opens up with the following character introduction of Fitzgerald’s semi-autobiographicalprotagonist,AmoryBlaine.

AmoryBlaineinheritedfromhismothereverytrait,exceptthestrayinexpressiblefew,thatmadehimworthwhile.Hisfather,

Line anineffectual,inarticulatemanwithataste(5) forByronandahabitofdrowsingoverthe

EncyclopediaBritannica,grewwealthyatthirtythroughthedeathoftwoelderbrothers,successfulChicagobrokers,andinthefirstflushoffeelingthattheworldwashis,

(10) wenttoBarHarborandmetBeatriceO’Hara.Inconsequence,StephenBlainehandeddowntoposterityhisheightofjustundersixfeetandhistendencytowaveratcrucialmoments,thesetwoabstractionsappearing

(15) inhissonAmory.Formanyyearshehoveredinthebackgroundofhisfamily’slife,anunassertivefigurewithafacehalf-obliteratedbylifeless,silkyhair,continuallyoccupiedin“takingcare”ofhiswife,continuallyharassed

(20) bytheideathathedidn’tandcouldn’tunderstandher.ButBeatriceBlaine!Therewasawoman!

Earlypicturestakenonherfather’sestateatLakeGeneva,Wisconsin,orinRome

(25) attheSacredHeartConvent—aneducationalextravagancethatinheryouthwasonlyforthedaughtersoftheexceptionallywealthy—showedtheexquisitedelicacyofherfeatures,theconsummateartandsimplicity

(30) ofherclothes.Abrillianteducationshehad—heryouthpassedinrenaissanceglory,shewasversedinthelatestgossipoftheOlderRomanFamilies;knownbynameasafabulouslywealthyAmericangirlto

(35) CardinalVitoriandQueenMargheritaandmoresubtlecelebritiesthatonemusthavehadsomecultureeventohaveheardof.ShelearnedinEnglandtopreferwhiskeyandsodatowine,andhersmalltalkwasbroadened

(40) intwosensesduringawinterinVienna.AllinallBeatriceO’Haraabsorbedthesortofeducationthatwillbequiteimpossibleeveragain;atutelagemeasuredbythenumberofthingsandpeopleonecouldbecontemptuous

(45) ofandcharmingabout;aculturerichinallartsandtraditions,barrenofallideas,inthelastofthosedayswhenthegreatgardenerclippedtheinferiorrosestoproduceoneperfectbud.

(50) InherlessimportantmomentsshereturnedtoAmerica,metStephenBlaineandmarriedhim—thisalmostentirelybecauseshewasalittlebitweary,alittlebitsad.Heronlychildwascarriedthroughatiresome

(55) seasonandbroughtintotheworldonaspringdayinninety-six.WhenAmorywasfivehewasalreadya

delightfulcompanionforher.Hewasanauburn-hairedboy,withgreat,handsome

(60) eyeswhichhewouldgrowuptointime,afacileimaginativemindandatasteforfancydress.Fromhisfourthtohistenthyearhedidthecountrywithhismotherinherfather’sprivatecar,fromCoronado,where

(65) hismotherbecamesoboredthatshehadanervousbreakdowninafashionablehotel,downtoMexicoCity,whereshetookamild,almostepidemicconsumption.Thistroublepleasedher,andlatershemadeuseofitasan

(70) intrinsicpartofheratmosphere—especiallyafterseveralastoundingbracers.So,whilemoreorlessfortunatelittlerich

boysweredefyinggovernessesonthebeachatNewport,orbeingspankedortutoredor

(75) readtofrom“DoandDare,”or“FrankontheMississippi,”Amorywasbitingacquiescentbell-boysintheWaldorf,outgrowinganaturalrepugnancetochambermusicandsymphonies,andderivingahighlyspecialized

(80) educationfromhismother.“Amory.”

“Yes,Beatrice.”(Suchaquaintnameforhismother;sheencouragedit.)“Dear,don’tthinkofgettingoutofbedyet.

(85) I’vealwayssuspectedthatearlyrisinginearlylifemakesonenervous.Clothildeishavingyourbreakfastbroughtup.”“Allright.”“Iamfeelingveryoldto-day,Amory,”she

(90) wouldsigh,herfaceararecameoofpathos,hervoiceexquisitelymodulated,herhandsasfacileasBernhardt’s.“Mynervesareonedge—onedge.Wemustleavethisterrifyingplaceto-morrowandgosearchingfor

(95) sunshine.”Amory’spenetratinggreeneyeswould

lookoutthroughtangledhairathismother.Evenatthisagehehadnoillusionsabouther.

1.Beatriceisbestcharacterizedas(A)privilegedandeccentric.(B)mean-spiritedandhaughty.(C)wealthyandindustrious.(D)misanthropicanditinerant.

2.Lines1–3(“Amory...while”)moststronglysuggestthat(A)Amoryandhismothersharemanyunfavorablequalities.(B)Amory’sbestcharacteristicscamefromhismother.(C)Amory’sgreatintellectandpersonalitycamefromhisfather.(D)Amoryhasastrikinginterestingenetics.

3.Thestyleofthesecondparagraph(lines22–49)isgenerally(A)educationalandmorose.(B)intellectualandnebulous.(C)idealisticandoptimistic.(D)emphaticanddescriptive.

4.Asusedinline31,theword“passed”mostcloselymeans(A)spent.(B)gaveup.(C)tossed.(D)agreed.

5.ThepassageimpliesthatBeatricemarriedStephenforwhatreason?(A)Truelove(B)Becauseshesettled(C)Becauseshewasforced

(D)Becauseofvengeance

6.Whichoptiongivesthebestevidencefortheanswertothepreviousquestion?(A)Lines3–10(“Hisfather...O’Hara”)(B)Lines15–21(“Formany...her”)(C)Lines50–53(“Inher...sad”)(D)Lines84–87(“Dear...up”)

7.Amory’supbringingandeducationcanbestbedescribedas(A)demanding.(B)scholarly.(C)exhausting.(D)unique.

8.Whichoptiongivesthebestevidencefortheanswertothepreviousquestion?(A)Lines15–21(“For...her”)(B)Lines41–49(“Allin...bud”)(C)Lines72–80(“So...mother”)(D)Lines84–87(“Dear...up”)

9.Asusedinline63,theword“did”mostcloselymeans(A)made.(B)traveled.(C)caused.(D)organized.

10.Amory’srelationshipwithhismotheris(A)traditionallypious.(B)unusuallyfriendly.(C)blatantlydisrespectful.(D)coldanddistant.

Questions11–21arebasedonthefollowingreadingselectionandaccompanyingmaterial.

What’sNotto“Like”?

ElizaJenningsisexcitedtobeapartofhercompany’sfirstevercommunityevent.ComegetyourfacepaintedandtryZing—

Line theneweight-hourenergyshot!#zing(5) #LawrenceCoFieldDay#lovemyjob

TomWillishasapositionopenforadministrativesecretary—2yearsofexperiencepreferred.Messagewithanyinquiries.

SherrySwansonneedshelpwiththenew(10) softwareprogram.Alert:Technophobia!Can

anybodyexplain?

Iftheeruptionofsmartphoneshasbeenthevanguardofanything,itisthenearsocietaltakeoverofsocialmedia.Withinthe

(15) workplace,mostsupervisorsquicklyblocksiteslikeFacebook,Twitter,andInstagramfromcompanycomputers,andforgoodreason.Productivityislikelytodecreaseifthreehoursofaneight-hourworkdayarespent

(20) “liking”and“tweeting”and“pinning.”Thenthereiscompanybad-mouthingtoconsider.Inthedigitalage,nothingonesaysordoesorrecordsonlineisprivate—nothing.Ittakesonlyafewincidentstorealizethatasocial

(25) mediapageisnotthebestplacetopostone’saggravationwithcompanypolicies,toshareconfidentialbusinessplans,ortoannounceone’sforthcomingresignation.Andwitheventhemostresponsiblestaff,thereisthe

(30) heightenedpossibilityofvirusesorhacking.Simplyput,manyemployersdecidetheconsoutweighthepros.Manyofthesesameadministrativeboards

agreethatsocialmediaoutsideofworkis(35) justasharmfulandmakeaddendumsto

employeecontractsoutlawingallmentionofworkand/orcolleaguesviaonlinesocialoutlets.Otherjobsgofarther,demandingthatone’ssocialmediapersonalitiesalign

(40) withthecorporatevaluesmaintainedintheworkplace.Onemayfacedisciplinaryactionandeventerminationifapageappearsindecentoroffensive.Still,whenbringinginnewhires,employersviolatetheirownembargo

(45) andcheckoutprospectiveemployees’pages,quicklydisqualifyingapplicantswhomaynotseemtofitthecompanyculture.Socialmediahascertainlyalteredtoday’sworkforce,andmanywouldarguethatthechange

(50) hasn’tbeenforthebest.Yet,theexamplesabovepaintanother

picture—onewheretheworkforceisactuallyimprovedbytheopencommunication,widernetwork,freeadvertising,andincreased

(55) accessibilityofsocialmedia.Infact,therearemanyreasonswhyanemployershouldhesitate

tobanallsocialnetworking.Forsome,theuniqueabilityofsocialmediatomarketcompanyservicesandextendcompanyreputation

(60) isindispensable.Manystartupbusinessesfindthattheysimplycannotcompetewithoutasocialmediapagetodelivertheirmissionandbroadentheircontacts.Itcansimplybethebesttoolavailableforadvertising,

(65) marketing,expansion,andcustomerfeedback.Likewise,itprovidesanunrivaledmediumformarketresearch.Networkingexpansionisn’ttheonlyplus.

Whileitispossiblethatsocialmediacould(70) creatediscord,itisjustaslikelytopromote

collaborationandsolidaritywithincompanyculture.Neverbeforehasitbeenaseasyforcolleaguestolinkup,interact,andinitiatefriendships.Thebenefitsofhaving

(75) employeeswhoknowoneanotheranddeveloprespectforoneanotherareendless.Acongenialicebreaker,FacebookandTwitterpagesareknownforbringingtogetherformerstrangersandallowingacquaintancestorealize

(80) similarinterests.Acorporateculturethatembracesaffinityandbreaksdownbarrierstoallowopenandconstructivediscussionisinafarbettersituationthanonethatdoesn’t.Surprisingly,socialmediahasalsobeen

(85) connectedtobettercompanyretention.Sometechnologicallysavvyemployershavecreatedcompanypageswherestaffcanmakeannouncements,shareideas,discussproblems,andcongratulateoneanotheronexcellent

(90) work.Thepagebecomesaspacewherecolleaguescansupportoneanother,butalsowherethecompanyitselfcanshowappreciation.Feelingsofopenness,teamwork,andaptrecognitionkeepgoodworkershappy

(95) andintheirpositionslonger.Moreso,someofficesreportthatemployeeswhousesocialmediaareactuallymoreproductive,withoccasionaltweetsandstatusupdatesprovidingamuch-neededbreakinanotherwise

(100) monotonousworkday.Socialmediaintheworkplacehasgotten

abadrap;inmanyways,itdeservesit.Buttheroleitcanplay—whenembracedappropriately—

innetworking,collaboration,and

(105) retentionprovesthatitisn’tassimpleasthat.Likeanynewandrapidlychangingtechnology,itwilltaketimeandadaptabilityforitsadvantagesandpitfallstobeclear.Thesmartcompanywillfinditnecessarytoconsider

(110) theimplicationssocialmediapresentsforitsfuture—isitreallysomethingthatcanjustbeignoredorbannedaltogether?

Percentagesofmembersofeachdemographicgroupwhousesocialnetworkingsitesin2014.

Source:PewResearchCenter

11.Whichofthefollowingstatementsbestexpressesthethesisofthepassage?(A)Socialmediahasalreadyproventobeoneofthemostvaluableworkplacetools.(B)Socialmediashouldnotbedisregardedasapotentiallyvaluabletoolintheworkplace.(C)Therisksofsocialmediaarefartoogreattoallowitintheworkplace.(D)Employeesshouldbeabletodecideforthemselveshowtobestusesocialmediawhileworking.

12.Whichoptiongivesthebestevidencefortheanswertothepreviousquestion?(A)Lines12–21(“Ifthe...consider”)(B)Lines33–43(“Many...offensive”)(C)Lines90–100(“Thepage...workday”)(D)Lines101–112(“Social...altogether”)

13.Thestructureoftheessayisbestdescribedasa/an(A)analysisoftheprosandconsofanissue.(B)argumentinfavorofachangefromthemainstream.(C)critiqueofthelatestmediaresearchonatopic.(D)aseriesofinterestinganecdotes.

14.Themostlikelypurposeoflines1–11isto(A)introducethethesisoftheessaybygivingthreemajorpointstobeanalyzedgoingforth.(B)hookthereader’sinterestwithconcreteexamplesillustratingtheapplicabilityofthepassage’s

topic.(C)connecttothesentencethatfollowsbyprovidinginstancesofcommonsmartphonelanguage.(D)drawupontheauthor’spersonalexperiencestoconnectwithsimilarexperiencesofthereaders.

15.Lines51–55(“Yet...media”)primarilyfunctionto(A)digressfromthethemeofthepassage.(B)challengetheargumentthatfollows.(C)explainhowcertainsocialmediacompaniescametodominatethemarketplace.(D)provideamajortransition.

16.Thepassagesuggeststhatwhattypeofbusinesswouldmostlikelybenefitfromutilizingsocialmedia?(A)Alarge,expandingbusiness(B)Awell-establishedbusiness(C)Asmall,growingbusiness(D)Amid-sized,industrialbusiness

17.Whichoptiongivesthebestevidencefortheanswertothepreviousquestion?(A)Lines23–28(“Ittakes...resignation”)(B)Lines33–41(“Manyof...workplace”)(C)Lines51–57(“Yet...networking”)(D)Lines60–63(“Many...contacts”)

18.Asusedinline67,theword“medium”mostcloselymeans(A)best.(B)means.(C)middle.(D)standard.

19.Asusedinline90,theword“space”mostcloselymeans(A)area.(B)dimension.(C)clearing.(D)separation.

20.Basedontheinformationinthetable,arandomlyselectedpersonwithwhichofthefollowingcharacteristicswouldbemostlikelytouseasocialnetworkwebsite?(A)A45-year-oldmanwhohasanadvancedgraduatedegree.(B)A35-year-oldmanwhohasamaster’sdegreeinengineering.(C)A25-year-oldwomanwholeftcollegewithoutcompletingherdegree.(D)A15-year-oldgirlwhoisasophomoreinhighschool.

21.Whatstatement,iftrue,wouldbestconnecttotheinformationinthegraphtoexplainwhatthepassagestatesaboutstartupbusinessesinlines60–66(“Manystartup...feedback”)?(A)Thoseintheagegroup18–29arebyfarthemostlikelytobeinterestedintheproductsofstartup

businesses.(B)Asmoreconsumersusesocialmediasites,theydevelop“adblindness,”tuningoutinformational

appealsthatdistractfromtheirprimaryfocus.(C)Startupbusinessestypicallyhavemoreyoungpeopleaspartoftheirworkforce.(D)Venturecapitalinvestorsareinterestedinreviewingdetailedfinancialstatementsofstartups

beforemakinginitialinvestments.

Questions22–31arebasedonthefollowingreadingselections.

Twoscientistspresenttheirviewsonvaccinations.

PASSAGE1

Smokingtobaccocausescancer.Itcausessmallcellandsquamouscellcancersofthelung,aswellasoralandnasopharyngeal

Line carcinoma,gastrointestinalcarcinoma,and(5) cancersofhalfadozenadditionaltissues.It

ishighlycorrelatedwiththesecondleadingcauseofdeathintheUnitedStates,andthedecisiontonotsmokereducesone’sriskoflungcancerbymorethanhalf.Thisinformation

(10) willsurprisenoone.Itis,therefore,bafflingthat1in5Americanscontinuestousetobacco.Yetthereisonebehaviorontherisethatisstillmorecontrarytohumanhealththansmoking—andperhapsmore

(15) inexplicableaswell,asitcannotbeexplainedawaybychemicaldependence,norbysocialcustom.Ispeak,ofcourse,ofthedecisiontonotvaccinateone’schildren.Thereisapparentlyabeliefamongthese

(20) erringindividualsthattheailmentsagainstwhichtheCentersforDiseaseControlrecommendswevaccinatearesomehowlessseriousthanwe,asasociety,havebeenledtobelieve.Thereisabelief,moreover,that

(25) becausethesediseasesareeasilypreventablethroughmodernmedicalscience,theymust,implicitly,beeasilytreatableaswell.Letusclarifythismatter.Notlongbeforeitwaseradicatedby

(30) vaccination,smallpoxviruserasedentireculturesontwocontinents,wherefatalityratesroseashighas90%.Forthosewhosurvived,itwasacauseofpermanent,oftendebilitatingdisfigurement.PriortotheHiB

(35) andDTaPvaccines,epiglottitiscausedbyHaemophilusinfluenzae,anddiphtheriacausedbyCorynebacteriumdiptheriaewerebothexceedinglycommoncausesofdeathinyoungchildren,largelybecauseoftheir

(40) tendencytodeveloprapidlyandobstructtheairway.Deathtypicallyoccurswellbeforemedicalattentioncanbeaccessed,andthus,inspiteofallourmodernmedicaladvances,thesediseasesremainjustasdireasthey

(45) weretwohundredyearsearlier.Wecouldcontinuethislist.Inthepast,

forinstance,rubellawasthemostcommoncauseofcongenitaldeafness,andmumpsamajorcauseofsterility.Childrenwho

(50) survivedthemeasles,meanwhile,ranandstillruntheriskofthevirusreemergingyearslaterassubacutesclerosingpanencephalitis,afrequentlyfatalinfectionofthebrain.Thepoint,however,isclear.Vaccinationisneither

(55) aconspiracy,normerelyamatterofmodernconvenience;norisitentirelywithoutrisk.Butvaccination,plainlyput,istheonlyeffectivemedicalinterventiontosafeguardourchildrenfromsomeofthedeadliestandmost

(60) virulentdiseasesknowntoman.

PASSAGE2

Letusdispenseentirelywiththefallaciesandthedelusions;thereisnotananalyticallycrediblesourceontheplanetthatwilldefendalinkbetweenvaccinesandautismspectrum

(65) disorder.Butformanyconscientiousparentswhochoosenottovaccinatetheirchildren,thedecisionisbasednotondelusion,butuponasimple,mathematicalreality.Philosophically,medicineispremisedona

(70) balancebetweenbeneficence,andnonmaleficence.Thatistosay,foramedicalinterventiontobedeemedethicalandappropriate,therisksofnottreatinganindividualmustalwaysoutweightherisksinherentinthe

(75) treatmentitself.Riskaccompanieseverymedicalintervention,andvaccinationisnoexception.Specifically,inacertainsubsetofindividuals,exposuretoeitherthegelatin

oreggproteincomponentsusedtostabilize(80) vaccinescanresultinalife-threatening

anaphylacticreaction.Additionally,so-called“live-attenuatedvaccines”,includingthoseforvaricella,rotavirus,andMMR,containlivingstrainsoftheircorrespondingpathogens

(85) that,throughgeneticmodification,havebeenrenderedpathologicallyinert.Rarely,theseattenuatedstrainsmayundergomutation,andrevertbacktotheirpathogenicforms.Immunocompromisedindividuals,

(90) suchasthoseinfectedwithHIV,areparticularlyatriskforthisdangerousoutcome.Whileitistruethatadversevaccine

eventsareuncommon,itisequallytruethat,withintheAmericanpopulation,most

(95) diseasesagainstwhichvaccinationsprotectareuncommon—moreuncommon,infact,thantheincidenceofadversevaccineevents.Thus,solongasthemajorityoftheAmericanpopulationremainsvaccinated,anunvaccinated

(100) individualwillbewell-protectedthroughaphenomenondescribedas“herdimmunity.”Thedecisionnottovaccinate,therefore,hangsuponanappreciationofthedynamicbalancebetweenbeneficenceand

(105) nonmaleficenceasitpertainstotheindividual.Whilesomemayarguethattakingadvantageofsuchastrategyviolatesanethicalobligationtosociety,onecannothelpbutwonder:whenpresentedwiththenumbers,

(110) willtheseindividualstrulyvaluethehealthofsocietyabovethatoftheirchildren?Theirmath,Iwouldwager,justdoesn’taddup.

22.Withrespecttotheauthor’sargumentasawhole,lines1–12(“Smoking...tobacco”)moststronglyserveto(A)showtheobviousabsurdityofapersonaldecisiontosetuptheargumentthatfollows.(B)highlighttheprimarytopicoftheessay.(C)providekeystatisticsonbothtobaccoandvaccinationresearch.(D)illustratetheirrationalityofwidespreadtobaccousegiventhelatestscientificresearch.

23.Theprimarypurposeoflines19–27(“Thereis...aswell”)isto(A)citescholarlyevidenceinsupportoftheauthor’sthesis.(B)presentthealternativeviewsthattheauthorwilllaterdissect.(C)underscoretheauthor’sfundamentalrespectforopposingviewpoints.(D)giveavitalclarificationtotheauthor’sargument.

24.Asusedinline30,theword“erased”mostcloselymeans(A)transported.(B)eliminated.(C)fought.(D)affected.

25.TheauthorofPassage2wouldmostlikelyagreethatanunvaccinatedindividualwouldbemostlikelytosurvive(A)undernocircumstances.(B)ifheorshetookpreventativemeasuresbasedonalternativemedicine.(C)ifheorshewerecarefultoremainimmunocompromised.(D)inasocietywherevirtuallyeveryoneelseisvaccinated.

26.Whichoptiongivesthebestevidencefortheanswertothepreviousquestion?(A)Lines61–65(“Letus...disorder”)(B)Lines77–81(“Specifically...reaction”)(C)Lines87–91(“Rarely...outcome”)(D)Lines98–102(“Thus...immunity”)

27.Asusedinline61,thephrase“dispenseentirelywith”mostcloselymeans(A)fundamentallyunderstand.(B)physicallyremove.(C)somewhatignore.(D)doawaywith.

28.Whatistheoverallrelationshipbetweenthetwopassages?(A)Passage1stronglydisagreeswiththetoleranceofnon-vaccinationinPassage2.(B)Passage1usesamorescientificapproachwhilePassage2ismoremathematical.(C)Passage2advocatescarefulcost-benefitanalysiswhilePassage1advocatesdecisivepolicies.(D)Passage2attemptstoexplainaphenomenonthatPassage1deemsinexplicable.

29.TheauthorofPassage2wouldmostlikelyrespondtothefinalsentenceofPassage1(lines57–60)bystatingthatit(A)contradictswidespreadthinking.(B)needsanimportantqualification.(C)isoverlyinfluencedbypopularopinion.(D)isinsufficientlyparanoid.

30.Whichoptiongivesthebestevidencefortheanswertothepreviousquestion?(A)Lines65–68(“Butfor...reality”)(B)Lines71–75(“Thatis...itself”)(C)Lines81–86(“Additionally...inert”)(D)Lines92–97(“Whileit...events”)

31.TherespectiveattitudesoftheauthorsofPassage1andPassage2towardparentswhochoosenottovaccinatetheirchildrenarebestdescribedas

(A)hatredandappreciation.(B)forgivenessandintolerance.(C)loathingandgratitude.(D)contemptandunderstanding.

Questions32–41arebasedonthefollowingreadingselection.

In 1862, RalphWaldo Emerson delivered the excerpt below as part of a lecture called “AmericanCivilization”attheSmithsonianInstitutioninWashington,D.C.

AtthismomentinAmericatheaspectsofpoliticalsocietyabsorbattention.Ineveryhouse,fromCanadatotheGulf,thechildren

Line asktheseriousfather,—“Whatisthenewsof(5) thewartoday?andwhenwilltherebebetter

times?”Theboyshavenonewclothes,nogifts,nojourneys;thegirlsmustgowithoutnewbonnets;boysandgirlsfindtheireducation,thisyear,lessliberalandcomplete.All

(10) thelittlehopesthatheretoforemadetheyearpleasantaredeferred.Thestateofthecountryfillsuswithanxietyandsternduties.Wehaveattemptedtoholdtogethertwostatesofcivilization:ahigherstate,wherelaborand

(15) thetenureoflandandtherightofsuffragearedemocratical;andalowerstate,inwhichtheoldmilitarytenureofprisonersorslaves,andofpowerandlandinafewhands,makesanoligarchy:wehaveattemptedtohold

(20) thesetwostatesofsocietyunderonelaw.Buttherudeandearlystateofsocietydoesnotworkwellwiththelater,nay,worksbadly,andhaspoisonedpolitics,publicmoralsandsocialintercourseintheRepublic,nowfor

(25) manyyears.Thetimesputthisquestion,—Whycannot

thebestcivilizationbeextendedoverthewholecountry,sincethedisorderofthelesscivilizedportionmenacestheexistenceof

(30) thecountry?Isthissecularprogresswehavedescribed,thisevolutionofmantothehighestpowers,onlytogivehimsensibility,andnottobringdutieswithit?Ishenottomakehisknowledgepractical?tostandandto

(35) withstand?Isnotcivilizationheroicalso?Isitnotforaction?hasitnotawill?…AmericaisanotherwordforOpportunity.Ourwhole

historyappearslikealasteffortoftheDivineProvidenceonbehalfofthehumanrace;

(40) andaliteralslavishfollowingofprecedents,asbyajusticeofthepeace,isnotforthosewhoatthishourleadthedestiniesofthispeople.Theevilyoucontendwithhastakenalarmingproportions,andyoustillcontent

(45) yourselfwithparryingtheblowsitaims,but,asifenchanted,abstainfromstrikingatthecause.[…]Emancipationisthedemandofcivilization.

Thatisaprinciple;everythingelseisan(50) intrigue.Thisisaprogressivepolicy;—puts

thewholepeopleinhealthy,productive,amiableposition,—putseverymanintheSouthinjustandnaturalrelationswitheverymanintheNorth,laborerwithlaborer.

(55) ThepowerofEmancipationisthis,thatitalterstheatomicsocialconstitutionoftheSouthernpeople.Nowtheirinterestisinkeepingoutwhitelabor;then,whentheymustpaywages,theirinterestwillbetoletit

(60) in,togetthebestlabor,and,iftheyfeartheirblacks,toinviteIrish,German,andAmericanlaborers.Thus,whilstSlaverymakesandkeepsdisunion,Emancipationremovesthewholeobjectiontounion.Emancipation

(65) atonestrokeelevatesthepoorwhiteoftheSouth,andidentifieshisinterestwiththatoftheNorthernlaborer.[…]Theendofallpoliticalstruggleistoestablish

moralityasthebasisofalllegislation.It(70) isnotfreeinstitutions,‘tisnotarepublic,‘tis

notademocracy,thatistheend,—no,butonlythemeans.Moralityistheobjectofgovernment.Wewantastateofthingsinwhichcrimeshallnotpay.Thisistheconsolation

(75) onwhichwerestinthedarknessofthefutureandtheafflictionsoftoday,thatthegovernmentoftheworldismoral,anddoesforeverdestroywhatisnot.

32.Whatisthegeneralpurposeofthispassage?(A)Toargueinfavorofemancipation(B)Toillustratetheevilsofslavery(C)TohighlighttheeconomicproblemsintheSouth(D)Toarticulateacomprehensivetheoryofmorality

33.WhatisEmerson’soverallattitudetowardpolitics?(A)Itspracticeishisgreatpassion(B)Itismerelyameanstoanend(C)Ittakesonlessimportanceinwar(D)Itwillultimatelysettlethequestionofslavery

34.Whichoptiongivesthebestevidencefortheanswertothepreviousquestion?(A)Lines20–25(“Butthe...years”)(B)Lines37–39(“Our...race”)(C)Lines62–64(“Thus...union”)(D)Lines69–72(“Itis...means”)

35.Lines6–9(“Theboys...complete”)mostclearlyindicatethat(A)interestsinchildhoodentertainmentchangedgreatly.(B)warledtoafocusonthebarenecessities.(C)childrenbecamelessinterestedinstimulation.(D)emancipationwasnotapriorityfortheyoung.

36.Asusedinline12,theword“stern”mostcloselymeans(A)playful.(B)terrifying.(C)serious.(D)pointless.

37.Whatisthepurposeofthequestionsinlines26–36?(A)Torhetoricallyemphasizeaneedforaction(B)Toencouragereaders’intellectualcuriosity(C)Tocritiquethereasoninggiveninthepreviousparagraph(D)TosummarizethethoughtsofEmerson’smentor

38.Emersonframesthephilosophicalstruggleunderlyingthenation’sconflictasonebetween(A)geographicalneedsversusinternationalrecognition.(B)monetaryconcernsversuspoliticalconsiderations.(C)traditionalprecedentversusmoralnecessity.(D)skepticalquestioningversuspiousobedience.

39.Whichoptiongivesthebestevidencefortheanswertothepreviousquestion?(A)Lines1–6(“Atthis...times”)(B)Lines9–11(“All...deferred”)(C)Lines37–43(“Ourwhole...people”)(D)Lines57–62(“Now...laborers”)

40.Asusedinline40,theword“following”mostcloselymeans(A)ensuing.(B)succeeding.(C)resulting.(D)obeying.

41.Whatoptionbestsummarizestheparagraphinlines56–67?(A)EmancipationwillleadtothepoliticaldominanceofEuropeanimmigrants.(B)Emancipationwillensureanequitableredistributionofincomeacrosstheraces.(C)AnendtoslaverywillcausetheSouthernConstitutiontobeamended.(D)AnendtoslaverywillrealignSoutherneconomicinterestsinfavorofunion.

Questions42–52arebasedonthefollowingreadingselectionandaccompanyingmaterial.

SearchingtheSkies

In1950,EnricoFermipositedthequestion,“Whereiseverybody?”whenconsideringtheapparentcontradictionbetweenhigh

Line estimatesofthelikelihoodoftheexistence(5) ofextraterrestriallifeandmankind’slackof

contactwith,orevidencefor,suchcivilizations.LaterreferredtoastheFermiParadox,hisprovocativequerywasfoundedontheassumptionthatsincetheSunisquitetypical,

(10) otherEarth-likeplanetssurelyexistandhaveintelligentlife,andbynow,shouldhavevisitedorcontactedEarth.Extraterrestrialintelligence,orETI,referstohypotheticalintelligentcivilizationsthatareassumedto

(15) existbasedontheexistenceofhumanintelligenceandthevastsizeoftheuniverse.WhilepopularandscientificopiniononETIvariesgreatly—fromcertaintytoskepticismtodownrightincredulity—thesearchforalien

(20) intelligenceisextensiveandsubstantive.Whetheryouanticipatethestringyworm

guyswithseriousfirepowerinMenInBlack;theeternallywiseJediMaster,Yoda;StanWinston’snightmarishpredators;orSteven

(25) Spielberg’ssweet-lovingE.T.;thesearchforintelligentlifeoutsideEarthison.SETI,or“thesearchforextraterrestrialintelligence,”isthecollectivenameforactivitiesundertakentoseekintelligentextraterrestrial

(30) life,andmostrecentlyinvolvesconstantmonitoringofelectromagneticradiationwithradiotelescopesinhopesofdetectingnon-naturalradioemissionsorothersignsoftransmissionsfromcivilizationsonother

(35) worlds.InMarch2014,UCBerkeleybegananall-skysurveyusingtheAreciboradiotelescope.

Althoughwehavebeenlisteningformessagessincethe1960s,therehavealsobeen

(40) recenteffortstocommunicatewithandpurposelysendoutourownmessages.ActiveSETIistheattempttosendmessagestointelligentextraterrestriallifeviaradiosignals.CETI,ontheotherhand,isanynumberof

(45) effortstocommunicatewithETIthatfocusesoncomposinganddecipheringmessagesthat,theoretically,couldbeunderstoodbyanothertechnologicalcivilization.AndthepursuitofETIcontactisnolongerlimitedto

(50) thefewandfarbetween.SETILive,launchedinFebruary2012,usesdatafromtheAllenTelescopeArraytoallowthepublictosearchradiosignalsthemselves.Manyastronomersandphysicistsattribute

(55) therenewedeffortstoestablishcontactwithaliencivilizationstothepresent-dayescalationinthediscoveryofexoplanets,orplanetsthatorbitastarotherthanourSun.AccordingtoNASA’sdata,asofJune2015,

(60) therehavebeen1,838confirmedexoplanets,where,just20yearsago,itseemedthatoursolarsystemwasdestinedtobetheextentofourplanetarydiscovery.AsignificantpartofthatescalationcanbeattributedtoNASA’s

(65) KeplerMission,anunmannedspaceobservatorycraftlaunchedin2009tofindEarthsizedandsmallerplanetsorbitingotherstars.Morethan800systemslikeourownsolarsystemwithstarsandorbitingplanetshave

(70) beenidentified.Sowhyisit,witharejuvenatedeffortto

findevidenceofextraterrestrialintelligence,thatFermi’squestionisstillsopertinent?Despitebillionsofdollarsandyearsof

(75) research,SETIhasnothingsubstantialtoshowforitself.Infact,theclosestthingtoETIcontactistheWow!Signal:astrongnarrowbandradiosignaldetectedin1977byJerryR.EhmanofOhioStateUniversity’s

(80) BigEarradiotelescopeproject.Ehmanwasabletosuccessfullyobservethesignalfora72-secondwindow,circlingitsnon-naturalwavesandwriting“Wow!”nexttoit—hisenthusiasmledtoitsname,butnottoany

(85) significantbreakthrough.Since1977,efforts

torelocatethesignalhavefailedagainandagain.ThetheoreticalexplanationsforFermi’s

paradoxdiffergreatly.Somesimplybelieve(90) thatfew,ifany,othercivilizationsexist.The

RareEarthHypothesissuggeststhatEarthisunique,andso,therefore,isintelligentlife.Otherstheorizethatintelligentlifehasatendencyofdestroyingitselfquickly;they

(95) hypothesizethatself-annihilationoccursbeforecontactcanbemade.Ontheotherhand,manypostulatethatETI’sdoexist,butweseenoevidenceforavarietyofreasons.Perhapswearetoofarapartinspaceortime.

(100) Perhapshumans,arelativelynewspecies,haven’tsearchedlongenough.Ormaybewearen’tlisteningproperly.Whatifourdistantneighborsareusingdifferentfrequencies?Regardlessofhowcertainoruncertain

(105) youarethatextraterrestrialintelligentlifeexists,theventuretosolveFermi’sParadoxisprevailing,andmanybelievethestakesarehigh.Somearguethattheenormousexpenseinvolvedinsuchprojectsisonlysurpassed

(110) bythefutilityofseekingalienswhenwehavehaddecadeswithoutsuccess;but,otherscounterthatthediscoveryof1,838exoplanetsishardlyunsuccessful.

NASASpaceProgramBudget

42.Whatisthepurposeofthispassage?

(A)Toadvocateforaparticularcourseoftechnologicalaction(B)Tosurveyvariousattemptstoresolveascientificdilemma(C)Todetailtheestablishedconsensusonaninterestingproblem(D)Toconsideralternativeapproachestoasocialissue

43.Accordingtothepassage,thegeneralscientificattitudetowardtheexistenceofextraterrestrialintelligenceisbestdescribedas(A)deeplypassionateandmostlycertain.(B)quiteinterestedbutcurrentlyunsettled.(C)somewhatpessimisticandratherfearful.(D)fundamentallyskepticalbutalwaysdogmatic.

44.Whichoptiongivesthebestevidencefortheanswertothepreviousquestion?(A)Lines1–7(“In1950...civilizations”)(B)Lines16–20(“While...substantive”)(C)Lines21–26(“Whether...on”)(D)Lines38–41(“Although...messages”)

45.WhichofthefollowingisthebestparaphraseoftheFermiParadox(line7)?(A)“Itseemsreasonablethatthereshouldbeextraterrestrialintelligence,sowhyhaven’twefound

it?”(B)“Itiscontradictorythatthereisbothconcreteevidenceinfavorofalienlifeanddirectevidence

againsttheirexistence.”(C)“Itisclearthatalienshavemadecontact,sowhywon’tthemajorityofhumanityacceptthis

obvioustruth?”(D)“Theuniverseissooldandlargethatthereshouldbeextraterrestrialintelligence;whywon’t

scientistsmakeanefforttolocateit?”

46.Asusedinline19,theword“incredulity”mostcloselymeans(A)curiosity.(B)anticipation.(C)wisdom.(D)disbelief.

47.Theexampleinlines72–83primarilyservestodemonstratethat(A)thereisdecisiveevidenceinfavorofalienlife.(B)thesearchforextraterrestrialintelligencehasbeenvirtuallyfruitless.(C)scientistsareredoublingtheireffortstobuildonEhman’sdiscovery.(D)astronomersarenotablefortheenthusiasmwithwhichtheyconducttheirobservations.

48.Thepurposeoflines99–103(“Perhapswe...frequencies”)isto(A)explainwhytheauthorisquitepessimisticaboutthepossibilityoffindingETI.(B)offersuggestionsthathavenotlikelybeenconsideredbyscientists.(C)givearecommendedcourseofactiontosolveascientificproblem.(D)elaborateonpossiblereasonswhywehavenotfoundevidenceofETI.

49.Asusedinline106,theword“venture”mostcloselymeans

(A)endeavor.(B)risk.(C)business.(D)speculation.

50.Basedonthetrendsinthegraphandtheinformationinthepassage,whichofthesebestrepresentsalogicalnextstepinthesearchforextraterrestrialintelligence?(A)Amannedspacestationthatwillmonitorradiowaves(B)Ananalysisofpreviousextraterrestrialcommunications(C)AnexploratorymissiontoanEarth-likeexoplanet(D)Aspaceshuttledeliveryofanadvancedouter-spacetelescope

51.Accordingtothegraph,betweentheyears2008–2015,thegrowthinwhichofthesebudgetcomponentsmostcloselymirroredthegrowthofNASA’sentirebudget?(A)ExplorationMissions(B)RoboticTechnology(C)Spaceshuttles/Stations(D)Aeronautics

52.SupposethatascientistwantsevidencethatwouldsupportNASA’sfundingdecisionswithrespecttospacestationsandspaceshuttlesasoutlinedinthegraph.Whichoptiongivesthebestevidencefromthepassage?(A)Lines38–41(“Although...messages”)(B)Lines50–53(“SETILive...themselves”)(C)Lines63–70(“Asignificant...identified”)(D)Lines76–80(“Infact...project”)

Ifyoufinishbeforetimeiscalled,youmaycheckyourworkonthissectiononly.Donotturntoanyothersection.

SCORINGAPPROXIMATIONThistablegivesyouanestimateofhowyourperformanceontheReadingsectionwillcontributetoyouroverallEvidence-BasedReadingandWritingscore.Keepinmindthateachtestwillbecurved,makingthe number of questions needed for a particular score dependent on the test that day. This is the bestestimate we can give you based on (1) previous SAT curves and (2) the fact that guessing is nowpermittedontheSAT.

Questionsoutof52answeredcorrectlyEstimatedoverallsectionscore(between

200–800)

800

52

750

49

700

46

650

43

600

40

550

37

500

33

450

30

400

26

350

20

300

13

250

7

200

0

PracticeTest3

65MINUTES,52QUESTIONS

Directions:Eachpassageorpairofpassagesisaccompaniedbyseveralquestions.Afterreadingthepassage(s),choose thebestanswer toeachquestionbasedonwhat is indicatedexplicitlyorimplicitlyinthepassage(s)orintheassociatedgraphics.

Questions1–10arebasedonthefollowingreadingselection.

Finnegan—Ashortstory

DavidBensonwasatimidboyborntoparentswhohadlongsincestoppedworryingabouthavingchildren.WillieandLouisehad

Line threegrowngirls,theyoungestwasnineteen(5) andleavingtocosmetologyschooltheyear

Davidwasborn.Louisehadsuspectedanarsenalofhealthissuesbeforerealizingshewaswithchild,andeventhen,shewaitedanotherthreeweekstotellherunsuspecting

(10) husband.Longago,whenhehadjusttakenover

thefarmandhisbodywasstrongandhisdreamswereconsiderable,Williehadwantedasonmorethananythingelse.Hehadhopes

(15) ofexpandinghisproperty,becomingarichman,andgainingrespectintown—thesewereallthingsthatnevercamewhichheplannedtopassontothesonwhocamefartoolate.BythetimeDavidwasborn,more

(20) thanhalfthefarmhadbeendividedandsold,andWilliekeptonlyahandfulofhiredhandstotendhissmallshareofthelandwhilehedrovesemi-trucksfiftyhoursaweektosupplementhismeagerincome.

(25) Davidwasanoversensitive,misunderstoodboy.Helearnedquicklythathismotherwasfartootiredtolovehimasshehadthegirls.Hisfather,oftenabsent,seemeddistantandbegrudgingaroundDavid.Soitwas

(30) thatDavid,fromtheageoffouron,oftenwanderedthefarmalone,contemplatingtheburlypigsorcollectingberriesandnutsorsimplydoinghisbesttoavoidthesinistersilencethatwashome.

(35) WhenDavidwasten,hefeltthathewouldhavehisfirstadventure.Mr.Harding,anold

friendofhisfatherwhohadaboyjusttwoyearsolderthanDavid,offeredtoletDavidstaywiththemintown.It’dbeeasieron

(40) Louiseifshedidn’thavetodrivetheboytoandfromschool,andDavidwouldbehappierhavingMichaeltokeephimcompany.DavidcouldspendweekendsonthefarmandhelpWilliewiththechores,ofcourse.

(45) Forthefirsttimeinhislife,Davidfeltthatwhateverhewasmissinghewassuretofind;whateverwasthevoidthatlingeredaboutthefarmitwassuretodisappearintown.ButwhateveritwasthatDavidwaslooking

(50) for,hedidn’tfinditwiththeHardings.Mr.HardingwasaboisterousbankerwhofoundDavid’sshynessunbearable.Mrs.Hardingwasaratherlargewomanwhospentninetenthsofherdaycookingandgrewsolemn

(55) whenDavidrefusedsecondhelpings.AndinMichael,Davidfoundonlyacombativestrangerwhoheldasingularinterest:ridinghisbicyclearoundthetownsquareinsearchofthecovetedSaraRidenour.

(60) Davidhadjustturnedfourteenwhenwalkingbackfromschool,heheardthepatheticwhimper.Thepuppywassmall—obviouslymalnourishedandfeeble—andmuchtooyoungtobeawayfromitsmother.David

(65) removedhisjacketandcoddledthepupagainsthischestashewalkedbrisklybacktotheHardingsrehearsingwhathemightsay.Tohissurprise,Mr.Harding’sonlyrequestswerethatDavidkeepthefrailanimalinhis

(70) ownroomanddisposeofitonceitwaseitherhealthyenoughtoliveonitsownordead.Theseconditionsseemedquitefairto

David;anddayandnight,hededicatedhimselftorestoringthehealthofFinnegan,

(75) theboy’sveryfirstfriend.Thepuppysleptonhischest,suckledmilkfromabottle,andmoanedsoftlywhenDavidwipedhisfailingbody.Intheend,David’sdevotionwasnotnearlyenough,andjustsixdaysafterhis

(80) rescue,thepuppy’sunderdevelopedorgansfailedhim.David’sanguishwaspalpable,andMrs.Harding,inararemomentofcompassion,suggestedthatMichaelhelpDavidbury

thepooranimalinapropermanner.(85) Ahalf-mileintothewoodsonthewest

sideoftown,MichaeldugaholewhileDavidweptinconsolably,clutchingthetinyshoeboxmadecoffin.Whentheholewasplentydeep,Michael,embarrassed,excusedhimself

(90) toallowDavidaminutealonetodisposeofhisbelovedcompanion.Afterfifteenminutes—longafterMichaelhadexpectedDavidtotrudgebackoutofthewoods,muddyandsobbing—Michaelwalkedannoyinglybackto

(95) theburialsite.HeplannedontellingDavidfranklythatthiswasnowaytoactaboutasillydog.Instead,MichaelfoundtheholestillemptyandDavidnowheretobefound.Later,thepolicewouldmakehimrepeatthestory

(100) againandagain.

1.Whichchoicebestsummarizesthepassage?(A)Aboyisupsetoverhisdog’sdeathandgoesmissing.(B)Twoparentscontemplatetheirregretsandunfilleddreams.(C)Amisunderstoodboyfindspurposeandjoyonlytoloseit.(D)Itillustratesthedifferencesbetweenlifeonafarmandlifeintown.

2.Theprimarypurposeofthefirstsentenceofthepassageisto(A)demonstratetheagedifferencebetweenDavidandhissiblings.(B)scientificallyexplainhowDavidwasgeneticallypredisposedtowardshavingamoreintroverted

personality.(C)giveareasonforwhyDavidwenttolivewiththeHardingfamily.(D)giveinsightintoDavid’spersonalityandhisparents’stateofmind.

3.ThepassageindicatesthatwhenDavidwenttolivewiththeHardings,hethoughtMichaelwas(A)helpfulandfriendly.(B)distantandquarrelsome.(C)embarrassedandunemotional.(D)athleticandobservant.

4.Whichoptiongivesthebestevidencefortheanswertothepreviousquestion?(A)Lines39–42(“It’dbe...company”)(B)Lines55–59(“Andin...Ridenour”)(C)Lines81–84(“David’s...manner”)(D)Lines85–88(“Ahalf-mile...coffin”)

5.Inline65“coddled”mostnearlymeans(A)spoiled.(B)humored.(C)cosseted.

(D)indulged.

6.Theprimarypurposeofparagraphssixandseven(lines60–84)isto(A)demonstratethatMr.Hardingisfairandjust.(B)provideajustificationforDavid’s“oversensitive”mindset.(C)givereasonsastowhyDavidissoupsetafterthepuppy’sdeath.(D)showthatDavidinitiallymisjudgedMrs.Harding’spersonality.

7.Basedontheinformationinthepassage,whatisthebestdescriptionofwhatDavidwaslookingforatthebeginningofparagraph5(lines49–59)?(A)Ahomeintown(B)Anewfriend(C)Aprosperousfamily(D)Apetofhisown

8.Whichoptiongivesthebestevidencefortheanswertothepreviousquestion?(A)Lines45–48(“Forthe...town”)(B)Lines49–52(“But...unbearable”)(C)Lines68–71(“Tohis...dead”)(D)Lines72–75(“These...friend”)

9.TheinformationinparagraphtwomostclearlyimpliesthatWillie(A)isamuchbettersemi-truckdriverthanafarmer.(B)soldtoomuchofhisfarmtopassanytohisfamily.(C)wouldhaveaccomplishedhisdreamsifDavidwasbornearlier.(D)hasnotbeensuccessfulinexpandingthefarm’ssize.

10.Asusedinline81,theword“palpable”mostcloselymeans(A)concealed.(B)credible.(C)noticeable.(D)believable.

Questions11–20arebasedonthefollowingreadingselectionandaccompanyingmaterial.

ADemocraticDuel

Ifoneweretosetouttoformanationbasedondemocraticprinciples,therewouldbeessentiallytwopathstotake:presidential

Line orparliamentary.Bothholdtheirown(5) intermsofadvantagesanddisadvantages,

andbothpossesstheendorsementofgreatprosperousnations.Parliamentaryisthefarmorecommonorder,butmanyattributeitsprevalencetothelegacyoftheBritishEmpire

(10) ratherthantoitssuperiority.

Thedisparitybetweenthetwoishardlysubtle.Inapresidentialsystem,theexecutiveandlegislativebranchesofgovernmentarecompletelyindependentofoneanother,

(15) suchasintheUnitedStatesofAmerica.ThePresident,electeddirectlybythepeople,isanationalfigurethatisatoncetheheadofgovernmentandstate,butisseparateanddistinctfromCongress,thelawmaking

(20) body.Ontheotherhand,aparliamentarysystemisafusionofexecutiveandlegislativepowerswiththeexecutive,mostoftencalledPrimeMinister,beingamemberofParliament.Inthelatterarrangement,members

(25) ofParliament,thelegislativeassembly,areelectedbythepeople,butthenchooseamongstthemselvesthemostfittobeexecutive.Mostoften,amonarch,likeinGreatBritain,isgiventheresponsibilityofheading

(30) thestateandbeingtheiconofnationalceremony.Otherdivergenceoccursinthetermlimits

andstandardsofaccountabilityinwhichtheexecutiveisheldto.Forinstance,the

(35) presidentialmodelallowsforfixedtermsandscheduledelections.Assuch,thePresidentenjoystheassuranceofasecureterminwhichheorshecanworktoestablishandmeetgoalsforthenation’sadvance.Onlyin

(40) situationswherethePresidentisfoundguiltyofseriouscrimeswillheorsheberemovedfromofficebeforetheendoftheterm.Conversely,thePrimeMinisterissubjecttomuchmorescrutinyandjobinsecurity.

(45) LegislatureswithinthismodelofgovernmentareexpectedtoquestionthePrimeMinisterdirectlyonaweeklybasisandareabletoremovetheexecutiveanytimeconfidenceislostinhisorherability,character,orjudgment.

(50) Hence,parliamentarysystemsaresubjecttorandomelectionsthatcanmoreeasilyreplaceanunfitleader.Advocatesofthepresidentialmodelposit

thatitismoredemocraticbecausethepeople(55) themselveschoosetheirexecutive.Theycontend

thatthefixedtermsevenallowforsomestabilitythattheparliamentaryorganization

doesnotallow.Likewise,afurtherbenefitliesintheseparationofpowerssubjectto

(60) checksandbalances,inwhichtheexecutiveandlegislativebodiesareabletomonitoroneanotherandassurethatpowerisnotcentralized.Yet,criticsofthesystemallowthatthePresident’snationalstatusaffordstendencies

(65) towardauthoritarianism.Furthermore,asarule,deadlocksorstalematesaremuchmorecommonwithinasystemthatoftenhasexecutiveandlegislativebodiesunderthecontrolofdifferentparties.Cynicstherefore

(70) suggestthatthepresidentialmodelisprivytodiscordandinefficiency—nottomention,presidentsaredifficulttoremovewhenthoughttobeunfit.Thoseinfavoroftheparliamentarymodel

(75) believethatthefusedgovernmentallowsforunityandharmonythatisallbutalienwithintheformer.Notonlyisitfasterandeasiertopasslegislation,butmoreoftenthannot,thegovernmentoperatesmorecooperatively.

(80) SinceitisveryunlikelythattherulingpartywillchooseaPrimeMinisterfromanotherparty,theexecutiveandlegislativebodiesarealmostalwaysworkinginaccord.Moreover,whenaPrimeMinisterisdecidedunfit,heor

(85) shecanberemovedandreplacedrightaway.Thissystem,theyargue,ismuchmoreefficientandlesspronetocorruption.Yet,noteveryoneagrees.Withthelegislatureholdingsupremepower,criticscontendthat“tyranny

(90) ofthemajority”isalltoolikely.Notonlyarethepeoplenotdirectlyelectingtheirleader,butthereisalsonobodytoopposeorvetolegislationpassedbyParliament.Theminoritypartieshavevirtuallynosayandtheruling

(95) partyoftheassemblycaneasilymanipulatewhenthePrimeMinisterwillbereplacedandwhenelectionswillbeheld.Thetwosystemsrepresentverydifferent

approachestodemocracy,andassuch,operate(100) so.Onemustconsiderbothdemocratic

idealsandeffectivenessindecidingthebestwaytoselectaleader.Whilepresidentialelectionscanbecomefeeblepopularitycontests,centralizedpowerisarealconcern.

11.Whatstatementbestsummarizesthepassage?(A)Asurveyoftwogovernmentalformsthatfocusesprimarilyonthedifferencesbetweenthem(B)Asurveyoftwogovernmentalformsthatfocusesprimarilyonthesimilaritiesbetweenthem(C)Ananalysisofwhetheraparliamentaryorpresidentialsystemisabetterfitforaparticular

country(D)Ananalysisofwhetheraparliamentaryorpresidentialsystemisabetterfitforseveraldifferent

countries

12.Theapproachoftheauthorisbestdescribedas(A)neutralandpessimistic.(B)passionateandstrong-willed.(C)analyticalandobjective.(D)technicalandpredisposed.

13.Theauthorstatesthatapossiblereasonforthegreaterpracticeofparliamentarythanpresidentialdemocracyis(A)majorityrule.(B)historicalinheritance.(C)legislative-executiveunity.(D)centralizedauthority.

14.Whichoptiongivesthebestevidencefortheanswertothepreviousquestion?(A)Lines7–10(“Parliamentary...superiority”)(B)Lines20–24(“Onthe...parliament”)(C)Lines58–63(“Likewise...centralized”)(D)Lines88–93(“Withthe...Parliament”)

15.SupposeacountrywithaparliamentarysystemandacountrywithapresidentialsystemwerechoosingrepresentativestoaglobalsportscompetitionliketheOlympics.Basedontheparagraphinlines11–31,whichrespectivegovernmentalofficialsfromtheparliamentaryandthepresidentialsystemwouldbethemostdesirableandfittingrepresentatives?(A)PrimeMinister,Congressperson(B)Judge,MilitaryGeneral(C)King,President(D)Queen,Senator

16.Asusedinline39,theword“advance”mostcloselymeans(A)payment.(B)progress.(C)spread.(D)increase.

17.Theparagraphinlines53–73primarilyservesto(A)exploretheprosandconsofthepresidentialmodel.(B)compareandcontrastthepresidentialandparliamentarymodels.(C)highlightthesuperioraspectsofthepresidentialmodel.(D)givespecificexamplesofcountriesthatpracticepresidentialpolitics.

18.Asusedinline70,theword“privy”mostcloselymeans(A)susceptible.(B)privileged.(C)concealed.(D)open.

19.Itismostreasonabletoinferthatthoseconcernedabouta“tyrannyofthemajority,”asdescribedinlines89–90,areafraidthatwhatwouldbelikelytooccurinsuchasituation?(A)Adictatorwillemerge.(B)Therightsofmostcitizenswillberespected.(C)Governmentwillstopfunctioningefficiently.(D)Minorityinterestswillbeignored.

20.Whichoptiongivesthebestevidencefortheanswertothepreviousquestion?(A)Lines45–50(“Legislatures...judgment”)(B)Lines80–85(“Sinceit...away”)(C)Lines93–97(“Theminority...held”)(D)Lines98–104(“Thetwo...concern”)

Questions21–30arebasedonthefollowingreadingselectionandaccompanyingmaterial.

Humanity’sCode

Aproteinisalarge,complexmacromoleculecomposedofoneormorelongchainsofaminoacids.Proteinsare15–25%nitrogen

Line andanequalamountofoxygen,andare(5) presentinandvitaltoeverylivingcell.They

areessentialforthestructure,function,andregulationofthebody’stissuesandorgans.Asamatteroffact,proteinsholdtogether,protect,andprovidestructuretothebodyof

(10) amulti-celledorganism.Furthermore,theyareresponsibleforcatalyzingandregulatingthebodychemistry.Yet,beforeFrederickSanger—oneofonlytwopeopletoeverreceivetwoNobelPrizesinthesamecategory—

(15) littlewasknownaboutproteinsandthesequenceoftheiraminoacidchains.FrederickSangergraduatedwithadoctorate

inbiochemistryfromSt.John’sCollegein1943,wherehehadspentthreeyears

(20) researchingthemetabolismoftheaminoacidlysine.Yet,itwasn’tuntilhisworkwithinsulinthatSangerdifferentiatedhimselfinthefieldofchemistry.Hisfirsttrueaccomplishmentoccurredwhenhesuccessfully

(25) determinedthecompleteaminoacidsequenceofthetwopolypeptidechainsofbovineinsulinAandBintheearly1950s.Hisresearchprovedthatproteinshaveadefinedchemicalcomposition,andheultimately

(30) concludedthateveryproteinhadauniquesequence.In1958,SangerwasawardedtheNobelPrizeinChemistryforshowinghowaminoacidslinktogethertoforminsulin,and,therefore,providingthetoolsforscientists

(35) toanalyzeanyproteininthebody.Muchlater,afterhisretirement,hewoulddescribehimselfas“justachapwhomessedaboutinalab.”Fouryearslater,Sangertookapositionas

(40) theheadoftheProteinChemistryDivisionontheMedicalResearchCouncil,wherehebegantoworkonthesequencingofribonucleicacid.Hedevelopedmethodsforseparatingribonucleotidefragmentsgenerated

(45) withspecificnucleaseswhichtriggeredthediscoveryofformylmethioninetRNA,responsibleforinitiatingproteinsynthesisinbacteria.Yethisearlierworkwithinsulinhelpedhimtoformanddeliberateon

(50) ideasofhowDNAcodesforproteins.WhenheturnedtosequencingDNA—theblueprint-likemoleculethatcarriesthegeneticinstructionsforalllivingorganisms—SangercollaboratedwithAlanCoulsontopublish

(55) the“PlusandMinusTechnique,”asequencingprocedurehedevelopedtodeterminetheorderofthechemicalbasesadenine,thymine,guanine,andcytosinewhichspelloutthegeneticcodeforalllivingthings.

(60) Whenhedevisedamoreefficientmethodforreadingthemolecularlettersthatmakeupthegeneticcodein1977,hechristeneditthe“SangerMethod.”The“SangerMethod”allowslongstretchesofDNAtoberapidly

(65) andaccuratelysequenced,whichearnedhimhissecondNobelPrizeinChemistryin1980.Heemployedhisinventiontodecipherthesixteenthousandlettersofmitochondria.Moresignificantly,thismethodeventually

(70) allowedscientiststodecodethethreebillionlettersofthehumangeneticcode,giving

sciencetheabilitytodistinguishbetween

normalandabnormalgenes.Inthesameway,Sanger’sworkdirectlycontributedto

(75) thedevelopmentofbiotechnologydrugslikehumangrowthhormone.In1986,thecelebratedchemistaccepted

anOrderofMerit.Shortlyafter,hehelpedopentheSangerInstituteoutsideof

(80) Cambridge,whichisnowoneoftheworld’slargestgenomicresearchcenters.SangerdiedinNovember2013;hisobituarydocumentedhissuprememodestyinanautobiographicalaccountofhimselfas“academicallynot

(85) brilliant.”Atanyrate,Sanger’sresearchpromptedthedecodingofthehumangenome.

NobelPrizeWinnersasof2013

21.Theorganizationofthepassageis(A)somewhatchronological.(B)mostlychronological.(C)somewhatargumentativelysequenced.(D)mostlyargumentativelysequenced.

22.Asusedinline5,theword“vital”mostcloselymeans(A)vibrant.(B)essential.(C)biological.(D)dynamic.

23.Accordingtothepassage,Sanger’sattitudetowardhisownaccomplishmentscouldbestbedescribedas(A)humiliated.

(B)humble.(C)confident.(D)arrogant.

24.Whichoptiongivesthebestevidencefortheanswertothepreviousquestion?(A)Lines12–16(“Yet...chains”)(B)Lines31–35(“In1958...body”)(C)Lines73–76(“Inthe...hormone”)(D)Lines81–85(“Sanger...brilliant”)

25.WhichoptioncouldbestbecitedasevidenceinsupportoftheclaimthatSangerwasconfidentinthesignificanceofhisresearch?(A)Lines23–27(“Hisfirst...1950s”)(B)Lines39–43(“Four...acid”)(C)Lines60–63(“Whenhe...Method”)(D)Lines73–76(“Inthe...hormone”)

26.Sanger’squoteinlines37–38(“just...lab”)hasatonebestdescribedas(A)playful.(B)somber.(C)bombastic.(D)careless.

27.Asusedinline67,theword“employed”mostcloselymeans(A)tried.(B)hired.(C)created.(D)utilized.

28.Theprimarypurposeoflines69–76(“More...hormone”)isto(A)elaborateonthepracticalapplicationsofadiscovery.(B)anticipateandaddressobjectionstotheauthor’sthesis.(C)explainSanger’sprimarymethodsofresearch.(D)showthenegativesideeffectsofSanger’sfindings.

29.Itismostlikelythatoneofthe“other”countriesthathasthemostNobelPrizewinnershasapercentageofthetotalnumberofprizewinnersinwhatrange?(A)Between12%and33%(B)Between11%and12%(C)Between6%and11%(D)Between0%and6%

30.WhatistheprobabilitythatarandomlyselectedNobelPrizewinnerfromthesetofwinnersfromGermanyandtheUnitedStateswillbefromGermany?(A)33(B)0.33(C)11

(D)0.25

Questions31–41arebasedonthefollowingreadingselections.

Below is an excerpt adapted from Booker T.Washington’s notable “Atlanta Exposition Speech” in1895.The second ispart of a1903 response, titled“OfMr.BookerT.WashingtonandOthers,”byW.E.B.DuBois.(Ashistoricaltexts,theseuseantiquatedlanguage.)

PASSAGE1

Ourgreatestdangeris,thatinthegreatleapfromslaverytofreedomwemayoverlookthefactthatthemassesofusaretolive

Line bytheproductionsofourhands,andfailto(5) keepinmindthatweshallprosperinproportion

aswelearntodignifyandglorifycommonlaborandputbrainsandskillintothecommonoccupationsoflife…Noracecanprospertillitlearnsthatthereisasmuchdignity

(10) intillingafieldasinwritingapoem.Itisatthebottomoflifewemustbeginandnotthetop.Norshouldwepermitourgrievancestoovershadowouropportunities.Tothoseofthewhiteracewholookto

(15) theincomingofthoseofforeignbirthandstrangetongueandhabitsfortheprosperityoftheSouth,wereIpermitted,IwouldrepeatwhatIsaytomyownrace.“Castdownyourbucketwhereyouare.”Castitdownamong

(20) the8,000,000Negroeswhosehabitsyouknow,whoseloyaltyandloveyouhavetestedindayswhentohaveprovedtreacherous[meant]theruinofyourfiresides.[…]

(25) Whiledoingthisyoucanbesureinthefuture,asyouhavebeeninthepast,thatyouandyourfamilieswillbesurroundedbythemostpatient,faithful,law-abidingandunresentfulpeoplethattheworldhasseen.

(30) Aswehaveprovenourloyaltytoyouinthepast,innursingyourchildren,watchingbythesickbedofyourmothersandfathers,andoftenfollowingthemwithteardimmedeyestotheirgraves,sointhefutureinourhumble

(35) way,weshallstandbyyouwithadevotionthatnoforeignercanapproach,readytolaydownourlives,ifneedbe,indefenseof

yours,interlacingourindustrial,commercial,civilandreligiouslifewithyoursinawaythat

(40) shallmaketheinterestsofbothracesone.Inallthingsthatarepurelysocialwecanbeasseparateasthefingers,yetoneasthehandinallthingsessentialtomutualprogress.

PASSAGE2

…BookerT.Washingtonaroseasessentially(45) theleadernotofoneracebutof

two—acompromiserbetweentheSouth,theNorth,andtheNegro.NaturallytheNegroesresented,atfirstbitterly,signsofcompromisewhichsurrenderedtheircivilandpolitical

(50) rights,eventhoughthiswastobeexchangedforlargerchancesofeconomicdevelopment.TherichanddominatingNorth,however,wasnotonlywearyoftheraceproblem,butwasinvestinglargelyinSouthernenterprises,

(55) andwelcomedanymethodofpeacefulcooperation.Thus,bynationalopinion,theNegroesbegantorecognizeMr.Washington’sleadership;andthevoiceofcriticismwashushed.

(60) Mr.WashingtonrepresentsinNegrothoughttheoldattitudeofadjustmentandsubmission,butadjustmentatsuchapeculiartimeastomakehisprogrammeunique.Thisisanageofunusualeconomicdevelopment,

(65) andMr.Washington’sprogrammenaturallytakesaneconomiccast,becomingagospelofworkandmoneytosuchanextentasapparentlyalmostcompletelytoovershadowthehigheraimsoflife.Moreover,this

(70) isanagewhenthemoreadvancedracesarecominginclosercontactwiththelessdevelopedraces,andtherace-feelingisthereforeintensified;andMr.Washington’sprogrammepracticallyacceptstheallegedinferiority

(75) oftheNegroraces.Again,inourownland,thereactionfromthesentimentofwartimehasgivenimpetustoraceprejudiceagainstNegroes,andMr.WashingtonwithdrawsmanyofthehighdemandsofNegroesas

(80) menandAmericancitizens.InotherperiodsofintensifiedprejudicealltheNegro’s

tendencytoself-assertionhasbeencalledforth;atthisperiodapolicyofsubmissionisadvocated.

(85) […]Mr.Washingtondistinctlyasksthatblack

peoplegiveup,atleastforthepresent,threethings—First,politicalpower,Second,insistenceoncivilrights,Third,highereducation

(90) ofNegroyouth—andconcentratealltheirenergiesonindustrialeducation,theaccumulationofwealth,andtheconciliationoftheSouth.

31.Asusedinline8,theword“common”mostcloselymeans(A)shared.(B)public.(C)ordinary.(D)universal.

32.Lines14–17mostpreciselyreferto(A)invaders.(B)foreigners.(C)immigrants.(D)travelers.

33.Thegeneralpurposeoftheparagraphinlines25–43istoargueinfavorof(A)foreignhostilitycoupledwithastrongdefense.(B)immigrationrestrictionscoupledwitheducationalopportunities.(C)nationalunitycoupledwithracialseparation.(D)ethnicloyaltycoupledwithbettercareforthesick.

34.Lines52–56moststronglyimplythattheNorthwasmostconcernedwith(A)ethicalconsiderations.(B)commercialadvancement.(C)religioustruth.(D)geographicalawareness.

35.Passage2moststronglysuggeststhatWashingtonencouragesAfrican-Americansto(A)fightforuniversalequalitybetweentheraces.(B)settleforlessthantheyrightfullyshould.(C)ignoreeconomicgoalsinfavorofmoralones.(D)deceiveotherswithrespecttotheirtrueloyalties.

36.Whichoptiongivesthebestevidencefortheanswertothepreviousquestion?(A)Lines47–51(“Naturally...development”)(B)Lines56–59(“Thus...hushed”)(C)Lines69–73(“Moreover...intensified”)

(D)Lines78–84(“Mr....advocated”)

37.Asusedinline66,theword“cast”mostcloselymeans(A)event.(B)constraint.(C)throw.(D)direction.

38.Whichsentencebestsummarizestherelationshipbetweenthepassages?(A)Passage1advocatesacourseofactionthatPassage2expressesasinsufficient.(B)Passage1presentsempiricaldatathatPassage2attemptstorefute.(C)Passage1arguesagainsttheeventualgoalslaidoutinPassage2.(D)Passage1ismoreidealisticwhilePassage2ismorepragmatic.

39.Basedonthepassages,whatWashingtonwouldmostlikelydefineasAfrican-American“compromise,”Duboiswouldmostlikelydefineas(A)obedience.(B)negotiation.(C)treason.(D)persistence.

40.Whichoptiongivesthebestevidencefortheanswertothepreviousquestion?(A)Lines14–18(“To...race”)(B)Lines25–29(“While...seen”)(C)Lines44–47(“Booker...Negro”)(D)Lines60–63(“Mr....unique”)

41.WhichselectionfromPassage1givesthemostdirectresponsetothelastparagraphofPassage2(lines86–93)?(A)Lines8–13(“Norace...opportunities”)(B)Lines19–23(“Castit...firesides”)(C)Lines30–32(“Aswe...fathers”)(D)Lines40–43(“Inall...progress”)

Questions42–52arebasedonthefollowingreadingselectionandaccompanyingmaterial.

Influenza

ItisapestilencethathasharriedcivilizationssinceatleastthetimeofHomer.What’smore,ithasdonesowithsuchroutine

Line periodicitythat,inourmodernageofannual(5) inoculations,theenduringdangerofthis

diseasehasgrownalltooeasytotakeforgranted.InfluenzaowesitsnametophysiciansoftheItalianrenaissance,whobelieveditwascausedbyinauspiciousastrological

(10) “influences.”Today,ofcourse,weknowittobetheresultofinfectionbyoneofseveralcloselyrelatedstrainsofvirus.However,unlikeothervirusesforwhichvaccinesareavailable—severalofwhich,through

(15) tenaciouspublichealthefforts,havebeeneradicatedworldwide—influenzaremainsaperennialmenace,andduetotheuniquenatureofitsgenome,isunlikelytoeverbecompletelyconquered.

(20) Traditionally,outbreaksofinfluenzaareclassifiedaseither“epidemic,”inwhichtheincidenceofthediseaseincreasessignificantlywithinagivencommunity,or“pandemic,”inwhichtheincidenceincreases

(25) overamuchlargerregion,suchasacontinent.Whilesuperficiallythedistinctionmayseemarbitrary,infactitreflectstwowell-delineatedfacetsoftheinfluenzavirusreplicationprocess.IntheNorthernhemisphere,

(30) “fluseason”spansfromNovembertoApril,andrepresentsanannualrecurrenceofinfluenzaepidemicsamongcommunitiessituatedinthispartoftheworld.Pandemicoutbreaks,thoughnotnearlyascommon,

(35) alsoseemtofollowanapproximateepidemiologicalpattern,typicallyoccurringaboutthreetimespercentury.Inthe20thcentury,theseoutbreaksincludedSpanishFluin1918,AsianFluin1957,andHongKongFlu

(40) in1968.Ofthethree,SpanishFluwasbyfarthemostdevastating.Withanestimatedmortalityashighas100million,itsdeadlinesswasonparwiththatoftheinfamousBlackPlague,whichravagedEurasiainthe

(45) MiddleAges.“Antigenicdrift”and“antigenicshift”are

thetwochiefprocessesthroughwhichinfluenzacircumventsouradaptiveimmunity,andarethoughttobethecausesofepidemic

(50) andpandemicinfluenza,respectively.Tounderstandthesetwoprocesses,itisnecessarytohaveaworkingknowledgeofthevirusitself.Therearethreeknownspeciesofinfluenzavirus—influenzaA,B,andC—each

(55) ofwhichconsistsofeightsegmentsofRNAcontainedwithinaproteincapsid,which

inturnissurroundedbyalipidenvelope.Collectively,theseRNAsegmentscodeforelevenproteins;twoofwhich,uponsynthesis,

(60) areexpressedontheenvelope’sexterior.Thesetwoproteinsareknownashemagglutinin(HA),andneuraminidase(NA).Intermsofthevirallifecycle,HAisresponsibleforattachingtosugarresiduesthatcoatthe

(65) cellsofourrespiratorytracts.Oncethevirushasinfectedacellandreplicatedwithinitsnucleus,NAcleavestheseresidues,allowingthevirustospreadfurtherthroughoutthebody.

(70) BecauseHAandNAaretheoutermostviralproteins,itisspecificallyagainstthesetwo“antigens”thatourwhitebloodcellscreateantibodies.Furthermore,amongthediversestrainsofinfluenza,genetically

(75) encodeddifferencesexistinthetypesofHAorNAexpressed.Thisallowsscientiststosub-classifystrainsbasedonthespecificantibodiesproducedagainstthem.Forinstance,theH1N1strainwasresponsible

(80) forbothSpanishFlu,aswellastheSwineFlupandemicof2009,whileH5N1causedtheAvianFluepidemicof2004.Randompointmutationtothegenes

encodingHAandNAisonewayinwhich(85) thesesubtypesevolve,andcan,moreover,

interferewiththeefficacyofourantibodies.Theaggregationofmanypointmutationsovertimeisreferredtoasantigenicdrift,andeventuallyresultsinrenewedvulnerability

(90) toviralstrainsagainstwhichanindividualwaspreviouslyimmune.Notably,influenzaAlackstheabilitytoproofreadandcorrectitsgeneticmaterialduringreplication,andasaresult,ispronetoamuchhigherrateof

(95) mutationthanotherspeciesofinfluenza.Forthisreasoninparticular,influenzaAisresponsibleforthevastmajorityofannualepidemics.Todate,16HAand9NAsubtypeshave

(100) beenidentified,onlyafractionofwhicharecurrentlyinfectioustohumans.However,becausetheinfluenzagenomeissplitintosegments,whenananimal—abird,for

instance—isco-infectedwithastrainspecific(105) toitsspecies,aswellasonecapableof

infectinghumans,thesegmentsmaybecomeintermixedduringreplicationinaprocesscalled“viralreassortment.”WhenthegenesimplicatedinreassortmentincludeeitherHA

(110) orNA,antigenicshiftoccurs,andtheresultingviralparticleswillexpressnovelproteinstowhichtheentirehumanraceisvulnerable.

Thetableshows,foreachhumanoutbreakofinfluenza,relevantepidemiologicaldata,andtheviralsubtypeinvolved.

HumanInfluenzaOutbreaks

42.Thestructureofthepassageisbestdescribedasa(A)broadsurveyfollowedbyatechnicalanalysis.(B)historicaloverviewfollowedbyalogicalargument.(C)generalcritiquefollowedbyexperimentalsummaries.(D)persuasivepresentationfollowedbyaresearchsummary.

43.Lines12–19(“However...conquered”)moststronglysuggestthatinfluenza(A)willcontinuetobeathreatdespitescientificadvances.(B)canbefullyeradicatedwithsufficientresearchfunding.(C)isuniqueamongdiseasesintheseverityofitssymptoms.(D)hasbeeneliminatedasapervasivethreattohumanity.

44.Asusedinline21,theword“epidemic”wouldbestdescribewhichofthefluoutbreaksinthetable?(A)1889(B)1957(C)2009(D)2013

45.Basedonthepassage,wouldantigenicdriftorantigenicshiftresultingreaterfundamentalchangestogeneticstructure?

(A)Antigenicdriftbecauseitresultsinincreasingvulnerabilitytoviruses(B)Antigenicdriftbecauseitcaneasilyspreadthroughoutthebody(C)Antigenicshiftbecauseitentailsgeneticreplication(D)Antigenicshiftbecauseitinvolvesinterspeciesgenomeexchange

46.Whichoptiongivesthebestevidencefortheanswertothepreviousquestion?(A)Lines61–65(“These...tracts”)(B)Lines65–69(“Once...body”)(C)Lines83–91(“Random...immune”)(D)Lines101–108(“However...reassortment”)

47.Theprimarypurposeoftheparagraphinlines83–98isto(A)explainhowHAandNAantibodiesleadtogeneticmutationsresultinginflu.(B)contrasttheprocessofantigenicdriftwiththatofantigenicshift.(C)describethemechanismwherebyaparticularflutypebecomesquiteharmful.(D)criticallyrespondtowidespreadmisconceptionsaboutfluvaccines.

48.Asusedinline92,theword“ability”mostcloselymeans(A)aptitude.(B)capacity.(C)skill.(D)talent.

49.Giventhedatainthetable,whichofthesefluoutbreaksmostlikelyresultedinthegreatestnumberofdeaths?(A)Russian(B)Asian(C)HongKong(D)Avian

50.Basedonthetableandthepassage,whichfluoutbreaks(givenbyyearofoccurrence)wouldmostlikelyresultinthehumanbodyproducingsimilarchemicalstofightthem?(A)1889and1957(B)1918and2009(C)1968and2013(D)2005and2013

51.Whichoptiongivesthebestevidencefortheanswertothepreviousquestion?(A)Lines37–45(“Inthe...Ages”)(B)Lines53–57(“Thereare...envelope”)(C)Lines78–82(“Forinstance...2004”)(D)Lines108–112(“When...vulnerable”)

52.Accordingtotheinformationinthetable,whichoftheseoptionsgivesthemostlogicalpossiblereasonthattheflusof2005and2013resultedinrelativelyfewcases?(A)Thesestrainsoffluaretransmittedviabloodratherthanthroughthemorecontagiousrespiratory

method.

(B)Asia,andparticularlyChina,havelowerpopulationdensitythantheglobalnorm.(C)Thosehumansinfectedweremorelikelytodiebeforetheycouldtransmitthedisease.(D)Thereservoirofthehumaninfluenzaoutbreakhadbirdsasitssource.

Ifyoufinishbeforetimeiscalled,youmaycheckyourworkonthissectiononly.Donotturntoanyothersection.

SCORINGAPPROXIMATIONThistablegivesyouanestimateofhowyourperformanceontheReadingsectionwillcontributetoyouroverallEvidence-BasedReadingandWritingscore.Keepinmindthateachtestwillbecurved,makingthe number of questions needed for a particular score dependent on the test that day. This is the bestestimate we can give you based on (1) previous SAT curves and (2) the fact that guessing is nowpermittedontheSAT.

Questionsoutof52answeredcorrectlyEstimatedoverallsectionscore(between

200–800)

800

52

750

49

700

46

650

43

600

40

550

37

500

33

450

30

400

26

350

20

300

13

250

7

200

0

DiagnosticTest

ANSWERSEXPLAINED

1.(B)Generally,thispassagecontrastsFanny’sexpectationsofherparentstotherealitiesshefindsuponreturninghome,makingchoice(B)correct.Herdiscussionisofherfamilyratherthanwithherfamilyandfriendsasin(A).(C)isadetailofthepassage,butnotitsmainidea.Furthermore,althoughshecontemplateshermother’sincongruitywithhereconomiccircumstance,thisisneitherthemainpointofthepassagenordoessheextendthedeliberationtoherself,making(D)incorrect.

2.(D)Fannyfindshome“theveryreverseofwhatshecouldhavewished,”andfindsherparentsunaffectionateandslovenly.Thebestoptionischoice(D)sinceherattitudetowardsthemcanbedescribedasdisappointment,andsheprovidesseveralreasonstosupportherfeelings.Sheisnotaffectionate,jealous,ordisrespectfultowardthem.

3.(C)Lines11–18depictFanny’sfatherasamanwithoutambition,curiosity,orknowledge“beyondhisprofession,”makingoverlypragmaticthecorrectanswer.Pragmaticmeans“practicalandrealistic,uninterestedinideasortheories,”soherfather’ssimple-mindednessfitsthisdescription.Shefindshimdullratherthaninterestingasin(A).(B)describeshispersonality,butnothisintellectualinterests.Anderuditemeans“cultured”or“well-educated,”making(D)theoppositeofFanny’sdescriptionofherfather.

4.(A)ThesentencebeforeindicatesthatMrs.PricenevershowedFannymorekindnessthanshedidonthatfirstday.Thesentenceafterstatesthat“shehadneitherleisurenoraffectiontobestow.”So,itcanbeinferredthatthe“instinctofnature”thathadtobesatisfiedwashermaternalfeeling.Itisnotrelatedtosurvival,acceptance,orjustice.

5.(C)Lines31–35giveevidencethatshedidnotoffermoreaffectionorkindnesstoFanny.Lines83–86supporttheideathatshedidnottrytogettoknowherorbefriendherinanyway.So,(C)isthebestchoice.(B)and(D)areoppositeofMrs.Price’streatmentofFanny.Thereisnoevidenceof(A).

6.(B)Lines33–35providedirectsupportthatMrs.PricedidnotincreasehertendernesstowardFanny.(A)referstoherrelationshipwithherfatherratherthanhermother.(C)referstohermother’shouseholdmismanagementratherthanherrelationshipwithFanny.(D)comparesMrs.Pricetohersisterandagainreferstoherdisorderandinefficiency.

7.(D)Basedonthefinalparagraph,Fannyis“anxioustobeuseful,”soitwouldbeappropriatetosaysheiseagertobea“helpfulcontributor.”Choice(A)istooextreme.Choices(B)and(C)incorrectlydescribecharacteristicswhichshemistakenlyhopedtofindinhermother.

8.(D)TheselinessuccessfullyillustrateFanny’sdesiretocontributeathome.Noneoftheotherchoicesareapplicabletoheraspirationswithinherparents’household.Instead(B)and(C)refertoFanny’sunfavorabledepictionofhermother,and(A)statesFanny’sdisappointmenttowardherhomeingeneral.

9.(D)EngagedaccuratelyreferstoMrs.Price’sabsorptionwithhersons.Itisappropriatetosaythey“engaged,”orpreoccupied,herattention.Noneoftheotherwordsfit;thesonsneither“stay,”“reside,”nor“dwell”herworryorcare.

10.(A)Lines59–73compareMrs.Pricetohersisters,indicatingthatMrs.PriceismoreinclinedtotheidlenessofLadyBertramthanthevigorofMrs.Norris,sochoice(A)iscorrect.Choice(B)falselyimpliesthatMrs.PriceandMrs.Norrisarecomparable,choice(C)suggeststhatMrs.Priceistheassiduousone,andchoice(D)wronglyindicatesthatMrs.Price’snegativequalitiesmakehermorerespectful.

11.(B)ThepurposeofthispassagecanbesaidtoraiseawarenessandaddresstheissuessurroundingapoliticallydisinclinedyouthinAmerica.Whilethegenerationisskepticalofthetwo-partysystem,theauthordoesnotarguetoendit.Likewise,thepassagedoesn’tgivesupportformandatoryvotingmeasures.Finally,(D)isincorrectbecausethepassageprovidesevidencethatmillennialsareactuallyavoidingpoliticaloffice.

12.(C)Concernedisthebestwordtodescribethenarrator’stone.(A)istooextreme.(B)istooneutral.And(D)isfartoopositive.

13.(A)Ascribeisaverbthatcanbedefinedas“attributeto,”andistheprecisewordmeaning.Here,thepassageissayingthatitwouldbemisguidedtoattributethegeneration’spoliticalindifferencetoanessentiallyunconcerneddisposition.Itisnotaccurateto“draw,”“blame,”or“dispute”theyoungergenerationascareless.

14.(C)Giventhecontextofthepassage,itissafetoassumethattheauthorbelievestheyoungergenerationisconcernedwithsolvingsocialissues,butpreferstodosooutsideofpoliticalspheres.Thepassagearguesagainst(A).Choices(B)and(D)arerefutedinlines26–40.

15.(B)Lines34–37specificallystatethatAmericanyouthare“lookingatotherwaysoftacklingsociety’sproblems,”providingdirectevidenceforthepreviousquestion.(A)givesthelinesthatintroducethefocusofthepassage.(C)referstothedecliningopiniontowardtheAmericanqualityoflife.(D)considerstheopinionthatavoidanceofthetwopoliticalpartiesisapositivetrend.

16.(C)Solvingfitsheresincethislinereferstoeffortstotackleproblems.Itismisleadingtostatethattheyoutharebeginningordiscoveringsociety’sproblems.Likewise,itopposesthenarrator’spositiontosaytheyaredismissingtheproblems.

17.(A)Lines55–68providestatisticsandrecentevidencetosubstantiatetheauthor’sclaim,making(A)thecorrectchoice.Choice(C)istempting,butisonlyadetailoftheparagraph.Choices(B)and(D)arenotaddressedwithinthepassage.

18.(C)Lines77–80serveastheauthor’sacknowledgementofapossibleobjectionthatwouldfindamoveawayfrompartisanshipapositivetrendinAmericanpolitics,so(C)isaccurate.Theselinesshowanopposingview,rulingout(A),anddonotgointodetailsasin(B).Finally,choice(D)isincorrectsincethelinesdonotincludeacitation.

19.(D)Thelinesareclosesttooneanotherin2008,so(D)iscorrect.Theotherchoicesprovideyearswherethedeviationbetweenvotersisgreaterthanitisin2008.

20.(B)Inlines31–34,theauthoraccountsforparticulareventsthat“generateaninfluxinyouthparticipation,”providingevidencethatthecandidacyofBarackObamamostlikelyaccountsforanincreaseinyoungvoterturnoutin2008.Choices(A),(C),and(D)allindicateadecliningtrendofvoterturnoutinthemillennials,andsowouldnotaccountforthe2008increase.

21.(A)Thispassageopenswithaprefaceongermsandantibioticsgenerallybeforemovingintospecificexamplesofbeneficialbacteriaandofinstanceswhenantibioticshavebeenharmfulratherthanhelpful.Choice(A)istheonlyoptionthatcapturesthatstructure.(B)and(D)failtorecognizetheintroduction,while(C)doesn’tconsidertheuseofexemplification.

22.(D)Line1referstoan“elementarytruth,”sofundamentalisthecorrectchoice.Scholasticwouldindicatethatthetruthwasconcernedwitheducation,while(B)and(C)arenotsynonymouswithelementary.

23.(A)Thesecondparagraphexpressesaturningpointwithintheintroductionwheretheauthorsuggeststhatwehavebeentooadamantinourgermophobia,making(A)correct.Thereisnoevidencefor(B).Choice(C)iscontradictivebecausethevalidconcernsofgermophobicswouldsupportratherthanrefutegermtheory.Andchoice(D)wouldinaccuratelyindicatethatthepurposeofthisparagraphwastoshowtheeverydayinstancesofgermtheory;infact,thisparagraphgivesahistoricalreminiscenceofgermtheorybeforepositingtheargumentthattheeliminationofallgermsisnotnecessarilythebestapproachtohumanhealth.

24.(B)Lines39–58furnishadirectexampleofthestatementmadeinlines35–38thatpointsoutthemetabolicandimmunologicaladvantagesofsomenaturalbacteria.Hence,(B)iscorrect.(A)and(D)misunderstanddetailsofthepassage.(C)isappealingbecauseitistruethatthisexamplecouldbeusedtoaddressanobjectiontowardallgermtheory,butthisparagraphismoreinterestedingivingthatillustrativeexampleratherthanrespondingtoobjections.

25.(D)Thisismostclearlyseeninlines102–106whereClostridiumdifficileisgivenasareal-lifeexampleofthe“direconsequences”thatoccurwhenantibioticsdisruptourbody’snaturalecologicalbalance.Choices(A),(B),and(C)provideexamplesofbeneficialbacteria,butaren’tusedtoillustratethedangersofantibiotics.

26.(D)Theselinesspecificallyaddresstheartificialupsettingofourbody’secologicalbalancethatoccurswithanoveruseormisuseofantibiotics;therefore,theyserveasdirectevidencetothepreviousquestion.Theyservetoallowthereadertounderstandapotentiallyfatalconditionthatemergeswhen“goodbacteria”havebeenwipedout.Theotherchoicesgiveevidenceforbeneficialbacteriaindigestivehealthandinfightingdisease,allowingthereadertounderstandthecrucialroleofourbody’snaturalmicrobiome.However,theydon’tdirectlyaddressthepotentialharmsofantibiotics.

27.(C)Dependsworksbestheresincethelinereferstoecologicalbalancebeingcontingentontwothings.Itwouldbeimpropertosaythebalancefulcrums,analyzes,orjointsontwothings.

28.(B)Fromthetable,wecanseevariationsinmicrobiotabasedontheirdietsandlocations.Sinceecologicalbalancedependsonthediversityofone’smicrobiomeandthefactorsthatinfluencethatpopulation,itmakessensethatthistablewouldbehelpfulinstudyingthatbalance.Germophobiareferstoafearofgerms,whichisnotwhatthetableisabout.Fromthepassage,weknowthatourgastrointestinalbacteriahelpwith“bloodclotting,”butthereisnoevidencethatthistablecouldhelptostudythistopic,sinceitisnotthefocusofthetable.Thetabledoesn’taddressbeneficialbacteriaversus“pathogenicbacteria”orinfectiousbacteria.

29.(D)Lines84–93directlygivethefactorsthataffectecologicalbalance,andthereforeprovidetheevidenceforthepreviousquestion.Sincethetableshowsthevariablediversityofmicrobiota,andecologicaldiversitydependsonthevariablediversityofmicrobiota,wecaninferthatthistablewouldbeuseful.Theotherchoicesfailtomentionconceptsthataredependentonthediversityofone’smicrobiome.

30.(B)Thebestapproachtothisquestionistoexaminethetableforwherethenumbersareverydifferentbetweentheindicatedcountries.Aquicklookhasnoticeablevariationsintwomicroorganisms:BifidobacteriaandClostridia.Therefore,choice(B)iscorrectbecausethepercentagesalonecouldbeusedtodifferentiatepersonsbycountryofresidence.

31.(A)Toapproachthisquestion,weshouldexaminethetableforinstanceswheredietchoicedoesnotmakeasignificantdifferenceinthepercentageofparticularmicroorganisms.Ofthefourchoicesgiven,thesmallestvariationinpercentagesoccursinBacteroideswherethereisa0.5%,0.4%,and1.4%change,respectively.

32.(C)ThepurposeofReagan’sspeechcanbesaidtoargueforaneweconomicpolicyandimmediateaction,so(C)isthecorrectchoice.Althoughheusesresearchandstatisticsforevidence,hispurposeisnottopresentthosefindings,buttousethemlogicallytopersuadeCongress.Finally,heisneithersurveyingapopularaudiencenordirectinghispersuasiontowardeconomistsonly.

33.(A)Haughty,or“arrogant,”anddismissivedon’tdescribeReagan’sfirmsenseofurgency.(C)isn’tappropriatebecauseheaffirmsthat“wecanactwithhope.”AndwhileReaganiscertainlyerudite,or“knowledgeable,”hisattitudeisnoturbane,or“suave.”So,thecorrectansweris(A),withempatheticmeaningtheabilitytounderstandanother’sexperiencesandemotions.

34.(D)SevereistheappropriatedefinitionsinceReaganisreferringtoa“punishing”riseinthepriceofgoodsandservices.(B)and(C)areoppositeofhowReaganmightdescribetheuncheckedincrease.Asfor(A),thoughtheeffectsoftheinflationmaybesad,inflationitselfisbetterdescribedasacriticalcondition.

35.(A)Inregardtounemployment,ReaganstatesthatmillionsofAmericans“whowanttobeproductive”areoutofwork.Thereisnoevidenceforchoices(C)and(D).While(B)istempting,Reaganalsomentionsthatinflationishardtokeepupwith,so(B)isnotlikely.

36.(B)Reaganposesunemploymentasareasonbehindtheneedforaneweconomicplaninlines21–24,so(B)isthecorrectanswer.Theselinesindicatethattheunemployedarenotwithoutworkbychoice.Options(A),(C),and(D)allrefertootherrationalesfortheproposedplan:interestrates,earning,andregulations,respectively.

37.(B)Inlines29–31,ReaganquotesanAmericanworkertoillustratetheincongruityofthehourlywage.Sinceanecdotalmeansan“accountbasedonpersonalstoryorexperience,”(B)isaccurate.Theselinesspecificallydonotcontain“statistics”oraddressacounterargument.Moreover,choice(C)iswrongbecauseratherthanseparatingthem,Reaganusesfactsdirectlyafterthequotetosupportthepersonaltestimony.

38.(C)Besuretoreadthequestionclosely.Whilethecurrentsituationcouldbedescribedasbothinflationaryandunsound,theunderlyingstructureoftheeconomy,accordingtolines70–74,isstillintact.So,(C)iscorrect.Likewise,Reagansuggeststhattheeconomyneedstorefocusandreprioritize,so(D)wouldn’tfit.

39.(D)TheselinesdirectlyindicateReagan’sbeliefthat,“Therehasbeennobreakdownofthehuman,technological,andnaturalresourcesuponwhichtheeconomyisbuilt.”Thus,theyprovideevidencetohisthoughtsregardingtheeconomy’sfoundation.Choices(A)and(B)refertothecurrentproblematicsituationratherthantheunderlyingstructureofthenation’seconomy.(C)isReagan’scalltoaction.

40.(B)Thebestapproachtoaquestionlikethisistoconsiderthepurposeoflines41–56inthecontextoftheentirepassage.ReaganspeaksdirectlytoCongresshere,providingvisualsandempiricalevidenceofjusthowbadthecurrentsituationis.So,(B)istheonlychoicethatindicateshisintentiontoillustrateseverity.

41.(A)Choice(A)isaccuratebecauseReagan’slistingofseveralfacetsoftheAmericanworkforceshowsthatregulationsareaffectingeveryone.Thisparagraphisaddressingregulationratherthanunemploymentasin(B).Reaganisproposingthattheeffectsareharmfulratherthanbeneficialasin(C).Additionally,theaudience,asdescribedintheintroduction,isCongressratherthanrepresentativesfromtheworkforceasin(D).

42.(A)DecreasedworksherebecausethislineindicatesthatAmericanproductivityhas“declined.”(B)and(D)aresynonymous,butsuggesta“rotting”thatisn’tasprecise.And(C)wouldimplythatproductivityhasfailedaltogether,whichisnotthepointthespeakerismaking.

43.(B)Lookatthesurroundingcontext.Theauthorstatesthatjustbecausecornsyrupisnaturaldoesnotmeanitissafe,andthengoesontotalkaboutanotherexampleofanaturalcomponentwhichprovesharmful.So,(B)iscorrectsincetheselinesrefutetheideathatwhateverisnaturalmustbesafe.Cornsyrupisalreadysensible,sowewouldn’tneedtointroduceapracticalapplicationasin(A).Theauthorisn’tusingsolaninetodefinesomethingelseasin(C).Andwhilethelinesdodrawattentiontoaharmfulelementofpotatoes,itisonlytoprovethatnaturecanbetoxic.

44.(A)Digressmeanstodeviatefromthetopic,so“stray”istheclosestmeaning.Inline16,theauthorisreferringtothefactthatheorshehasstrayedfromthetopic,andthentransitionsbackintocornsyrup.Sincethepotatoexampleisneitherananalysisnoracontradiction,wecanruleout(B)and(C).Choice(D)mightbetempting,butthecontrastingconjunctionbutshouldhintthatwearereferringtowhatwasjustmentionedratherthanwhatiscoming.

45.(B)Lines37–52discussthethirdstepofglycolysisandhowitworksintandemwiththebody’scurrentlevelsofglucose.So,dynamic,orina“stateofconstantactivityandchange,”isappropriate.Artificialmeans“fake,”whilearbitrarymeans“random.”Static,or“unchanging,”istheoppositeofdynamic.

46.(C)Accordingtothefirstauthor,consumingyoursugarsfromfructoseeventuallycausesobesityandotherhealthissues.Thus,(C)iscorrectbecauseitistheonlychoicethatconsidersthelong-termeffectsoffructoseaccordingtothefirstpassage.

47.(D)Lines62–68statethatourbody’suseoffructose“predisposesonetoobesity,diabetesmellitus,andahostofotherdangerousmetabolicdisorders,”andtherefore,providedirectevidenceforthepreviousquestion.(A)referstotheeffectsofsolanine,notcornsyrup.Andchoices(B)and(C)describethethirdstepofglycolysis,whichweknowfromthepassageisactuallybypassedbyfructose.

48.(C)Here,theauthorisstatingthatfearsandskepticism“persist”becauseunnaturalnessisillogicallylinkedtounhealthiness.So,they“continue”orcarryon.“Persevere”isanothermeaningofpersistthatreferstoadeterminationtocontinuedespitedifficulty.And(B)and(D)donotmeanpersist.

49.(B)Inlines100–115,Passage2statesthatthehealthrisksaremisdirectedatcornsyrup;realistically,theyareassociatedwithfructose,whichisprevalentin“canesugar,honey,andagavenectar”aswell.So,(B)iscorrect.Cornsyruphascomparableorevenlesseramountsoffructosethantheauthor’sotherexamples,so(A)isnotsupported.(C)misunderstandstheargument—fructose,notcornsyrup,isproblematic.And(D),althoughtrue,doesnotaddressthefactthatthehighamountsoffructoseinotherfoodstuffscontradictthecaseagainstcornsyrup.

50.(C)Bothauthorsdebunktheconnectionbetweennaturalandhealthy,so(C)iscorrect.Onlypassage2explicitlyconsiders(B),andimplicitlyconsiders(A),since“engineered”wouldbesimilartogeneticmodification.Andsincemetabolismisthenameforhowourbodiesfunction,itisanimprecisechoice.

51.(A)Lines6–15arguethatjustbecausecornsyrupisnaturaldoesnotmeanitissafeforhumanconsumption.Lines75–83extendthesecondauthor’saggravationatthepopularmisunderstandingthatunnaturalissynonymouswith“unhealthy.”Hence,bothofthesesetsoflinesindicatetogetherthattheauthorswouldagreethatafoodisnotnecessarilyhealthyjustbecauseitisorganic.

52.(A)Thefirstauthor’sargumentisbasedonglycolysisandtheprocessesofthebody.Meanwhile,thesecondauthorconsiderstheproductionofcornsyrupandthefructoselevelsinotherfoodstuffs.So,(A)istheappropriatechoice.

Chapter1.ReadingStrategies

AnswersExplained

1.(B)Therewillbe10ofthesetypesofquestionsontheSATReadingsection.Toanswerthisone,gobacktoline10,andconsiderthecontextsurroundingtheword:Timepasses.Therefore,youcansitstillinachairnottravelinginthreedimensions,buttravelinginspacetime.Wethinkoftimeaspassingforward.StephenHawkingexplainsthisasthree“arrowsoftime”.

Trytocomeupwithyourownsynonymforthewordbeforeyouexaminethechoices—thatway,youwon’tbetrappedbythehighlypersuasivewronganswers.Passinginthiscontextmeans“going”or“progressing”sincethesentenceisexpressingthenatureoftime.Thebestanswerismoving,sinceitcomesclosesttomeaning“going”or“progressing.”Itisnot(A),living,becausetimeisnotalivingthing,andisnotdescribedinthisway—evenmetaphorically—inthesurroundingsentences.Itisnot(C),becausethetypeofmovementthattimeundergoesisnotaphysicalthrow,butanabstractprocess.Itisnot(D),becausealthoughsucceedingcanmean“following,”passingisdescribinganongoingprocess,notaseriesofseparateevents.

2.(C)StartbygoingbacktotheparagraphthatreferstoEinstein’sthoughtsontimetravel:theonethatisinlines17–27.Thisparagraphsummarizeshisviewsabouttimetravel,buttoseewhatmostpeoplewouldthink,weshouldlookatthebeginningofthenextparagraph.Lines28–29state:Thisisn’ttheexcitingkindoftimetravelfromthesci-fibooksandmovies,though.Wewouldpreferourtimetraveltobeinstantaneousandnotlimitedtothefuture.

ByusingWeandour,theauthorisimplyingthatthesesentencesreflectthefeelingsthatmostpeoplewouldhaveaboutEinstein’sideasabouttimetravel.TheauthorthinksthatbecauseEinstein’sconceptionoftimetravelonlyinvolvedtravellingforwardintotimemoregradually,itwouldnotbeasexcitingassciencefictionconceptionsoftimetravelthatentailtravellinginstantaneouslyintothedistantpast.So,mostpeoplewouldbe“disappointed”withthisideaoftimetravel.Itisnot(A)becausethisportionofthepassagedoesnotindicatehopeoroptimismaboutwhatwouldcomefromthissortoftimetravel.Itisnot(B)becausederisionhasfartoonegativeaconnotationtobefittinghere.Itisnot(D)becausewhileconceptsoftimetravelmaybedifficulttograsp,thetextprimarilyindicatesthatmostpeoplewouldunderstandthisidea,butwouldbedisappointedthatitdidn’tmeettheirhopesofwhattimetravelwouldbe.

3.(D)Therewillbe10“CommandofEvidence”questionsontheSATReadingtest.Determinewhichofthechoicespresentstextualevidencetosupporttheanswertothepreviousquestion.Sincethesequestionsgohand-in-handwiththequestionsthatprecedethem,givequestionslike#2plentyoffocusbecauseifyoumiss#2,youarelikelytomiss#3.If#2isgivingyoutrouble,checkouttheanswersto#3toseeifanyoftheselectedlinespointyouintherightdirection.Lines28–29useinclusivelanguagetoindicatethattheauthorbelievesthatmostpeoplewouldbedisappointedthatEinstein’sconceptionsoftimetravelfallshortofthedreamsofsciencefiction.Itisnot(A)becauselines13–16focusonStevenHawking’sviewsontime.Itisnot(B)becauselines19–21provideexplanatoryinformationaboutEinstein’sideasbutnotonhowpeoplewouldreacttothem.Itisnot(C)becauselines25–27alsocontinuetogivedetailsaboutEinstein’sthoughts,butnotonthoseofthegeneralpublic.

4.(A)Forquestionsaboutthepassageasawhole,besureyouchooseananswerthatistrueforthepassageinitsentirety,notjustforsomesmallpartofthepassage.Incorrectanswersonquestionslikethesewilloftenbetrueaboutspecificcomponentsofthepassage,makingthemverypersuasive,butwillfailtocapturethebigpicture.“Informedcontemplation”isthebestofthesedescriptions,sincethenarratorspeculates(contemplation)onthefascinatingpossibilitiesoftimetravelwhilegroundingthesethoughtsinthetheoriesofrenownedscientificminds(informed).Itisnot(B)becausethenarratorpresentsanumberoftheoriesasbeingpossiblewithoutdismissinganyofthemoutright.Itisnot(C)becausewhilethereissomeimaginativemusingaboutthepossibilitiesoftimetravel,thereisasignificantemphasisonwhatisrealisticallypossiblegiventhelatestscience.Itisnot(D)sincewhilethereisquiteabitofscience,timetravelispresentedasmuchmoretheoreticalthanfactual.

AnswersExplained

1.(B)Becertaintocarefullyreadaquestionlikethis—thequestionasksyoutodeterminewhytheauthorisusingaparticularword.Todeterminehowthewordisbeingused,wemustconsidersufficientcontext:Annawasanxious.Herfatheroughttohavereturnedfromhistraplinefourdaysago,butthesnowshadbeenheavierthanexpected.Probably,hewasjustcaughtontheothersideofthepassuntilthestormcleared—probably.Kolyastirredthekettleslowly,mixingtogethertheblood-redbeets,andtheice-whitecabbage.

Typically,“probably”isusedtoconveyrelativecertainty.Inthegreatercontextofthisparagraph,however,weseethatAnnaisanxiousaboutwhetherherfatherwillreturnfromhistrapline.Sheisrepeatingprobablyinawaythatisquitetellingaboutheruncertaintythathewillcomeback.Itisnot(A)becauseifshewereconfidentabouthisreturn,shewouldn’tneedthisrepetitiveself-talkforreassurance.Itisnot(C)becausethereisnoindicationthatsheisfeelinglikefightinganyoneoranything.Itisnot(D)becausethisistooextreme—shehasnotyetgivenuphope.

2.(C)ConsidertheevidenceintheparagraphsthathighlightKolya’spointofview.Thepointofviewinthefirstoftheseparagraphsismostclearlyseeninlines23–24:Itpleasedhimtobelongtosoremoteaprovince;cutofffromtheuglinessofcities,andthesouthernroads.

Thisisclearlyamoreappreciativepointofviewsinceheispleasedtobefarawayfromdistanttroubles.

Thepointofviewinthesecondoftheseparagraphsismostclearlyseeninline31:

AndKolya,forthefirsttime,cametoknowthecostoflivingattheedgeoftheworld.

Thisisamoremelancholypointofview,becauseKolyarealizes,basedonalltheobservationsbuildinguptothissentence,thathislocationkeepshimfromhavingmanytypesoffoodthatwouldmakelifemucheasier.

Itisnot(A)or(B)becausethepointofviewshiftsoverthecourseofthesetwoparagraphsandisnotconsistentthroughout.Itisnot(D)becausethefirstoftheseparagraphsismorepositiveinitsoutlookthanthesecond.

3.(D)Thisquestionasksyoutopickwhichchoicegivesthebestplotsummaryofthepassage.Ifthistypeofaquestiongivesyoudifficulty,itcanbeverysensibletowaitonituntilafteryouhaveansweredmorespecificquestions.Intheprocessofansweringthespecificquestions,youwillpickupontheoverallplotofthepassage,especiallysincethequestionsaregenerallyintheorderofwheretheyarefoundinthetext.(D)isthebestanswerbecausewehaveaccesstobothKolyaandAnna’sinnerthoughtsandtheirconversations.Theirthoughtsandconversationsfocusontheirpresentstateofaffairsandalsotheirexperiencesandthoughtsinthepast.Itisnot(A)becauseAnnaandKolyaaremorepassiveintheirapproachtothetroublestheyface.Itisnot(B)becausethisdoesnotaddresswhathappensinthepassageoverall.Itisnot(C)becausethistooonlycomprisesasmallpartofthepassage.

AnswersExplained

1.(B)Therelationshipsbetweenthetwopassageswillgenerallybemoresubtle—itishighlyunlikelythatthepassageswillbediametricallyopposedtooneanother.Assuch,youwillneedtobethoughtfulwhenitcomestodeterminingthenatureofthepassages’relationship.Thesepassagesbotharguethattheelectoralcollegeisabadidea,buttheyusedifferentsortsofargumentstomaketheircases.(B)iscorrectbecausePassage1highlightsthemanyrisksoffraudulentvotingthattheelectoralcollegemaycause,andPassage2drawsmuchmoreattentiontothelegacyoftheFoundingFathersoftheUnitedStates.Theanswerisnot(A)becauseneithercitesanyoneasapoliticalauthority.Itisnot(C)becausePassage2ismorerespectfultowardtheFoundingFathers,referringtothemas“greatmen.”Itisnot(D)becausethereisnothinginPassage2abouteconomicconsequences.

2.(D)TheauthorofPassage1emphasizesproblemswithvotingproceduresintheelectoralcollegeprocess,andmakeshercaseusingmorequantitativeandanalyticalapproaches.Reviewthetextcitedinchoice(D):Considerthatit’stheoreticallypossibletoreceivejust11votes,haveyouropponentreceive200million,andstillwintheelectionundertheelectoralcollege.

Thisstatementisanalyticalandquantitative,makingitstylisticallymostlikewhatisfoundinPassage1.Choice(A)isquitemetaphorical,choice(B)ismorehistorical,andchoice(C)isratherironic.

3.(B)Besureyoureadthequestioncarefullyandpickupontheworddisagreed.Sincebothpassageauthorsopposetheelectoralcollegesystem,someonewhodisagreedwithbothpassageauthorswouldwanttociteinformationthatmoststronglysupportedtheelectoralcollege.(B)isthebestoption,sinceoneconsequenceofelectoralcollegevotingisthatinthe1912electioninwhichthereweremultiplecandidatesandnocandidatewonamajorityofpopularvotes,thevictoriouscandidatedidreceiveavastmajorityofelectoralvotes.Thisobservationcouldbelogicallytiedtotheelectoralcollegeleadingtomorenationalunitythanwouldotherwisebethecase.Theanswerisnot(A),(C),or(D)becausetheseideaswouldsupportthetwopassageauthors,albeittodifferentdegrees.

AnswersExplained

1.(C)“Synthetic”istheoppositeoforganic,soitrefersto“conventional,”manmadeapproachestofarming.Theauthorshowsbothprosandconsofthesyntheticapproach,arguingthatwhileitmaynotbeashealthyorenvironmentallysafe,itismuchcheaperthantheorganicapproachandcanpositivelyimpactglobalhunger.(A)isincorrectbecausetheauthordoesnotbelievethatsyntheticagricultureisinferiortoorganic.(B)isincorrectbecauseorganicishealthier,butnotaccessibletothepoor.And(D)doesn’tworkbecausethepassagearguesthatbothmethodsarescientific.

2.(A)Line5referstoBorlaug’scontributiontoworldwidepovertyandhunger,statingthatheextendedthefarmingmethods“alreadycommonintheUnitedStatesandBritain.”Sincethisisreferringtotheirprevalence,widespreadisthecorrectchoice.Lowlymeansoflowimportance.Communalandcorporatewouldimplythatthemethodswereliterallysharedamongmembersofacommunity.

3.(B)Since(A),(C),and(D)areallstatedwithinthepassage,choice(B)isthecorrectanswer.

4.(B)Accordingtotheauthor,organicfarming“encouragespolyculture,”soitproducesmorediversity.Thepassagestatesthatconventionalfarmingismoreproductiveandcentralized.Thereisnoevidencefor(C)eitherway.

5.(C)Lines32–34comparetheindividualfocusofconventionalfarmingtothemulti-cropenvironmentoforganicfarming,makingitdirectlyapplicabletothepreviousquestion.(A)merelyillustratesBorlaug’scontributionviatheGreenRevolution.(B)suggeststhatorganicfarmingusessciencetomimicandprovokenaturalprocesses.(D)considersthattheexpenseoforganicagriculturedoesn’tallowitacomprehensivemarket.

6.(A)Lines28–38explainthesciencebehindorganicagricultureinresponsetothepreviousclaimthatitoccursnaturally,so(A)isthebestanswer.(B)and(C)aredoneinotherpartsofthepassage.And(D)isuntrue.

7.(C)“Symbiotic”canbedefinedasthecloseassociationoftwoorganismsinamutuallybeneficialrelationship.Inline36,weseethatmicroorganismsformsymbioticrelationshipsbetweenfungusandplantrootstonaturallybreakdownorganicmatterandreplacenutrients.Therefore,evenwithoutknowingthedefinition,wecaninferthatthisrelationshipisfavorabletoall.(B)providesanoppositemeaningfromthis.(A),whiletrue,isnotdescriptiveoftheadvantageousrelationship.(D)isincorrectbecausethereisnoindicationthattheseorganismshaveexperiencedahardshipthatdemandsanagriculturalresolution—theyaresimplyhelpingoutoneanother.

8.(D)Theselinesgoontoshowthesciencebehindorganiclivestock,so(D)capturestheirpurpose.Choices(A),(B),and(C)areallassuming.

9.(C)Thequestionisaskingyoutofindthegreatestdeviationbetweenacrop’sorganicandnon-organicprice.Since,organicsoybeanismuchmoreexpensivethantheothercropprices,wecanseethatthisdivergenceisgreatest.

10.(D)Thegraphshowscropprices,andineverycase,organicpricesarehigherthanconventionalpricesforthatsamecrop.So,thegraphsupportslines46–47,whichstatethatorganicfarmingismoreexpensive.(A)illustratestherecentgrowthoftheorganicmarket,while(B)and(C)givedetailsoforganictechniques.

11.(C)Sinceorganicisclearlymoreexpensive,wecanruleout(A)and(B).Moreover,lookingatthegraph,theorganicnumbersareapproximatelydoublethatofthenon-organicnumbersforthesamecrop.(D)ismuchtooextremeforthedataprovided.

AnswersExplained

1.(D)Theauthorusesparticulargeographicalexamplestoillustratetheextensiveramificationsassociatedwithresidentialsegregation.So,(D)isthecorrectanswer.Theauthordoesn’tusefirst-hand(i.e.,fromtheauthor’sviewpoint)hypotheticalsituationsorstatisticaldataasin(A)and(B).Likewise,thepassagedoesn’taddressobjections,butinsteadfocusesonthebutterflyeffectofmunicipal,state,andfederalregulations.

2.(A)Lines1–7introducethetopicofresidentialsegregationbyplacingthereaderintoahypotheticalsituationinwhichheorsheisintheslumsandfeelsuncomfortable.Thisisonlyaneffectivedeviceifthereadercanrelate.Therefore,choice(A)iscorrect.Thepassagedoesn’tprovideevidencefor(C)or(D).Finally,theauthordoesnotconfineheraudiencetothoseconcernedabouttheinnercities,asin(B).

3.(D)Here,theauthoroffersanexplanationfortheconditionoftheslum-likeneighborhoods,i.e.,residentialsegregation,andthendefinesit.Instancesof(A)and(B)comelaterinthepassage,and(C)isnotsufficientlyaddressedinthepassage.

4.(C)Theauthorarguesthathistoricalracismandsegregationpurposefully“prohibitedblackfamiliesfromaccumulatinghousingequity,”andthecontinuationofcertainpolicies“enforcethecontinuanceof‘ghettos.’”Therefore,theauthorwouldsaythattheslumswerecreated(C),intentionally,ratherthanrandomlyoraccidentally.Belligerentlymeans“aggressively”andisn’taspreciseofawordchoice.

5.(C)Lines53–56statethat“ahistoryofresidentialsegregationhaspurposelyimposed”slum-likeconditionsindesignatedareas.Thus,theselinesgivedirectevidenceoftheauthor’sopiniononhowslumswereformed.(A)discussesthesadlivingstandardsinlowresourcedareas.(B)considersthefar-reachingconsequencesofexclusionaryzoning.Finally,(D)positsthat,asaresultofresidentialsegregation,thoselivingintheslumsareoftenmistakentocausetheundesirableconditionsthattheyarethemselvesvictimsof.

6.(B)Potentialisanaccuratesynonymsincethislinerefersto“prospective”homebuyersorrenters.Thisworddoesn’trefertotheireconomicsituation,asin(A)and(D).Norisitdefinitivetosaythesebuyers/renterswereperceived,whichmeanstobecomeawareof.

7.(B)Theauthorcontendsthatonereasonbehindthelackofresourcesindesignatedneighborhoodsistheirtendencytobezonedas“commercial”or“industrial,”whilewhiteneighborhoodsarezonedas“residential.”Therefore,wecanassumeslum-likeneighborhoodsaremorelikelytobeusedforthingsbesideshousing,asin(B).

8.(D)Lines45–48indicatethatzoningcategoriesinfluenceavailableresources,andthatblackneighborhoodsareoftenlabeled“commercial”or“industrial,”providingsupportforthepreviousquestion.Choice(A)referstotheauthor’sopeningvisualoftheslums.(B)iswheretheauthorposesthequestionthatshehopestoshedlighton.And(C)divesintotheeffectsofurbanrenewalplansinChicago.

9.(A)Accordingtothecontext,urbanrenewalplanscreatedanarrayoftroubleforresidentiallysegregatedneighborhoods,includingpolicedepartmentswho—withoutadequatestaffandresources—“respondedtoviolencewithviolence.”Wecaninferthatthismeansthepolicefeltunabletocontrolcertaincircumstancesand,instead,reactedaggressively.

10.(A)Inthespecifiedlines,theauthorreferstoanumberofcausesthathaveoverlappedwithoneanothertocreate“areasofpoverty,unemployment,andlackofnecessaryresources.”So,whilethecausesaremanyandmultifaceted,theresultisclear:theslums.(B)isincorrectbecausetheauthorbelievesitcouldbe“acombinationofallthesefactorsandmore.”Likewise,(C)and(D)disregardthemyriadoffactorsthatmaycontributetoslum-likeconditions.

Chapter2.WordsinContext:StrategyandPractice

AnswersExplained

1.(C)Here,naturereferstoMike’scharacterordisposition,making(C)theclosestwordchoice.(A)and(D)areothermeaningsofnature.(B)isanappealingchoice,butusuallyreferstoatemporarymoodratherthanacombinationofcharacteristicsthatmakeupone’spersona.

2.(A)Nosyisthemostprecisechoicegiven.Sincesheisaskingmanypersonalquestions,wecaninferthatnosyismoreaccuratethancurious.Choice(B)wouldn’tworkbecausethismeaningisreferringtoanadjective,whilethischoiceisanoun;itwouldhavetobe“interfering”tobeconsidered.And(D)referstoanotherdefinitionofpryorlever.

3.(A)Willnevercompleteshisassignmentsontime,sowecaninferthattheintendedmeaningislazyasinchoice(A).(B)refersmoretosleepinessthanidleness.(C)wouldindicatethatWillwasunintelligent,andisnotameaningoflackadaisical.(D)isanearantonym,sinceitreferstoeagernessandintensity.

4.(D)AnothernameJohnny’ssistermightcallhimiscoward.(A)istempting,butreferstohisphysicalstrengthmorethanhiscourage.(B)referstoanotherdefinitionofchicken.AndwhileJohnnymaybepetrified,chickenisreferringtohislackofcourageratherthanhisfear.

5.(D)Sincetheboyisnotboastful,wecaneliminate(A)and(C)whichrefertoarrogance.Glorifiedreferstosomethingbeingidolized,oftenoverlyso.Instead,itisappropriatetosaythattheboyisconfidentor“self-assured.”

6.(D)Approachthiscarefully;thedressismadepoorlyandEmilywishesshewentsomewheremoresophisticatedlikeaboutique.Therefore,wecaninferthatitisa“cheap”dress,perhapsbothinpriceandcomposition.(A)referstoarefundorapaybackafterpurchase.(B)isanothermeaningofdiscount:tooverlookordismiss.(C)istrueofthedress,butisnotameaningofdiscounted.

7.(D)Here,discountedmeans“discredited”or“dismissed.”(A)referstoalowerprice.(B)isnotameaningofdiscount.(C)isanounwhereweneedaverb.

8.(B)Strongisreferringtotheconcentrationoftheacid,so(B)iscorrect.(A)and(D)refertoothercommonusesofstrong.(C)meanspersistentordetermined.

9.(A)Itisaccuratetosaythattheactress“couldnotbereached.”(B)doesn’treflectmeaning,butaddsassumption.(C)indicatesthattheactresswasindecent.Cognizantmeans“tobeawareof.”

10.(B)Thislinesays“Joeysetupapranktoexacthisrevengeonhisoldersister,”sotheclosestmeaningfortheunderlinedportionis(B).(A)is“toinflictharmonbehalfof,”soitwouldonlybeappropriatetosay,“Joeysetouttoavengehimself.”(C)addsassumption,namelythatJoeyiscruel.(D)isaslangtermforrevenge,butitisn’tprecisetosay“titfortatonhisoldersister.”

11.(C)Leewaymeans“freedomforerror”or“flexibility.”So,thejudgeisallowingsomelibertythatshemightnotnormallyallowonthegroundsthattheprosecutorisnew.(C)evokesthissameintendedmeaning.(A)speaksmoreofanopportunitythanalenience.Carteblanchemeans“freereinorfullauthority.”Andamnestyisapardonforpastconvictions.

12.(A)Apprehensionmeans“anticipationorsuspense”andistheappropriatewordmeaning.Itdoesn’tmakesensetosay“waitedwithimpatiently.”(C)indicatesliteralbreathingtroubles.Andcessationisdefinedas“theendorterminationofsomething.”

13.(C)Here,currentisreferringtothefast-movingriverwater,so(C)isappropriate.(A)and(D)evokeothermeaningsofcurrent.Andwhile(B)istrue,itreferstotheriverasawholeratherthantothemovingwater.

14.(C)Usecontextclues—EmilyisnervousaboutherinvitationbecauseofherlastencounterwithKayla.So,shehas“doubts.”(A)and(B)refertoothermeaningsofreservations.And(D),whileappealing,iswrongbecauseyoucannot“haveskeptical”;ratheryouwould“beskeptical.”

15.(A)Mr.Daviscannotapproveof,or“endorse,”thestudents’newmediumofexpression.(B)indicatesthathecannotmakeanagreement“withthestudents”andis,therefore,poorlywordedandimprecise.(C)and(D)arenotmeaningsofsanction.

16.(C)Thisonecanbetricky.(C)iscorrectbecausethisusageofentertainingmeans“thinkingofoftenordeeply.”Choice(B)doesnotworksinceitisunlikelythathewouldhavetheself-controltokeepthesefeelingssecret.Choices(A)and(D)aretheoppositeofwhatthesentencemightconvey.

17.(C)Here,leanreferstothemeat’sreducedcaloriesorreducedfat.(A)meanstough;(B)referstobeingangled.Andwhile(D)isquitetrueofthemeat,itisnotwhattheauthorissaying;“nutritious”isnotameaningoflean,butapossibleimplicationofit.

18.(C)Evadedmeans“avoided”andisthebestmeaning.(A)isanothermeaningofskirted.(B)assumesaroundaboutroute.(D)hasthecorrectidea,butitisimpropertosay“theycarefullyavoidanceofherquestions.”

19.(A)Here,Mariaiswillingtodothedifficultworkbecausesheseesthelong-termresults,making(A)correct.(B)and(C)indicatenearoppositemeanings.And(D)wouldhavetobe“adheredto”inordertobeconsidered.Still,itwouldmeanthatsheactedinthewayrequired,butnotsignifyapersonalwillingness.

20.(A)Thecontexttellsusthattheringisnotsilver,but“madeofbasemetals,”soyoumightbethinkinginferior.(A)means“lowly”or“inferior.”Whileitmightbecorrecttosaytheringis“metallic”or“artificial,”thosearenotmeaningsofbase.And(D)referstoanotherdefinitionofbase,asinafoundation.

21.(D)Breakmeansapauseorgap,so(D)iscorrect.(A)indicatesinactivity.Andchoices(B)and(C)representothermeaningsofbreak.

22.(B)Accordingtothecontext,thebirdsarebeingsoloudthatWrenishavingtroublesleeping.So,(B)iscorrect.(A)referstoconfusionandupheaval.(C)isaspecificprolongedsoundthatisimprecisehere.(D)means“fraudordeception.”

23.(B)Forthisquestion,readthelinewitheachanswer.ItreferstoMelinda’slocation,so(B)iscorrect.(A)and(D)implytheothermeaningofrespect,asin“respectingandshowingreverenceforsomeoneorsomething.”

24.(C)Here,trackrecordrefersto“pastperformance”asin(C).ItwouldbeinaccuratetosayMel’s“indicator”or“achievement”wasfraughtwithawardsandhonors.Andwhilearesumecouldconsistofthosethings,itreferstoaspecificdocumentoneusesfornetworkingandhiring.

25.(A)Thismeaningoffreeisindicatingtheabsenceofcost,so(B)compedimpliesthatsomeoneelsepaidforit,oritwasgivenawayforaspecialreason.(C)and(D)referenceothermeaningsoffree.

26.(C)WecansaythatJosiah“experienceddifficultywith”thetest.(A)and(B)wouldchangetheintendedmeaning,and(D)wouldimplythatthetestactuallycausedharmtoJosiah.

27.(C)WeknowthatBillyhas“accurateinformation,”souponreferstochoice(C).(A)isanoppositemeaning.(B)meansthathewasactuallystandingonorpositionedontheevents.And(D)wouldeitherimplythathewas“runningoncurrentevents”or“fashionableonthem,”neitherrepresentativeoftheintendedmeaninghere.

28.(A)Thinkaboutthemeaning.Evidencewasintroducedsothatitcouldbetakenintoaccount.(A)capturesthatmeaningbest.Choices(C)and(D)areclose,butnotcorrect.Weneedbothideas—thatitwasputforwardandthatitwasconsidered—tobeprecise.

29.(A)Grandpahasaroutinethatheis“unwillingtochange.”Choices(B)and(D)areoppositemeaningsofwhatthesentenceistryingtoconvey.(C)impliesthatheismorallyopposedtocontemporarysocialcustoms.

30.(C)Here,thewritersexperiencedresistancewhenpeoplewereuncomfortablewithchange,so(C)isthebestwordmeaning.(A)and(B)changethemeaningofthesentence,while(D)istooextremesinceitdenotesviolence.

31.(B)“Space”istheintendedmeaning.Choices(A)and(D)refertoothermeaningsofplot.Andwhilethespacesinacemeteryarecalled“plots,”thegraveyarditselfisnot.Additionally,thereisnoevidencethatthistypeofspaceiswhattheauthorintends.

32.(B)Substituteyourownsynonymforpresent;youmightsay“there”or“inattendance”asin(B).(A)impliesthatthethreehundredviewerswereallintroduced.(C)relatestopresentwhenitinvolvesagiftorreward,andagainchangesthemeaningtosignifythattheviewerswereallgivenawards.(D)referstopresentasin“thecurrenttimeratherthanpastorfuture.”

33.(D)Clenchedistheappropriatewordchoice.Choices(A)and(C)areotherdefinitionsoftense.Meanwhile,choice(B)invokesabackandforthmotion.

34.(A)Meritmeans“valueorworth,”andisthecorrectchoicehere.Pricewouldrefertotheextrinsicvalueratherthantheintrinsic.Choices(C)and(D)refertoasetofvaluesormoralstandards.

35.(C)Here,pitchedjustmeansthatCarl’sbodyisfallingforwardasin(C).Choices(A)and(D)useothermeaningsofpitch.(B)couldeitherinvolvehittingsomethingoraconditionofbraindamagecausedbyinterruptionofbloodflow.Eitherway,itisnotameaningofpitched.

36.(D)Kelliwascausedtoact,or“forced”tosell.Choices(A),(B),and(C)allrefertoothermeaningsofdroverelatedtooperatingavehicle.

37.(A)Choices(B)and(C)couldbothbetrueofthedog,butarenotmeaningsofpanted.(D)isameaningofpanted,butdoesnotfitthecontext.

38.(D)Besuretoconsidercontext.Monica’sroommatehasideasthatarevaguelybasedonfact,butthenareexaggeratedormistakensothattheycannotbetestedorrationalized.Therefore,theyarenonsense.Indubitablemeans“impossibletodoubt.”(B)ismuchtoomild.And(C)referstoabunkbed,whichisnottheintendedmeaningofthesentence.

39.(B)Here,lawnisprecise.(A)and(C)areindicationsofyardasaunitoflength.(D)isinaccurate;itcanbepartofayard,butisnotsynonymouswithyard.Also,itwouldbeunusualtomowagarden.

40.(B)Substituteageneralanswerbeforelookingatthechoices.Simonwasbroughtup,or“raised,”inasmalltowniswhatthelineisintendedtoconvey.Itdoesinvolvehisgrowingup,buthewasnot“grown”likeacrop.(C)isanothermeaningofrear.(D)changesthemeaningofthesentence.

41.(C)A“live”showindicatesthatitwasnotpre-recorded,buttelevisedasithappenedinrealtime.Choice(C)istheclosestmeaningsinceitreferstoaspontaneousornotprepared-ahead-of-timeperformance.(A)isadifferentdefinitionoflive,asin“survival.”(B)isanoppositemeaning.(D)referstolive,asinlivelyorvigorous.Evenifyoudidn’tknowthemeaningofimprovise,youcouldlikelyeliminatetheotheranswers.

42.(C)Here,thecaptainistellingRogertogotoand“operate”thesteeringwheeloftheboat.Choices(A)and(D)inaccuratelyimplythatthecaptainiscallingRogera“man.”And(B)isnotameaningofman.

43.(C)Thisonecanbetricky.Ifyoureadcarefully,youcanseethatthesentenceisreferringtothenewspapercompany,ratherthanthepaperitself.Whiletheinformationcouldhavebeenwithinanarticleandwrittenbyajournalist,neitherofthose“print”thepaper,andsoareimprecise.(D)isawrittenaccountandacommonnamefornewspapers,butagain,isnotindicatingthecompany.

44.(C)Substituteyourownsynonymhere.Rossineededto“get”or“reserve”aticket.(A)and(D)aredifferentmeaningsofbook,while(B)isnotameaningofit.

45.(A)Here,Andreiselongatinghisnecktoseefarther,so(A)isthemostaccuratewordmeaning.(B)wouldmeanthatheisactuallycausingmagnification,aswithamicroscope.(C)isanoppositemeaning.Andchoice(D)isnotameaningofcranedandchangesthemeaningofthesentenceentirely.

46.(B)Accordingtothecontext,Yusefishopingtheshorewillslowdowntheboat’sspeed.So,(B)isappropriate.(A)and(C)arewhatwilleventuallyhappen,butnotwhatheisdoingatthispointofthesentence.

47.(D)Aquoteis“anestimatedfiguregivenforthecompletionofaprojectorservice.”So,(D)iscorrect.(A)canbeappealing,butquotesarenotthesameandsometimesvarygreatlyfromactualexpense.Choices(B)and(C)denoteothermeaningsofquote.

48.(B)Here,thecaseisreferringto“anoccurrencethatisbeingtriedincourt,”so(B)iscorrect.(A)and(C)areothermeaningsofcase,anddefendantisrelatedtothetrial,butiscertainlynotthetrialitself.Andwhileitwouldmakesensetosaythenoteswereconcerningthedefendant,itwouldultimatelychangethemeaningofthesentence.

49.(D)Here,mightreferstopotentialso(D)iscorrect.(B)and(C)pointtoothermeaningsofmight.(A)wouldalterthemeaningtoinvolvepastperformanceorevents.

50.(A)Putinyourownsynonym.Wecanseefromthecontextthatbeatreferstothedrummershittingtheirinstrumentstoplayarhythm.So,(A)isthebestchoice.(B)isfartooviolentandextremeinitsconnotation,usingawordassociatednotwithmusicbutwithphysicalfighting.(C)and(D)signifyothermeaningsofbeat—“defeatinganopponent”and“beingtiredout,”respectively.

Chapter3.GraphAnalysis:StrategyandPractice

AnswersExplained

1.(D)Besuretoreadtheanswerchoicescarefully.Choices(A),(B),and(C)allmakeassumptions.While“falls”accountforfourtimestheamountofbraininjuriesthan“assault,”thatdoesnotnecessarilymeantheyhappenfourtimesasoften.Likewise,(B)introducesexternalfactorslikeseatbeltusethatwecannotprovideevidencefor.Andjustbecausetheyarelabeled“Other,”itdoesnotmeanthecausesofbraininjuriesareunknown.(D)istheonlydefinitechoicebasedonthegraph;togetherthetwocategoriesmakeup31%oraboutone-thirdoftraumaticbraininjuries.

2.(B)15outof100canberewrittenas15/100or15%.Therefore,(B)isthecorrectanswerbecause“unintentionalblunttrauma”is15%ofthechart.

3.(A)Thebarsofthegraphfollowthekeychronologically,sothefourthcategory,“umpire,”isthehighestlikelihoodwith98.3%.

4.(C)Again,onlyusetheevidencegiven.Wecannotmakeconclusionsaboutanyemploymentoutlooksasin(A)and(B).Norcanwemakeassumptionsaboutthenumberofpeoplecurrentlyemployedintheseprofessionsasin(D).However,wecanusethegraphtoshowthatjanitorialjobsaremorelikelytobecompletedbymachinerythanhistorianjobs,making(C)correct.

5.(B)1.2isthreetimes0.4,making(B)correct.Youcanmovethedecimalsifitmakesiteasiertothinkabout:12=3×4.

6.(B)Alineofbestfit,oratrendline,isastraightlinethatbestrepresentsthedatavaluesgiven.Sincethislinewouldhaveadownwardslopeinthegraphshown,(B)istheaccurateanswer.

7.(B)Accordingtothegraph,beefconsumptionwaslowestin2014.

8.(C)From2006to2007,beefconsumptionremainedsteadyatabout28billionpounds,so(C).

9.(D)While(B)lookspersuasive,beverycarefultoconsidertheunitsalongthey-axis.Theunitsstartat26,200insteadof0,soallofthesemountainsarerelativelyclosetooneanotherinheight,making“noneoftheabove”correct.

10.(D)Thisgraphshowsthenamesandtheheightsoftheworld’stallestmountains,so(D)istheonlyanswerforwhichwehavedirectevidence.(C)canbetempting,butheightisnotnecessarilycorrelatedwiththedangerofclimbing.Forinstance,Lhotsecouldpossiblyhavesaferpathsandbeinamoretemperateclimate,whileNangaParbat,thoughmuchsmaller,couldprovideonlyriskyclimbingandexposuretodangerousweatherandwildlife.

11.(B)Toapproachthisquestion,findwherethetwolinesarenearesteachother.Thus,(B)iscorrect.

12.(C)Choice(C)istheonlyanswerevidencedbythegraph.Bothtypesofinstitutionshaveincreaseddramaticallyinpricesince1982.(A)doesnotnameasingleyear,butatwo-decadespan,sowecannotknowtheannualincreasesperyearduringthatrange.In2012–13,aprivateeducationwasjustover,notunder,twicethecostofapubliceducation,making(B)wrong.Andwehavenoinformationfor2013–14,butcouldprojectthatitwouldcontinuetoriseratherthandrop.

13.(B)Thereisnoevidencetosupport(B)sincethegraphdoesnotaddressinflationandwearenotprovidedanyotherinformation.Theremainingchoicescanallbeprovenbythegraph.

14.(A)AfghanistanandEthiopiahavethelowestliteracyrates,withfemaleratesbeingabout24%and41%,respectively.SinceAfghanistanisnotachoice,(A)isthecorrectanswer.

15.(B)Accordingtothegraph,about52%ofmeninAfghanistanareliterate,comparedto24%ofwomen,sothisisroughlytwiceasmuch.Thus,(B)isaccurate.

16.(D)Themaleliteracyrateisslightlyhigherthanthefemaleliteracyrate,rulingout(A).(B)doesn’tworkbecausetheratesareeveninNorthKorea.Andoverall,Mexico’snumbersareslightlyhigherthanIran’sasin(C).Sincetherearethreecountrieslistedwherefemaleliteracyisequaltomaleliteracyandtheothersevenoptionsallhavehighermaleliteracy,wecansay7/10or70%ofthecountrieshaveagreatermaleliteracyrate.

17.(B)Toapproachthisquestion,lookforachunkthatmakesupaboutone-fourthoftheentirepiechart.(B)isthebestanswer.(C)ismuchbiggerthanafourthandtheotheroptionsareobviouslyless.

18.(C)Thecherrysectionofthepiechartislargerthanthecoconutcreamsection.Thisistheonlyoptionforwhichthegeneralrelationshipholds.

19.(D)Tofindthis,drawlinesfromFebruaryovertothey-axistoseethattherevenueforbeveragescoversfromabout35to47,meaning$12,000.

20.(B)Producehadarevenueof$30,000accordingtothegraph,andthetotalrevenuewasrightat$100,000.So,forJanuaryproducemadeup30/100or30%oftotalrevenue.

21.(A)Thetotalrevenueismeasuredbythetopcurveofthegraph,andthetopcurvewasthelowestinMarch,making(A)correct.

22.(C)ThelargestdropwasbetweenWeeks7and8wherethegraphwentfrom11hoursto3hours.

23.(A)Tocalculatetotalhours,findthehoursforeachweek.Youshouldfindthat3+5+7+2+6+8+11+3=45.

24.(A)Accordingthegraph,only2hourswerespentwatchingTVinWeek4,makingthatthelowestofallthegivendata.Thereisnoconcreteevidencefororagainstchoices(B),(C),and(D).

25.(C)Thisquestionisaskingaboutpercentageratherthandepth.BetweenJulyandAugust,sedimentmadeup100%ofgroundcovering,sincetheotherelementswerenotpresent.

26.(A)Thesedimentcurvefluctuatesbetween12and14inchesallyearlong,making(A)thecorrectchoice.(D)maybeappealing,butbecareful;theoverallgroundcoveringincreases,buttheamountofsedimentstaysprettysteady.

27.(B)InApril,theicecurvetakesupthe6to8inchpartofthegraph,soitsdepthisapproximately2inches.

28.(A)NoticetheNOTinthequestion.Sincethisgraphisdealinginpercentages,thereisnoevidencefortheactualclasssizeorwhatmightconstitutealargeclass.

29.(A)Inthisgraph,“green”takesup10%while“brown”takesup60%.Therefore,theratiois1:6orone-sixth.

30.(D)Ages0–14makeupthecategoryclosesttothex-axis,whichmakesiteasytoseethatZimbabwehasthehighestpercentagewithabout38%.

31.(B)Accordingtothegraph,Armenia’spercentageofpopulationbetweentheagesof15and24isabout15%,or34–19.

32.(A)Thegraphdoesnotprovideevidenceonhealthcareorbirthrates,rulingoutchoices(B),(C),and(D).Additionally,wecanconfirm(A)byseeingthatabout17%ofthepopulationismadeupofthose65andolder.TheonlycountryclosetothispercentageistheUnitedStateswithabout15%ofthepopulationbeinginthiscategory.

Chapter4.PracticeExercisesofIncreasingDifficulty

PRACTICEEXERCISEA

ANSWERSEXPLAINED

PassageA1

1.(A)Thepassageasawholedescribestheunusualexperiencesofaboyinthe“SatisHouse”andhowthoseexperiencescometochangehislife.Thepassagedoesnotgiveevidencethattheboyseducesthegirl.(C)and(D)arerelatedtodetailswithinthepassage,butnotaccuratesummariesofthemainidea.

2.(B)Thepassageprogresseschronologically,beginningwithPip’sfirsttimemeetingMissHavishamandEstella,andmovingtosequentialmeetings.Itisnotstructuredaroundspacesorthesignificanceofdetails.(D)istempting,butincorrectbecausethepassageistoldinpasttenseratherthan“flashbacks,”andthereisnoevidencethatthenarrator’sreflectionshavebeenalteredbytime.

3.(C)Lines3–7providethebackstoryforhowPipcametothehouseofMissHavisham.(A),(B),and(D)allcomelater.

4.(D)LayingisthebestchoiceheresinceitreferstoMissHavisham’selbow“resting”onthetable.

5.(B)MissHavishamisdescribedbythenarratorasstrange,sounusualiscorrect.(A)istheoppositeofstrange.(C)and(D)don’tworkbecausetheroomandthebridearedullandcolorless.Similarly,wordslikegrim,seclusion,andwitheredgiveevidencethatMissHavishamisanythingbutlively.

6.(B)givesdirectevidencethatthenarratorbelievesMissHavishamisstrangelooking.(A)givesdetailoftheappearanceofthehouseratherthantheinhabitant.(C)describesMissHavisham’semotionalstateratherthanherappearance.(D)doesn’trefertoMissHavishamatall.

7.(A)Thiscanbeseenbestinlines39–41and52–54.Pipbecomesawareofhislowlinessandbecomesdissatisfiedwithhimself,even“ashamedofhome.”(B)and(C)areoppositeofhisfeelings.Finally,(D)hasapositiveconnotationthatdoesnotmakesensehere.

8.(D)Lines52–54refertoPip’sfeelingsuponleavingMissHavisham’safterinquiringaboutEstella.He“wenthomedissatisfiedanduncomfortable.”(A),(B),and(C)don’tdiscussPip’sfeelingsabouthissocialand/oreconomicsituation.

9.(C)TheselinesrefertoPip’sfirsttimeleavingthegloomyhouseofMissHavisham.Here,hefeelsillateaseanddisquieted,becominginsecurewithhimselfforthefirsttimeinhislife.Theotheroptionsdon’tfitthismoodaccurately.

10.(D)Troubledisthebestchoice.(A)istoostrongofaword.(B)and(C)inaccuratelyportraythenarrator’sshameasapowerorstrongpoint.

PassageA2

1.(A)SojournerTruth,thespeakerofPassage1,isadamantthatwomencandothethingsmencando,anddothemwithoutaman’shelp.Thus,itisaccuratetosaysheisemphaticabout(A).Shedoesnotbelievewomenaredependentonmenorconfinedtothehomeasin(B)and(C).(D)isnotdiscussed.

2.(B)Predicamentfitsherebecauseline3referstowhitemenbeinginafix,oradifficultsituation.Choices(A),(C),and(D)refertosynonymsoffix,butdon’tpreciselyfitthismeaning.

3.(A)Lines15–16indicatethatmeasureofintellectisirrelevantonthebasisthatoneshouldbeallowedtopursuewhateveritistheyarecapableofpursuing.Forthespeaker,evenifsheisnotasintellectualassome,sheshouldbeaffordedthelittlesheisqualifiedfor.Thus,(A)istheaccuratechoice.

4.(A)Truth’sfirstreferencetoreligioninvolvesthereversalofanobjectionthatwomenareinferiorbecauseJesusChristwasaman.Inreply,shestatesthatChristcamefrom“Godandawoman,”attestingtowomen’ssignificance.Next,sheusesthestoryofEvetocontendthatwomenarestrongenoughtoupsetandrepairtheworld.Choice(A)istheonlyanswerthataccuratelyinterpretsheruseofreligiontoexhibitwomanlyimport.

5.(C)Lines20–22statethatthefirstwoman“wasstrongenoughtoturntheworldupsidedownallalone,”andthattheunitedfemalepopulationiscertainlystrongenough“toturnitback.”Thus,theselinesprovideevidenceofwomen’sstrengthandgivetheanswerforthepreviousquestion.Choices(A),(B),and(D)areincorrectbecausetheydonotrefertothespeaker’suseofreligiousteachings.

6.(B)Inlines29–31,CattarguesthatCongress’sapproachtosuffragecompels,orforces,womentobegtheirintellectualinferiorsforfreedom.Itisnotprecisetosaywomenaresuggested,demonstrated,orlistedtobegtheirinferiors.

7.(D)Accordingtothesecondpassage,“Thetimeforwomansuffragehascome.”Hence,immediatelyisthecorrectanswer.Noneoftheotherchoicesrefertoinstantaneousaction.

8.(D)Lines42–44statethatideasaremorepowerfulthanCongress,andthatthetimeforwomen’ssuffrage“hascome.”Hence,thesearethelinesthatprovidedirectevidenceforthepreviousquestion.(A)referstothelimitationsonwomen’sfreedoms.(B)indicatesthecurrentattitudeofwomen.And(C)urgesCongresstoactontheirpromises,butdoesn’tspecifyatime.

9.(B)Toparaphrase,Cattproposesthattheideaofsuffragewillonlygetstronger,whilethepoliticalpartythatrefusestoacknowledgeitwillfade.So,(B)istherightchoice.(A)istoolukewarm.(C)istooextreme.AndwhileCattisacknowledgingthedyingoutofapartywhorefusestoadoptwomen’sfreedoms,sheisnotdiscussinginternalsecurityasin(D).

10.(C)Bothspeakersareadvocatingforwomen’srighttovote,soequalityiscorrect.Hierarchyreferstoanunevenrankingsystem.Repressionis“tocontrolorholdbackbyforce.”Andisolationmeans“astateofsolitudeorseparation.”

11.(A)Truthadvocatesforgeneralsuffrage,andsheincludesallwomeninherspeech.Cattspecifiesatypeofwomanandcallsforimmediateaction.Bothwomenarepassionateandearnest.So,(A)isaccurate.ItiswrongtocharacterizeTruthascasualorCattaswithoutfirmgoals.AndwhileCatt’sgoalsarespecifiedanddetailed,Truthisnotlessopinionated.Andfinally,(D)isincorrectbecauseinnowayisTruthlessintense.

PassageA3

1.(C)Thepurposeofthispassageistwofold.Theauthorisinformativeaboutearthquakesgenerally,andalsofocusesmorespecificallyononeearthquakewhichdevastatedthenationofHaitiin2010.Thepassagedoesn’tfocusonHaiti’seconomyorculture,doesn’tdiscussseveralmajorearthquakes,anddoesn’tgetintothescienceofseismographs.

2.(B)Accordingtothepassage,amoderateearthquakewillcausedamagetoonlyolderbuildings,whileastrongearthquakewillcausedamageinmostbuildings.However,collapseisnotcommonuntilthelevelofamajorearthquake.So,(B)istheonlyplausibleanswer.

3.(C)Theauthorpointsoutthatbuildingscanavoiddamageinastrongearthquakeiftheyare“builttobe‘earthquakeresistant.’”Thus,(C)iscorrect.Whilechoices(A),(B),and(D)couldallbetrue,theyarenotevidencedinthepassage.

4.(C)Lines34–37specificallystatethatbuildingswhichare“earthquakeresistant”willremainintactwhilemostotherbuildingssufferrealdamageinastrongearthquake.(A)discusseslightearthquakesthatdonotcauseinfrastructuraldamage.(B)indicatesthatmoderatelevelquakeswillonlydamageunsoundbuildings.And(D)referstotheseverelevelofearthquakesthatwillcollapsebuildingsdespitetheirstructure.

5.(A)Mildisanappropriateword,sincethislinerefersto“slightearthquakes.”Theotherchoicesdonotaccuratelydepictalevelofseverityormagnitude.

6.(B)Accordingtotheauthor,lackofsoundinfrastructurewithinHaitiresultedinamajorearthquakein2010causingaberrant(unusualorabnormal)damage.Thepassageprovidesnosupportfor(A)or(D).Andwhile(C)explainswhyHaitiwouldbemorelikelytoexperienceearthquakes,thequestionisfocusedonwhyanearthquakeofagivenmagnitudecausedmoreseveredamagethanwouldhavebeenthecasewithanearthquakeofthesameintensityinanotherlocation.

7.(D)Lines46–50returntothetopicofHaiti,statingthat“thedamagewasmoreseverethanonewouldexpectduetothetypesofbuildingscommoninHaiti.”So,theselinestestifytothepreviousquestion.(A)underscoresthewidespreaddeathanddestructioncausedbytheearthquakein2010,butdoesn’taddresswhythedamagemightbeunexpectedforthequake’smagnitude.Finally,choices(B)and(C)merelyaddresstheclassificationsfordifferentlevelsofearthquakes.

8.(D)Line42refersto“greatearthquakes”whichcausemajordamage.Bigistheonlywordchoicetodepictanimmensesizeandstrength.AndunlikethepreviousWordsinContextquestion,thereisnotahigherlevelthatcouldbeconfusedwithbig.Boundlesscouldrefertoanimmensesizeorspace,butinaccuratelysuggeststhattheearthquakeisunlimited.

9.(B)A9.5earthquake—thehighestonthegraph—occurredinChilein1960,making(B)thecorrectoption.

10.(D)Permanentchangestothegroundgeographyonlyoccurinearthquakesatalevelof9.0orhigheraccordingtolines43–44.Thegraphonlyshowsthreeinstancesofsuchmagnitude,inChile,Indonesia,andJapan.

11.(B)Accordingtothequestion,theRichterscaleoperatesonascalethatmultipliesashakingamplitudeby10foreveryincrement.So,iftheHaitianearthquakewasa7.0andtheJapaneseearthquakewasa9.0,thereisatwo-incrementdifference—or10×10.Thus,(B)isthecorrectanswer.

PassageA4

1.(A)TheauthorconsidersthefateoftheEuropeanUnion,specificallydeliberatingonthecurrentconflictregardingGreece.So(A)iscorrect.Theauthordoesnotargue(B)ortakeupeconomictheoryasin(C).Last,Russiaisbutadetailwithinthepassage.

2.(D)ParagraphoneemphasizestheunpredictableviolenceofanentirecenturyinEuropewhichmakesitssubsequentsolidaritymoreunlikely.Sincetheauthoronlycoversonecentury,(A)isnotaccurate.Choices(B)and(C)misunderstandtheliteraryreferencetobetheauthor’spoint,whenreallyitisadevicetoinitiatethetopic.

3.(A)Lines6–9arethelinesthatdirectlyfollowthefirstparagraphandsuggestfascinationthatsuchaviolentinteractioncouldresultintheEuropeanUnion.Thus,theysupportthepreviousquestion.(B)introducesthecurrentconflictwithintheE.U.(C)detailsthedecisionthatmustbemadetoaddressthecurrentconflict.And(D)describesRussia’sunreliabilityasanally.Sotheremainingchoicesdonotaddresshowthefirstparagraphworkswithrespecttotherestofthepassage.

4.(C)Lines10–19givedetailsofthecurrentconflictinEuropeusingpersonificationtogenerallyoutlinewhat’sgoingon,so(C).Thedialogueisnotrepresentativeofactualconversations,rulingout(A)and(B).Andfinally,thepassagedoesn’trefertomilitaryconflictasin(D).

5.(D)Arbitercanbedefinedasapersonorpartywhohasauthorityoverorsettlesadispute.Inline16,theauthorpositsthemajorityofEuropeancountriesas“somewherebetweencharacterwitnessandneutralarbiter.”Punditmeansexpert.Legaterefersto“anemissary”or“representativeofthepope.”And(C)doesn’tsuggestanyauthority.

6.(B)Theauthorsuggeststhatthemostalarmingpossibilityofthecurrentconflictis“animpressionableGreece”thatwouldtakeboldmeasurestogainfriendsandresources.ThereisobviousuncertaintyaroundtheEUmodel,so(A)isnonsensical.And(C)and(D)arenotevidencedbythepassage.

7.(D)Lines37–38directlystatethatthebiggestworryinvolvesaGreece“desperatebothforalliesandaccesstotheircoffers,”therebyprovidingevidenceforthepreviousquestion.(A)simplystatesthediscrepancyofavolatilecontinentunifying.(B)referencesthecrossroadswheretheE.U.isnow.And(C)isamerespeculationofhowGermanycouldrespondifthingscontinuethewaytheyare.

8.(B)Lines25–27compareGreece’sexitto“thefirstdominotofallinaseriesofdevelopmentsthatcoulddestabilizetheregion.”Hence,wemightsaythattheGreekexitcouldprovokeotheradversecircumstances.Thematterisunresolvedatthewritingofthispassage,so(A)wouldn’twork.(C)ismuchtooextreme.Andfinally,thepassagedoesn’ttakesidesso(D)isincorrectaswell.

9.(C)Unfortunateworksherebecause“perverted”isusedtodescribethecommoncurrencythatGermanycouldimaginablyneglectinfavoroftheirownindependentcurrency.Choice(C)istheonlywordchoicethatsuggestsundesirability.

10.(B)Lines44–48posethequestionofwhethertheE.U.willsurvive.Anappropriateparaphrasingcanbefoundinchoice(B).Thepassagedoesnotindicateanarmsraceasin(A).TheconcernisnotactuallywhetherlectureswillbegivenabouttheE.U.,butinsteadifitwillbecomesomethingpurelyhistorical.Andfinally,thequestionofwhetherGreeceisaffordedaloanwasrisenearlier,andisasub-questiontothislargerquestion.

PassageA5

1.(B)Thepassageisveryinterestedinconveyingtheextensiveimpactsofthevastfieldofengineering,andissupremelyappreciativeofengineeringcontributionsthusfar.So,(B)isright.Whiletheauthordoesbringupengineeringasacareerfieldandoffersexamplesofbiomedicalcontributions,theyaredetailsratherthanthemesofthepassage.(D)isnotsupportedbythepassage.

2.(D)Lines10–11statesthatengineershaveexemplified“howtobegoodstewardsofourresources.”Beforethisstatement,theauthordiscusseshowagricultural,biological,andenvironmentalengineershavehelpedtocreateacleaner,safer,andall-aroundhigherqualityenvironmentforus.Hence,guardian,or“protector”istheclosestmeaning.Curatorsarekeepersofmuseums.Finally,bothattendantsandassistantsindicatehelpers,sotheyaretootepidforthiscontext.

3.(A)Theauthorwouldsurelyagreethatengineeringcanbeincorporatedintoeveryone’slifesinceheorsheseesitasanexpansivefieldwithunlimitedaffiliations.Heorshedoesnotadvocateforonebranchoveranother,rulingout(B).WhilethepassagedoesreferencetheWrightbrothers,theauthordoesn’tcompareintellectasin(C).Andtheauthordoesn’taddresscollegescholarships,making(D)irrelevant.

4.(C)Lines46–49indicatethatdespiteone’sinterestsordesires,thereissomeapplicabilitywithinthefieldofengineeringforeverybody,evenjustforfun.Thisprovidesevidenceforthepreviousquestion.(A)suggeststheexpansivenessofengineeringcareers.(B)deliversanintriguingchallengethatspecificengineersface.And(D)discussesrewardsofengineering.Althoughallthreeoftheseotherchoicesadvocateforengineering,noneaddressengineeringasahobby.

5.(B)Lookdirectlybeforethissentenceanddirectlyafterit.Dr.Seussisusedasaset-upfortheinfiniteareasandinterestsoftheengineeringprofession.Hence,(B)istheaccuratechoice.Theauthordoesnotbecomereminiscentasin(A).Althoughthebookisusedasatransition,thechangeoftopicwassignaledintheprevioussentence.Andsincehyperbolemeans“exaggeration,”wecanrulethisoutonthegroundsthattheauthorisnotexaggeratingthevariousfieldsofengineering.

6.(B)Lines21–23statethat“theoptionsaremany”forprofessionalengineers.Hence,(B)iscorrect.Whiletheotherchoicesareaddressedwithinthepassage,theyarenotengagedwithintheselines.

7.(C)Thequestionisaskingwhichlinesprovethatengineeringhasmadedailylifeenjoyable.Lines36–45considertheimplicationsofengineerson“communications,computing,andsports,”focusingontheentertainmentindustry.Paragraph2definesengineeringmorebroadly.Paragraph3expressestheenticementofthecareeritself.AndParagraph5describeshowengineeringcanbepartofeveryone’slifeevenifitisn’tone’scareerchoice.

8.(D)Inthisparagraph,theauthorisexplainingtheintellectualstimulationoflifeasanengineer.WhenheorshereferencestheNationalAcademyofEngineering,itistoprovideanauthenticexampleofthisstimulation.So,(D)iscorrectbecauseheorsheismakingacaseandaddingcredibility.

9.(D)Paragraph4discussesthecontributionsengineershavemadetomakinglifemoreenjoyable,specificallyinthedailyareasof“communications,computing,andsports.”Hence,(D)isright.Theauthorhasmovedawayfromdiscussingengineeringspecificallyasacareer,rulingout(A).And(C)isincorrectbecausetheauthorisconsideringhowengineersfromvaryingfieldshaveimpactedcommunicationsandsports,ratherthanfocusingontwotypesofengineers.Choice(B)istempting,buttheauthor’spurposeistoshowhowentirelyengineershaveimpactedcertainaspectsofourdailylife,nottoillustratehowourliveswouldbedifferentwithoutthem.

10.(A)Lines54–55statesthatwithoutengineers,“ourliveswouldbemoreimpoverished.”Thebestwordmeaningis(A),emptier.Insolventmeans“bankrupt.”Unproductiveindicates“barrennessorunprofitability.”Indigentis“poorandneedy.”So,alltheremainingchoicesimpreciselyimplythatoureconomicsituationswouldbedifferentwithoutengineers,whichisnotthepointthepassageismaking.

PRACTICEEXERCISEB

ANSWERSEXPLAINED

PassageB1

1.(A)ThepassageasawholeisJaneEyre’srecountofadreamandaseeminglysupernaturaleventthatoccurreduponherawakening,bothterrifyingomensthatsuggestanunhappyanduncertainfuture.(C)and(D)don’tdiscussthismainidea.(B),althoughtempting,isincorrectbecauseJaneissharingherexperienceratherthanaskingforadvice,andbecauseonlyoneofherforewarningstoriesissaidtobeadream.

2.(A)Mr.Rochester’sattitudetowardJanecanbeinferredthroughhislackofparticipationandinterest,andbyhisassumptionthattheintruderisaghostorfigmentofJane’simagination.Heinterruptsher,contradictsherdescriptions,andneversaysmorethanonesentenceatatime.Althoughheisskeptical,heisnotdepressedasin(B).Heshowsneitherpanicnorfearasin(C).Andthoughhelistenstohertelling,Mr.Rochesterdoesn’tdisplayanygenuinecuriosityinwhatJanesaysasin(D);instead,heseemsdeterminedtodismissherworries.

3.(C)Inthiscase,Janeisreferringtothefactthatshemust“holdonto”the“unknownlittlechild.”Theotherchoicesfailtocapturethepointthatshecannotlaydownthechild.

4.(B)Lines7–8statethatJaneisfirstmadeawareofMr.Rochester’spresencewhenshehears“thegallopofahorse.”

5.(A)TheselinesgivedirectevidencethatwithinherdreamJanefirstperceivesMr.Rochesterthroughasenseofhearing.(B)describesherattempttoseehimafterhearinghishorse.(C)tellshowherdreamends.(D)describestheappearanceofthemysteriousandfrighteningintruderwhotearsupherveil.

6.(D)Line18isusedtotransitionfromherdreamtotheghastlyandinexplicableoccurrenceafterwards.Itisnotadescriptionasin(A)and(C).Norisitademonstrationofrespectsince“sir”iscommonthroughoutthepassage,andthislineismoreaboutmovingfromoneideatothenext.

7.(A)Entityreferstoanobjectthatexistsinitself,likeabody,being,creature,ororganization.Thisistheonlychoicethatfitsforminthiscontext.Creationreferstoaformationratherthanaform,and(C)and(D)areincorrectchoicesbecausetheydonotrefertobeingsthatcanemergefromacloset.

8.(C)Lines26–28listthemostlikelyindividualstobeinJane’sroom.ConsideringMr.Rochester’sdismissalandindifferencetoJane’snarration,theselinesservetoruleouthisanticipatedresponseandattempttorevealtheseverityoftheincident.Itisalistratherthanadescription,so(A)and(B)don’twork.And(D)isn’tcorrectbecausethisoccurrenceisafter,notduring,Jane’sdream,andcanbeaccountedforbythetornveil.

9.(B)Thefigureisdescribedas“tallandlarge,”“fearfulandghastly,”witha“savageface”and“redeyes.”So,choice(B)ismostfitting.(A)isincorrectbecausetheformisdistinctivefromalltheindividualsinthehouse.(C)canbeeliminatedbecausethewomanisportrayedasabominableandimposingratherthanweak.AndalthoughJanecomparesthewomanto“theVampyre,”thereisnoevidencethatsheactuallythinksthewomanisabat.

10.(C)Lines43–44giveJane’sdescriptionoftheintruder’sfeatures,andsoserveasthebestevidenceforthepreviousquestion.(A)isadescriptionofThornfieldHallwithinJane’sdream.(B)iswheretheformfirstemerges,butisnotyetdescribed.(D)isametaphorusedtodepictthehorroroftheghost-likeintruder,butisnotveryusefulindepictingthewoman’sfeatures.

PassageB2

1.(A)Accordingtothepassage,whenmeatisheatedtheproteinsdenatureand“theirquaternary,tertiary,andsecondarystructuresbreakdown.”So,wecanassume(A),theprimarystructure,isleastaffected.

2.(D)Lines13–15specificallyaddresswhichstructuresareimpactedbycookinginordertomakethemeattender.Assuch,theyprovidedirectevidencetothepreviousquestion.Answerchoices(A),(B),and(C)afforddetailsofeachstructure,butdonotaddresswhicharebrokendownduringcooking.

3.(C)Lines12–13canbesaidtotransitionfromthedescriptionoftheproteinorganizationinmeattotheeffectsofcookingonthosestructures.Thus,(C)iscorrect.Asaquestionthatmovesfromoneideatothenext,itwouldbeinaccuratetosaythissentenceisadefinition,explanation,oranalysis.

4.(B)“Loosened”istheclosestmeaning,sinceline13referstotheinteractionsbecoming“weaker.”Inthefollowingsentences,theauthordescribesthisasabreakingdownwhereproteinsbecome“looseandstretchedout.”(A)and(D)inaccuratelyexpressanexhaustingloss.And(C)wouldindicatethatthebreakingdownofproteinsarousespity.

5.(C)Negligiblemeansinconsequential,sothisquestionisaskingwhichapproachwouldnotchangetheendresult.(A)removesmoisture,soitisconsequentialevenifitisanundesirableeffect.(B)locksinmoisture.And(D)wouldstilldenaturetheprotein,impactingtendernessandmoistness.Accordingtotheauthor,searingmeatdoesn’tinfluencetenderness,andso,(C)isthecorrectanswer.

6.(D)Lines29–30underscoretheauthor’sassertionthatsearedmeatbehavesthesameasnon-searedmeat,whichisevidencethatsearingisaninconsequentialpracticeincookingmeat.Choice(A)impliesthatovercookingreducesmoisture.Choice(B)addressesthefactthatacidsandbasescandenaturemeat,despitebeingundesirableinfoods.Andfinallychoice(C)provesthatbriningincreasesmoisture.Hence,allthreechoicesareconsequentialandarenotappropriateanswersforthepreviousquestion.

7.(D)Inline39,theauthorusesanidiominanironicway,statingthatdespitethedissension,chemistryisextremelyapplicabletocooking,inferringthatthisissomethingtothinkabout.(A)incorrectlyunderstandsitasstimulatingone’sappetite.(B)and(C)inaccuratelyassumeactionratherthancontemplation.

8.(B)Theauthordiscussesthecolorofmeatinlines31–36,attributingrednesstomyoglobincontent.Accordingly,cookingaltersthecompositionofmyoglobintotransformthecolortobrown.(A)isassociatedwithtenderness,notcolor.(C)referstotheorganizationoftheproteinstructurethatbreaksdownfortenderness,butagainisnotassociatedwithcolor.Finally,choice(D)isnotmentioned.

9.(B)Lines19–20statethathighamountsofcollageninconnectivetissuesareassociatedwithtoughercutsofmeat.Thegraphconsidershowmuchmeatisproducedviadifferentcookingmethods.Therefore,wearelookingforwhichmethodwouldeliminatethemostconnectivetissuesincethisiswhatmakesthemeatespeciallytough.So,(B)iscorrect.

10.(D)Broilingmeatyieldsabout85%,whilebraisingmeatyieldsabout66%.So,choice(D)istheonlyplausibleoption.

PassageB3

1.(C)Thebestapproachtothisquestionistolookatthecontext.Theauthorisdiscussingtheteacherasaguidewhoarousescuriosity,sowecaninferthatthisphrasedoesnotactuallymeanachangeinlocationoranovelundertakingasin(A)and(B).Instead,theauthormostnearlymeansthatateachercanencouragenewideasasin(C).Choice(D)isappealing,buttheauthorisreferringtoquestioningone’spreconceivedvaluesmorethanone’schoices.

2.(D)Lines16–17refertothefirstauthor’sopinionthataqualityeducationcreatesalife-longpersistentquestioner.Thus,theauthorwouldthinkcuriosityisthemostimportantthingforateachertoinspireinstudents.TheauthorofPassage2mightsay(A),butnottheauthorofPassage1.Furthermore,theauthorwouldwantstudentstosearchfortruthwithacertaindiligence,butonlyasaresultoftheircuriosity.

3.(C)Accordingtolines32–35,theauthorofPassage2hopestopreparestudentsforcollegeandthejobmarket,sochoice(C),bothprofessorsandemployers,makessensehere.

4.(B)Theselinesgivedirectevidence,advocatingforanendresultwherestudents“walkoutfeelingconfidentandpreparedwhetheritbeforgraduationtests,college-entranceexams,orthejobmarket.”ThisstatementsupportstheideathattheteacherwritingPassage2wouldvaluetheskillsetsdesiredbybothcollegeprofessorsandworkforceemployers.

5.(A)Idealisticmeans“relatingtonobleprinciples”or“conceivedasastandardofperfection.”Itiscorrectheresincetheauthorisindicatingthatherjobdescriptionseemsveryromantic,butisinfactverypragmatic.Whileitcouldmakesensetosayherjobsoundspassionate,loving,oremotionalinothercontexts,noneofthesechoicesmakesthecontrastbetweenhowitsoundsandhowitisputintopractice.

6.(C)Lines36–41conveytheauthor’sprioritytopreparestudentsforthechallengesahead“atallcosts.”So,wecanassumethattheauthorplacesanemphasison(C),rigorousexpectations.Thereisnoevidencethattheauthorvaluesphysicalexertionasin(A).Choice(B)morecloselyresembleswhattheauthorofPassage1mightstress.Finally,(D)usuallyreferstoaprocessbywhichonesupportsandgiveslogicalevidenceforanargumentorposition.

7.(D)Thisquestionisjustaskingfortheinstancesinthegraphwhereaskillsetislabeledatahigherimportancebycollegeprofessorsthanitisbyemployers.Boththethirdandfourthcolumnshaveahighervalueforthecollegethanworkforce,sochoice(D)iscorrect.Theotherchoicesallincludeaskillsetthatismorevaluedintheworkforce.

8.(D)Passage1canbegenerallycharacterizedashigh-mindedandvisionarysinceitrendersaphilosophyofeducationbasedoninspiration.Passage2isarealisticapproachtoadefined,butlimitedcurriculum.Sinceloftymeans“exalted”andpracticalmeans“hands-onorsensible,”choice(D)isaccurate.

9.(B)Besuretonoticethe“respectively”inthequestion,meaningthattheanswersmustfollowintheorderalreadymentioned.Thefirstauthorcouldbeaddressinganyoneinterestedineducation,whilethesecondauthorseemstobespeakingspecificallytoparentsofpotentialorcurrentstudents.Thus,(B)isanappropriatechoice.Cosmopolitanmeans“worldly”or“cultured.”Choices(C)and(D)havepossiblecharacteristicsflipped.

10.(A)Toapproachaquestionlikethis,breakitintotwoparts.Lines42–44indicatethattheauthorofPassage2prioritizestestpreparationandattributesstandardizedprowessforfuturesuccess.Now,considerhowtheauthorofPassage1mightrespond.Inlines3–4,welearnthattheauthorthinksadesiretolearnisfarmoreimportantthantheknowledgeitself,andinlines12–14,thataclassroomthatfailstoinspire,failsaltogether.So,(A)withdisgust,depictshowheorshemightrespond.Concernandambivalencearebothtooindifferent,whileappreciationisanoppositeresponsefromwhatwemightexpect.

11.(C)Choices(A),(B),and(D)allstateandsupporttheauthor’sposition,butdonotaddresshowheorshemightviewanopposingphilosophyofeducation.(C)istheonlyoptionthataddresseshowtheauthorfeelstowardadverseclassroomenvironments.

PassageB4

1.(B)Thepurposeofthispassageistoexploreresearchregardingthesciencebehindlove,so(B)iscorrect.Theauthordoesnotaddressmatchmakingasin(A).Nordoesheorshespendtimeaddressingbothsidesasin(C).Finally,thepassageisn’targuingforsocialreformorimprovement.

2.(D)Thefirstparagraphisverypersonalandemotionalinitsapproachtodescribinglove.Thesecondtakesaveryscientificandfactualapproachtolove.Sincethefirstisnotbasedonprinciplesofmorality,ethicalisaninaccuratedescription.(B)takesanopiniononwhichbasisismoresignificant,assuminganemotional,heartfeltdescriptionofloveistrivialorfoolish.And(C)indicatesthatthetechnicalapproachismisguided.Thus,(D)istheonlyaccuratechoice.Subjectivemeansbasedonpersonalexperience,whileobjectivemeansbasedonarepresentationoffacts.

3.(A)Line3iswithinthetraditionalthoughtprocesssurroundingwhathappenswhenonefallsinlove,andstatesthatthecouplehad“chemistry.”Here,theworddoesnotrefertoabranchofscience,buttoanaffinityorattractionbetweenpeople.

4.(D)Thequestionisaskingwhatdevicetheauthorgenerallyusestobuildacase.Theauthordoesn’tusepersonalstoriesoffallinginloveasin(A).Nordoesthepassageconcernitselfwithfindingerrorsinlogicorpresentingrhetoricalquestions.However,(D)iswidelyusedthroughoutthepassage.

5.(A)Oxytocin,accordingtothepassage,isknownasthe“lovehormone”andisassociatedwithattachment.Thus,itiscrucialforlong-lastingromanticlove.(B)isconnectedwithfeelingsoflustordesire.And(C)and(D)areconnectedwithattractionand“feelingsofgiddiness.”

6.(B)Inlines23–26,theauthorexplainsanexpert’sresearchonthethreebrainsystemsforloving,connectinglong-lastinglove,orattachment,with“oxytocinandvasopressin.”Hence,theselinesgiveevidenceforthepreviousquestion.(A)and(C)refertochemicalsthatinduceexcitementandattraction.(D)namesallthechemicalsinthebraindirectlyassociatedwithlove,butdoesnotdiscusswhichchemicalsareconnectedtolonger,seriousrelationships.

7.(B)Supplementistheclosestwordchoice.Lines21–22indicatesresearchthataffirmsthat“wechoosepartnerswhosebiologicalchemicalscomplementourown.”So,“complement”withinthecontextmeans“gowelltogether”ratherthanpraise,reveal,ormerelyinteractwith.

8.(A)Inlines42–43,theauthorsuggeststhatloveisoftencomparedtochemicaladdiction—inpoetry,music,andscience.Assuch,(A)correctlydepictsthepurposeofthelines.Theotherchoicesdon’tconsiderthevarietyofusesforthemetaphor.(B)incorrectlyinfersthatthemetaphorwithintheartsiscontributingtoscience.(C)isinaccuratebecausetheauthorisnotcallingthemetaphorflawed.Finally,(D)iswronginthatitattributesthemetaphortostylisticaspirations.

9.(C)Thegraphshowsmenciting“firstsight”atabout15%andwomenciting“firstsight”atabout8%.So,the11%figureinlines50–52mostlikelyaccountsforbothmenandwomen,sinceitwouldaveragethesenumbersout.

10.(D)Accordingtothegraph,themajorityofmenfallinlovebythe6monthmark,making(D)correct.Approximately14%ofmensaid“firstsight,”17%saidafter“threemonths,”and38%saidafter“6months.”14+17+38=69,whichisclearlyamajority.Thenumbersaresuchthatyoucaneasilyestimatethatitwouldbeamajority,sinceyouwon’thaveacalculatoravailableontheReadingTest.

11.(A)Thequestionisaskingifthereisconflictbetweenthegraphandthepassage.(A)istheonlychoicewhichpresentsatimeframeforevaluatingapartner,namely3minutes.So,sincethemajorityofpersonssurveyeddon’tfallinlovewithin3minutes,thegraphcouldpossiblyconflictwithFisher’sperspective.Choices(B),(C),and(D)donotspecifyatimeframeforthesciencebehindlove,sotheydonotopposethegraph’sfindingsinanyway.

PassageB5

1.(A)Tosummarize,theauthorusestherecentfinancialcrisistoexplorethesystem’sunjustnature,making(A)thecorrectchoice.Accordingtotheauthor,thesystemismoreatfaultthantheconsumers,rulingout(B)and(C).Choice(D)isincorrectbecausethepassagemakesnomentionofhow“universalprofitability”couldhaveoccurred.

2.(B)Subprimeloansarethoseextendedwith“littletonochance”ofrepayment.Yet,theauthorstatesinlines26–29thatthebanksoverlookeddefault,thinkingthattheextremeinterestswouldstillensure“hugeprofits.”Hesuggeststhatthebanksknewmanywouldn’tbeabletopaybacktheirloans,making(A)wrong.(C)and(D)arenotsupportedbythepassage.

3.(C)Lines26–29depictthe“banks’deludedthoughtprocess”inregardtosubprimeloans,andso,givedirectevidenceforthepreviousquestion.(A)introducesthethirdtypeofhumaninteractioninwhichonepartyadvancesatanother’sexpense.(B)givesanexampleofthattypeofinteraction.Andfinally,choice(D)presentsthemethodbywhichaveryinsightfulfewwereabletobenefitfromthecrisis.

4.(A)Inline15,theauthorreferstoasellerriddinghimselfofa“bloatedinstrument”whichwillcertainlydevalue.Hence,overvaluedisaprecisewordchoice.Whiletheotherchoicesaresynonymsofbloated,theydonotcapturetheauthor’smeaningasrelatedtoinflatedvalue.

5.(B)Inlines20–21,theauthorattributestheCrashof2008to“wildspeculation”andadisregardfor“soundfinancialtheory.”(C)and(D)canberuledoutbecausetheyarenotdiscussedintheselinesanddivergefromtheauthor’sargument.Choice(A)isappealing,butnoticethattheauthorclaimsthemortgageindustrywasthebestandbiggestexampleofthesefailures,so(B)isthedefinitechoice.Likewise,“homeowners”isinaccurate,becausethebanksownedthehomesuntiltheborrowerscouldpaybacktheloans.

6.(A)Theauthorexplainsthislineinthefollowingsentences,statingthatamajorityofborrowersdefaultedandthehouses’valuesplummeted,so(A)isthecorrectchoice.Thelinedoesn’trefertoactualphysicaldestructionorconstructionasin(B)and(C).Similarly,weknowvaluesgreatlyfell,making(D)incorrect.

7.(C)Inthisparagraph,theauthorindicatesthatafew“sages”wereabletoreversetheconsequencesofthefinancialcrash,while“theguardiansofthemoneyboxes”mistookcoalfordiamonds.Sincethesagesprofitedandtheguardiansdidnot,wecanassumetheguardiansarethosebankswhoweresupplyingsubprimeloansand“lostbillions.”

8.(B)Abstrusemeansdifficulttounderstand,soperplexingisthecorrectchoice.Thiswordiscoupledwithasynonymthatmighthelp:“convoluted,”or“complicated,”suggeststhemeaningofabstruse.Sinistermeans“menacing”andistoonegative.Subrosareferstosomethingbeingdoneinsecret.Andweightymeans“heavyorimpactful.”

9.(C)Theauthordefineszerosumasaninteractionwhere“anythinggainedmustcomeatthelossofanother.”(C)istheonlyoptionwhereonepartydoesnotsufferatthehandsofanother;itisanexampleofhissecondinteractiontypeinwhicheverybodywins.

10.(A)Lines3–5provideaverysimilarexampleofan“everybodywins”situation,andsogiveevidenceforthepreviousquestion.Choices(B)and(D)provideinstancesof“zerosum”sinceonepartyflourisheswhileanotherloses.Choice(C)isirrelevant.

PRACTICEEXERCISEC

ANSWERSEXPLAINED

PassageC1

1.(B)Basedonthecontextoftheexcerpt,thenarratorisdeliberatingonthe“mirthless”ceremoniallifeofapriest—thepaththathasbeendesignatedforhim—andanunpremeditateddestiny“elusiveofsocialorreligiousorders.”Hischoiceisnotbetweenholinessandcriminality,asin(A).Nordoesitconsistofmemoriesofdegeneracyasin(D).Finally,(C)isincorrectbecausehisdecisionisaboutmorethansimplelogicalconsistency—hewantshischoicetobeconsistentwithhisauthenticdesires.

2.(C)Thenarrator’scontemplationisabuildupofthoughtsandemotionsthateventuallyleadstohisrealizationinlines39–42.Itisnotasthoughhisthoughtsprogressspatially,sinceheisnothavingobservationsbasedonwhatheisgraduallyseeinginspace.Thepassageisnotstructuredwithregardtothepassageoftime,sinceithasflashbacks.Itisalsonotanargumentasmuchasaninternalreflection.

3.(D)Inthefirstparagraph,thenarratorattributeshis“unrest”tothesmells,sounds,andgloomyimagesofhistimeatClonglowes.Inthesecond,heimagineswhatlifewillbelikeifhecontinuestopursuepriesthoodandfindshisprognosisrepulsive.Noneoftheotherchoicesreflectsthesethoughtprocessesaccurately.

4.(A)Lines26–29givethemostdirectevidencetohowthenarratorunderstandspriests:“eyeless”and“sour-favoured”individualswith“suffocatedanger.”So,hewoulddescribethemasrepressingemotions,notcelebratingorexpressingthem.(C)doesn’tworkbecausethenarratorthinkstheangerisstillpresent,justsuffocated.

5.(D)Lines26–27bestdepictthenarrator’sconceptionofrepressedemotionswithintheclergy.(A)showsanxietyanddiscontent,butonlyinrememberingapiouseducation.(B)isareflectionofthenarrator’sthatshowsdivergencebetweenhisinnatecharacteristicsandthemanifestationsofpriesthood.(C)isahypotheticalimaginingofhisownselfinpriesthood,butdoesnotgiveevidenceofhowpriestshandleemotion.

6.(B)Inconsideringthecommunalnecessitiesofcontinuinghispiouseducation,thenarratorquestionsthevalueheplacesfirstonhisshynessandthenhispridefulseclusion.Thisisnotaphrasethatexpressesapproval,disinterest,oroptimism.

7.(B)Thisisbestseeninlines41–42whenthenarratorsayshewilllearn“apartfromothers”or“himselfwandering.”(A)describeshispreviouseducation,withwhichheisunhappy.Thereisnoevidencefor(C)or(D).

8.(D)givesthebestevidencefortheeducationalstylethatthenarratorwantstopursue,andthathethinkswillworkbestforhim.(A)describeshishostilitytowardaformalreligiouseducation.(B)isanegativeimagethenarratorconceiveswhenheimaginespursuingareligiouseducation.(C)doesn’tgiveevidenceaboutatypeofeducation.

9.(B)Thesurroundingcontextsupportstheideathatthepriestlylifedoesnotappealtothenarrator.Itmakessensetosaytheeffortsofthepriesttopersuadehimdidnotaffecthim“deeply.”

10.(A)Attheconclusionofthepassage,thenarratoryieldstohisfate.Heis“stillunfallen,butabouttofall,”sothebestdescriptionofthistonewouldberesignation.Terroristoostrong.Corruption,althoughrelatedtosin,isnotdepictiveofthenarrator’stone.Anddecreasingwouldbeusedimproperlyheresincethetoneisnotlesseninginsizeorstrength.

PassageC2

1.(D)Thephraseisprovidedasarelevantandrecognizableexampleinaseaofverytechnicalanalysis.Esotericmeanslikelytobeunderstoodbyonlyaspecializedaudience.(A)and(C)involveconceptsintroducedlaterinthepassage.Choice(B)isincorrectbecausetheexampleisusedtoextendunderstandingtolaypeopleratherthanimplytheycannotunderstandit.

2.(C)Fromthecontext,weunderstandthatsurfactantsreducesurfacetensionandcaninterferewiththestabilityofanoil-waterfilm.Therefore,ifitspropertiesallowasurfactanttointerferewithforcesthatmaintainintegrity,wecanassumetheproperwordchoiceiscohesion,or“theactofstickingtogether.”Rectitudereferstoanothermeaningofintegrity,like“virtue.”Decadenceis,then,theoppositeofrectitude.Andsolidarity,whileatemptingchoice,usuallyreferstounityamongagroupofindividuals.

3.(B)Lines11–16attributesurfactants’manyapplicationstotheiruniquepropertyofbeingamphiphilic.Whiletheyareusedoftenasemulsifiers,“emulsion”isnotthedistinctionthatleadstotheirpopularitybutamanifestationofthatdistinction.(A)refersspecificallytobiosurfactants,and(D)appliestosoapbutnotallothersurfactants.

4.(A)Lines13–16statethat“surfacetensionloweringagentshavefoundahostofapplicationsindiversecommercialproducts”asaresultofauniqueproperty.Theuniquepropertyhereisthattheyareamphiphilic,sotheselinesprovidedirectevidenceforthepreviousquestion.

5.(B)Thequestionisaskinghowmuchofeachsurfactantexists,andsincethemainobstacletobiosurfactantsisisolatingthemforindustrialproduction,wecanassumetheyarenotyetusedonalargescaleandwouldlikelybelesscommon.Then,accordingtolines46–48,detergentsarethemostproducedanddistributedofallcommerciallysynthesizedsurfactants.So,choice(B)iscorrect.

6.(A)isthecorrectchoicesincetheauthormakessuretodifferentiatesoapsanddetergentsdespitethefactthatthey“aresometimesusedinterchangeably.”(B)and(C)givemisleadinginformationthatdoesn’taccountforthepassage’sexplanationthattheyareneitheridenticalinstructurenorincreation.(D)iswrongbecauselines61–62attributetheobstacletoisolatingbiosurfactantsonalargeenoughscale.

7.(C)Lines37–38statethatthetwosurfactantsareerroneouslyusedinterchangeably,butthatunlikesoaps,“detergentsarenotsynthesizedbysaponification.”Then,theauthordevotesaparagraphtothedifferencesofdetergents.So,theselinesgivedirectevidenceforthepreviousquestion.(A)iswherethepassagegivestheoriginofthewordemulsion.(B)referencesthoselineswhichexplainthestructureofsoap,while(D)referenceslinesthatstatethatbiosurfactantsarestructurallydistinct;however,neitherchoicedifferentiatessoapsfromdetergents.

8.(B)Line53statesthatthecontroversyarounddetergentsis“unlikelytobesettled,”soresolvedisthebestanswerchoice.Itisnotaccuratetosaythecontroversyisunlikelytobefirm,disturbed,ormobilized.

9.(B)Soapbubbleswithaconcentrationof8.5%surfactantwouldmakeupalinebetweenthe5%and10%linesonthegivengraph.So,at10milliseconds,thatlinewouldbesomewherebetween30and45,making(B)theaccuratechoice.

10.(A)Thequestionissimplyaskingforthelinesonthegraphwhichshowthegreatestdifferenceinsurfacetension.Asweseeinthegraph,whenthepercentageislower,thechangesinsurfacetensionaregreater.So,since(A)hasthesmallestpercentages,itisthecorrectanswer.

11.(D)Don’tbeintimidatedbythisquestion.Toapproachit,lookatthex-axisofthegraphandnoticethattheincrementsfollowanexponentialscale,i.e.,theunitsaremultipliedbytenratherthanfollowingastandardlinearpattern.Wecaninferthatthislogarithmicscalewouldbeusedtoexpressalargerangeofquantities—inthisinstance,toseethechangesoverawiderangeoftime.So,(D)iscorrect.Alinearscalecouldstillbeaccurate,rulingout(A).Aproportionalrelationshipwouldnotrequireascaletocoversuchwideexpanse,so(B)isinaccurate.Andwecanassumethescalewaschosenforutilityratherthantoimpressreadershipasin(C).

PassageC3

1.(C)Asseeninlines3–8,Descartesbelievesthathehasacceptedfalseknowledgeandbuilthisownprinciplesonuntruths,soheisskeptical.Dogmaticreferstoaninclinationtolaydownprinciplesastruth—thisisthetypeofknowledgehewantstogetridof.Hisabilitytoacknowledgehisownpreconceivednotionsshowsthatheisnotnaïve,orcredulous.Andwhilefallacymaybewidespreadorpopular,thatdoesnotillustratehisattitudetowardknowledge.

2.(D)Lines8–13refertothefactthatDescarteshaswaiteduntilanoptimalagetotrytobreakdownhisfalseprinciples,making(D)accurate.(A)and(B)aredoneinotherlines,and(C)isnotsupportedbythepassage.

3.(B)Here,theauthorisindicatingthathehaswaiteduntilthisagepurposelytoensurethatheisready.So,(B)correctlydesignatesadvancedasrelatedtotime.Theotherchoicesdon’trefertotheauthor’sage.

4.(C)Lines20–21statethat,atminimum,Descartescanrejectaconceptifhefinds“somegroundfordoubt.”Hence,(C)ishisthresholdfordismissal.(A)and(B)suggestthatDescarteswouldhavetofindamoreconsiderableflaw.Finally,dismissingallknowledgeasin(D)isn’thisgoal.

5.(C)TheselinesgivedirectevidenceofhowDescartesplanstoapproachanyandallknowledgeclaims.Hestatesthathedoesn’thavetoprovethemwhollyfalse,butinsteadcanrejectanywithaprecariousexistence.Choices(A)and(B)arerelatedtotheactualobjectiveofriddinghimselfofadoptedbeliefs,butdonotconsiderhisrequirementtowhatisfalse.And(D)iswhereheexplainstheabsurdityofrebukingthemostfundamentalknowledge.

6.(D)TheselinesrefertoDescartes’acknowledgementthattherearesometruthsthatare“manifestlyimpossibletodoubt.”Asexamples,Descartespresentssensoryobservationslikehislocation,clothing,andcurrentactivitiesasoccurrencesofknowledgethathecannotrefute.So,(D)isright.Heisemphasizingthecertaintiesofhumanperception,ratherthanthepitfalls.Additionally,heisnotreferringto“logicalreasoning”or“tools,”butmerelytofundamentalsenses.

7.(A)HierarchalaccuratelydepictsDescartes’beliefthatknowledgeisbasedonanorderofrank.Tohim,certainformsofknowledgearesuperiortoothers;thoseofparticularsubserviencearebasedwhollyonfalse,predeterminedopinions.(B)isincorrectbecauseDescartesstatesthatweformprinciplesbasedonthoseopinions,sotheyareconnected.Indubitablemeans“unquestionable.”AndnonexistentwouldimplythatDescartesdoesn’tbelieveinknowledgewhatsoever.

8.(A)Inlines7–8,Descartesrealizesthathemustpurgeanyperemptoryknowledgeinordertosucceedinthesciences,sowecansafelyinferthathebelievesknowledgeofthesciences—foundedonproventruths—tobeofhigherquality.(B)canberuledoutbecauseitspecifiesthatitistimetoact,butdoesn’taddressanystructureofknowledge.(D)refersonlytoDescartes’examplesofsensorytruths.Choice(C)isappealingbecauseitsignifiesDescartes’intentiontodisprovefoundationalbeliefs,ratherthangothrougheachopinion.However,itdoesn’thintathowhemightgoaboutorderingclaimsofknowledge.

9.(B)Lines37–38areadepictionofwhateveritisthatcloudsthebrainoftheinsanetomakethemthinktheyarerichwhentheyarepoor,ortheyareclothedwhennaked,etc.So,itisrelatedtohallucinogenicvisions,notdemonicpossession,chronicdepression,ormathematicallogic.

10.(D)ExcessiveworksheresinceitisreferringtoDescartes’acknowledgementthatitwouldbeirrationaltoquestionsomeformsofknowledge,suchasthosebasicsensoryperceptions,especiallyifhedidsototheextentthatthepeoplewhohavedelusionsmentionedpreviouslyinthisparagraphdid.(A)isanotherdefinitionofextravagant,butreferstoexpensiveelegance.(B)istheoppositeofthiswordmeaning.And(C)doesnotreflectanextremeexampleasintended.

PassageC4

1.(B)Thispassageisexplanatoryandinformative,butfailstoargueorgivemedicaladvice.Theauthordoescomparehistoricalscientificparadigmstomorerecentdiscoveries,butthesearenotopposingsomuchasreplacingtheoutdatedbeliefs.

2.(B)Thefirstparagraphintroducesthetopicofgeneticmaterial,statesthattheresearchupuntilnowis“staggering,”andindicatesthatthereisstillalotmoretodiscover,so(B),providesawidercontext,istheappropriatechoice.(A)isadetailbutnottheobjectiveoftheselines.Theopeningparagraphmaystatethatthereare“manymysteries,”butitdoesnotillustrateourlimitedknowledgeinanywayasin(C).Itisnot(D)becausethereisnotadiscussionoftheauthor’sfeelingstowardadvancement.

3.(C)Accumulatedmakesthemostsenseherebecausewearediscussingthevolumeofknowledgethathasbeencompiledinthelastfiftyyears.Nooneisinquiring,fusing,orweighingtheknowledge.

4.(A)Inlines21–24,theauthordescribestheprocess,statingthattheintronsare“removedanddegraded”so(A)isthecorrectanswer.Theexonsaretranslated,butthewordexcisedisspecificallyreferringtotheintrons.

5.(D)Accordingtotheauthor,“itremainstobeseen,”sounsettledisthebestdescriptionfortheconnectionbetweenalteredsplicingandcancer.Skepticalmeansdoubtfulandistoonegativehere.Assuredismuchtooconfident.Andmalignantmeansdiseasedoruncontrollableandisusuallyassociatedwiththesevereeffectsofcancer.

6.(C)Wecanfindthenumberofproteinsbymultiplyinghowmanygenestherearebyhowmanyproteinstheycodefor.Line54tellsusthatthisparticulargene—thatcodesfor38,000proteins—is“anextremeexample.”Therefore,ifthegenomehasabout15,000genes,itishighlyunlikelythattheyallencodeforthatmanyproteins.Hence,wecanassumethatthetotalnumberofproteinsislessthan15,000×38,000.Infact,evenifthegeneswereallsimilarlyextreme,(C)wouldstillbecorrectbecausethetotalproteinswouldbeequalto15,000×38,000.

7.(B)Fromthecontext,weknowthatpre-mRNAdevelopsafterthetemplateDNAandcancodeforavarietyofproteinsthroughalternativesplicing.So,lookingatthetable,wecanassumethattheisoformsaredifferentcodesthatoccurredduringsplicing,making(B)thecorrectanswer.Itisn’t(A)becausethereisnoevidencethatthecodesalwaysoccurinthesamepattern.And(C)and(D)incorrectlyhavethepre-mRNAdevelopingfromtheisoformsratherthanviceversa.

8.(D)Lines37–39explainthephenomenonofalternativesplicinginaccessiblelanguage.Sincethisgivesdirectevidencethatstrandsofpre-mRNAundergosplicingtocodefordifferentproteins,itissupportiveofthepreviousquestion.Choices(A)and(B)arenothelpfulbecausetheysimplydefinethedifferentregionsofagene.(C)isn’tcorrectbecause,althoughitintroducespre-mRNA,itdoesn’tdiscussthesplicingthataccountsforthedifferentisoformsinthepreviousquestion.

9.(A)Lines49–54introducethescientificideathatalternativesplicingisessentialtomaximizingtheeconomyofgeneticmaterialandmaintainingdiversity.Sincethetableexhibitsseveralisoformsthatarisefromsplicinginonegene,itcanreasonablybeinferredthatthesesamescientistswouldsupportthedata.Itisn’t(B)becausethetabledoesn’tshowtheprocessandbecausethescientistsaremoreinterestedinitsimplicationsfortheeconomyofgeneticmaterialanddiversity.

10.(A)Accordingtolines46–48,thetraditionalunderstandingofgenes“heldthateachcodingregionwasresponsiblefortheexpressionofasingleprotein,”soabelieverinthistheorywouldassumethattheoneα-tropomyosingenewouldcodeforoneprotein.Thereisnoevidencetosupporttheotheranswerchoices.Likewise,theninealternativeformsinthetablemightbeanticipatedbyanup-to-datescientist,butnotsomeonewhosupportedthehistoricaltheoryofDNA.

11.(D)TheselinesservetodefinethetraditionaltheoryofDNAthatismostusefulinansweringthepreviousquestion.Theauthortellsusthatwepreviouslyentertainedasimplifiedunderstandingthatgenescodedforasingle,distinctprotein;thisreasoningsupportstheideathatsomeoneunderthatimpressionwouldexpectonlyonetranscriptinthetable.(A)and(B)providethebackgroundandframeworkforthetopic,butdonotaddressthehistoricaltheoryspecifically.(C)isnotcorrectbecausetheselinesmerelydiscusstheunresolvedpatternsfoundwithinsplicingandcancer.

PassageC5

1.(B)Inlines14–16,theauthorofPassage1makesitclearthatwhatqualifiesasliterature“arousesemotion,”butalsostatesinlines22–25thatemotioninandofitselfisnotenough.Therefore,answer(B)iscorrectbecausealthoughareader’semotionalresponseiscritical,itisnotthesolestandard.

2.(D)Lines24–25statethatreaderresponsealonecannotclassifyawork“asoneofthegreats.”TheselinesdirectlycontendthattheauthorofPassage1wouldargueforamoreprecisestandardthanemotionalresponse.(A)and(C)restatetheauthor’sargumentthatliteraturedemandsobjectivity.Choice(B),thoughtempting,expressestheauthor’sopinionthatliteratureshouldhelpthereadertounderstandtheworldbetter,butdoesn’tgiveevidenceofhisorherfeelingstowardemotionalreaction.

3.(C)Literaryrelativismworksherebecauseline2isreferringtoaninstancewherewritingiswithoutmeasureandis,therefore,preferential.Relativismistheconceptthatknowledgeandtruthdonotexistoutsidecultureandsociety,sochoice(C)wouldrenderanalternativewherethequalityofliteraturevarieswithperception.(A)isanelementthatshouldbeconsideredinmeasuringliterature,accordingtothefirstpassage.(B)isincorrectbecausethealternativewouldbesubjectivetoallinterpretation,notjustartisticinterpretation.Finally,linguisticimpartialitywouldimplyaneutralitytowardlanguage,so(D)iswrong.

4.(A)Artisticmeritreferstoexcellenceorworthinessofthearts,soqualityisthecorrectchoice.Choices(B)and(C)wouldberepetitive,andchoice(D)wouldindicateartisticintegrityratherthanvalue.

5.(D)TheauthorofPassage2believesthatthequalityofwritingis“entirelysubjective,”andbaseshisorherargumentoninconsistentgrading.So,choice(D)wouldruleouttheevidenceandunderminetheauthor’sargument.(A)wouldsupportPassage1.(B)and(C)don’taffecttheargumenteitherway.

6.(B)Toparaphraselines40–42,theauthorofPassage2understandsthatbasicacademicwritingrequiresatleastalooseorganization,whileanythingbeyonditissansguideline.Onlychoice(B)considerstheauthor’sacknowledgmentofcertaincontextualrequirementsdespitehisorheropinionthatwritingisgenerallywithoutmeasure.

7.(A)Accordingtolines49–51,areader’sevaluationis“contingentnotonlyonthereader,butthereader’smood,location,andevenonwhatthereaderhasrecentlyread.”So,wecaninferthatheorshewouldbelieveagradewoulddependonbothmoodandsettingasinchoice(A).Anevaluator’straining,pastfeedback,orfamiliaritywithdifferentrubricswouldnotinfluenceareader’sresponseasmuchasdispositionand/oratmosphere.

8.(D)Theselinesdirectlysupporttheauthor’sopinionthatseveralfactorsinfluencereaderresponse,so(D)istheproperoptionbecauseitistheonlyonethatconsidersaninstructor’sinconsistency.Choices(A)and(B)refertotheauthor’spersonalexperienceswithdiscrepancyinwriting.Choice(C)indicatesthatonlywritingextremescanbeagreedon.

9.(B)Contingentisdefinedas“subjecttochance”or“dependenton,”so(B)isthecorrectanswer.Itisnotappropriatetosaywritingstandardsaregrouped,random,ordisappointedonmultiplefactors.

10.(C)Bothauthorsacknowledgethatsomewritingisparticularlybadasin(C).Passage1contendsthatstandardsarepreciseandclearlymeasurable,whilePassage2grantsthat“particularlyterrible”and“particularlybrilliant”writingaretheonlyareasofconcession.Onlythefirstauthorwouldagreewith(A).Sincebothauthorsshowastrongopiniononthematter,neitherwouldsupport(B).Finally,thereisnoevidenceineitherpassagefor(D).

11.(A)Thefirstpassagearguesthatliteratureisatermreservedforwritingthatisartisticallysuperiorandtimeless,pursuing“beauty,purpose,andmeaning.”Thesecondpassage,ontheotherhand,postulatesthatstandardsofevaluationareillusoryandbiased.Therefore,(A)istheonlyoptionthatcapturestheoverallrelationshipbetweenthetwopassages.

PracticeTest1

ANSWERSEXPLAINED

1.(A)Thispassagecanbeparaphrasedbestbychoice(A).(B)isnotsupportedsincetherunningawayhasnotyethappened.(C)istempting,butultimatelyincorrectbecauseMr.Shelbyisnottryingtomakeaprofit,buttopayoffadebt.(D)isadetailbutnotasummary.

2.(C)FromtheconversationbetweenHaleyandMr.Shelby,itisclearthatHaleyisfirstandforemostabusinessman.Moreover,hewon’tbepersuadedtojustacceptTomaspaymentforthedebt,so(C)isanaccuratedepictionofhischaracter.Heisneitherempathizingnorpassive.Infact,heisnearlyuncompromising.Finally,choice(B)won’tworkbecausewecaninferbyhisnegotiationskillsthatheisnotanamateur.

3.(A)IncludecouldbeasubstituteforflinginsincethissentencereferstoHaley’srequestthatMr.Shelbyaddthesmallboytothedeal.Hurlwouldincorrectlyindicatetheactofthrowing.Involveistooweakforthiscontext.AnditwouldbenonsensicaltosaythatHaleywantedMr.Shelbytorelatetheboy.

4.(B)ThereisevidenceinthepassagethatMr.Shelbyisreluctanttosellhisslaves,anddoesn’twanttoseparatethechildfromhismother.Yet,histreatmentoftheboyintheseparticularlinesismorefittingforanadoredpetthanahumanchild,makingchoice(B)correct.ThepassagedoesnotsupporttheideathatMr.Shelbyisdishonestoraggressive.Andfinally,choice(D)iscontradictivesinceimpartialmeans“neutralordisinterested.”

5.(D)Itemmostcloselyfitsthemeaningofarticleinthisline,sinceitreferstoElizaasanobject.(A),(B),and(C)arenotfittingsincetheydonotrefertothewomanasacommodityasintendedbythespeaker.

6.(C)Lines84–86indicatethatuponhearingthenews,Tomisshockedanddejected.Despondencyreferstohopelessnessandjoylessness.Hisreactionisneitherpeaceful,relieved,norplayfulso(A),(B),and(D)areincorrect.

7.(D)Tom“collapsed”and“sunkhishead,”sotheselinesprovidedirectevidenceforthepreviousquestion.(A)and(C)refertoMr.Shelby’swife’sandEliza’sreactions,respectively.And(B)describesTom’sstatebeforehearingthenews,ratherthanafter.

8.(B)Asevidencedbylines41–42,Mr.ShelbyisclearandresolutethatElizawillnotbesold.Since,heagrees—albeitgrudgingly—tothesaleofTomandEliza’sson,itcanbeinferredthatheplacesahighervalueonEliza.ThepassagedoesnotprovideevidenceeitherwayforhisopinionofChloe.

9.(C)Lines37–42givethemostdirectevidenceofMr.Shelby’sdeterminationtokeepElizaregardlessofwhatHaleymightoffer.Choice(A)suggestsaslightfondnessforTom,andchoice(B)exhibitsasadisticaffectionforthechild;butinbothcases,Mr.Shelbystillagreestosellthem.(D)isirrelevantasitjustrevealsAuntChloe’ssurpriseatElizashowingupatnightunannounced.

10.(A)Itissafetoassumethatthe“lighttap”isindicativeofEliza’sdiscretion,asshemustbeverysecretiveaboutherplanstorunaway.So,itisfittingtosaythatsheistryingtoavoiddetectionasinchoice(A).(C)isincorrectbecauseweknowsheriskedvisitingTomspecificallytosharethenews.Choice(B)wronglyassumeshercautionisrelatedtooffendingTomandChloebyvisitingthemsolate.Finally,choice(D)inaccuratelyassumessheisworriedmoreaboutbreakingthelawthanbeingcaught.

11.(B)Lines13–19,lines47–50,andlines63–65allsuggestthattheaudienceisunknowinglymanipulatedbyadvertising.Theauthorappearstobespeakingtoageneralaudience,ratherthanafinanciallyeliteoneasin(A).Likewise,thereisnoindicationofwhattheaudiencemightfeeltowardstheirchildrenasin(D).(C)canbetemptingbecausetheauthorstatesthattheaudiencethinkstheyarelessinclinedtofoolerythanpreviousgenerations,butnotnecessarilyinregardtopolitics.

12.(D)Theauthorbeginsconversationally,hintingatalltheaspectsofculturethatareinfluencedbyadvertisingbeforemovingintoadetailedanalysisofadvertisingschemes.Rapportreferstoacloserelationshiporamutualunderstanding,making(D)correct.Neitherthebenefitsnorthehistoryofadvertisingisthoroughlyaddressed.And(B)isinaccuratebecausethetermsaredefinedbeforetheyareusedincontext.

13.(B)Thesewordsareusedtodenotefashionsubculturesthattheauthorusestointroducethemanifestationsofeffectiveadvertisinginaconsumeristsociety.Theyarenotusedtosuggestareformistagendaasin(A)or(C).Andtheauthorisactuallyimplyingthatthelabelsareconventional,making(D)incorrect.

14.(C)Lines110–113serveasawarningtothereadersto“usecaution”andnottoassumetheyare“unaffected.”So,distrustfulisindicativeofhowtheauthorwouldadviseconsumers.(A)istoopositive.(B)referstothepartofthemindoneisnotfullyawareof,butdoesnotdescribeanattitude.(D)meansaggressiveorhostile,andistooextremelynegative.

15.(C)Intheselines,theauthorurgesthereadertobeskepticalofadvertising,givingclearevidencethatoneshouldnotcandidlytrustwhateverissaidordepicted.(A)isarhetoricaldevicetoappealtothereaders.(B)merelystateshowubiquitousadvertisingis.(D)isanappealingchoicebecauseitdescribeshowaparticularschemeofadvertisingcreatesanattitudeinaconsumer,buttheauthorwouldcautionagainstthisreaction.

16.(C)Thisisastatisticintendedtoprovethatadvertisingispervasiveandsuccessful,so(C)iscorrect.Theotheroptionsdon’tconsiderthecontextaroundtheselinesinwhichtheauthortriestopersuadethereaderthatadvertisingiseffectivedespitecommonopinionthatoneisimmunetoinfluence.

17.(C)Methodsfitsthemeaningbestsincethisreferstoothermechanismsusedforadvertising.Choices(A)and(D)referto“amidwaypoint,”while(B)means“aspan,”or“thevariationbetweenupperandlowerlimitsorvalues.”

18.(C)Thisisapercentage-of-changeproblem.Tocalculatepercentchange,youfindthedifferencebetweenthevaluesandthendividebytheoriginalvalue.Here,thecombinationofpersonalandpsychologicalfactorsmakesup40%,comparedtothe60%ofculturalfactors.60–40=20and20dividedbytheoriginal40%gives0.5.Thisisequivalentto50%.

19.(A)Fromthegraph,weknowthatculturalfactorsaremosteffectiveinpersuadingconsumers.Fromthepassage,lines92–109,weknowthataffectiveconditioningassociatespositivefeelingsthroughappealingimagerywithproducts.Therefore,wecanassumethatacaradvertisementwouldusefeel–good,culturallyrelevantimagestotransferthosefeelingstothecaritself,making(A)correct.(D)isanexampleoflogicalpersuasionratherthanaffectiveconditioning.Andchoices(B)and(C)failtotransferfeelingsthroughpleasantscenes.

20.(A)Theselinesgivedirectevidenceofimagesthatanadvertisementmightdeploywhenhopingtoinfluenceconsumersthroughaffectiveconditioning.Choices(B)and(C)refertoanauthoritativesourceusedbytheauthortoillustratetheeffectsofaffectiveconditioning,butdonotdiscusshowitworks.And(D)isincorrectbecauseitsimplyconcludesthepassagewithawarningfromtheauthor.

21.(C)Ifaconsultingfirmwereadvisingarangeofindustriesonadvertising,itwouldbeimportantthatthisgraphbemorespecifictoeachparticularindustry.Forinstance,anentertainmentfirmmightusethesefactorsdifferentlythanacorporatebankingfirm.Additionally,psychologicalfactorsmightinfluenceaneducationalfirm’sconsumerstoagreaterextentthanaclothingfirm’s.(C)istheonlychoicethataddressesthegraph’sambiguitytowarddifferentindustries.Althoughprecisionisneeded,(A)failstogiveanynewinformation.(B)mightbehelpfuliftheindustriesweretargetingcertainsocialclasses,butthereisnoevidenceforthat.And(D)wouldonlybehelpfuliftheindustriesweretargetingcertainlocationsandiftherewasresearchtoprovegeographysignificantlyaltersaperson’sconsumerbehavior,bothofwhicharenotsupported.

22.(C)Itmightbesaidthatthepurposeofthispassageistoinform,making(C)correct.Afterintroducinghemoglobin,theauthorrelaysinformationaboutitsvitalroleinthehumanandthendiscussesthedisordersthatarisewhenitsproductiongoesawry.Theauthordoesnottakeapositionorappealforchange,making(A)and(D)incorrect.Similarly,thestyleisinformativewithoutraisingquestions,asin(B).

23.(A)Notorietyreferstofame,soitisthecorrectchoice.Infamyisaclosesynonym,butismoreassociatedwithbeingwellknownforabaddeed,whichmakesitincorrect;althoughthedisordersassociatedwitherroneoushemoglobinmaybeinfamous,hemoglobinitselfisnot.Personagerefersspecificallytoafamousperson.Festivityreferstoacelebration.

24.(D)Lines20–22statethatacombinationofpolypeptidechainsaccountsforthevarietiesofhemoglobin.Choices(A)and(B)refertocharacteristicsofthehemethateachtypeofhemoglobinhasincommon.Andchoice(C)isimprecisebecausemetalloproteinreferstocollectiveheme,nottothedistinctchainsassociatedwiththeglobin.

25.(D)Globin,thevariablepartofhemoglobin,isdefinedintheselinesandattributedtoeachtype’s“uniquecharacteristics,”givingdirectevidenceforthepreviousquestion.Choice(A)istempting,butitdefineshemoglobin’spartsmoregenerally.(B)and(C)bothrefertotheheme,metalloprotein,whichalltypesshare.

26.(B)Lines44–53statethatbecauseofhemoglobin’simportance,itisnotsurprisingthatitsmalfunctionscauseseriousdisorders.Yet,itissurprisingthatitserrorsaresocommonandoccurinsomanypeople.Therefore,choice(B)iscorrect.(A),althoughtempting,iswrongbecausetheauthorisnotsurprisedthatthedisordersarecausedbyhemoglobin,butthattheyaresofrequent.Thedisordersareinherited,notcontagiousasin(C).Andthepassagedoesnotgiveevidencefor(D).

27.(C)Widespreadmakesthemostsensehere.Theauthorstatesthattheerrors“areamongthemostcommon,”andfollowsthestatementwithempiricalevidenceoftheprevalenceofthesedisorders.Hence,he/sheisreferringtohowrecurrentandextensivetheyare.Communalmeans“tohaveincommon”or“share.”Lowlyindicates“inferiorityorbaseness.”Andfamousrefersto“beingwellknown.”

28.(C)Lines60–72considerthegeographicalspecificityofthalassemiadisordersandexplainthatitisnotcoincidentalbutlinkedtoevolution’sdefenseagainstmalaria,making(C)accurate.Thedistributionisnotlinkedtomigrationasin(A),orrelatedtoculturalphenomenaasin(B).Finally,(D)isincorrectbecausethereisnoconnectionmadetomedicalinnovationintheselines.

29.(A)Theselinescontendthattherangeofeffectsassociatedwiththalassemiaissobroadbecausethereisaredundancy,namely,fourcopiesoftheαglobingene.So,wecaninferthatthedisorderwouldbemoreuniform,orconsistent,iftherewerefewercopies.Theparagraphevenstatesthattheseverityisbasedonhowmanycopiesofthegenearedefective.(B)wouldnotchangethescopeofintensity.(C)wouldlikelyincreasethevariation.(D)canberuledoutbecausenotenoughevidenceisgiventoinferwhatmightdecreasethedissimilarity.

30.(B)Accordingtotheparagraph,onedeletioncausessubtlesymptomswhilefourdeletionsarelethal.Hence,wecansaythatasthenumberofdeletedgenesincreases,sodoestheseverityofthedisease.Thisrelationshipisproportional.Inversewouldimplythatasoneincreases,theotherdecreases.And(C)and(D)don’taccuratelydescribeacorrespondingrelationship.

31.(A)Thex-axisofthegraphbeginslongbeforebirth,withtheveryfirstweeksofpregnancy,sowearelookingforevidencetosupporttheideathathemoglobindevelopsrightaway.Lines5–7areappropriatebecausetheyrefertoitsappearance“withintheveryfirstweekofembryogenesis.”Neitherhemoglobin’scharacteristics(B)northeprevalence(C)andseverity(D)ofitsassociatedgeneticdisorderscanaccountforthegraph’sgestationaltracingofit.

32.(B)Tofindtheclearestindicationthatachildwasborntwomonthspremature,wewouldwanttodeterminethelevelofaparticularhemoglobinproteinthathadaneasilymeasureableanduniquevalueat7monthsofgestation—betaisapproximately20%atthispoint,andthisistheonlytimethatitmeetsthelevel.Itisnot(A)becausethealphamaintainsits7monthpercentforsometime.Itisnot(C)becausegammaisatits7monthpercentbothat7monthsandat5months.Itisnot(D)becausezetaceasestobeafactorafter3andhalfmonthsofgestation.

33.(C)JohnMuirisverypassionateinhisargumenttosavetheforests.Heisnotmerelytellingastoryordescribingnature.Additionally,hedoesnotmakeapointtoexaminecurrentknowledgeotherthantoconsidertheexpertiseofothercountries.

34.(A)Lines89–91bestillustrateMuir’stoneofurgency.Heisneitherarrogantnorcheerful.Andwhilethecurrentignorancesurroundingenvironmentalconservationmaymakehimdepressed,hispassageisacalltoactionratherthanahopelessrant.

35.(B)Lines17–19statesthatthewhiteman’saxesealedthefateofthetrees,sosettledismostappropriate.Thisuseofsealedrefersto“concludingorguaranteeingafate,”ratherthan“afasteningorclosing,”asin(A).Likewise,itisnotprecisetosaythatthewhitemanauthenticatedorkilledthedoomofthetrees.

36.(D)Accordingtothepassage,theEastandMidwestregionsoftheU.S.hadalreadybeenplundered,so(D),ThePacificRegion,iswhatisleftunharvested.

37.(D)Lines49–55indicatethattheharvestinghas“enteredthelastofthegreataboriginalforestsontheshoresofthePacific.”Assuch,theselinesgiveevidenceofwhichregionisstillabundantinforestsandprovidetheanswerforthepreviousquestion.Choices(A),(B),and(C)illustratewhichareashavebeenharvested,butdonotaddresstheregionwhichhasnot.

38.(C)Lines59–62specificallystatepossibleconsequencesofoverharvestingintheformof“scarce”timberwithoutnature“lefttorestinorprayin.”Thus,thenarratorappealstoeconomics,leisure,andreligion.Nootherchoiceencapsulatesallthreeofhisappeals.

39.(A)AccordingtoMuir,thegovernmenthasbeenineffectiveinconservingitsforests.Hence,wecanassumeithasnottakenmeasurestocreatelegislation,orinterveneinanyproductiveorliberalmanner.

40.(D)Lines81–84addressMuir’sevaluationofthegovernmentasa“foolishspendthrift”thathasleftitsforests“tobesoldandplunderedandwastedatwill.”Therefore,theselinesspecificallydescribehisunderstandingofthegovernment’sroleinforestmanagementuptothispoint.(A)affirmsMuir’sopinionthattheUnitedStateshadthebestforests.(B)referstothelandthathasbeendeforestedrecklessly.And(C)conveysaneffortbycitizensandconservationiststostoptheirresponsibledestructionoftheforests.

41.(B)Lines70–80contendthatothernationshavetakenmoreresponsibleapproachestonature.Specifically,MuirreferencesPrussiawhich“haslearnedthattheforestplaysanimportantpartinhumanprogress.”(A)and(C)arenationsthatMuircallsbarren,orbleakandunproductive.And(D)isnotaseparatecountry,butastatewithintheU.S.

42.(C)Devastationistheprecisechoice,sincelines89–91refertodesperatemeasuresthatmustbetakentoavoidruin.Heisadvocatingthatweavoiddevastationofthenationalforests,ratherthanthehostilityorignoranceofthem.Undoingisatemptingchoice,butusuallyreferstoaperson’sdownfall.

43.(B)Thequestionisaskingforthefirstauthor’sopiniononalternativeenergy.Accordingtolines1–6,theauthorbelievesoilwillbegoneshortly,andwemuststoprelyingonitorwitnesscompletechaos.Thus,theauthoradvocatesclearlyforapursuitofalternativeenergy.(A)wouldimplythattheauthorisuncertainbecausethechoiceiscomplex.(C)wouldinferasimilarinexactness.(D)istheopinionofauthor2ratherthanauthor1.

44.(C)Lines30–31referstothe“breakingoffatintothreeseparatechains,”soseparatingistheclosestwordmeaning.Floutingmeansdefying.Eliminationwouldimplythatthefatwasremovedinsteadofdivided.Andcategorizingisaclosesynonym,butindicatesaclassifyingofparts.

45.(D)Lines44–49callforimmediateaction,making(D)correct.Specifically,thepassagestates,“Thetimeforalternativefuelexplorationisnow.”

46.(D)Lines59–61metaphoricallysuggestthatagreatnumberofattemptsensureseventualsuccess,so(D)iscorrect.(A)istheoppositeofthissuggestion.Choices(B)and(C)failtocapturetheauthor’sideathateventhemosthaphazardeffortswilloccasionallyresultinawin.

47.(A)Lines82–83referto“apocalypticforecastingofpeakoil,”andcontrastitwiththecurrenthealthyoutput.So,wecaninfertheauthorissayingthateventhoughpeoplehavebeenpredictingthedepletionofoilforalongtime,itisstillgoingstrong.Hence,pessimisticisthebestchoice.Choices(B),(C),and(D)donotaccountforexpectationofabadoutcome.Furthermore,(D)wouldimplythatthepredictionswerepurposelyinsincere.

48.(B)Bycomparingoilpricestowaterprices,theauthorisattemptingtoshowhowlowthedemandforoilisinregardtoitssupply,making(B)right.Passage2isnotconcernedwiththedemandforwaterasin(A).(C)isfalseandunsupported.(D)isincorrectbecausetheauthordoesn’tattempttocompareutility,justprice.

49.(A)Youshouldworkbackwardsonthese.First,lines83–84statethat“petroleumoutputishealthy.”Now,considerhowthefirstauthormightrespond.Choices(B),(C),and(D)arenotvariable:petroleumisnotrenewable,alwaysgeneratesgreenhousegases,andislessexpensivethanethanol.(A)worksbecausethefirstauthorwouldwanttoconsiderhowmuchoilisavailabledomesticallysinceitimpactsourself-sufficiency.

50.(B)Lines19–21specificallyindicatethattheauthorofPassage1isinterestedinenergythatisnotonlyrenewable,butalsodomesticallymade.(A)introducesethanol,while(C)declaresalternativeenergiesmoreexpensivethangasoline.(D)isatemptingchoicebecauseitreferencesself-sufficiencyasadesirablequality,buttheselinesserveasmoreofanevaluationofrisksthantoshowspecificallywhatkindoffuelthatmostintereststheauthor.

51.(C)Again,workbackwards.Inlines78–81,Passage2questionswhywearesoadamantaboutreplacingsomethingthatwestillhave.Now,considerwhatlinesinPassage1couldsupporttheideathatweshouldsticktooilforthetimebeing.Lines21–25affirmthatethanolwouldnotonlybemoreexpensivethangasoline,butalsoraisefoodprices.So,theselinesserveaseffectivereasoningofwhywemightconsiderstickingwithoiluntilwedon’thaveachoicebuttouseanalternative.

52.(C)Passage1believesthatoilwillbegonesoonandarguesforanimmediatepursuitofalternativefuels.Passage2acknowledgesthatwewillhavetoreplaceoileventually,butsaysthereisnohurry.Hence,(C)istheappropriatechoice.Thepassagesarenotmainlyconcernedwitharguingaboutemissionsorfunding.(B)isnotmentionedineither.

PracticeTest2

ANSWERSEXPLAINED

1.(A)Beatriceisdescribedasexceptionallywealthy,charming,andcultured;yet,theauthormakessuretodepictherassuperficialandidiosyncratic.Eccentricmeans“unconventional”or“unusual,”so(A)accuratelyexpresseshercharacter.Beatriceiscertainlyhaughtyor“arrogant,”butsheisnotmean-spiritedorhateful.Sheiswealthy,butnothard-working.Anditinerant,meaningtraveling,describesBeatrice,whilemisanthropic,meaning“unsociable,”doesnot.

2.(B)TheopeninglinesstatethatthetraitsthathavemadeAmoryworthwhilecamefromhismother.(A)isincorrectbecausethenarratorassertsthatAmory’sbestqualitiesarethosethathereceivedfromBeatrice,i.e.,hismom.(C)isincorrectbecauseStephenpasseddownonlyhisheightandindecisivenessaccordingtolines11–15.(D)isnotsupportedwithinthepassageatall.

3.(D)ThesecondparagraphisacompellingdescriptionofAmory’smother,Beatrice.Emphaticmeans“forcefulordonewithemphasis,”soitexemplifiestheparagraph’sstyle.Itisnotgloomyandeducationalasin(A),vagueandintellectualasin(B),ornaïveandhopefulasin(C).

4.(A)Lines30–49refertohowBeatrice’syouthispassed,sospentistheappropriatechoice.Itisnotcorrectusagetosayshegaveup,tossed,oragreedheryouth.

5.(B)Lines50–53claimthatBeatricemarriedStephenbecauseshewas“weary”and“sad,”making(B)thecorrectanswer.Thereisnoevidencethatshefellinlovewithorwasforcedtomarryhim.Likewise,vengeanceimpliesretaliationorapunishmentforawrongdoing,whichisnotmentioned.

6.(C)TheselinesprovidethemostdirectevidenceforBeatrice’smaritalmotivations.(A)referstoAmory’sfather’schoicesratherthanhismother’s.(B)describestherelationshipofAmory’sparentsaftertheyaremarried.Andchoice(D)isunrelatedasitisadialoguebetweenBeatriceandAmoryregardingthemeritsofinactivity.

7.(D)Lines72–80indicatethatAmory’supbringingisunlikeother“littlerichboys”andthathiseducationis“highlyspecialized,”makingchoice(D)correct.Choices(A)and(C)arenotsupportedsincethepassagedescribesAmory’syouthasoneofadvantageandleisure.AndwhileAmorywascertainlyeducated,hisknowledgecamefromatypicalexperiencesratherthanthemoreconventionalscholarlymethods.

8.(C)Lines72–80providethebestevidenceforAmory’sone-of-a-kindchildhood.Thenarratordifferentiateshimfromeventhosechildrenofhissamesocialandeconomicclass,makingitclearthatAmory’sexperienceisanomalous.Choice(A)referstoStephen’sroleinthefamily.(B)isincorrectbecauseitaddressesBeatrice’seducationratherthanherson’s.Finally,(D)isanexampleofBeatrice’squirkymoralitythatsheattemptstopassontoherson,butisnotreflectiveofhisupbringingoreducationmoregenerally.

9.(B)Inthiscontext,thenarratorisreferringtothefactthatAmoryandhismotherwanderedthecountryforseveralyearsofhischildhood,sotraveledmakesthemostsensehere.Itisnotaccuratetosaytheymade,caused,ororganizedthecountryinherfather’sprivatecar.Thefollowingsentencesrefertoacoupleofthelocationstheyvisitedduringtheirtravels.

10.(B)Amory’srelationshipwithhismothercanbebestseenthroughtheirdialogueattheendofthepassage.Amory’suseofhismother’sfirstnameindicatesthattheiraffiliationisunceremonious,whileheranxietyoverhimrisingtooearlyshowsthatitisakindlyrelationship.So,(B)worksbesthere,wherefriendlyrefersto“approachableandinformal.”Usinghismother’sfirstnameisfarfromtraditional,asin(A).Andsinceshefindsitagreeable,thereisnoevidencethatAmoryisdisrespectfulasin(C).Choice(D)canbetemptingbecauseAmorynotonlyusesBeatrice’sfirstnameinaddressingher,butthenarratoralsotellsusthatAmoryholdsnoillusionsabouthisvainmother.However,coldordistantistoonegative.

11.(B)Thepassagearguesthattheadvantagesofsocialmedianeedtobeconsideredintheworkplace,so(B)isthecorrectchoice.Theauthorprovidesevidenceof(A)tosupportthethesis,butitisnotthethesisitself.(C)representsageneralargumentthatthepassagequestions.And(D)isnotdiscussed.

12.(D)Lines101–112makeuptheconclusionparagraphwheretheauthorarguesthat“itisn’tassimple”asacomprehensiveprohibitionofallsocialmedia,andsupporttheauthor’sargumentthatsocialmediacanbeveryusefulintheworkplace.Choices(A)and(B)indicatethestandardnegativeviewofsocialmediaatwork,whilechoice(D)merelygivesoneexampleofitsutility.

13.(A)Theauthorisinterestedinshowingbothsidesofthesocialmediaconflict,anddoessowithoutadvocatingforchange,butinsteadencouraginganinclusiveapproachtothedecision.Choice(A)istheonlychoicethataddressesboththebenefitsanddisadvantages,whilestayingtruetotheauthor’sinformativecaution.

14.(B)Lines1–11provideexamplesofsocialmediauseintheworkplacethatcouldbepotentiallyvaluabletothecompany,so(B)iscorrect.Theselinesdonotprovidepointstobelateranalyzed,examplestoalignthereaderwithsmartphonelanguage,orpersonalexperiencesoftheauthor.

15.(D)Theselinesprovideatransitionfromthecommonargumentsagainstsocialmediatothediscussionoftheprefacedexamplesthatposeitspossibleperks.(A),(B),and(C)don’taccountforthattransition.

16.(C)Theauthor’snumberoneadvantagetosocialmediauseisnetworkexpansion,sowecaninferthatagrowingcompanythatneedstoextenditsprofessionalnetworkingandmarketingwouldbenefitthemostfromsocialmedia.Whiletheauthorwouldsupporttheideathattheotherchoicescouldbenefitaswell,theimpactisnotasclear.

17.(D)Lines60–63statethat“startupbusinesses”specificallyfindsocialmediauseful,andtherefore,supportthepreviousquestion.Choices(A)and(B)refertoundesirablesocialnetworking.Finally,(C)suppliesthetransitiontotheprosofsocialmedia,butdoesnotsuggestaparticularbusinessthatcouldprofitfromit.

18.(B)Line67statesthatsocialmediacanbe“anunrivaledmediumformarketresearch,”making(B)theprecisewordmeaning.Itisnonsensicaltosaysocialmediaprovidesanunrivaledbestormiddle.Andwhilestandardisthenextbestchoice,itrefersmoretowhatisacceptedasbest,ratherthananavenuefordoingsomething.

19.(A)Areaisthecorrectchoice.Here,thepassagepresentsasituationwhereacompanyusesasocialmediapageasa“space”forstaffsupportandappreciation.Itisinaccuratetosaythataclearingorseparationiscreated.Andwhiledimensionisatemptingchoice,itimpreciselyrefersto“measurement”or“alevelofexistence.”

20.(C)Thegraphindicatesthatwomen,thoseintheagegroupof18–29,andthosewhohavecompletedsomecollegearethemostlikelycategorizationstousesocialnetworkingsites—choice(C)istheonlyoptionthatfitsallthesecriteria.Choices(A)and(B)bothhavemales,andchoice(D)usesanagegroupthatisoutsidetherangegivenbythegraph.

21.(A)Thepassageusestheselinestoexpressthatasocialmediapresenceisessentialforstartupcompaniestogrow.Sincethegraphindicatesthatthoseintheagegroupbetween18–29arethemostlikelyagegrouptousesocialmediasites,itisreasonabletoconcludethatifstartupswanttoreachthecustomersmostlikelytopurchasetheirproducts,theymusthaveasocialmediapresence.Itisnot(B)becausethiswouldmakecreatingsocialmediaappealsfarlesspromisingasamarketingstrategy.Itisnot(C)or(D)becausethereisnoindicationaboutthedemographicsofthesegroups,makingitimpossibletousetheinformationinthegraphtoconnecttothem.

22.(A)Lines1–12donotparticularlyspeaktowardtheauthor’sargument,butinsteadareusedtoshowhowapersonalchoicecanputoneatveryserioushealthrisks.Sincetheauthorwagesthatthedecisiontonotvaccinateisevenmoreridiculousthanthedecisiontosmoke,theselinesareeffectiveatsettinguptheargument.Lungcancerisnottheprimarytopicnordotheselinesgivestatisticsforvaccinationresearch.Andwhilechoice(D)istrue,itdoesnotshowhowthelinescontributetotheauthor’sargumentasawhole.

23.(B)Lines19–27expresstheperceivedviewsofthosewhodonotvaccinate,sothattheauthorcangoontorefutetheseviews,making(B)correct.Theselinesdonotsupportorclarifythepassageargument,noraretheyconnectedwitharespectforopposition.Infact,theauthorispassionatelyagainstthechoicetonotvaccinate.

24.(B)Here,theauthorstatesthatsmallpoxviruswasextremelydangerous“beforeitwaseradicatedbyvaccination,”resultinginentireculturesbeingeliminatedintheAmericas.Thecontextdoesn’tsupportthattheviruswastransportedinanyway.Similarly,whileitmaybeaccuratetosayvaccinationsfoughtandaffectedcultures,neitherwordchoiceindicatesthedestructionofwholecultures.

25.(D)AccordingtoPassage2,unvaccinatedpersonsareparticularlysafeinavaccinatedpopulationbecausetheyareprotectedfromthevirusesthemselvesthrough“herdimmunity”andfromadversevaccineevents.

26.(D)Lines98–102statethataslongasAmericansingeneralarevaccinated,the“unvaccinatedindividualwillbewell-protected,”making(D)thecorrectchoice.Choice(A)rulesoutwhattheauthorseesasadelusionalreasonfornotvaccinating.Andchoices(B)and(C)discusstherarerisksofvaccinating,butdon’talludetothecircumstancesunderwhichtheunvaccinatedaresafer.

27.(D)Doawaywithworksbestherebecausethelineissayingthattheauthorwantstoruleout“fallacies”and“delusions.”Theauthorisnotspendingtimetryingtounderstandthedelusionsasin(A).(B)istooliteral.(C)istoolukewarm.

28.(D)Toapproachthisquestion,considerthatthefirstauthorfindsnoexcuseforavoidingvaccinations.Ontheotherhand,thesecondauthorshowsthatchoosingnottovaccinateone’schildrencanbeanevensaferchoiceinaplacelikeAmericawherethemajorityofthepopulationisvaccinated.Thus,(D)istheonlychoicethatdescribesarelationshipwherePassage2explainsanunforgivableinstanceinPassage1.

29.(B)Thefinalsentenceofthefirstpassagesuggeststhatvaccinationsaretheonlyoptiontoprotectourchildren.Passage2acknowledgestheneedforgeneralvaccination,butultimatelyunderstandsthatinavaccinatedpopulation,aparentcanactuallyprotectachildfrommorecommonrisksviathechoicetonotvaccinate.So,(B)isrighthere.Bothagreethatvaccinationismorewidespreadthannot.AndtheauthorofPassage2stillrecognizestheneedforthegeneralpopulationtobevaccinated,rulingouttheotheroptions.

30.(D)Lines92–97refertothesecondauthor’sacknowledgementthattherisksofvaccinesareactuallymorecommonthanthediseasestheyprotectagainstinAmerica.Therefore,theselinesprovideevidenceforhowtheauthormightrespondtothestatementthatunvaccinatedchildrenarelessprotected.(A)introducesthefoundationforaparent’schoicetonotvaccinatebutdoesn’tgivedetail.(B)broadlyconsidersrisks,butagaindoesn’taddresstherisksofvaccination.And(C)isadescriptionofonevaccinationriskthatdoesnotincludeitsprevalenceincomparisontothevirusitvaccinatesagainst.

31.(D)Passage1seesnoreasonforaparenttonotvaccinate.Passage2acknowledgesavalidreasonforaparenttonotvaccinate.Hatredandloathingaretoostrongforthefirstauthor’sdisdain,butsurelyheorshedoesn’tmakeroomforforgiveness.Likewise,appreciationorgratitudewouldbetoopositiveforthemerecomprehensionofauthor2,whileintoleranceismoredescriptiveofauthor1.

32.(A)Emerson’spurposeistopresentsupportforEmancipationasin(A).Whileslaveryisobviouslyrelated,thispassagedoesn’tworktoillustrateitsevilsasin(B).Likewise,choice(C)isadetailusedtoargueforEmancipation,butisnottheobjective.Andfinally,moralityisthebasisforEmancipationaccordingtoEmerson,butagain,isrelatedtohisgeneralpurposeasathinkerratherthanthepurposeofthepassageitself.

33.(B)Thiscanbestbeseeninlines68–72,whichstatethatpoliticalstruggleisnottheend,“butonlythemeans.”Thus,(B)isthecorrectchoice.Thereisnoevidenceforchoice(A).Emersonwoulddisagreewithchoice(C)accordingtolines1–2.Andfinally,asfor(D),Emersonbelievesamoralgovernmentwouldsettlethequestionofslavery,butnotnecessarilyanypoliticalbody.

34.(D)Lines69–72indicatethat,forEmerson,Americandemocracyisnotimportantinandofitself,butonlyusefulinachievingabiggeraim.Thataim,hecontends,ismorality;andinthisspecificinstance,emancipationrepresentsthemoralchoice.Therefore,theselinesbestexpresshisoverallattitudetowardtheroleofpolitics.(A)and(C)givedetailedsupportforemancipation,while(B)suggeststhatAmericaisadivinepower’slastchanceatgettinghumanityright.

35.(B)Inthecontextofthefirstpassage,theselinesservetoillustratetheeffectsoftheCivilWar.Welearnthattheatmosphereisoneof“anxiety”and“sternduties.”Choice(B)istheonlyoptionthatfocusesonthemisfortunesofchildrenasbeingcausedbythewar.

36.(C)Accordingtolines11–12,thestateofthecountryinduces“sternduties,”soseriousisthecorrectchoice.Acountryatwarwouldnotconnecttoanythingplayful.(B)istoonegative.Choice(D)wouldinaccuratelyindicatethatwhat’satstakeispointless.

37.(A)Thequestionsinlines26–36rhetoricallyappealtoasenseofdutyandjusticeassociatedwithAmericandemocracy,makingchoice(A)correct.Wordsintheselineslikepractical,heroic,action,andwillemphasizetheauthor’surgency.Choice(C)isincorrectbecausethequestionsexpandonthepreviousparagraph’sreasoningratherthancritiqueit.And(B)and(D)arenonsensicalbecausethisdeviceisreflectingontheapplicationofdemocracyandcallingforaction,notarousingcuriosityorsummarizinganother’sthoughts.

38.(C)Toapproachthisquestion,considerhowEmersonintroducestheconflictofAmericanCivilizationastheincompatibilitybetween“twostatesofsocietyunderonelaw.”He,then,makesthecasethatdemocracyhasaresponsibility,thatfollowingthetraditionofslaveryisnottheAmericandestiny,andthat“emancipationisthedemandofcivilization.”Hence,wecansaythatheframestheconflictbetweenwhatislong-establishedandwhatisethical.(A)isincorrectbecauseinternationalrelationsarenotthoroughlydiscussed.AccordingtoEmerson,bothaspectsof(B)wouldbesolvedbyemancipation.(D)usesdetailsofEmerson’sargument,butdoesnotconsidertheunderlyingaspects.

39.(C)Choices(A)and(B)introducethenation’sstateofrestlessdisillusionment.Choice(D)givesanexampleofhowemancipationwillsolidifytheNorthandSouth.Noneoftheseoptionsprovidesevidenceofaphilosophicalstruggleframingthenation’sdecision.(C)istheonlyanswerthataddresseshowEmersonunderstandsthefundamentalconflict—anopportunityformoralitythatdivergesfrom“aliteralslavishfollowingofprecedents”—andthus,affirmsthepreviousquestion.

40.(D)ObeyingisappropriateheresinceEmersonisarguingthata“followingofprecedents,”orarepeatingofhistory,isn’tbefittingofAmericancivilization.Ensuingmeans“tohappenasaresult.”Succeedingreferstocomingaftersomethingintime.Likewise,resultingisasynonymoffollowingthatdoesn’tindicateaconformingtopasttradition.

41.(D)Toparaphraselines55–56,emancipationwillchangeSoutherncultureinwaysthatwillunifythenation.Thus,(D)bestcapturesthisidea.Emersononlymentionsimmigrantstoarguethatemancipationopensuplaboroptions.Choice(B)isadistortedexaggeration.ASouthernConstitution,asin(C),wasnevermentioned.

42.(B)Considerthattheauthorposesaquestionandthentalksaboutvariouswaysscienceisattemptingtoanswerthisquestion.So,(B)accuratelydescribestheauthor’spurpose.Theauthordoesnotargueonewayortheotherasin(A).Thereisnotanestablishedconsensus—infact,opinionsdiffergreatly,rulingout(C).Andtheissueitselfismuchmorescientificthansocialasin(D);furthermore,hedoesn’tconsideranyalternativeapproaches.

43.(B)Theauthorallowsthatthesearchisextensive,butthatopinionvaries“fromcertaintytoskepticismtodownrightincredulity,”givingevidencefor(B).Theotheroptionsinaccuratelyindicatethatthereisascientificagreementofsomesort.

44.(B)Lines16–20providedirectevidenceofanunsettledattitudetowardaquestionthatisbeingexploredintensely.Hence,theyworktosupportthepreviousquestion.(A)iswheretheauthorposesthequestionconcerningETI.(C)consistsoftheirregularimaginingsofwhatETIcouldlookandactlike.Finally,(D)referstonewmethodsofETIresearch.

45.(A)Line7referstothephenomenondescribedintheprecedinglines,namelytheinconsistencyinnothavinganyproofwhatsoeverofotherintelligentlifeinauniversesovast.(A)accuratelyparaphrasestheFermiParadox.Choices(B)and(C)implythatwehavealreadyhadcontactwithalienlife.Meanwhile,choice(D)proposesthatwearenotsearching.

46.(D)Incredulityistheinabilitytobelievesomething.However,evenifyoudidn’tknowthat,thislineisaspectrumofopinionaroundtheexistenceofalienlife.So,ifthespectrumstartsatcertainty,wecaninferthewordwearelookingforhastheoppositemeaning.Hence,(D)iscorrect.(A)istoomild,and(B)isnotanoppositeofcertainty.(C)isnotanattitudetowardsomething,butimpliesgoodjudgmentandintelligence.

47.(B)Lines76–81provideevidenceforthestatementdirectlyabove:“SETIhasnothingsubstantialtoshowforitself.”Theselinesgoontoshowhowa72-secondanomalyin1977thatwasneverrelocatedistheclosestthingwehavetoevidenceofoutsidelife.Hence,(B)iscorrect.(A)impliesthatthisevidenceprovedalienexistence,whichisfalse.Andwehavenoevidencefor(C);weonlyknowthateffortshavefailedinthepast.(D)iswrongbecauseitassumesthatthesignificanceofthisexampleisEhman’s“Wow!”ratherthanitsfeeblerankastheclosestwe’vecometoproof.

48.(D)Lines99–103givepossibleexplanationsforwhywehavenotcomeintocontactwithETI’seveniftheydoexist.Thereisnoevidenceastotheauthor’sopinion,asin(A).Choices(B)and(C)areincorrectbecauseneitherarecommendationnorsuggestionisgivenintheselines.

49.(A)Endeavorworksbesthere,sincethecontextrefersto“theventuretosolveFermi’sParadox.”Thelineissayingthattheattempttosolvethequestionisongoing,sotheotherchoicesareimprecise.(B)and(D)implybiastowardtheundertaking.Choice(C)couldonlybelogicalifweweresaying“thebusinessofsolving,”not“thebusinesstosolve.”

50.(C)Sincethegraphshowsmoreandmoreofthebudgetbeingdesignatedtoexplorationmissions,wecanassumethenextstepwouldbe(C).Weknow“exoplanets”arebeingdiscoveredonanalmostweeklybasisfromthepassageinformation,sothisrepresentsalogicalinference.Fundingisdecreasingfor(A)and(D).Andweknowfromthepassagethat(B)doesnotexist.

51.(D)Thesumofthefourcurveswillgivetheentirebudget,sothebudgetwentfromabout18millionto21million.Wecanruleout(B)and(C)sincethesecomponentsshowadecrease.Finally,thegraphindicatesthatexplorationmissionsdoubledfromabout5million(18–13)to10million(21–11),whichisnotrepresentativeofthemoregradualincreaseintheprogrambudget.Aeronautics,ontheotherhand,wentfromabout5to8million,andmaintainedaboutathirdoftheentirebudgetconsistently,soitiscloselymirroredwiththebudgetitself.

52.(C)Toapproachthisquestion,considerthatfundingformannedspaceshuttlesandstationshasdeclineddramatically.So,wearelookingforevidencetoexplainthatdrop.Lines67–70offerpossiblesupport,sincetheydetailanescalationin“unmanned”spaceobservatories.ItisplausibletoinferthatNASAdoesn’thavetodevotespendingtomannedstationswhenunmannedstationsarenowmorecommon.(A)referencesinitiativestowardsendingourownmessages.(B)focusesontelescopesandradioresearch.Andchoice(D)occurredin1977,soitwouldn’tbehelpfulwiththegivengraphdata.

PracticeTest3

ANSWERSEXPLAINED

1.(C)Theshortstorytracesthelonelychildhoodofatimidboywhofailstofitinorfeelloveduntilhefindsafriendinanillpuppy.Whenthepuppytragicallydies,theboymysteriouslydisappears.(B)and(D)relatetodetailsofthestory,butarenotmainideas.Andalthough(A)istrue,itfailstocapturethegistoftheentirepassage.

2.(D)Lines1–3provideexpositionthatexplainswhyDavidandhisparentsarethewaytheyare,so(D)isthebestchoice.Hissiblingsarenotintroduceduntilthefollowingsentence,making(A)incorrect.(B)statesthatheistimid,butthelinesdonotofferascientificexplanationofthegeneticreasonsbehindthis.Choice(C)movinginwiththeHardings,referstoanoutcomeofDavid’spersonalityandhisparents’stateofmind,ratherthanacause.

3.(B)Lines56–57refertoDavid’sopinionofMichaelasa“combativestranger”whodoesn’tsharehisinterests.Therefore,distantandquarrelsomedescribehisimpressionmostaccurately.(A)istheoppositeofthisimpression.ItistruethatMichaelismomentarilyembarrassedanddoesn’tshowtheaffectionforthedogthatDaviddoes,butthesearenotrepresentativeofhispersonalityasawhole.Andthereisnoevidencefor(D).

4.(B)TheselinesbestsupporttheclaimthatDavidfindsMichaeloveralldistantandquarrelsome.(A)givesjustificationsforDavid’smovetotown.(C)referstothereactiontothedog’sdeath.(D)describestheoneinstancethatMichaelisrecruitedtohelpDavid.Hence,(A),(C),and(D)donotindicateDavid’sgeneralimpressionofMichael.

5.(C)Cossetedmakesthemostsenseheresinceoneofitsdefinitionsis“tocuddleorcaresslovingly.”Choices(A),(B),and(D)canbeeliminatedbecause,althoughtheyarewaysDavidmaytreatthedog,theyarenotreflectiveofhishuggingtheanimaltohischestasinline65.

6.(C)Paragraph6referstoDavid’sdiscoveryoftheunhealthyanimalthatquicklybecomeshisfriendandthesolefocusofanotherwiseemptylife.Paragraph7describeshisdevotiontothepup,andhisagonyoveritsdeath.So,choice(C)isthebestoption.(A)and(D)aredetails,butareunrelatedtotheprimarypurpose.(B)isincorrectbecauseDavid’sactionsareaneffectofhissensitivity,ratherthanacauseofit.

7.(B)Lines49–59expressthatdespiteDavid’snewarrangement,heisstillunhappyandmisunderstood.ItmakessensetoinferthatbecausehedidnotfindacompanioninMr.Harding,Mrs.Harding,orMichael,thatwhathewantedmostwasafriend.(A)and(C)areincorrectbecausetheseareboththingshedoesgetwithhismove,butthatdonotfillhisvoid.Finally,althoughthepupbecomesafriendandallowshimtoexperiencethecompanionshipheismissing,itisnotessentialthattheintimacybeformedwithapet.

8.(D)TheselinesgivedirectevidencethatwhateverDavidwaslookingforisfulfilledthroughthefriendshipwithhisnewpet.(A)expresseshisunfulfilledhopethatmovingtotownwouldsomehowhelpgivehislifemeaningandhappiness,butdoesnotdefinehoworwhatmightdothat.(B)reiteratesthathishopeswereunfulfilled.(C)explainsMr.Harding’sreactiontoDavid’snewpet,butfailstoprovidesupportforthepreviousquestion.

9.(D)TheauthordescribesWillie’sfaileddreamsforaprosperousfarminparagraph2,sochoice(D)iscorrect.(A),(B),and(C)usedetailswithintheparagraphtomakeunsupportedassumptions.

10.(C)NoticeablemakesthemostsenseindescribingDavid’sanguishsincethisuseofpalpablemeans“obviousandtangible.”Choice(A)meanstheopposite.(B)and(D)areotherpossiblemeaningsofpalpable,butdon’tfitthiscontext.

11.(A)Thispassagemaintainsaninformativeandobjectivetone,analyzingbothformsofdemocracywithouttakingsides,rulingout(C)and(D).(A)isamoreaccuratechoicethan(B)becausetheauthorfocusesonthedifferences,statingthat,“Thedisparitybetweenthetwoishardlysubtle.”

12.(C)Sincetheauthorsustainsanattitudethatisbothinformativeandimpartial,(C)isthecorrectanswer.Whiletheauthoriscertainlyneutral,heorsheisnotpessimistic,passionate,orpredisposedabouteithersystemofgovernment.

13.(B)Thepassagestatesthattheprevalenceofparliamentaryruleisconnectedto“thelegacyoftheBritishEmpire,”sotheansweris(B).Choice(A)isanaspectofpresidentialrule.(C)and(D),althoughfeaturesofaparliamentarysystem,arenotstatedasreasonsforitspopularity.

14.(A)Inlines7–10,thepassagegivesitsonlymentionofwhichsystemismorecommon,alongwithapossibleexplanation.Accordingly,(A)iscorrect.(B)givesadetailofhowparliamentarysystemsarerun.(C)actuallyreferstoabenefitofthepresidentialsystem.And(D)providesacritiqueofparliamentaryrule.

15.(C)Fromthecontext,weknowthatpresidentialsystemsexpectthePresidenttobebothgovernmentleaderandnationalfigure.Contrastingly,aparliamentarysystemhasagovernmentexecutiveseparatefromthenationalicon,mostcommonlyamonarch.So,(C)isthecorrectchoice.

16.(B)Line39referstothenation’sadvance,soprogressistheonlyappropriateanswer.ThepassageisnotindicatingthatthePresidentisworkingtowardamonetaryoranimperialisticgoal.

17.(A)Theindicatedparagraphaddressestheprosandconsofthepresidentialmodel,making(A)correct.(B)isnotaccuratebecausetheparliamentarymodelisnotmentioneduntilthenextparagraph.(C)isimprecisebecauseitprovidesdetailsoftheaforementionedparagraphbutdoesnotencompasstheperceiveddisadvantagesaswellasthebenefits.(D)iswrongbecauseitdoesn’tgivespecificexamplesofcountriesfollowingthissystemintheparagraph.

18.(A)Inline70,privyreferstothepresidentialmodel’spredispositiontodiscordandinefficiency.Hence,wecouldsaythesystemissusceptible,meaning“vulnerable.”Choice(D)isanearsynonym,butnotaspreciseawordchoice.

19.(D)Lines88–97depictsomenegativefeaturesoftheparliamentarymodel,statingthatsincepoweriscentralized,“tyrannyofthemajority”isprobable.Thisconcernisrelatedtotheabsenceofachecks-and-balancessystemandaccordingtolines92–97canresultinminoritypartiesbeingignored.Thisconcernreferstothemajoritypartyratherthanasolerulerasin(A).Itisnonsensicaltosaycriticswouldbeconcernedwhenhumanrightsarerespectedasin(B).Finally,(C)isrelatedtoproductivity;amajoritycouldbebothproductiveandunjust.

20.(C)Theselinesgivedirectevidenceofwhatmighthappenunderthe“tyrannyofthemajority.”Choices(A)and(B)givestrongpointsratherthanconcernsoftheparliamentarysystem.And(D)referstotheauthor’sneutralconclusion.

21.(B)OtherthantoprovidedefinitionsandrefertoSanger’sownretrospect,thispassageisstructuredchronologically,movingsequentiallythroughtimeinregardtoSanger’saccomplishments.Thiscanbecategorizedas(B),mostlychronological.

22.(B)Lines4–5statesthatproteins“arepresentinandvitaltoeverylivingcell.”Thelineisindicatingthatproteinsarenecessaryorfundamental,hence,essentialisthecorrectword.Itisnotappropriatetosayproteinsarevibrant,biological,ordynamictoeverycell.

23.(B)Sanger’sownattitudeisreferencedtwicewithinthepassage:onceinreferencetohisfirstNobelPrizeandonceinhisobituary.Bothsentimentsindicatemodestyorhumility,making(B)correct.(A)isnotevidenced.(C)isanearantonym,and(D)isadefiniteantonym.

24.(D)Lines81–85documentSanger’saccountofhimselfas“academicallynotbrilliant,”andtherefore,providedirectevidenceforhishumblenature.(A)introducesSanger.(B)explainswhyhereceivedhisfirstNobelPrize,butnothisreactiontoit.And(C)referstotheimpacthisworkhadonfuturediscoveries.

25.(C)ThisquestionisaskingforlinesthatindicateSangerwasoptimisticorassuredinhiswork.Lines60–63arethebestevidenceastheselinesrefertothefactthathenamedhismoreefficientsequencingmethodafterhimself.Wecanassumehewantedittobeassociatedwithhisnamebecauseheknewithadgreatimplicationsforchemistry.TheotheroptionsgivefactsaboutSangerbutdonotreflectanyofhisownfeelingstowardhiswork.

26.(A)Lines37–38isSanger’sself-reflectionregardingtheworkthatearnedhimaNobelPrizein1958.Heishumorousandhumble,sotheonlyadjectivethatisaccurateis(A),playful.Sombermeans“grave,”whilebombasticis“pompous.”Sanger’squoteisperhaps“carefree,”butnotcareless,whichmeans“indifferentorunworriedtothepointofnegligence.”

27.(D)Line67saysthatSangeremployedor“used”hismethodtodecodemitochondria.Thus,(D)istheprecisewordchoice.Hesucceeded,so(A)isimprecise.Themethodwasalreadycreated,so(C)ismisleading.And(B)isanothermeaningforemployedthatreferstoemploymentinaworkforce.

28.(A)Lines69–76outlinesomeofthewaystheSangerMethodcruciallyimpactedthefieldofchemistry.Here,theauthorshowshowSangerusedhismethod,howotherscientistsusedhismethod,andtheeventualoutcomesofthatcombinedresearch.Thus,(A)iscorrect.Theselinesdon’taddressobjectionsornegativeconsequence.So(B)and(D)arewrong.Choice(C)isalsoincorrectbecausethelinesdon’trefertohismethodologyasmuchastotheimplicationsofhisdiscoveries.

29.(D)Thisquestionreferstothegraph.SincethegraphhasspecificcategoriesfortheUnitedStates,UnitedKingdom,Germany,andFrance,wecaninferthesearethecountrieswiththelargestnumbersofwinners.Therefore,tobeputinthe“other”category,thecountrywouldhavetohaveapercentagelowerthanthatof6%.So,(D)iscorrect.

30.(D)Toapproachthisquestion,findtheratioofGermanytoGermanyplustheUnitedStates.Together,GermanyandtheUnitedStatesmakeup44%ofthegraph,sotheratiowouldbe11:44or1:4.Theratio1:4isthesameas¼or0.25,asin(D).

31.(C)ThislinereferstoWashington’sclaimthateconomicgainwillcomeaswe“putbrainsandskillintothecommonoccupationsoflife,”soordinaryisthecorrectsynonym.Intheverynextlines,wecanseethatWashingtoniscomparingaveragedutiestomoreelevatedambitions.Assuch,wecaninferthathemeanstypical,blue-collaredwork.Theotherchoicesgivesynonymstocommon,butsuggestcommunality.

32.(C)Lines14–17pointto“theincomingofthoseofforeignbirthandstrangetongueandhabits,”meaningforeignimmigrants.(A),(B),and(D)donotaddressthepermanenceofthepersonscomingtoliveandworkinanothercountry.

33.(C)Toparaphrase,Washingtonsuggeststhathistoricalloyaltyisagoodreasonforwhitestotrustblacks,andthengoesontoinsinuatethatsocialseparationisfineifitisaccompaniedwitheconomicunity.So,(C)accuratelydepictsthegeneralpurposeoftheselines.Heisnotarguingforhostilityorsuppressiontowardforeigners.Norishisargumentfoundedoncaringforthesick.

34.(B)Lines52–56statethattheNorthyearnedfor“peacefulcooperation”becausetheywere“wearyoftheraceproblem”and“investinglargelyinSouthernenterprises.”Thus,wecaninfertheNorthisconcernedwithcommercialadvancementinsteadofmoralityasin(A)ortruth(C).Additionally,thereisnoevidencethattheconcernlaywithlocationaldistinctionasin(D).

35.(B)DuBoisstatesthatWashingtonrepresents“adjustmentandsubmission,”and“acceptstheallegedinferiorityoftheNegroraces.”Hence,(B)iscorrect.DuBoisfindsWashington’sargumentflawedbecauseofitsneglectfor(A).Similarly,(C)isincorrectbecauseDuBoisbelievesthatWashingtonisyieldingmoralityinfavorofeconomicadvancement.Thereisnoevidenceforchoice(D).

36.(D)Lines78–84specificallystatethatWashington“withdrawsmanyofthehighdemands,”advocatingfora“policyofsubmission.”TheselinesprovidedirectevidenceofDuBois’perspectiveonWashington’sefforts,andsosupportthepreviousquestion.Choices(A)and(B)communicatevaryingpublicresponsetoWashington,ratherthanDuBois’ownview.And(C)impliesthatthecurrenttimeisapivotalmomentinhistory,makingWashington’smenialpursuitsthatmuchmoreinexcusable.

37.(D)Directionistheappropriatewordchoice.Here,DuBoisarguesthatWashington’smethodtakesaneconomiccast.Thelineisnotreferringtoaneconomicevent,constraint,orthrow,butinsteadreferstoaninclinationorpath.

38.(A)Washingtonisconcernedwithadvocatingforawayforward,whileDuBoisisinterestedincriticizingwhatheseesasWashington’simmoralcompromise.So,(A)isright.Passage1doesn’trelyonstatisticsmaking(B)incorrect.(C)and(D)havetherelationshipreversed.

39.(A)Toapproachthisquestion,considerthatwhatWashingtonconsidersadvancement,DuBoislabelscompromise.Thus,wecaninferthataleveldownforWashingtonwouldevenbeafurtherrepressionaccordingtoDuBois,so(A)iscorrect.(B)isasynonymforcompromise,andclearlytheauthorsdon’thavethesameopinion.(C)istooextreme.(D)inaccuratelyindicatesthatWashingtonismoreprogressivethanDuBois.

40.(D)TheselinesgiveDuBois’opinionofWashington’sdispositionas“theoldattitudeofadjustmentandsubmission,”andsocanbeusedtosupporttheideathatWashingtonislessliberal-minded.Hismoreconservativedemeanorgivesustheevidencetoanswerthepreviousquestion.Choices(A)and(B)givedetailsofWashington’sargument,butdon’tgivethereaderaclearideaofhowthetwoauthorsdiffer.And(C)ishowPassage2introducesWashington,butdoesnotprovideinsightofDuBois’attitude.

41.(A)Lines86–93statethatWashingtonasksblackstogiveup“politicalpower,”“civilrights,”and“highereducation”infavoroffarlessloftygoals.IfWashingtonweretorespondtoDuBois,hemightrespondwithlines8–13wherehearguesthatblackswillnotprosperuntiltheyembracecommonlabor.(B)and(C)refertoWashington’spleaforworkonbehalfoftheAfrican-Americanpopulation.(D)provesDuBois’pointratherthanrespondingtoit.

42.(A)Thispassagecanbeclassifiedstructurallyasamovefromageneralsummaryoftheinfluenzavirustoaveryscientificanalysisofhowitoperatesandevolvesatthemolecularlevel.Hence,(A)isthebestanswer,withsurveymeaninga“generalexamination”ratherthan“agroupofquestionsgiventoawideaudiencetogatherinformation.”Theauthorisnotarguingasin(B),orpresentingresearchsummariesasin(C)and(D).

43.(A)Lines12–19refertothefactthatinfluenzaremainsaviablethreat,andbecauseofitsuniquedisposition,islikelytostaythatway.Choices(B)and(D)statetheopposite,arguinginsteadthatthevirusislikelytobekilledoffandisessentiallyharmless.(C)isnotcorrectbecauseinfluenza’sinabilitytobeconquered,notitsseverityiswhatmakesitunique.

44.(D)Theauthordefinesepidemicasbeingcontainedtoagivencommunity,anddifferentiatesitfrompandemic,inwhichtheincidencesoccursoveramuchlargerregion.Fromthetable,wecanseethatonlytheoutbreaksin2005and2013wereconfinedtoaregion,whiletheotherswentglobal.

45.(D)Lines106–108describethegeneticchangesassociatedwithantigenicshift,statingthatvia“viralreassortment”theparticleswillproducecompletelynewproteins“towhichtheentirehumanraceisvulnerable.”Whiletheantigenicdriftcanalsoresultinrenewedvulnerability,itistoviralstrainsofwhichonewaspreviouslyimmune.Therefore,thechangeswouldnotbeasgreatasthoseassociatedwithantigenicshift.(Anotherwaytothinkaboutitisthatgeneticdriftresultsfromrandompointmutations,whicharethesmallestchangespossible.)Likewise,theauthorusestheexampleofagenomesplitinabirdthatiscapableofinfectinghumansmaking(D),ratherthan(C),thecorrectchoice.

46.(D)Lines101–108givetheinterspeciesgenomeexchangeexampleandexplain“viralreassortment,”successfullyprovidingevidenceforwhytheantigenicshiftresultsingreaterfundamentalchangesthanantigenicdrift.Choices(A)and(B)describetheoutermostviralproteins,butdonotaddressdriftvsshift.Andchoice(C)isincorrectbecauseitreferstoantigenicdriftonly,whichdoesnotaltergeneticstructureassignificantly.

47.(C)TheindicatedparagraphexplainsantigenicdriftandsuggeststhatinfluenzaAisparticularlydangerousbecauseit“ispronetoamuchhigherrateofmutation.”Therefore,itcanbesaidthattheauthor’spurposeistoexplainthereasonswhyonespeciesofinfluenzavirusisespeciallydamaging,asin(C).(A)incorrectlyunderstandsantigenicdriftasresultinginflu,wheninrealityitisaviralstrandthatundergoesmanypointmutationstorenewvulnerability.Andthisparagraphdoesn’ttalkaboutantigenicshiftorvaccinations,so(B)and(D)canberuledout.

48.(B)CapacityfitsherebecausethelinereferstoinfluenzaA’slackofabilitytocorrectitsgeneticmaterial.Thistypeoffluvirussimplydoesn’thavethecapabilitytodoit.Theotherwords,althoughsynonymsofability,refertoatendencytoperformwellratherthanactualabilitytoperformatask.

49.(B)Choices(C)and(D)canbeeliminatedbecauseHongKongFluresultedinonlyalessthan0.1%fatalityrate,andAvianFluconsistedonlyof840cases.Sincetheremainingtwochoiceshavesimilarfatalityrates,wecanassumethattheoutbreakwhichaffectedmorepeopleresultedinthegreaternumberofdeaths.AsianFluisestimatedtohaveaffectedapproximately3.1millionmorehumansthanRussianFlu.

50.(B)Accordingtolines76–82,scientistsareabletoclassifystrainsofinfluenzabasedontheantibodiesproducedtofightthem.So,wecaninferthatoutbreaksclassifiedasthesamesubtypewouldcausethesamereactioninthehumanbody.(Italsosaysinlines78–82thatbothoutbreakswerecausedbyH1N1).Withinthetable,theonlyoutbreakswithmatchingsubtypesoccurredin1918and2009.

51.(C)Lines78–82indicatethat“SpanishFlu”and“SwineFlu”werecausedbyidenticalstrandsoftheinfluenzavirus.Theselinescanbeusedintandemwiththetabletoanswerthepreviousquestion.(A)merelylistswell-knownfluoutbreaks.(B)givesthethreespeciesofthefluvirus.(D)referstotheimplicationsofantigenicshift.

52.(C)Thebestwaytoapproachthisquestionistolookforanomaliesinthetablefortheoutbreaksof2005and2013.Thesetwoinstanceswerecontainedgeographicallyandheldveryhighfatalityratescomparedtotheotheroutbreaks.Therefore,wecaninferthathighfatalityratesallowedthediseasetodieoutbeforereachingaglobalscale.Thereisnoevidencefor(A)or(B).And(D)isnotaccuratebecausetheAsianFlualsohaditssourceinbirds,butaffectedcolossalnumbers.